Você está na página 1de 97

P

02 – O tom predominante no texto é:


a) crítico, pois, sob o pretexto de mostrar a possibilidade de
um suicídio, o autor transmite sua visão de mundo, que
pode ser depreendida das ações atribuídas aos
ORTUGUÊS - EEAR personagens.
1/2001-TURMA A b) melancólico, visto o tema central versar sobre o estado
depressivo em que se encontram as pessoas que vivem
Um dedo na multidão nas grandes cidades.
(Dorival Coutinho da Silva) c) pessimista, pois é evidente a disposição de espírito do
Um homem vai se atirar do sétimo andar de um edifício autor, que encara tudo pelo lado negativo, como comprova
em pleno centro de São Paulo. a predominância de substantivos e adjetivos desse mesmo
Não é o primeiro. valor.
Nem será o último. d) trágico, já que a ação do homem suicida infunde piedade e
Afinal nem todos os homens são poetas. terror na multidão que se aglomerava na praça.
Que os poetas desafogam na lira os desencantos.
03 – A última estrofe do texto permite a seguinte leitura:
Um homem comum vai jogar as migalhas da vida a) A indiferença da multidão desmotivou o homem anônimo a
Aos pombos famintos jogar-se do sétimo andar.
Que se aglomeram. b) A melancolia e o desapontamento da multidão se devem
ao fato de a vida voltar à rotina, visto que o homem não se
O grupo que ensaia capoeira na praça jogou do sétimo andar.
Ao som do berimbau c) A multidão, dispersa devido à melancolia do dia-a-dia, não
Deixa de ser atração. se importa com o homem que tenta se jogar do sétimo
Todo mundo quer ver o homem que vai pular andar.
Do sétimo andar. d) A vida só continua porque o homem não se suicidou.
Doida escalada aos prédios vizinhos, O melhor ângulo,
Posições estratégicas... 04 – Diz-se do texto, quanto à estrutura, que: se caracteriza
Quem sabe escapará algum detalhe como descritiva, pois detém-se na observação dos detalhes do
Nos noticiários de amanhã? centro de São Paulo.
a) não se caracteriza como narrativa, uma vez que foi escrito
O estafeta liga para a repartição: em verso e detém-se ao lirismo do poeta.
“Que venham logo! Pois um homem vai pular do sétimo andar. b) se caracteriza como narrativa, que se utiliza de recursos
E tem até TV!” descritivos e reflexivos para reforçar seu conflito.
c) não se pode defini-la, pois apresenta trechos descritivos e
Expectativa pesada. Olhos fincados no sétimo andar. dissertativos.
Uma palavrinha ao companheiro do lado, Um acocorar para
prender 05 – “Do Paraíba as águas a rolar
os cadarços, Uma visita aos sanitários... A tua história bem longe irão levar
Nem cogitar! Dos teus alunos e mestres desta casa
Aparecida há de sempre se orgulhar.”
Tampouco amendoins e pipocas O texto utiliza-se de uma figura de linguagem que se
Antecipam os louros na espera. caracteriza como conotativa. É ela:

Mas agora um dedo na multidão, a) prosopopéia.


Entre mil indicadores algozes, eretos, b) eufemismo.
Pressiona os três dígitos c) hipérbole.
Da salvação. d) metáfora.
E só depois de uma operação súbita
Audaciosa 06 – No período “O Rio é um feriado. São Paulo é uma
Precisa segunda-feira.” (Arnaldo Jabor), a figura de linguagem
Remove-se o trágico misantropo encontrada é a:
Para o interior do prédio a) catacrese.
(Mas não de si mesmo). b) metáfora.
c) onomatopéia.
Cortinas cerradas. d) prosopopéia.
Silêncio na praça.
O homem não se jogou do sétimo andar 07 – Em “As águas do rio gemiam alto, soluçando entre seixos”
E a multidão (Guimarães Rosa), temos a figura de linguagem:
se dispersa a) catacrese.
Melancólica b) metonímia.
Desapontada c) prosopopéia.
Porque a vida continua.... d) onomatopéia.

As questões de 01 a 04 referem-se ao texto acima. 08 – Ler os enunciados abaixo e assinalar a opção que
preenche, respectivamente, as lacunas:
01 – “Mas agora um dedo na multidão, I – A _________ do IBGE de fazer um ___________ é de
Entre mil indicadores algozes, eretos, grande utilidade para o País.
Pressiona os três dígitos II – A __________ de terras aos posseiros foi decidida pela
Da salvação.” Assembléia Legislativa na última ___________ .
A idéia deduzida no trecho acima encontra-se na III – __________________ as razões pelas quais cometera o
alternativa: crime, o réu acabou sendo _______________ pelo tribunal,
a) Entre aqueles mil indicadores, somente um apontava seu que o julgou inocente.
dedo para a salvação do homem.
b) Entre mil dedos na multidão, apenas um se compadeceu a) tenção – censo / cessão – sessão / Discriminadas –
daquele homem comum e acionou o corpo de bombeiros. descriminado
c) Naquela multidão, somente uma pessoa continuava b) tenção – senso / seção – sessão / Descriminadas –
indiferentemente a pressionar os dígitos de seu telefone discriminado
celular. c) tensão – censo / cessão – sessão / Descriminadas –
d) Uma pessoa apenas, no meio daquela multidão de discriminado
algozes, recusou-se a pressionar os dígitos da salvação. d) tensão – senso / sessão – seção / Discriminadas –
descriminado

1
17 – Observar as frases:
09 – Marcar a opção em que as palavras seguem, I – Se eu atirar, te faço um furo no peito seu ordinário. Agora
respectivamente, as mesmas regras de acentuação das mexe aí para ver só, se eu não te mando pro inferno.
palavras destacadas em: “É incrível como certos políticos II – Naquela noite, quando se preparava para ir pro cinema, a
exercem forte influência sobre grande parte da população.” empregada veio correndo lá de dentro, apavorada.
a) util – forceps – assembleia III – O Almirante, satisfeito consigo mesmo, guardou a arma e
b) rouxinol – insolito – benção foi pro cinema. Quando voltou, o sabiá tinha desaparecido.
c) automovel – exercito – femea IV – — Pois tu vais botar o sabiá na gaiola outra vez,
d) carretel – revolucionario – etiope vagabundo. Vai botar o sabiá lá, vai me pedir, desculpas
por tentar roubá-lo.
10 – No texto V – — Agora, suma-se, mas lembre, sempre, que esta arma é
“Onde estão os meus verdes? 45. Eu explodo essa sua cabeça, se o vir perto de minha
Os meus azuis? casa outra vez.
O Arranha-Céu comeu!” (Mário Quintana)
Assinalar a opção cujas frases estão corretamente pontuadas:
Quanto ao processo de formação de palavra, o termo destacado
acima caracteriza-se como: a) I, III e IV.
a) derivação imprópria. b) II, III e V.
b) derivação regressiva. c) II, III, IV e V.
c) justaposição. d) I, II, III, IV e V.
d) parassíntese.
18 – “A vida é combate
11 – O termo sublinhado é um modificador do verbo em: Que aos fracos abate
a) O jogo levou meio dia para acabar. E aos fortes e aos bravos
b) Naquela época, andava cheio de esperança. Só pode exaltar.” (Gonçalves Dias)
c) O resultado da prova foi melhor do que eu esperava. Quanto à função sintática, o termo grifado, no trecho acima,
d) É um privilégio trabalhar próximo a pessoas tão cultas. classifica-se como:
a) objeto direto.
12 – Apresento-lhe Maria Lúcia. b) núcleo do sujeito.
Faço tudo pela felicidade de Maria Lúcia. c) adjunto adnominal.
Se juntarmos as duas sentenças numa única, usando o d) predicativo do sujeito.
pronome relativo, teremos:
a) Apresento-lhe Maria Lúcia, por cuja felicidade faço tudo. 19 – Assinalar a alternativa incorreta quanto à concordância
b) Apresento-lhe Maria Lúcia, cuja felicidade faço tudo por verbal:
ela. a) Acontece que não chegaram as cartas.
c) Apresento-lhe Maria Lúcia, a qual faço tudo por sua b) Daquela massa inerte é que nascerá todos os seres
felicidade. viventes.
d) Apresento-lhe Maria Lúcia, a quem faço tudo pela c) Haviam conseguido cargos importantes na política.
felicidade dela. d) Seis bilhões de pessoas habitam o planeta Terra.

13 – Assinalar a única opção em que a colocação pronominal é 20 – “Para entender os problemas brasileiros, precisamos ter
inadmissível de acordo com a norma culta: uma visão ampla de nossa situação no contexto mundial.”
a) Acusaram-no de caluniador. No período acima, a oração sublinhada expressa a idéia
b) Não respondiam, embora me ouvissem. de:
c) Nos últimos dias, me aconteceram coisas incríveis. a) causa.
d) Ele não se conformou por nos opormos a suas idéias. b) concessão.
c) conclusão.
14 – Observando a norma culta da linguagem, qual das opções d) finalidade.
abaixo não permite voz passiva?
a) A imobiliária do meu tio alugou todas as casas velhas da 21 – Quanto à concordância verbal, assinalar a forma correta:
vila. a) A maioria do povo parece ignorarem a medida de sua
b) Napoleão queria a suas tropas como a um irmão. mediocridade.
c) Os criminosos recebem o merecido castigo. b) A maioria do povo parecem ignorarem a medida de sua
d) Nós arrumamos o salão para a festa. mediocridade.
c) A maioria do povo parece ignorar a medida de sua
15 – Indicar a opção em que se verifica a voz passiva: mediocridade.
a) Após um longo período de recessão, admitiram-se novos d) A maioria do povo parecem ignorar a medida de sua
empregados naquela fábrica. mediocridade.
b) Cumprimentaram-se os adversários antes de a partida
iniciar. 22 – Observar a oração: “Tarde da noite, entrou pé ante pé.”. O
c) Os amantes despediram-se no aeroporto entre beijos e verbo destacado é intransitivo. Assinalar a alternativa que
abraços. apresenta também um verbo intransitivo:
d) Os candidatos queixaram-se da extensão da prova. a) Trabalhou a escultura com afinco.
b) Os operários pareciam felizes ontem.
16 – Marcar a frase em que a omissão da pontuação acarreta c) “Quebrei a imagem dos meus próprios sonhos.”
mudança de sentido: d) Existem reformas pendentes nas áreas política e
a) A chuva de ontem foi tão forte, que os bairros periféricos econômica.
ficaram inundados.
b) As obras governamentais são regularmente interrompidas, 23 – ANULADA
já que não há uma continuidade na realização dos projetos.
c) Os manifestantes fizeram uma rebelião, a fim de que as 24 – Assinalar a alternativa em que a regência está correta:
autoridades tomassem as devidas providências quanto ao a) As pessoas, naquele dia, foram amáveis; simpatizei-me
reajuste salarial. com elas.
d) Todos acreditavam no mestre, que lecionava há mais de b) Depois de tantos fracassos, ainda custei a reconhecer o
quinze anos. problema.
c) Nunca me esquecerei o seminarista batendo na porta da
sala do reitor.
d) Procedeu-se à apuração e o vencedor foi mesmo o Dr.
Lupicínio.

2
25 – Assinalar a opção que preenche corretamente as lacunas d) “Façamo-nos também desse modo videntes: olhemos
das frases abaixo: devagar para a cor das paredes, o desenho das cadeiras, a
I – Vamos _____ outras partes. transparência das vidraças, os dóceis panos tecidos sem
II – Vossa Senhoria não pode prender-me _____ toa. maiores pretensões.”
III – O abaixo-assinado vem dar os parabéns _____ Vossa
Senhoria. 03 – Com relação ao conteúdo do último parágrafo, é possível
IV – Era melhor pensar numa cama igual _____ de Seu Tomás. se fazer a seguinte interpretação:
a) as – à – a – à a) o verdadeiro sentido das coisas encontra-se em suas
b) às – a – à – à formas e cores; ou seja, em seu valor estético.
c) às – à – a – à b) a harmonia das proporções e a beleza exterior das coisas
d) às – a – a – a muitas vezes não corresponde ao seu verdadeiro sentido.
c) assim como os pintores e fotógrafos, devemos desenvolver
1/2001-TURMA B nossa faculdade de visão sobrenatural por meio da
observação.
d) é preciso saber que a carga de experiências contidas nas
Da solidão
coisas humildes independe do trabalho humano.
Há muitas pessoas que sofrem do mal da solidão. Basta que
em redor delas se arme o silêncio, que não se manifeste aos
04 – A frase que apresenta um defeito de estilo – a ambigüidade
seus olhos nenhuma presença humana, para que delas se
– é:
apodere imensa angústia: como se o peso do céu desabasse
a) Diante do ocorrido entre você e seu filho, fiquei sem saber
sobre sua cabeça, como se dos horizontes se levantasse o
se o que eu disse concorreu para agravar a situação.
anúncio do fim do mundo.
b) A clareza é essencial a todo escritor, pois facilita para
No entanto, haverá na terra verdadeira solidão? Não
quem lê a percepção rápida do pensamento.
estamos todos cercados por inúmeros objetos, por infinitas
c) A prosa ou o poema devem ser extravasados numa
formas da natureza e o nosso mundo particular não está cheio
linguagem espontânea, colorida como a fala.
de lembranças, de sonhos, de raciocínios, de idéias que
d) Meu pai recebeu uma carta em que o irmão lhe contava
impedem uma total solidão?
como sua mulher sofrera um acidente de automóvel.
Tudo é vivo e tudo fala, em redor de nós, embora com vida e
voz que não são humanas, mas que podemos aprender a
05 – Observar:
escutar porque muitas vezes essa linguagem secreta ajuda a
I – “Ando tão à flor da pele
esclarecer o nosso próprio mistério. Como aquele sultão
que qualquer beijo de novela me faz chorar
Mamude, que entendia a fala dos pássaros, podemos aplicar
ando tão à flor da pele
toda nossa sensibilidade a esse aparente vazio de solidão: e
que teu olhar flor na janela me faz morrer.”
pouco a pouco nos sentiremos enriquecidos.
(Zeca Baleiro)
Pintores e fotógrafos andam em volta dos objetos à procura
O eu-lírico se apresenta muito sensível, provavelmente por
de ângulos, jogos de luz, eloqüência de formas, para revelarem
estar apaixonado ou carente.
aquilo que lhes parece não só o mais estético dos seus
II – “Passas sem ver teu vigia
aspectos, mas também o mais comunicável, o mais rico de
catando a poesia
sugestões, o mais capaz de transmitir aquilo que excede dos
que entornas no chão.”
limites físicos desses objetos, constituindo, de certo modo, seu
(Chico Buarque)
espírito e sua alma.
O eu-lírico expressa a indiferença da amada, que o inspira a
Façamo-nos também desse modo videntes: olhemos
criar.
devagar para a cor das paredes, o desenho das cadeiras, a
III – “Se a noite e o dia passassem
transparência das vidraças, os dóceis panos tecidos sem
como nuvens, sem cadeias,
maiores pretensões. Não procuremos neles a beleza que
e os instantes da memória fossem vento nas areias!”
arrebata logo o olhar, o equilíbrio de linhas, a graça das
(C. Meireles)
proporções, muitas vezes seu aspecto  como o das criaturas O eu-lírico expressa o desejo de que o dia e os instantes
humanas  é inábil e desajeitado. Mas não é isso que sejam mais duradouros.
procuramos, apenas: é o seu sentido íntimo que tentamos IV – “Eu deixo a vida como deixa o tédio
discernir. Amemos nessas humildes coisas a carga de Do deserto o poento caminheiro.”
experiência que representam, e a repercussão, nelas sensível, (Álvares de Azevedo)
de tanto trabalho humano, por infindáveis séculos. O eu-lírico sofre por deixar a vida.
Cecília Meireles
Quanto à linguagem conotativa, estão corretas as
As questões de 01 a 03 referem-se ao texto acima. afirmativas:
a) I e II. c) II e IV.
01 – De acordo com o ponto de vista adotado no texto, é b) I e III. d) III e IV.
correto afirmar que:
a) a verdadeira solidão existe somente no nosso mundo 06 – “Mesmo com toda fama
particular, pois lembranças, sonhos, raciocínios e idéias Com toda Brahma
são conceitos abstratos. Com toda a cama
b) a solidão é um estado de espírito em que algumas pessoas Com toda a lama
se encontram quando estão angustiadas. A gente vai levando
c) a solidão é, para muitas pessoas, a ausência de A gente vai levando
manifestação humana ou simplesmente a presença do A gente vai levando essa chama.” (Chico Buarque)
silêncio.
d) a solidão é o estado ou a situação ideal para pintores e As palavras grifadas estão usadas no sentido conotativo,
fotógrafos transmitirem o espírito e a alma dos objetos. constituindo:
a) duas metonímias.
02 – O texto utilizado como unidade de análise é uma b) uma metáfora e uma hipérbole.
dissertação subjetiva sobre a solidão. Assinalar a alternativa c) uma metonímia e uma metáfora.
que não se aplica diretamente ao enunciador do texto: d) uma metáfora e uma prosopopéia.
a) “Há muitas pessoas que sofrem do mal da solidão. Basta
que em redor delas se arme o silêncio, que não se
manifeste aos seus olhos nenhuma presença humana, ...”
b) “Tudo é vivo e tudo fala, em redor de nós, embora com
vida e voz que não são humanas, mas que podemos
aprender a escutar porque muitas vezes essa linguagem
secreta ajuda a esclarecer nosso próprio mistério.”
c) “ ... podemos aplicar toda a nossa sensibilidade a esse
aparente vazio de solidão: e pouco a pouco nos sentiremos
enriquecidos.”
3
07 – “O cafezal é a soldadesca verde 13 – Assinalar a alternativa em que o conectivo “e” não pode ser
que salta morros na distância iluminada interpretado com valor adversativo:
um dois, um dois, de batalhão em batalhão, a) Amou muito e não foi amada.
na sua arremetida acelerada contra o sertão.” b) O telefone tocou e, como estava magoada, ela não
(Cassiano Ricardo) atendeu.
c) Estudou Português muitos anos e desconhece o léxico da
Nos versos assinalados, ocorrem, respectivamente: Língua.
a) metáfora e prosopopéia. d) Escreveu uma carta para matar a saudade e colocou-a no
b) comparação e prosopopéia. correio imediatamente.
c) comparação e hipérbole.
d) metáfora e eufemismo. 14 – Estabelecer a correlação entre o sentido e o termo grifado.
Em seguida, assinalar a alternativa que apresenta a seqüência
08 – Colocar C para certo e E para errado com relação à correta:
transposição do discurso direto para o indireto. 1  Causa 2  Comparação 3  Conformidade
I – ( ) Amável, a senhora me perguntou: ( ) Confesso que eu escrevo de palpite, como outras pessoas
 Seu pai como vai? tocam piano de ouvido.
Amável, a senhora me perguntou como ia meu pai. ( ) Vinham em bandos, descansavam e, como em redor não
II – ( ) Ela disse ontem a seu filho: havia comida, seguiam viagem para o Sul.
 É necessário que você faça as compras. ( ) Como era muito seco de maneiras, tinha inimigos que
Ela disse ontem a seu filho que era necessário que ele chegavam a acusá-lo de bárbaro.
fizesse as compras. ( ) Não me respeitava a adolescência , como não respeitava a
batina do irmão...
III – ( )  Conta-me o que há, minha filha  pedia-lhe o pai na ( ) O diabo não é tão feio como o pintam.
carta. a) 2  3  1  1  2 c) 2  1  1  2  3
O pai pediu, na carta, que a filha lhe conte tudo o que b) 1  2  3  2  3 d) 3  1  2  1  1
há.
IV – ( ) Antonieta lhe comunicou inesperadamente: 15 – Preencher as lacunas com a forma adequada dos verbos
 Irei embora desta casa. entre parênteses. A seguir, asssinalar a alternativa que contém
Antonieta lhe comunicou inesperadamente que iria a seqüência correta:
embora daquela casa. I – Se eu os ____ amanhã, poderei dar-lhes o recado. (ver)
Assinalar a alternativa que contém a seqüência correta: II – Se nós as ____ amanhã, poderíamos dar-lhes o recado.
a) E – C – C – E c) E – C – E – E (ver)
b) C – E – C – C d) C – C – E – C III  ____, por meio desta, informar-lhe sobre o assunto vigente.
(vir)
09 – Assinalar a alternativa que apresenta erro de concordância IV – Quando ____ para cá, não sabíamos que a situação seria
verbal: essa. (vir)
a) No Congresso, mais de um parlamentar se ofenderam na
tumultuada sessão de ontem. a) vir – víssemos – vimos – viemos
b) Luxo, riqueza, dinheiro, nada o tentavam naquele momento b) ver – víssemos – viemos – vimos
de decisão. c) visse – virmos – vimos – viemos
c) “As Memórias do Cárcere” são indispensáveis a quem d) vir – virmos – viemos – vimos
acredita na dignidade humana.
d) O Amazonas é um dos rios que cortam a floresta 16 – Considerar os períodos I, II e III pontuados de duas
equatorial. maneiras diferentes.
I – Carlos, o professor de História, pediu-me o livro.
10 – Em cada par, há uma oração certa e uma errada em Carlos, o professor de História pediu-me o livro.
relação à sintaxe de colocação pronominal. II  O piloto nervoso cometeu um erro grave durante a corrida.
I – A - Em se tratando de doença grave, o melhor é levá-lo para O piloto, nervoso, cometeu um erro grave durante a corrida.
o hospital. III  A poluição ambiental, infelizmente, vem crescendo nas
B - Convidar-me-iam para a recepção se encontrassem-me. grandes cidades.
II – A - Nada contentá-lo-á enquanto não tiver a paz interior. A poluição ambiental infelizmente vem crescendo nas
B - Se conseguisse chorar um pouco, sentir-se-ia mais grandes cidades.
aliviado. Com a alteração da pontuação, houve mudança de sentido
III – A - Se se vive triste sem amor, por que não amar? em:
B - A festa, que realizar-se-á na próxima semana, promete
ser um sucesso. a) I e III. c) I e II.
b) II e III. d) I, II e III.
Estão corretas:
a) IA, IIA e IIIB. c) IB, IIA e IIIB. 17 – Observar:
b) IB, IIB e IIIA. d) IA, IIB e IIIA. “ ... não sei se é esta toalha.”
Qual das alternativas abaixo apresenta oração com a
11 – Assinalar a alternativa em cuja frase a colocação mesma classificação da destacada acima?
pronominal segue a norma padrão: a) Se for possível, gostaria de acompanhá-lo.
a) “Me dá um cigarro.” b) Ficaremos decepcionados, se a prova for adiada.
b) “Amo-te como se ama todo o bem/ Que o grande mal da c) Os pedidos serão atendidos, se bem elaborado o ofício.
vida traz consigo.” d) Na verdade, gostaria de perguntar se ainda me considera
c) E eu direi-lhe, no entanto, que não aceito sua posição. seu amigo.
d) Ah! Não roubou-me tudo a negra sorte!
Inda tenho este abrigo. 18 – Observar a concordância nominal nas orações abaixo:
I  Anos depois, bastantes verdadeiros se tornaram também
12 – Assinalar a opção que completa correta e outros avisos de meu pai.
respectivamente as lacunas abaixo: II  Bastantes verdades experimentei anos depois do aviso que
O pai, _____ era fortemente preso às tradições e _____ meu pai me deu.
princípios defendia veementemente, prendia as meninas ao III  São estudiosos as alunas e os alunos deste curso.
sair para trabalhar, _____, não tendo como cuidar delas, IV  O advogado considerou perigosos o argumento e a
temia por sua honra. decisão.
a) que – cujos – pois
b) o qual – cujos os – já que Estão corretas as frases:
c) que – de cujos – pois a) II e IV apenas. c) I, II e III.
d) o qual – por cujos – que b) II, III e IV. d) I, II, III e IV.

4
2/2001-TURMA A
19 – Assinalar a alternativa em que os vocábulos estão
corretamente acentuados:
a) Sabíamos que vocês têm dinheiro para reconstruir o Embora muitos setores públicos ainda não se tenham dado
edifício. conta, a máquina governamental não é uma entidade
b) Todos crêem que a inocência vai guia-lo. autônoma: sua existência só se justifica em função das
c) Convém que só lhe deêm o cartão de crédito após a necessidades dos cidadãos. Há um contrato social, implícito no
maioridade. fato de uma pessoa fixar residência em determinada cidade, que
d) Eles vão propôr dividí-los em grupos homogêneos. pressupõe direitos e deveres. Morar, comer, vestir, ser
transportado como gente até o trabalho, ter acesso à educação,
20 – Está sem erro de concordância a frase: a serviços de saúde e lazer são condições básicas de existência
a) Fazem dois anos que tento ingressar na EEAR. de qualquer cidadão. Pagar impostos, preservar o patrimônio
b) Houveram dias de desespero e revolta. público, respeitar o próximo, por outro lado, são suas obrigações
c) Os sentenciados houveram do juiz a comutação da pena. mínimas. Teoricamente, todos compreendemos isso. Em nível
d) Quando cheguei, já havia saído as pessoas? operacional é que desrespeitamos os contratos. A mesma
pessoa que amaldiçoa o prefeito pelas enchentes é a que varre
21 – Assinalar a alternativa que completa corretamente as a sujeira para dentro da boca-de-lobo ou cimenta todo o terreno
lacunas dos períodos a seguir: de sua casa, impedindo que a água se infiltre solo adentro. O
I – Não foi essa a obra ______ me referi. cidadão que se irrita com os ruidosos vendedores de pamonha
II – Há momentos na vida _______ nunca nos esquecemos. no sábado à tarde liga o som ao máximo enquanto lava seu
III – Os filhos _______ pais são disciplinados são mais felizes. carro no domingo de manhã. O outro, que acha as ruas
IV – O professor, _____ conhecimentos desconfiávamos, foi esburacadas, constrói uma edícula, secretamente, para evitar o
infeliz em sua aula. pagamento correto do IPTU.
a) à qual – de que – cujos – de cujos (Jaime Pinsky)
b) à que – que – dos quais – em cujos
c) a que – de que – os quais – cujos As questões de 01 a 03 referem-se ao texto acima.
d) à qual – que – cujos – de cujos
01 – De acordo com o texto, é dever do cidadão
22 – Mesmo atentos à aula, os alunos não assimilaram o a) ligar o som ao máximo enquanto lava o carro.
assunto, pois a linguagem do professor não era acessível a b) ser transportado como gente até o trabalho.
eles. c) pagar impostos para melhorar a cidade.
a) A frase apresenta dois erros de regência nominal. d) achar as ruas esburacadas.
b) A frase apresenta-se correta quanto à regência nominal e
verbal. 02 – Assinale a alternativa que explica o sentido da passagem:
c) A frase apresenta um erro de regência nominal e nenhum “Teoricamente, todos compreendemos isso. Em nível
de regência verbal. operacional é que desrespeitamos os contratos.”
d) A frase apresenta um erro de regência verbal e nenhum de a) A nossa liberdade termina onde começa a dos outros.
regência nominal. b) O cidadão, que paga impostos, acha-se no direito de agir
conforme a sua vontade.
23 – Assinalar a alternativa que preenche corretamente as c) Todos fazemos parte de uma sociedade justa, pois temos
lacunas das seguintes orações: os mesmos direitos e deveres.
I – Não assisto ___ filmes de guerra ou de violência. d) Na teoria, conhecemos nossos direitos e deveres, mas na
II – Não gaste ___ vista: óculos a prazo. prática nem sempre os observamos.
III – Aliás, temos magníficas perucas ___ Luís XIV.
IV – De segunda ___ sexta não haverá expediente. 03 – Das atitudes abaixo, citadas no texto, assinale aquela em
que não há incoerência no comportamento das pessoas.
a) a – a – à – à b) à – a – à – à a) Fixar residência em determinada cidade e obter direitos e
c) a – à – à – a d) a – à – à – à deveres.
b) Reclamar das enchentes e varrer a sujeira para dentro da
24 – Observar os tipos de discurso empregados nos textos boca-de-lixo.
abaixo. c) Irritar-se com o barulho do vendedor de pamonhas e ligar o
I – “De noite, o rei ouvia os soluços da filha. De que adiantava a som ao máximo enquanto lava o carro.
coroa se a filha da gente chorava à noite?” d) Construir uma edícula secretamente para evitar o
II – “Ariosto Ribas já estava em Caruaru, há cinco anos, não pagamento do IPTU e reclamar dos buracos na rua.
saíra de casa uma só vez. Para que sair? Para aquelas
caras que odiava tanto?” 04 – Assinale a alternativa cujas palavras completam
III – “  Sempre é tarde para os amigos, replicou Sofia, em ar corretamente os espaços do texto abaixo.
de censura.” “Há estados que não ................... o que fazer com tanto
IV – “Uma semana depois, Virgília perguntou ao Lobo Neves, a dinheiro. Isso, nos Estados Unidos. .................. US$ 3 bilhões
sorrir, quando seria ele ministro.” num fundo dos estados que ..................... assistência a famílias
Pode-se dizer que, nos textos acima, temos, necessitadas. A culpa, em parte, é da prosperidade da era
respectivamente, discurso: Clinton. Em 1996, 12,2 bilhões de americanos pobres
a) direto, indireto livre, direto, indireto. dependiam dessa verba. Hoje, ................ 8 milhões.” (Época)
b) indireto livre, indireto livre, direto, indireto.
c) direto, direto, indireto, indireto livre. a) sabe – Sobrou – dão – restou
d) indireto, indireto, indireto livre, indireto livre. b) sabe – Sobraram – dá – restou
c) sabem – Sobrou – dá – restam
25 – Todos os substantivos estão corretamente pluralizados d) sabem – Sobraram – dão – restaram
em:
a) salários-família, salários-mínimo, segunda-feiras, salvos- 05 – “Amor é dado de graça
condutos. é semeado no vento,
b) os leva-e-traz, guardas-mor, peixes-boi, ferros-de-abrir- na cachoeira, no eclipse.”
lata.
c) os disse-me-disse, os reco-reco, vices-reis, aves-marias. A locução destacada acima classifica-se como
d) beija-flores, vaivéns, alto-falantes, tique-taques. a) adjetiva. c) conjuntiva.
b) adverbial. d) prepositiva.

5
06 – Das alternativas abaixo, uma está incorreta quanto à
flexão do substantivo. Assinale-a. 11 – Assinale a opção em que as vírgulas intercalam termos que
a) João era o único testemunha do acidente. desempenham a mesma função da que se verifica no seguinte
b) Na festa, o champanhe era a bebida preferida. período: “Todo o ajuntamento, uma vez que o comandante se
c) Para a garota, o dó era o pior dos sentimentos. aproximava, se pôs de pé.”
d) O marajá e a marani eram muito queridos pelos súditos. a) O discurso, para ser eficaz, deve adaptar-se ao auditório
ao qual é destinado.
07 – “O homem não é existência: é ausência. É a definição b) A poluição ambiental, disse o prefeito, é de
perfeita da falta de algo interior, indefinível e misterioso. Todos responsabilidade de toda a população.
nós, seres humanos, somos, no entanto, exceções, por nossa c) O juiz, cuja decisão deve ser imparcial, decidiu prorrogar o
individualidade e essência únicas; exceções de uma única prazo para anunciar a sentença.
regra, traço de igualdade, a que chamamos de solidão. A d) As tropas de choque, que se aproximavam rapidamente,
solidão é a regra de nossa existência. Em função dela apresentavam um contingente bastante superior ao do
buscamos viver, na tentativa incessante de nos completarmos.” adversário.
A abordagem do tema central permite afirmar que
12 – O covarde (Mino)
a) se utilizou o jogo de figuras por trás do qual o tema está Era uma vez um homem tão covarde,
implícito. tão covarde, tão covarde,
b) não há qualquer relação lógica entre os enunciados, uma que só chegava cedo
vez que se evidencia uma progressão temporal entre eles. porque tinha medo de chegar tarde.
c) são relatados aspectos simultâneos de um objeto particular Nunca disse a verdade,
concreto (o homem) situado num momento definido do como todo covarde,
tempo. mas, mesmo assim,
d) há predominância de conceitos abstratos, que são se alguma coisa negava
utilizados como recursos de argumentação para se fazer era por medo de dizer sim.
referência ao mundo concreto. E porque temia o umbigo
pelas costas apunhalou seu amigo.
08 – Na frase “Seus primos querem que convidem eles para o
passeio.”, há um erro no emprego do pronome. Assinale a Com relação ao texto acima, pode-se dizer que
alternativa que corrige esse erro. I – a oração do verso 3 é subordinada adverbial consecutiva.
a) Seus primos querem que convidem-os para o passeio. II – a oração do verso 3 funciona como principal para a oração
b) Seus primos querem que os convidem para o passeio. do verso 4.
c) Seus primos querem que convidem-nos para o passeio. III – as orações dos versos 4 e 10 são, respectivamente,
d) Seus primos querem que convidem-los para o passeio. coordenada explicativa e subordinada adverbial causal.
IV – as orações dos versos 6 e 8 são subordinadas adverbiais,
09 – Coloque 1 para descrição, 2 para narração, 3 para respectivamente, comparativa e temporal.
dissertação e assinale a alternativa com a seqüência correta. V – nos versos 4 e 10, há, ainda, oração subordinada
( ) A casa de Ramires era grande. O sobrinho brincava no substantiva objetiva indireta.
quintal sem saber que, naquele exato momento, o cão bravo
escaparia. Ninguém ouvia o pedido de socorro. Mais tarde, Estão corretas as afirmações de
quando todos chegaram, encontraram o menino estendido na a) I, III e V.
grama. Ele tinha estudado muito e de repente dormira. b) I, II e IV.
( ) A rua tinha aproximadamente cem metros e nela havia seis c) II, III e IV.
casas populares. Nos terrenos, o capim por crescer. Era uma d) III, IV e V.
linda manhã de verão, e as crianças brincavam de pega-pega. O
sol abraçava tudo com seus longos tentáculos de ouro. 13 – Assinale a alternativa que contém erro de grafia da palavra
( ) A educação brasileira está passando por uma crise que, “porque”.
com certeza, vai afetar a qualidade de vida da sociedade. Os a) “Você fez a viagem porque queria aumentar sua cultura?”
jovens ignoram a leitura de bons livros, os quais tornam-se b) “Sabe por que o jornalista é bom de papo? Porque vive
baús empoeirados que guardam grandes tesouros cujo valor só rodeado de assunto.” (Propaganda da Abril)
poderá ser reconhecido se aberto. Esse desprezo pela leitura c) “Diana tornou-se um fenômeno de popularidade por que
faz com que as pessoas fiquem alheias à discussão de representa duas importantes facetas da mulher moderna:
problemas sociais. uma triste e outra feliz.” (Veja)
a) 2 – 3 – 1 c) 2 – 1 – 3 d) “Físicos americanos do Laboratório Brookhaven acharam
b) 1 – 3 – 2 d) 3 – 1 – 2 indícios de uma nova partícula, o méson exótico, que se
soma às 200 outras da fauna nuclear. Ela explicaria por
10 – Assinale a opção em que não há conotação. que prótons e elétrons se unem no núcleo do átomo.”
a) “Tenho sonhos cruéis; n’alma doente (Istoé)
Sinto um vago receio prematuro.
Vou a medo na aresta do futuro, 14 – No texto abaixo há um discurso direto. Assinale a
Embebido em saudades do presente...” alternativa que o reescreve corretamente em discurso indireto.
“Outro dia o mestre quis
b) “O poeta é um fingidor ditar palavras, na sala.
Finge tão completamente Ao ler borracha com x,
Que chega a fingir que é dor disse ao guri:  pode usá-la.”
A dor que deveras sente.” a) Disse ao guri que podia usá-la.
b)  Pode usá-la, disse ao guri.
c) “pego a palavra no ar c) Disse o guri que a usasse.
no pulo paro d) Disse ao guri: use-a.
vejo aparo burilo
no papel reparo e sigo
compondo o verso”

d) “Tenho apenas duas mãos


e o sentimento do mundo,
mas estou cheio de escravos,
minhas lembranças escorrem
e o corpo transige
na confluência do amor.”

6
15 – Quanto à coesão, assinale a alternativa em que a
explicação a respeito da palavra grifada está incorreta. 20 – Assinale a alternativa que identifica, no texto abaixo,
a) “Rebeliões na Febem mostram que ela não só piora o retirado do Projeto de Constituição, as palavras que pertencem
problema, mas muitos carentes estão ganhando um futuro às classes gramaticais com função de relacionar, conectar.
em outras instituições.” (Veja) – O pronome está “Art. 240 – A educação, direito de cada um e dever do
retomando a palavra Febem. Estado, será promovida e incentivada com a colaboração
b) “Uma mostra inédita em Nova York revela a arte que os da família e da comunidade, visando ao pleno
egípcios faziam na época em que construíram suas desenvolvimento da pessoa e ao seu compromisso com o
grandes pirâmides.” (Veja) – O pronome está retomando repúdio a todas as formas de preconceito e discriminação.”
arte. a) de – e – com – a
c) “Uma reforma feita às pressas na Igreja de Nossa Senhora b) A – um – com – e
das Mercês, em Ouro Preto, quase provoca seu c) de – e – um – seu
desabamento.” (Veja) – O pronome está retomando d) A – com – a – seu
reforma.
d) “O escritor Adolfo Caminha chocou os leitores com seus 21 – “Do Paraíba as águas a rolar
personagens ambíguos.” – O pronome está se referindo ao A tua história bem longe irão levar
escritor Adolfo Caminha. Dos teus alunos e mestres desta casa
Aparecida há de sempre se orgulhar.”
16 – Leia os itens abaixo. A estrofe acima pertence a um hino escolar. Assinale a
I – Metáfora: “Seus braços, autênticos tentáculos, enfeixam alternativa correta com relação à classificação sintática dos
delitos variados.” termos retirados do texto.
II – Metonímia: “...ternos Armani e decotes Versace se dizem a) Do Paraíba — aposto especificativo
chocados com o noticiário...” b) Aparecida — adjunto adverbial de lugar
III – Antítese: “O grande nariz não está na favela do Rio nem na c) A tua história — objeto direto do verbo levar
periferia de São Paulo.” d) Dos teus alunos... casa — adjunto adverbial de companhia
IV – Metonímia: “Pois é preciso que a sociedade comece a
enxergar o nariz invisível que cheira na grande metrópole 22 – “Jardim da pensãozinha burguesa
como cúmplice da mão que segura a motosserra no Acre.” Gatos espapaçados ao sol.”
O termo em destaque nos versos de Manuel Bandeira significa
O exemplo dado condiz com a figura de linguagem somente tornar desenxabido, insosso, amolecido. Quanto à estrutura do
nos itens vocábulo, podemos dizer que
a) I e III.
b) II e IV. I – apresenta afixos.
c) I, II e III. II – seu radical é papa.
d) I, II e IV. III – çados é um sufixo que adjetiva o verbo espapar.
IV – há apenas o acréscimo do prefixo es ao radical papaçar.
17 – “Eu sei que vou sofrer
A eterna desventura de viver Está correto o que se afirma em
À espera de viver ao lado teu a) II e III apenas.
Por toda minha vida.” b) I e II apenas.
Assinale a alternativa correta com relação ao texto. c) IV apenas.
a) que é uma conjunção integrante que introduz uma oração d) I apenas.
subordinada substantiva subjetiva.
b) que é conjunção integrante que introduz uma oração 23 – Leia as frases abaixo e os comentários.
subordinada substantiva objetiva direta. I – “A lua recusou-se a passar por aquele lugar e escondia-se
c) de viver forma locução adjetiva que completa o sentido de entre as nuvens acinzentadas, quando se aproximava
desventura, palavra de valor relativo. delas.” Comentário  A lua se aproximava das nuvens,
d) À espera de é um exemplo de locução conjuntiva que por isso o verbo está no singular e o pronome elas, no
introduz uma oração reduzida de infinitivo. plural.
II – “A lua recusou-se a passar por aquele lugar e escondia-se
18 – “No ano que vem, você será uma pessoa do século entre as nuvens acinzentadas, quando se aproximavam
passado. Melhor começar a pensar novo.” (Veja, 24/05/00) dela.” Comentário  As nuvens se aproximavam da lua,
A oração reduzida destacada equivale à seguinte oração por isso o verbo está no plural e o pronome ela, no
desenvolvida: singular.
a) Comece, pois, a pensar novo. III – “Não se deve aplicar a pena de morte, e sim fazer um
b) Quem sabe, você comece a pensar novo! trabalho de recuperação dos marginais para reintegrá-los
c) Embora talvez você possa começar a pensar novo. à sociedade. Com isso, diminuiriam os processos na
d) Contudo, seria melhor que você começasse a pensar novo. justiça.” Comentário  O verbo diminuíram é intransitivo e
tem como sujeito os processos na justiça.
19 – Assinale a alternativa na qual a pontuação inadequada IV – “Não se deve aplicar a pena de morte, e sim fazer um
gera ambigüidade. trabalho de recuperação dos marginais para reintegrá-los
a) “Apreciando a capacidade notável que esse grande grupo à sociedade. Isso diminuiria os processos na justiça.”
de fiéis torcedores possui para avaliar o potencial de seus Comentário  O verbo diminuiria é transitivo direto: os
craques, vem com a incapacidade notória do mesmo para processos na justiça é objeto direto, e Isso é sujeito.
avaliar seus políticos, um ilustre pensador desconhecido
sugeriu a instauração da República Futebolística do Brasil.” Pode-se afirmar que
(Sandra Ap. Pereira) a) todos os comentários estão corretos.
b) “Diariamente desfilavam diante do portão aquelas mulheres b) os comentários I e III estão incorretos.
silenciosas e magras, latas d’água na cabeça. De vez em c) somente os comentários dos itens I e IV estão corretos.
quando surgia sobre a grade a carinha de uma criança, d) somente os comentários dos itens II e III estão corretos.
olhos grandes e atentos, espiando o jardim.” (Fernando
Sabino)
c) “Mas o que importa agora é mostrar que esse preconceito,
velhíssimo mas nem por isso menos injusto, se revela em
todos os campos em que o homem vem imperando através
dos séculos, e tem guarida até na linguagem.” (Adriano G.
Kury)
d) “No tempo em que eu freqüentava o Beco da Fome
Kissinger ainda freqüentava Harvard e nem sonhava com o
poder e a glória e a diplomacia a jato, mas que diabo.”
(L.F.V.)

7
24 – Leia com atenção: As questões de 01 a 06 referem-se aos textos acima.
I – “Em meio aos transeuntes, saímos abraçados de forma
colossal, como se fôssemos atores coadjuvantes na 01 – Em relação aos textos, é correto afirmar que:
recepção do Oscar.” I – os autores tratam da questão da fome utilizando-se da
II – “Ela era tão miudinha, que um dia seus pais pediram para mesma estrutura.
tirar uma fotografia com ela.” II – no texto 1 o tema é transformado em conflito humano,
III – “A mãe ligou para o filho. Disse-lhe que o pai havia morrido enquanto que no texto 2 é abordado enquanto problema.
e que estava precisando muito dele.” III – se trata de textos literários que demonstram emoções,
angústias e ansiedades do homem.
Os trechos acima não apresentam clareza, fato decorrente de IV – o texto 2 preocupa-se em analisar as causas da fome.
outros problemas detectados em cada um deles. Assinale a
alternativa que apresenta, respectivamente, os defeitos dos Está(ão) correta(s):
textos. a) II apenas.
b) I, II, III e IV.
a) Obscuridade, prolixidade e preciosismo c) I e III apenas.
b) Impropriedade vocabular, preciosismo e anfibologia d) II e IV apenas.
c) Falta de correlação de idéias, obscuridade e prolixidade
d) Impropriedade vocabular, falta de correlação de idéias e 02 – De acordo com o texto 2, a necessidade de matar a fome
ambigüidade faz com que os instintos primários exaltem-se no homem.
Assinalar a alternativa que comprova essa idéia, através de
25 – Quanto à concordância nominal, assinale a alternativa trechos do texto 1.
incorreta. a) “Resistiram à fraqueza, afastaram-se envergonhados, sem
a) “Alta noite, lua quieta, muros frios, praia rasa” (Cecília ânimo de afrontar de novo a luz clara...”
Meireles) — O adjetivo concorda em gênero e número com b) “Miudinhos, perdidos no deserto queimado, os fugitivos
o substantivo a que se refere. agarraram-se, somaram as suas desgraças e os seus
b) Passei dia e noite frios no Alaska. — Se os substantivos pavores.”
forem antônimos, o adjetivo irá obrigatoriamente para o c) “Sinhá Vitória beijava o focinho de Baleia, e como o focinho
plural (concordância gramatical). estava ensangüentado, lambia o sangue e tirava proveito
c) Tinha a face, os lábios e as orelhas enfeitados. — Quando do beijo.”
na oração ocorrem dois ou mais substantivos de ambos os d) “Levantaram-se todos gritando. (...) Aquilo era caça bem
gêneros, o adjetivo irá necessariamente para o masculino mesquinha, mas adiaria a morte do grupo.”
plural.
d) — Bom-dia! Vossa Magnificência está descansado? — 03 – Relendo o último parágrafo do texto 1, vê-se que Fabiano
Com expressões de tratamento, a concordância se faz readquire forças, sente-se novamente homem. Refletindo sobre
sempre considerando o sexo da pessoa a quem se refere o o que lhe trouxe novo vigor, de acordo com o texto, é possível o
pronome de tratamento. seguinte raciocínio:
a) o homem basta a si mesmo; nada se impõe sobre sua
2/2001-TURMA B condição de ser.
b) a condição de homem relaciona-se diretamente a fatores
Texto 1
que interferem nas condições do ser.
Entrava dia e saía dia. As noites cobriam a terra de chofre. A
c) a satisfação das necessidades fisiológicas não participa da
tampa anilada baixava, escurecia, quebrada apenas pelas
essência do ser.
vermelhidões do poente.
d) o que constitui a natureza humana é a interdependência
Miudinhos, perdidos no deserto queimado, os fugitivos
física e mental que em nada se relaciona com a natureza
agarraram-se, somaram as suas desgraças e os seus pavores.
animal, vegetal ou mineral.
O coração de Fabiano bateu junto do coração de Sinhá Vitória,
um abraço cansado aproximou os farrapos que os cobriam.
04 – O autor Josué de Castro afirma que o flagelo modifica o
Resistiram à fraqueza, afastaram-se envergonhados, sem ânimo
comportamento de toda uma área geográfica.
de afrontar de novo a luz dura, receosos de perder a esperança
No texto, flagelo — aquilo que castiga e tortura — pode ser
que os alentava.
entendido como
Iam-se amodorrando e foram despertados por Baleia, que
a) apenas a seca.
trazia nos dentes um preá. Levantaram-se todos gritando. O
b) apenas a fome.
menino mais velho esfregou as pálpebras, afastando pedaços
c) a fome e a seca.
de sonho. Sinhá Vitória beijava o focinho de Baleia, e como o
d) a miséria e o abandono.
focinho estava ensangüentado, lambia o sangue e tirava
proveito do beijo.
05 – O texto 2 tem como principal objetivo
Aquilo era caça bem mesquinha, mas adiaria a morte do
a) apenas escrever sobre a fome.
grupo. E Fabiano queria viver. Olhou o céu com resolução. A
b) mostrar as conseqüências da fome.
nuvem tinha crescido, agora cobria o morro inteiro. Fabiano
c) frisar a fome no Nordeste brasileiro.
pisou com segurança, esquecendo as rachaduras que lhe
d) alertar sobre o problema da fome no mundo.
estragavam os dedos e os calcanhares.
(Graciliano Ramos)
06 – Das afirmações abaixo, relacionadas ao texto 2,
I –- A fome muda o comportamento dos seres vivos.
Texto 2
II –- O homem, como qualquer animal, tem necessidade de se
Não é, somente, agindo sobre o corpo dos flagelados,
alimentar.
roendo-lhes as vísceras e abrindo chagas e buracos na sua
III –- Só os nordestinos passam pelo problema da inanição.
pele, que a fome aniquila a vida do sertanejo, mas, também,
IV –- A fome pode desagregar profundamente a personalidade
atuando sobre o seu espírito, sobre a sua estrutura mental,
humana.
sobre sua conduta social. Nenhuma calamidade é capaz de
desagregar tão profundamente e num sentido tão nocivo a
Está(ão) correta(s)
personalidade humana como a fome quando alcança os limites
a) I, II e IV apenas.
da verdadeira inanição. Fustigados pela imperiosa necessidade
b) II e III apenas.
de se alimentar, os instintos primários exaltam-se, e o homem,
c) I e III apenas.
como qualquer animal esfomeado, apresenta uma conduta
d) IV apenas.
mental que pode parecer a mais desconcertante. Muda o seu
comportamento como muda o de todos os seres vivos
alcançados pelo flagelo na mesma área geográfica.
(Josué de Castro)

8
07 – Assinalar a alternativa cujo trecho apresenta linguagem 14 – Assinalar a alternativa que completa corretamente as
conotativa. frases abaixo.
a) A independência portuguesa, que por tantos anos tendera I – Os teóricos, neste momento, _____________ os conceitos.
a realizar-se, retrocedia ainda uma vez; era um problema II – _____________ muito talento os meus amigos.
cuja solução já perto do seu termo devia tornar a ser III – A casa ele ____________ do necessário.
tentada de novo. IV – Espero que todos do grupo _____________ crédito ao novo
b) Ciência – Música pode melhorar ação do cérebro gerente.
Pesquisadores, reunidos nos EUA, revelam que som a) reveêm/ tem/ provém/ dêem
estimula certas áreas da mente. b) revêem/ têm/ provém/ dêem
c) “Eu canto porque o instante existe c) reveem/ têm/ provem/ deem
e a minha vida está completa d) revêem/ tem/ provêm/ deêm
Não sou alegre nem sou triste
sou poeta.” 15 – Com relação às palavras do texto abaixo, assinalar a
d) “Pensem nas crianças alternativa que contém a afirmação correta quanto ao processo
............................... de formação de palavras.
A rosa hereditária “Mas desconfio que toda essa conversa é feita apenas para
A rosa radioativa adiar a pobreza da história, pois estou com medo. (...) As coisas
Estúpida e inválida estavam de algum modo tão boas que podiam se tornar muito
A rosa com cirrose ruins porque o que amadurece plenamente pode apodrecer.”
A anti-rosa atômica...” a) Desconfio, amadurece e apodrecer sofreram derivação
parassintética.
08 – Assinalar a alternativa em que todos os vocábulos estejam b) Plenamente, conversa e pobreza são palavras derivadas
com a acentuação gráfica correta. por sufixação.
a) pôquer, hífen, ínterim, alcatéia c) Conversa sofreu derivação regressiva, e desconfio,
b) índex, rúbrica, míope, chevrolés derivação prefixal.
c) sótão, sangüíneo, diminuí-lo, arquetipo d) Ocorre derivação regressiva em conversa, e prefixal e
d) apóio (verbo), rouxinóis, seminú, fortuito sufixal em apodrecer e amadurece.

09 – Observar: 16 – “Fora da idéia de tamanho, as formas aumentativas e


I – — Não, disse-me ele, não me deve mais nada. diminutivas podem traduzir o nosso desprezo, a nossa crítica, o
II – Contarei-lhe o segredo, se puder. nosso pouco caso para certos objetos e pessoas.” (Evanildo
III – — Que Deus acompanhe-te por toda parte. Bechara)
IV – Nunca soubemos quem os roubava nas medidas. Tomando como base a definição acima, assinalar a alternativa
em que o aumentativo tem esse sentido pejorativo.
Quanto à colocação pronominal, estão corretas a) “Ele pegou um peixão! Quatro quilos!”
a) III e IV apenas. b) “Soltava uns dois espirros... assoava o narigão...”
b) II e III apenas. c) O dono da fazenda virou-se para ver que barulhão era
c) I e IV apenas. aquele.
d) I e II apenas. d) “... saía do mato uma mulheraça rúbida, de saias rufadas
de goma.”
10 – “Conjunção é uma palavra invariável que liga e relaciona
entre si duas orações completas ou incompletas.” 17 – Observar:
As palavras sublinhadas, na frase acima, podem ser I – Costumava assistir ao seriado todas as noites, após chegar
classificadas, respectivamente, como: do trabalho.
a) conjunção integrante / conjunção subordinativa. II – O comerciante não vencia pagar ao agiota as dívidas
b) pronome relativo / conjunção coordenativa. eternas, cujos juros eram exorbitantes.
c) conjunção integrante / preposição. III – Tendo esgotado a sua paciência, a mãe chamou a atenção
d) pronome relativo / preposição. do menino na frente de todos.
IV – “Estava à toa na vida,
11 – Assinalar a alternativa cujo verbo não é defectivo. O meu amor me chamou
a) Abole o cigarro. Pra ver a banda passar
b) Reavei o tempo perdido. Cantando coisas de amor”
c) “A fiança remiu-o da prisão.”
d) “Minha música não quer redimir mágoas.” Marcar a alternativa cujos pronomes substituem,
respectivamente, os termos grifados nos textos acima, de
12 – Assinalar a alternativa correta quanto à pontuação. maneira correta.
a) No dia seguinte, almoçamos num restaurante e tomamos a) lhe – lo – sua – la
três garrafas de vinho tinto; ao entardecer, num bar, fiquei b) lo – lhe – lhe – lhe
a alisar ternamente a sua mão fina, de veias azuis. c) a ele – lo – sua –lhe
b) Por mais distraído que fosses, leitor amigo, terias notado d) a ele – lhe – lhe – la
que, ele ficara sinceramente alegre, posto que contivesse a
alegria, segundo convinha a um filósofo. 18 – Observar:
c) E o globo da Lua, num dado momento parece roxo, “De manhã escureço
sangüíneo, como um vaso de sangue. Que singular De dia tardo
metamorfose, e que triste símbolo? De tarde anoiteço
d) O pai, frustrado e ambicioso sonha para o filho a profissão De noite ardo.”
que não conseguira ter. Essas eram as palavras de vovô...
Com relação aos termos da oração presentes na estrofe acima,
13 – Do alto, observavam-se as ruas e as casas; viam-se é correto afirmar que
também, nas praças, frondosas árvores. a) todos os versos possuem sujeito oculto “eu”.
A concordância do verbo com o sujeito na frase acima justifica- b) os adjuntos adverbiais complementam o sentido dos
se pela mesma razão que determina a concordância verbal em: verbos, que são transitivos indiretos, pois modificam-se ou
a) Cabia, na alma do sertanejo, o amor da mulher, o afeto dos transformam-se por ação do homem.
filhos e a amizade do irmão. c) os três primeiros versos apresentam sujeito inexistente,
b) Realmente procedem, em meio a tais abusos, tantas pois os verbos exprimem fenômenos da natureza.
queixas dos contribuintes. d) a estrofe é formada por orações sem sujeito, ainda que
c) Restabelecer-se-iam, de imediato, as ligações e os estas estejam na linguagem conotativa.
contatos, se houvesse técnico de plantão.
d) O diretor e seus assessores têm encontrado, nos últimos
meses, sérias falhas nos relatórios.

9
19 – Sem sofrerem modificações no sentido, os termos em 24 – Observar:
destaque de cada frase foram substituídos pelos verbos entre “O avião, um EA-6B Prowler, voava a apenas 90 metros
parênteses. Assinalar a alternativa em que essa substituição do chão, desrespeitando as regras de segurança para
obedece às regras de regência verbal. vôos de treinamento, que estipulam altura mínima de 300
metros do solo naquela região.”
a) Era difícil, para mim, acreditar no que aconteceu. (custar)
Eu custei a acreditar no que aconteceu. A vírgula que precede o pronome relativo confere a seguinte
b) Sentia-se inclinado pelas idéias daquele grupo jovem. interpretação ao período acima:
(simpatizar)
Simpatizava-se com as idéias daquele grupo jovem. a) o pronome relativo, nesse caso, restringe o sentido de seu
c) “Ele dava primazia ao sossego do campo e não à agitação termo antecedente, “vôos de treinamento”, o que significa
da cidade.” (preferir) dizer que essas regras não estipulam, necessariamente,
Ele preferia o sossego do campo que a agitação da cidade. apenas a altura mínima citada.
d) Assim que receber, irei reembolsar a meu irmão parte da b) a oração adjetiva explicita uma idéia que já se sabe estar
quantia que me foi emprestada. (pagar) contida no conceito de “regras de segurança para vôos de
Assim que receber, pagarei a meu irmão parte da quantia treinamento”, ou seja, que todas as regras, sem exceção,
que me foi emprestada. estipulam altura mínima de 300 metros do solo naquela
região.
20 – Observar a concordância nominal nas orações abaixo: c) por tratar-se de uma oração adjetiva explicativa, a idéia
I – Anos depois, bastantes verdadeiros se tornaram também nela expressa é atribuída a “vôos de treinamento”, os quais
outros avisos de meu pai. estipulam, sem distinção, altura mínima de 300 metros do
II – Bastantes verdades experimentei anos depois do aviso que solo naquela região.
meu pai me dera. d) a oração adjetiva em questão restringe o conceito de
III – São estudiosos as alunas e os alunos deste curso. “regras de segurança para vôos de treinamento”,
IV – O advogado considerou perigosos o argumento e a delimitando seu sentido; em outras palavras, o atributo
decisão. expresso nessa oração é próprio de apenas algumas
dessas regras que foram desrespeitadas pelo avião.
Estão corretas as frases
a) II, III e IV apenas. 25 – Observar:
b) I, II e III apenas. Reclinada molemente na sua verdejante colina, como
c) II e IV apenas. odalisca em seus aposentos, está a sábia Coimbra, a
d) I, II, III e IV. Lusa Atenas. Beija-lhe os pés, segredando-lhe de amor, o
saudoso Mondego. E em seus bosques, no bem
21 – “Já era noite. Parecia viável que todos entendessem que, conhecido salgueiral, o rouxinol e outras aves canoras
naquele momento, deviam-se lembrar de que nada é soltam seus melancólicos trilos. Quando vos aproximais
eternamente assim.” pela estrada de Lisboa, onde outrora uma bem organizada
Com relação ao texto acima, é correto afirmar que mala-posta fazia o serviço que o progresso hoje
a) de que nada é eternamente assim é oração subordinada encarregou à fumegante locomotiva, vede-la
adjetiva restritiva. branquejando, coroada do edifício imponente da
b) que todos entendessem é oração subordinada substantiva Universidade, asilo da sabedoria.
subjetiva. (O primo Basílio. São Paulo, Abril Cultural, 1979. p. 229-30.)
c) naquele momento é adjunto adnominal.
d) noite é sujeito do primeiro período. Com relação ao texto acima, apresentam correta classificação
sintática os termos presentes na alternativa:
22 – Flexionar os verbos entre parênteses na 2ª pessoa do a) “a Lusa Atenas”, “o saudoso Mondego”, “asilo da
singular do modo imperativo e assinalar a alternativa correta. sabedoria” são apostos.
I – (Opor) a perseverança às dificuldades. b) “o rouxinol e outras aves canoras”, “vos” e “fumegante
II – Não (desprezar) os conselhos dos velhos. locomotiva” são, respectivamente, sujeitos de “soltam”,
III – Odiar) o vício e (prezar) a virtude. “aproximais” e “encarregou”.
IV – (Evitar) o mal e (praticar) o bem. c) os pronomes oblíquos de “Beija-lhe” e “segredando-lhe”
desempenham a função de objeto indireto.
a) Opões d) são adjuntos adverbiais “em seus bosques” e “pela estrada
b) despreza de Lisboa”.
c) Odeia; preza
d) Evites; pratiques 1/2002-TURMA A
Kiki do rebolado
23 – Observar:
- Katia Killer? – perguntou a irmã de caridade entrando no
“Sou caprichosa, eu sei. Desce o pó sobre os móveis. Que eu
quarto, e Kiki bem percebeu que a freira nunca acreditara ser
colho na flanela. Escurecem-se as pratas. Que eu esfrego com
aquele seu nome verdadeiro.
a camurça. A aranha tece. Que eu enxoto. A traça rói. Que eu
- Sou eu – disse em tom angustiante; e aprumou-se na
esmago. O cupim voa. Que eu afogo na água da tigela sob a
cadeira.
luz.”
- Sempre resolvida a entregar o recém-nascido?
I – A pontuação do texto contraria as normas gramaticais, pois
- Sou mãe solteira, irmã – desculpou-se, agora os olhos
o pronome relativo deve pertencer ao mesmo período de
baixos – há muito saí de casa. Na profissão que abracei...
seu termo antecedente.
A freira persignou-se:
II – A pontuação adotada é estilística, o que não significa dizer
- Em nome de Deus, fiz tudo que pude! – olhou o alto,
que ela está gramaticalmente correta.
suspirou, olhou a moça. – Vou buscar o papel para você
III – Transformando-se a pontuação de modo a adequá-la às
assinar; o casal que vai adotar a criança quer o preto no branco.
regras gramaticais, o texto, dentre outras possibilidades,
Quando a irmã saiu, Kiki deslizou o olhar para as três
apresentaria seis períodos compostos.
camas já esticadas – a sua, a duma tal de Cida que, em
Está(ão) correta(s) a(s) declaração(ões)
situação igual, partira na véspera, e a da moça sendo cesariada
a) II e III apenas.
naquela hora: “Coitada! saíra há pouco, descorada de dor”.
b) I e III apenas.
Kiki levantou-se da cadeira onde estivera esperando;
c) II apenas.
encaminhou-se para a grande janela de vidraças abertas. A luz
d) I, II e III.
da manhã favorecia e nelas pôde espelhar-se de corpo inteiro:
os seios empinados marcando a malha preta, a minissaia
vermelha oferecendo ancas certinhas, aquelas que, reboladas
no palco, arrancavam mil exclamações. E assobios! Sorriu, na
certeza de que em nada desmerecera.
De repente, a irmã Dolores invadiu o quarto. Com aquele
sorriso claro entregou-lhe o bebê que carregava:
10
- Trouxe o menino para você conhecer; pode contar lá 05 – Assinale a alternativa cujo texto se caracteriza como
fora do garotão forte que teve! dissertação argumentativa.
Kiki, pega de surpresa, apanhara a criança. Agora estava a) “Logo à direita do corredor encontramos aberta uma larga
ali, desajeitada, sem querer olhar: porta, que dá entrada à sala de recepção, vasta e
- Não adianta, irmã; sou firme nas minhas resoluções. luxuosamente mobiliada. Acha-se ali sozinha e sentada ao
Mas, irmã Dolores, como um pé-de-vento, entrara e saíra. piano uma bela e nobre figura de moça.”
Desanimada, Kiki sentou-se, o bebê no colo. b) “Obedecendo à instrução do sertanejo, Luzia desceu pela
Na quietude, primeiro sentiu um calor desconhecido, não tortuosa ladeira, que ia no fundo da grota, e, sustendo-se
sabia se vindo da criança para ela ou dela para a criança. nos arbustos das margens para não escorregar, colhendo
Depois, o nenê resmungou e Kiki baixou os olhos. Então, foi flores silvestres, parando, a revezes, para desembaraçar
como se dali para trás nada mais importasse: as vestes dos espinhos que a detinham, chegou à
- Que coisa mais linda, meu Deus! – exclamou apertando garganta...”
o filho de encontro ao coração, os olhos rasos d’água – Ah!... c) “Os dois estudantes aproximaram-se de Augusto, que
queridinho... Ah! ... – disse, e as lágrimas rolaram. acabava de rogar à linda Moreninha a mercê da terceira
Quando se sentiu lúcida, ergueu-se. Numa bem- quadrilha.
aventurança abriu a porta, caminhou pelo corredor, alcançou a – Leva de tábua, disse Fabrício ao ouvido de Leopoldo... é
saída, fugiu para a rua, para a manhã vazada de sol, o filho a mesma que eu lhe havia pedido.”
aconchegado nos braços. d) “Embora cobrar dos governos estadual e municipal seja
(Lucília Junqueira de Almeida Prado, Depois do extremamente justo, não se pode negar que é chegado o
aguaceiro) momento de cada cidadão dar sua própria contribuição. O
terreno permeável em todas as construções para o
As questões de 01 a 04 referem-se aos textos acima. escoamento da água, a limpeza das ruas e dos bueiros,
evitando transbordamento, o lixo jamais jogado nas ruas
01 – A parte descritiva presente no oitavo parágrafo do texto são medidas simples, possíveis e baratas e que poderiam
serve para fazer a diferença entre um verão de tragédia e um verão
a) mostrar a satisfação da personagem com seu corpo e a civilizado.”
relação com a sua vida profissional, o que justifica sua
atitude, a princípio. 06 – “Deixe a meta do poeta, não discuta,
b) tornar o texto mais subjetivo, despertando o sentimento de Deixe a sua meta fora da disputa
repulsa do leitor para com a personagem. Meta dentro e fora, lata absoluta
c) mostrar que Kiki tinha um belo corpo, mas que lhe Deixe-a simplesmente metáfora.”
incomodava o fato de ele despertar o desejo dos homens. (“Metáfora” – Gilberto Gil)
d) expressar o orgulho da personagem por ter-se tornado
mãe tão jovem. Pode-se afirmar, quanto ao texto acima, que o autor
a) usa do recurso da repetição do vocábulo “meta” para
02 – “- Sou eu – disse em tom angustiante...” enriquecer o texto, ampliando-lhe o sentido.
A angústia da personagem provém do fato de que b) com a repetição do vocábulo “meta”, deixa,
a) Kiki queria ficar livre do hospital e voltar à sua rotina. intencionalmente, o texto com um defeito de estilo.
b) não via outra saída senão entregar o filho. c) faz um jogo de palavras inadequado, tornando o texto
c) não conhecia o casal que iria adotar seu filho. obscuro.
d) pensava em abandonar a carreira. d) ao usar o pronome oblíquo “a” (deixe-a), cria ambigüidade
no texto.
03 – A mudança de atitude da personagem, no final, deve-se
I. ao despertar de seu instinto maternal, ao segurar o filho 07 – “Recebeu o e-mail do colega comunicando que seu
no colo. trabalho havia sido publicado.”
II. à facilidade com que tinha dado à luz o filho, e ao bem-
estar no período pós-parto. Pode-se afirmar que, no texto acima, há
III. ao amor súbito pela criança, que superou as perspectivas a) ambigüidade, causada pelo uso inadequado do pronome
das dificuldades pelas quais passaria. possessivo.
IV. à pressão sutil da irmã de caridade sobre a jovem b) ambigüidade, por erro de regência verbal.
parturiente. c) perfeita clareza de sentido.
d) falta de clareza, devido à falta de sujeito expresso.
Está(ão) correta(s) a(s) afirmação(ões)
a) I, III e IV. 08 – “Foi em março, ao findar das chuvas, quase à entrada
b) I, II e III. Do outono,(I) quando a Terra, em sede requeimada,
c) II e IV. Bebera longamente as águas da estação (II),
d) III somente.  Que, em bandeira, buscando esmeralda e prata
À frente dos peões, filhos da rude mata (III),
04 – Analise os trechos abaixo e assinale a alternativa correta. Fernão Dias Paes entrou pelo sertão.(IV)”
I. “– Sou mãe solteira, irmã – desculpou-se, agora os olhos
baixos.” Analise os trechos acima assinalados quanto ao sentido
II. “Quando se sentiu lúcida, ergueu-se. Numa bem- denotativo e conotativo e assinale a alternativa correta.
aventurança abriu a porta, caminhou pelo corredor...” a) Apenas I e II têm sentido denotativo.
III. “... – exclamou apertando o filho de encontro ao coração, b) Apenas II e III têm sentido conotativo.
os olhos rasos d’água – Ah! queridinho... Ah!... disse, e as c) I e IV têm sentido conotativo; II e III têm sentido denotativo.
lágrimas rolaram.” d) Todos têm sentido conotativo.
IV. “... Kiki deslizou o olhar para as três camas já esticadas –
a sua, a duma tal de Cida que, em situação igual, partira 09 – Assinale a alternativa em que não há hipérbole.
na véspera, e a da moça sendo cesariada naquela hora...” a) A platéia estourava de tanto rir.
b) Morri de saudades de meus pais.
O sentimento de impotência de Kiki diante da situação está c) Você, estudante, tem um mundo pela frente.
expresso em d) Naquele confronto, Luís foi desta para melhor.
a) I e III. c) I e IV.
b) II e IV. d) II e III.

11
10 – Assinale a alternativa em que a passagem do discurso 17 – Assinale a alternativa em que os afixos estão destacados
direto para o indireto está incorreta. corretamente em todas as palavras.
a)  Ande depressa, menina, pediu a mãe.
A mãe pediu que a menina andasse depressa. a) PREVer – amoroSO – belEZA
b) Clotilde pensava: “O mundo ficará todo cheio de flores”. b) DESObediente – certEZA – preVER
Clotilde pensava que o mundo ficaria todo cheio de flores. c) paulaDA – cafeZAL – desanimaDOR
c)  Queremos paz no futuro, disseram os jovens. d) DESonesto – rancorOSO – laranjEIRA
Os jovens disseram que queriam paz no futuro.
d)  Direita, volver!  bradou o comandante da tropa. 18 – Marque a opção em que a palavra grifada não exerce
O comandante da tropa bradou direita, volver. função de adjetivo.
a) O jogo levou meio dia para acabar.
11 – Relacione os encontros vocálicos das palavras (coluna b) Naquela época, andava cheio de esperança.
direita) com sua classificação (coluna esquerda) e assinale a c) O resultado da prova foi melhor do que eu esperava.
alternativa que contém a seqüência correta. d) É um privilégio trabalhar próximo a pessoas tão cultas.
( 1 ) ditongo ( ) conveniente
( 2 ) tritongo ( ) fortuito 19 – Considerando o processo de formação de palavras,
( 3 ) hiato ( ) receoso enumere a coluna da direita de acordo com a da esquerda e
( ) Jaceguai assinale a alternativa com a seqüência correta.
( ) destrói 1 – prefixação ( ) esfarelar
a) 3 – 1 – 3 – 2 – 1 2 – regressiva ( ) desalmado
b) 2 – 3 – 2 – 1 – 2 3 – sufixal ( ) graciosa
c) 3 – 3 – 1 – 2 – 1 4 – parassintética ( ) antinatural
d) 1 – 1 – 3 – 2 – 3 ( ) debate
a) 3, 2, 3, 4, 1
12 – Relacione a coluna da direita com a da esquerda e assinale b) 4, 4, 3, 1, 2
a alternativa que contém a seqüência correta. c) 1, 4, 2, 1, 2
Observe que foram retirados os acentos gráficos de algumas d) 1, 2, 4, 2, 3
palavras, propositadamente.
( 1 ) oxítona ( ) caracteres 20 – Observe as frases abaixo e assinale a alternativa em que
( 2 ) paroxítona ( ) refem há a classificação correta dos adjetivos quanto à flexão de grau,
( 3 ) proparoxítona ( ) rubrica respectivamente.
( ) publico (adjetivo)
( ) interim I. “Era a mais bela! o seio palpitando.”
( ) arquetipo II. O Prefácio Interessantíssimo é de autoria de Mário de
( ) amendoim Andrade.
( ) latex III. O tempo é tão fugaz quanto incerta e curta é a vida.
a) 2 – 1 – 2 – 3 – 3 – 3 – 1 – 2 IV. Nossos antepassados eram menos preocupados com o
b) 3 – 2 – 3 – 2 – 3 – 2 – 1 – 1 meio ambiente do que nós.
c) 1 – 1 – 2 – 3 – 1 – 3 – 2 – 2 a) superlativo relativo de superioridade, superlativo absoluto
d) 2 – 2 – 3 – 1 – 2 – 1 – 3 – 1 sintético, comparativo de igualdade, comparativo de
inferioridade.
13 – “Este manual .................. informações importantes sobre os b) superlativo absoluto analítico, superlativo relativo de
produtos que ................ elementos prejudiciais à saúde. superioridade, comparativo de superioridade, comparativo
Portanto, ................. com a ajuda dele.” de igualdade.
Assinale a alternativa que preenche os pontilhados, na ordem c) comparativo de superioridade, superlativo absoluto
em que aparecem. analítico, comparativo de superioridade, superlativo relativo
a) contêm – contém – contem de inferioridade.
b) contém – contêm – contem d) superlativo relativo de superioridade, superlativo absoluto
c) contem – contem – contém analítico, comparativo de inferioridade, comparativo de
d) contém – contêm – contêm superioridade.

14 – Assinale a alternativa que preenche corretamente os 21 – (...)


pontilhados da frase abaixo, respectivamente. “Convive com teus poemas, antes de escrevê-los.
“Naquela noite ........................ as luzes se apagaram, voou Tem paciência, se obscuros. Calma, se te provocam.
porta adentro um .......................... de abelhas. Trocado Espera que cada um se realize e consuma
o .......................... , entretanto, a ....................... se retirou.” com seu poder de palavras
a) mal – enxame – fusível – colmeia e seu poder de silêncio.”
b) mal – enchame – fuzil – colméia (...)
c) mau – enxame – fusível – colmeia (C.D. Andrade)
d) mau – enchame – fuzil – colméia As palavras grifadas, no poema, classificam-se,
respectivamente, como
15 – Das alternativas abaixo, alteradas propositadamente,
apenas uma não apresenta erro no emprego da forma verbal. a) conjunção integrante, conjunção integrante.
Assinale-a. b) conjunção adverbial condicional, conjunção adverbial
a) Folha de São Paulo – não dá pra não lê. condicional.
b) Potentes como Antônio Ermírio de Moraes já vêm, com c) partícula expletiva, partícula expletiva.
certa simpatia, a candidatura de Ciro Gomes. d) pronome reflexivo, conjunção adverbial condicional.
c) O encontro histórico dos presidentes sul-americanos
evidencia a intenção brasileira de liderar a região. 22 – “A professora dizia que a rua tinha aquele nome em
d) É revoltante saber que profissionais de algumas áreas têm homenagem a um grande homem, digno de ser imitado. Nós a
de estudar dez anos ou mais para conseguir um salário de ouvíamos meio duvidosos. Para nós a rua era do pipoqueiro, do
cinco ou seis mil reais enquanto semi-alfabetizados ganhão sorveteiro, do vendedor de algodão-doce... De alguém que
milhares na TV. significasse muito, era o nome da rua.” (Elias José)
Os substantivos grifados nesse texto classificam-se,
16 – Assinale a alternativa em que o elemento mórfico em respectivamente, como
destaque está corretamente analisado. a) simples, derivado, composto e comum.
a) meninA – desinência nominal de gênero b) próprio, primitivo, simples e abstrato.
b) vendEste – desinência modo-temporal c) simples, derivado, próprio e concreto.
c) partIa – vogal temática da 2.ª conjugação d) comum, composto, derivado e comum.
d) amaSSem – desinência de 2.ª pessoa do plural

12
23 – Assinale o período gramaticalmente correto com relação
ao uso dos pronomes. 30 – Assinale a alternativa cujo termo destacado classifica-se
a) Não façam eu estudar tanto. como complemento nominal.
b) Entre eu e ele não há discórdias. a) Arrancaram-lhe as roupas.
c) Para mim ele não contou a história. b) Ela nunca lhe desobedece.
d) Mandaram eu entrar assim que me viram. c) A sentença foi-lhe favorável.
d) Júlio devolveu-lhe o livro emprestado.
24 – “Rasgue as minhas cartas e não me procure mais. Assim
será melhor, meu bem. O retrato que eu te dei se ainda tens não 31 – Assinale a alternativa em que há oração subordinada
sei, mas, se tiver, devolva-me.” adjetiva explicativa.
Nesse trecho da música de Renato Barros e Lilian Knapp, há a) A revista que você me emprestou ontem, eu a perdi.
mistura dos pronomes tu e você. Se se considerar apenas o b) O homem que fala, que persiste, que enfrenta as
pronome você, quantas palavras devem ser corrigidas? dificuldades triunfa na vida.
a) Duas. c) Todos sabem, meu amigo, que a verdade virá à tona
b) Três. quando menos esperarmos.
c) Quatro. d) O Brasil, que contém muitas riquezas naturais, atrai turistas
d) Cinco. do mundo inteiro.

25 – Em qual(is) sentença(s) abaixo o verbo grifado não é 32 – Assinale a alternativa em que o período é composto por
defectivo? subordinação.
I. Nessa época do ano, é comum nevar em algumas regiões a) A moça estremeceu, abaixou os olhos e não respondeu à
do país. pergunta.
II. As onças rosnam ao redor da cabana. b) Tudo acabou bem; eram, pois, infundados os nossos
III. A erupção vulcânica baniu da vila a maior parte de seus receios.
habitantes. c) Disseram-me que você é pessoa digna de confiança.
IV. Uma das testemunhas interveio a favor do réu. d) Geralmente os ignorantes falam muito; os sábios, porém,
a) III. preferem o silêncio.
b) IV.
c) I e II. 33 – “Maltrataram inocentes, mas ninguém quer depor.” (Elza
d) III e IV. Santana Silva)
Os verbos grifados têm, respectivamente, sujeito
26 – Leia as frases abaixo. a) indeterminado e simples.
I. As pessoas só confiarão em mim se eu sempre manter a b) oculto e indeterminado.
palavra. c) simples e oculto.
II. Quando ele se propor a nos auxiliar, estaremos mais d) indeterminado e oculto.
próximos de nossos objetivos.
III. Assim que os autores compusessem a peça, os ensaios 34 – “Vinhas fatigada e triste e triste e fatigado eu vinha...”
teriam início. (Olavo Bilac)
IV. Se contessem as águas, navegaríamos mais tranqüilos. No período acima, pode-se afirmar que os predicados são
V. Muitos propõem soluções, mas é preciso que se aventurem a) verbo-nominais, porque os verbos são de ação e vêm
a colocá-las em prática. seguidos de dois advérbios de modo.
Com relação à flexão das formas verbais, estão corretas as b) nominais, porque os verbos são de ação e vêm seguidos
frases de dois predicativos.
a) I e II. c) verbo-nominais, porque os verbos são intransitivos e vêm
b) II, III e V. seguidos de dois predicativos.
c) I, IV e V. d) verbais, porque os verbos têm sua significação completada
d) III e V. por dois objetos diretos.

27 – “Sê o que renuncia 35 – “O pequeno avião fez um pouso de emergência numa


Altamente: plantação próxima ao aeroporto. Ficaram feridos todos os seus
Sem tristeza da tua renúncia! ocupantes.” (Veja)
Sem orgulho da tua renúncia! Sobre esse texto, é incorreto afirmar que
Abre a tua alma nas tuas mãos a) “feridos” é objeto direto.
E abre as tuas mãos sobre o infinito b) “pequeno” é adjunto adnominal.
E não deixes ficar de ti c) “todos os seus ocupantes” é sujeito.
Nem esse último gesto!” (Cecília Meireles) d) “numa plantação” é adjunto adverbial de lugar.

Mudando-se a pessoa verbal da 2.ª do singular para a 2.ª do 36 – Assinale a alternativa em que a concordância verbal está
plural, os verbos grifados serão, respectivamente, incorreta.
a) sedes – abris – deixais ficar. a) Era João como o pai; e como o aconselhavam a devoção e
b) sejai – abrais – deixeis ficar. a pobreza.
c) sede – abri – deixeis ficar. b) No outro dia, cinco horas batia; Seu Chico aparecia de
d) sois – abri – deixeis ficar. novo no asilo.
c) Ninguém, na sua opinião, entre os vinte e poucos alunos,
28 – Em qual período abaixo há necessidade do emprego da tinha jeito para aquilo.
vírgula? d) A minha vocação, o meu gosto, o meu pendor para a coisa
a) Esses casos exemplares demonstram a utilidade da literária, tudo se revelou precisamente naquele dia.
campanha.
b) É preciso preservar a vida do elefante que é um grande 37 – Assinale a alternativa cujas palavras completam
mamífero. corretamente as lacunas da frase abaixo.
c) Chegou-se à conclusão de que o importante mesmo eram “ .............................. e ............................. pelos amantes dos
os benefícios proporcionados à população. espaços naturais afastados da trepidação urbana, a serra e
d) A humanidade espera que os homens de bom senso o parque do Gerês escondem sítios fascinantes e
cumpram a missão para a qual foram designados. comoventes na sua beleza.”
a) Afamado – procurado
29 – Assinale a frase cujas palavras destacadas são, b) Afamado – procurada
respectivamente, vocativo e aposto. c) Afamados – procurados
a) Joana, minha prima, está doente. d) Afamadas – procuradas
b) Mauro, telefone-me assim que Teresa chegar.
c) Alice, meu vizinho João comprou um computador.
d) Paulo disse que José, treinador do time, está desanimado.

13
38 – Assinale a alternativa que não admite ambas as regências. As questões de 01 a 04 referem-se aos textos acima.
a) Andava alheio a / de tudo.
b) Está apto a / para o trabalho. 01 – O título estabelece coesão com o restante do texto. Por
c) Sois propensos a / para o trabalho. isso, após a leitura, entende-se que "O mais terrível" é
d) Pedro ficou alucinado com / sobre a notícia. a) conhecer o drama do menor que vive nas ruas, e não
conseguir fazer nada para solucioná-lo.
39 – Assinale a alternativa que completa corretamente as b) não confiar no mercado, que pode oferecer boas
frases abaixo. oportunidades a todos.
O pobre operário preferiu a morte ............ vida. c) não ter paciência com a criança que pede esmolas.
Assistia- ............ o direito de reclamar. d) constatar que a solidariedade é antinatural e que deve ser
Perdi meu pai, ............ muito amava. evitada.
a) do que a – lhe –que
b) que a – o – a quem 02 – Observe as afirmações e assinale a alternativa correta.
c) a – o – que I. - O texto tem por objetivo principal enfatizar um fato ocorrido
d) à – lhe – a quem com o narrador num espaço e tempo definidos.
II. - A preocupação do texto é descrever um tipo comum na
40 – Indique, nas orações abaixo, o caso em que o acento nossa sociedade - a criança de rua - e os adultos que têm
grave, indicador da crase, não é facultativo. de conviver com elas.
a) Bons tempos aqueles em que vovó nos contava histórias III. - O texto objetiva refletir sobre um problema comum nos
referentes à sua infância! dias de hoje, valendo-se de um fato corriqueiro da cidade
b) Como a situação não agradasse a todos, foram até às grande.
últimas conseqüências. IV. - O texto disserta sobre a falta de compromisso da
c) O comitê esportivo entregou o troféu à Marlene, capitã do sociedade com relação ao problema da criança de rua
time de futebol feminino. brasileira.
d) Mal desci do ônibus e já me dirigi à casa de meu colega. Pode-se dizer que estão corretas as afirmações
a) I e IV. c) I e II.
1/2002-TURMA B b) III e IV. d) II e III.
O mais terrível 03 – No texto, pode-se deduzir que o narrador expressa alguns
O mais terrível não era a menina me chamando de "tio" e sentimentos seus sobre o problema abordado, a fim de
pedindo um trocado, ela de pé no chão no asfalto e eu no meu sensibilizar o leitor. Com isso, marque (V) para verdadeiro e (F)
carro de bacana. O mais terrível não era eu escolhendo a cara para falso, tendo em vista tais sentimentos revelados nos
e a voz para dizer que não tinha trocado, desculpe, como se a trechos abaixo; a seguir, assinale a alternativa com a seqüência
vergonha tivesse um protocolo que a absolvesse. O mais correta.
terrível não foi nem a naturalidade com que ela cuspiu na minha ( ) "O mais terrível foi que ela era tão pequena que a
cara. O mais terrível foi que ela era tão pequena que a cusparada não me atingiu." - piedade
cusparada não me atingiu. ( ) "Vai cheirar cola, para passar. Vai morrer, para esquecer.
Somos boas pessoas, bons cidadãos e bons pais, mas Ou vai crescer, para me matar na próxima esquina." - revolta
somos tios relapsos. Nossas sobrinhas e nossos sobrinhos ( ) "A família humana é um mito, o sofrimento alheio é um
enchem as ruas das nossas cidades, cercam nossos carros, estorvo e se a miséria a tua volta te incomoda, compra uma
invadem nossas vidas e insistem que são da nossa família, e antena parabólica." - resignação
não temos nada para lhes dar ou dizer, além da esmola e ( ) "Mas mais terrível do que tudo é eu ficar aqui, escolhendo
"desculpe". Na família brasileira "tios" e sobrinhos têm um frases para encher papel, até cuidando o estilo, já que é
diálogo de ameaça e medo, revolta e remorso, e poucas domingo. Como se fizesse alguma diferença. Como se isso
palavras. Nenhum consolo possível, nenhuma esperança, fosse nos salvar, o tio de sua impotência e cumplicidade e a
nenhuma explicação. O que dizer a uma sobrinha cuja cabeça sobrinha anônima de seu destino. Desculpe." - frustração
mal chega à janela do carro e tenta cuspir na cara do tio? Feio. a) V, V, F, V c) V, F, F, V
Falta de educação. Papai do céu castiga. Paciência, minha b) F, F, V, F d) F, V, V, F
filha, este é apenas um ciclo econômico e a nossa geração foi a
escolhida para este vexame, você aí desse tamanho pedindo 04 – As considerações feitas no terceiro parágrafo tornam
esmola e eu aqui sem nada para te dizer, agora afasta que abriu possível ler conotativamente seu fecho: "Afasta do carro que
o sinal. Não pergunte ao titio quem fez a escolha, é tudo muito abriu o sinal.", o qual representa, por isso,
complicado e, mesmo, você não entenderia a teoria. Vai cheirar a) a vitória dos mocinhos.
cola, para passar. Vai morrer, para esquecer. Ou vai crescer, b) a falência da família.
para me matar na próxima esquina. c) o desafio das novas gerações.
A História, dizem, terminou, e os mocinhos ganharam. Os d) o sofrimento dos realistas.
realistas, antiutópicos, os racionais. Ficou provado que a
solidariedade é antinatural e que cada um deve cuidar dos 05 – “Agora, o cheiro áspero das flores
apetites dos seus. Ou seja: ninguém é "tio" de ninguém. A leva-me os olhos por dentro de suas pétalas.”
família humana é um mito, o sofrimento alheio é um estorvo e se Quanto à linguagem figurada do trecho grifado acima, há nessa
miséria a tua volta te incomoda, compra uma antena parabólica. frase
Ninguém é insensível, dizem os mocinhos, mas a convivência a) uma sinestesia. c) uma hipérbole.
com a dor dos outros, que deviam ter nos educado para a b) um pleonasmo. d) uma onomatopéia.
compaixão, nos educaram para a autodefesa, para cuspir
primeiro. Os bons sentimentos faliram, dizem os mocinhos. 06 – Assinale
ANULADA a alternativa em que a colocação pronominal não
Confiemos o futuro ao mercado, que não tem sentimentos, que observa a norma culta, e sim a fala corrente, coloquial.
tritura gerações entre seus dedos invisíveis, pra que se a) O pai censurou-os por se estranharem tanto durante as
envolver? Afasta do carro que abriu o sinal. brincadeiras.
Mas mais terrível do que tudo é eu ficar aqui, escolhendo b) Naquela hora, só me ofereceram um copo d’água enquanto
frases para encher papel, até cuidando o estilo, já que é aguardava por duas horas a chegada de Luís.
domingo. Como se fizesse alguma diferença. Como se isso c) Não sei como se justificam perante o Diretor do colégio.
fosse nos salvar, o tio da sua impotência e cumplicidade e a d) João não se conformava de ter perdido a cerimônia de
sobrinha anônima do seu destino. Desculpe. casamento do seu melhor amigo. Dona Jandira, mãe do
Luís Fernando Veríssimo noivo, o chamaria ingrato.

14
07 – Dado o texto, 11 – Marque a alternativa em cuja frase se verifica o emprego
“Nunca ouvimos de Jacinto uma palavra áspera, uma lamúria, da pontuação de acordo com a norma culta.
nunca respondeu com irritação às crianças que o insultavam, a) A autora, sentia-se totalmente responsável por seus
impiedosas, quando passava embriagado. Bêbado, sorria personagens.
beatífico e acima de todas as misérias.” (Paulo Mendes b) “Morena tropicana eu quero teu sabor...”
Campos) c) Durante os últimos meses, a nação viveu hipnotizada.
d) Conrado, que morava na casa vizinha atravessava
Assinale a alternativa que traz uma seqüência incoerente para diariamente a cerca de fícus para falar comigo.
o texto.
a) Era um homem de andar vacilante, trôpego, de um blá-blá- 12 – Observe estes versos:
blá infinito, molengo e queixumeiro. “Um dia me disseram quem eram os donos da situação
b) Devido à bebida, falava numa língua quase ininteligível, Sem querer eles me deram as chaves que abrem essa prisão.”
mas que deixava a impressão de benignidade. (Engenheiros do Havaí)
c) Trôpego, articulava um trololó quase sem sentido e
gesticulava; gestos grandes, largos, prazenteiros. As palavras grifadas estão empregadas conotativamente.
d) De quando em quando, parava, respirava. E era só seus Assinale a alternativa que mostra seu significado no contexto.
olhos confusos cruzarem com os de alguém para alentar- a) chaves: consciência de ver o mundo criticamente
se e continuar numa falação pausada e sem fim. prisão: visão limitada da realidade
b) chaves: instrumentos que abrem portas
08 – Com relação ao trecho transcrito a seguir, assinale a prisão: recinto fechado; clausura
alternativa cujo comentário está correto. c) chaves: problemática a ser resolvida
“Com esta história eu vou me sensibilizar, e bem sei que prisão: problema em que eles se encontram
cada dia é um dia roubado da morte. Eu não sou um d) chaves: instrumentos que vão fechar a prisão
intelectual, escrevo com o corpo. E o que escrevo é uma prisão: limitação do homem
névoa úmida.”
a) É um texto tipicamente narrativo, pois apresenta um 13 – Assinale a alternativa cujas palavras preenchem
narrador-personagem e informações quanto ao tempo e ao corretamente as lacunas do texto abaixo.
lugar. “A maneira afável ____________ a moça o tratava tinha,
b) Apesar de apresentar um narrador-personagem, este não ______________ desvanecido completamente, ao menos
relata fatos, faz uma autodescrição de caráter dissertativo. embotado as suscetibilidades de sua consciência
c) O que está em evidência é o ponto de vista adotado pelo _______________ do ajuste que fizera com Lemos.”
narrador, cuja tese a respeito da escrita está fundamentada (José de Alencar,
em argumentos explícitos, característica de um texto Senhora)
essencialmente dissertativo. a) por que/ senão/ acerca
d) Não se pode tecer qualquer comentário quanto ao tipo de b) porque/ senão/ a cerca
texto, uma vez que o trecho é insuficiente para tal. c) porque/ se não/ acerca
d) por que/ se não/ a cerca
09 – Leia:
I. “A velha Totonha de quando em vez batia no engenho. E 14 – O substantivo pode figurar de várias maneiras na oração.
era um acontecimento para a meninada. Ela vivia de Assinale a alternativa que apresenta substantivos com as
contar histórias de Trancoso. Pequenina e toda mesmas funções sintáticas dos destes versos de Manuel
engelhada, tão leve que uma ventania poderia carregá-la, Bandeira.
andava léguas e léguas a pé...” (José Lins do Rego) - Vi uma estrela tão alta,
Esse texto é uma narrativa, pois há vários verbos de ação, Vi uma estrela tão fria!
e o narrador é observador. Vi uma estrela luzindo
II. “Dario vinha apressado, guarda-chuva no braço esquerdo Na minha vida vazia.
e, assim que dobrou a esquina, diminuiu o passo até
parar, enconstando-se à parede de uma casa. Por ela a) "Classificado do futuro
escorregando, sentou-se na calçada, ainda úmida de Vende-se vasinho de
chuva, e descansou na pedra o cachimbo.” (Dalton samambaia
Trevisan) - Apesar da presença de verbos de ação, que diretamente da mata amazônica" (Ulisses Tavares)
identificam uma narrativa, esse texto é descritivo, pois há b) "As nuvens são cabelos
elementos caracterizadores de Dario e do ambiente. crescendo como rios;
III. “Fala-se mal o português. Ou melhor, fala-se errado. são gestos brancos
Ninguém agüenta mais ouvir erros grosseiros do tipo da cantora muda;" (J.C.M. Neto)
‘houveram acidentes’ ou ‘é para mim fazer’. Para os mais c) 'Stamos em pleno mar... Doudo no espaço
letrados, essas agressões ao idioma têm ferido tanto os Brinca o luar - doirada borboleta -
ouvidos, que se decidiu partir para um contra-ataque.” E as vagas após ele correm ... cansam
(IstoÉ, 20/08/97) - O texto tem a intenção de expressar o Como turba de infantes inquietas." (Castro Alves)
ponto de vista do autor a respeito do assunto; logo, é uma d) "Deixei a cidade sumida no silêncio da madrugada."
descrição. (Raul Bopp)

Com relação às afirmações feitas sobre os textos, pode-se 15 – Foram retirados os acentos de algumas palavras das
concluir que frases abaixo. Assinale a alternativa em que há duas palavras
a) todas estão erradas. proparoxítonas.
b) apenas a III está correta. a) “Um revolver de ouro, marcas de muita magoa.”
c) apenas a II está errada. b) “O mais doce nectar é o que desejo
d) apenas a I está errada. O mais fundo vacuo.”
c) “Cristais diluidos de clarões alacres”
10 – A frase que apresenta um defeito de estilo – a ambigüidade d) “Não sei, não: se fossemos culpados, teriamos que nos
– é: retratar.”
a) A prosa ou o poema devem ser extravasados numa
linguagem espontânea, colorida como a fala. 16 – Assinale a alternativa em que todas as palavras estão
b) Diante do ocorrido entre você e seu filho, fiquei sem saber corretamente acentuadas.
se o que eu disse concorreu para agravar a situação. a) “Há anos raiou no céu fluminense uma nova estrêla.”
c) A clareza é essencial a todo escritor, pois facilita para b) E só ver com outros olhos pára poder também ver com a
quem lê a percepção rápida do pensamento. alma.
d) Meu pai recebeu uma carta em que o irmão lhe contava c) E vou tratando de trabalhá-lo para pôr as idéias em ordem.
como sua mulher sofrera um acidente de automóvel. d) Pôr esse caminho, passávamos para ver o Caapóra e o
Saci-Pererê.

15
17 – Numere os textos, obedecendo ao código abaixo: 1) formam verbos de substantivos ( ) legalidade, mansidão,
I- Discurso direto e adjetivos fraqueza, planície.
II- Discurso indireto 2) formam advérbios ( ) falsificar, clarear,
III- Discurso indireto livre de adjetivos lacrimejar, gaguejar
IV- Ausência de discurso 3) formam substantivos de outros ( ) capitalismo, budismo.
( ) “Caíra no fim do pátio, debaixo de um juazeiro, depois substantivos republicano, cavalaria.
tomara conta da casa deserta. Ele, a mulher e os filhos tinham- 4) formam substantivos ( ) livremente, especialmente,
se habituado à camarinha escura, pareciam ratos — e a de adjetivos firmemente, calmamente.
lembrança dos sofrimentos passados esmorecera.”
(Graciliano Ramos) a) 3, 2, 4, 1 c) 4, 1, 2, 3
( ) “ Minha mãe ficava sentada cosendo b) 4, 1, 3, 2 d) 2, 3, 1, 4
olhando para mim.
— Psiu... Não acorde o menino. 22 – Assinale a alternativa em que o vocábulo que funciona
Para o berço onde pousou um mosquito.” como conjunção.
(Carlos Drummond de Andrade) a) O jornal a que me refiro publicará seu texto.
( ) “Aprumou-se, fixou os olhos nos olhos do polícia, que se b) O importante é que todos se convençam de que é preciso
desviara. Besteira pensar que ia ficar murcho o resto da vida. lutar.
Estava acabado? Não estava. Mas para que suprimir aquele c) Foram inúteis os argumentos de que fizeram uso durante o
doente que bambeava e só queria ir para baixo?” debate.
(Graciliano Ramos) d) É importante saber conviver com as mudanças que
( ) “Cristiano foi o primeiro que travou conversa, dizendo-lhe acompanham a existência.
que as viagens de estrada de ferro cansavam muito, ao que
Rubião respondeu que sim...” 23 – Assinale a única alternativa em que não há adjetivo ou
(Machado de Assis) locução adjetiva.
a) “A vida é combate,/ Que aos fracos abate,/ Que os fortes,
Assinale a alternativa que contém a seqüência correta. os bravos,/ Só pode exaltar.” (Gonçalves Dias)
a) II, II, IV, III c) IV, I, III, II b) “Alguns dias dava-lhe gana de satisfazer o apetite,
b) I, I, II, IV d) III, I, IV, IV devorando lascas de pirarucu assado, com farinha d’água
e lata de marmelada...” (Inglês de Sousa)
18 – Assinale a alternativa cujo termo em destaque é advérbio. c) Para os índios, o ingresso na vida adulta é sinônimo de
a) Procuras o bem, e o bem terás. mudanças mais radicais. Eles precisam ter coragem e
b) Aconselhava-a para seu próprio bem. sangue- -frio para cumprir os rituais de passagem.
c) Compreenda-a bem, que sem ela tua vida será difícil. d) Para os brancos, penugens no rosto e menstruação
d) “Quero a primeira estrela que vier / para enfeitar a noite do representam “os rituais” de que necessitam para a
meu bem.” chamada vida de adulto.

19 – Com relação à regência verbal, a frase "A mulher? Ora, 24 – Observe:


quero-a e quero-lhe.", de Latino Coelho, Reclinada molemente na sua verdejante colina, como odalisca
a) está correta; o autor observou os diferentes significados do em seus aposentos, está a sábia Coimbra, a Lusa Atenas. Beija-
verbo, o que podemos ver pelos complementos verbais. lhe os pés, segredando-lhe de amor, o saudoso Mondego. E em
b) está incorreta; o verbo querer significa desejar e é seus bosques, no bem conhecido salgueiral, o rouxinol e outras
transitivo direto; trata-se de um "erro proposital"; o autor aves canoras soltam seus melancólicos trilos. Quando vos
intensifica o quanto é forte seu desejo pela mulher. aproximais pela estrada de Lisboa, onde outrora uma bem
c) está correta; o verbo querer, significando desejar, organizada mala-posta fazia o serviço que o progresso hoje
apresenta transitividade direta ou indireta, encarregou à fumegante locomotiva, vede-la branquejando,
indiferentemente; o autor faz uso dos dois complementos coroada do edifício imponente da Universidade, asilo da
para reforçar sua idéia. sabedoria.
d) está incorreta; o autor não poderia usar a transitividade (O primo Basílio. São Paulo, Abril Cultural, 1979. p. 229-30.)
indireta, uma vez que faz uso do objeto pleonástico na
forma de objeto direto. Os pronomes, cuja função essencial é denotar ou determinar os
seres, funcionam também como elementos coesivos, isto é,
20 – Observe: estabelecem a ligação entre os elementos de uma frase ou de
I. Um velho galo matreiro, percebendo a aproximação da um texto, a fim de manter a sua unidade. Leia o texto acima e
rapoza, impuleirou-se numa árvore. marque a alternativa em que o termo referente não é retomado
II. Tanta gente também nos outros ensinua pelo pronome indicado.
Crenças, religiões, amor, felicidade, a) “vede-la” – o pronome oblíquo “a” retoma o substantivo
Como este ascendedor de lampiões na rua! próprio “Coimbra”.
III. Consumido pela morte e pela sede, começou a exitar, não b) “onde” – o pronome relativo retoma o seu termo
sabendo se antes comia do feno e depois bebia da água antecedente “estrada de Lisboa”.
ou se antes sassiava a sede... c) “seus bosques” – o pronome possessivo refere-se a
“saudoso Mondego”.
Quanto à grafia das palavras nos períodos acima, há d) “que” – o pronome relativo refere-se ao termo antecedente
a) dois vocábulos errados em cada item. “serviço”.
b) vocábulos errados somente nos itens II e III.
c) um vocábulo errado no item I; um vocábulo errado no item 25 – Assinale a alternativa em que o pronome relativo foi
II; dois vocábulos errados no item III. utilizado incorretamente na junção das frases para formar um
d) apenas um vocábulo errado em cada item. único período.
a) A sábia natureza me abençoará eternamente. Dediquei a
21 – "Os sufixos são capazes de modificar o significado do ela minha longa vida.
radical a que são acrescentados. Sua principal característica, A sábia natureza, a que dediquei minha longa vida,
no entanto, é a mudança de classe gramatical que geralmente abençoar-me-á eternamente.
operam." (Gramática da Língua Portuguesa, Pasquale e Ulisses b) Vivo em sintonia com aquela centenária árvore. Sob sua
Infante). Relacione a coluna da esquerda com a da direita, rama jaz minha infância.
observando a mudança causada pelo sufixo e assinale a Vivo em sintonia com aquela centenária árvore, sob cuja
seqüência correta. rama jaz minha infância.
c) Aquele rio é saudosa lembrança. Em suas águas
douradas, deixei minha inocência.
Aquele rio em cujas águas douradas deixei minha
inocência é saudosa lembrança.
d) A velha e amada casa fica no alto da montanha, quase a
tocar o céu. Eu nasci naquela casa.

16
A velha e amada casa na cuja nasci fica no alto da 32 – Assinale a alternativa em que o emprego do pronome
montanha, quase a tocar o céu. demonstrativo está incorreto.
a) Esta empresa “Cosméticos Vida” agradece a essa loja a
26 – “Estamos ansiosos. Vão apresentar-nos a programação.” preferência dada e cumprimenta-a pelo novo ponto
comercial.
Assinale a alternativa que contém a relação implícita entre Atenciosamente, Diretor de Vendas
essas orações. b) De acordo com o que foi solicitado pelo Gerente dessa
a) Finalidade c) Temporalidade cadeia de lojas, esta empresa antecipará a entrega do
b) Causalidade d) Oposição material para o Natal.
ANULADA Atenciosamente, Diretor de Vendas
27 – Observe os termos grifados nas frases abaixo, quanto à c) “Laços de Família” e “Por Amor” apresentam a
função sintática por eles exercida. personagem Helena. Nessa, a personagem foi vivida por
Regina Duarte; naquela, por Vera Fischer.
I. Hillary Clinton, primeira-dama dos Estados Unidos está d) Essa seção solicitou que fossem enviados, com urgência,
lançando o livro “Convite à Casa Branca”. dois funcionários, até o último dia deste mês.
II. Saiba quais são as principais ferramentas para entrar no Concedido.
mundo do comércio virtual. Gerente administrativo
III. Bonito, cidade do Mato Grosso do Sul, é mais que bonita.
33 – “Uma formiguinha atravessa, em diagonal, a página ainda
28 – Assinale a alternativa cujo verbo se conjuga por haver. em branco. Mas ele, aquela noite, não escreveu nada. Para
quê? Se por ali já havia passado o frêmito e o mistério da
a) Provia a despensa toda vez que aparecia. vida...”
b) Revia os amigos, quando sentia saudade. Quanto à função sintática, os termos destacados, no trecho
c) Desavimo-nos, diariamente, por questões fúteis. acima, classificam-se como
d) Ele reavia suas forças, quando outra tragédia o acometeu. a) adjunto adverbial e núcleo do sujeito.
b) objeto direto e adjunto adnominal.
29 – Relacione as orações em destaque a seus significados e c) adjunto adverbial e núcleo do objeto.
assinale a alternativa em que a seqüência esteja correta. d) núcleo do adjunto adverbial e objeto direto.

(1) contraste, ressalva (3) conclusão 34 – A única maneira de ter sensações novas é construíres-te
(2) alternância (4) explicação uma alma nova. (...) E o único meio de haver coisas novas, de
sentir coisas novas é haver novidade no senti-las. Muda de alma
( ) A mim ninguém engana, que não nasci ontem. como? Descobre-o você.
( ) O instinto social não é privilégio do homem, antes, se nos Lendo com atenção o texto de Fernando Pessoa, que foi
depara nos próprios animais. modificado, percebe-se erro de
( ) "Já atravessa as florestas; já chega aos campos do Ipu." a) pontuação. c) concordância verbal.
( ) Ele é teu amigo: respeita-lhe, pois, a verdade. b) regência verba. d) concordância nominal.
a) 1, 3, 2 e 4 c) 4, 1, 3 e 2
b) 1, 2, 3 e 4 d) 4, 1, 2 e 3 35 – O programa de educação do Estado de Minas Gerais é
considerado inovador. Tem recebido referências elogiosas, a
30 – Observe: última ________ feita pelo Ministro José Serra, na sexta-feira
I. As paixões tornam os homens cegos. (...). A informação existente é a de que a campanha das etapas
II. A árvore ficou sem folhas. do programa ____________ por um grupo de pessoas entre
III. "Ninguém se fie da felicidade presente." ________ alguns empresários de Belo Horizonte. (Revista Veja,
adaptado)
É correto afirmar a respeito dos períodos acima que Complete os espaços em branco, observando os princípios da
a) em I há predicado nominal, pois nele há um verbo de concordância.
ligação. a) da qual – estão sendo feitas – o qual
b) o núcleo do predicado de II é sem folhas. b) das quais – está sendo feita – as quais
c) o núcleo do predicado de III é o verbo fie. O predicado, c) das quais – estão sendo feitas – o qual
portanto, é verbo-nominal. d) da qual – está sendo feita – as quais
d) o predicado de II é verbal, pois o verbo ficar é intransitivo.
36 – Assinale a alternativa que apresenta erro quanto à
31 – Classifique os verbos quanto à transitividade na frase: “Os regência nominal.
ilhais da fera arfam de fadiga, a espuma franja-lhe a boca, as a) Aquele que se acha ávido de sabedoria enriquece sua
pernas vergam, e os olhos amortecem de cansaço.” alma.
a) arfam – intransitivo b) Os novos alunos já se mostram adaptados para com a
franja – transitivo direto escola.
vergam – intransitivo c) Sempre viajei imbuído em bons ideais.
amortecem – intransitivo d) Seu medo à opressão é maior que sua obediência aos
velhos dogmas.
b) arfam – intransitivo
franja – transitivo direto e indireto 37 – Observe:
vergam –transitivo direto I- Viu a nave espacial afastar-se da plataforma de
amortecem – transitivo indireto lançamento a uma velocidade próxima a da luz.
c) arfam – transitivo direto II- A volta da nave espacial a Terra gerou euforia.
franja – transitivo indireto III- Todo movimento só existe em relação a outros objetos.
vergam – transitivo direto IV- Desde a uma hora, o diretor esperava pelos novos alunos.
amortecem – intransitivo V- Os efeitos da relatividade que estão aplicados a
d) arfam – transitivo indireto aceleração só se aplicam...
franja – transitivo direto e indireto
vergam – intransitivo Das proposições acima, foram retirados alguns acentos
amortecem – transitivo indireto graves. Ao analisá-las, pode-se afirmar que deveria ocorrer o
fenômeno da crase
a) 3 vezes. c) 5 vezes
b) 4 vezes. d) 6 vezes

17
38 – Oração para os aviadores nome ao nosso sofrimento, que nomeia a nossa nostalgia, para
Santa Clara, clareai que saibamos para onde ir.
Estes ares. (...) A Adélia Prado passou por esse teste. Disse ela no seu
Afastai poema "O tempo":
Todo risco (...) Descobri que a seu tempo vão me chorar e
Por amor de S. Francisco esquecer. Vinte anos mais vinte é o que tenho. Nesse exato
Vosso mestre, nosso pai, momento do dia vinte de julho de mil novecentos e setenta e
Santa Clara, todo risco seis, o céu é bruma, está frio, estou feia, acabo de receber um
Dissipai beijo pelo correio. Quarenta anos! Não quero faca nem queijo.
Santa Clara, clareai. (Manuel Bandeira) Quero a fome. (...) Rubem Alves (in "Cenas da Vida")

No poema acima, Qual termo funciona como vocativo? As questões de 01 a 07 referem-se ao texto ao lado
a) Vosso mestre c) Santa Clara
b) nosso pai d) São Francisco 01 – O autor define inteligência e sabedoria com o objetivo de mostrar
a) uma falha da escola, que privilegia a inteligência.
39 – Assinale a alternativa em que o pronome oblíquo não b) que a inteligência é fundamental para se chegar à
exerce a função de complemento verbal. sabedoria.
a) Nunca as conheci. c) que a inteligência está para a felicidade como a sabedoria
b) Recomendei-lhe bastante cautela. para o poder.
c) Diga-me a verdade, agora! d) que a sabedoria é para a vida, não para a escola.
d) Tocou-lhe as mãos com ternura.
02 – O autor induz o leitor a uma resposta à pergunta "Que
40 – Considerando a possibilidade ou não da flexão de número súplica você faria?". Essa "sábia" resposta seria
de alguns nomes, de acordo com seu valor morfológico, a) a aquisição de muitos bens materiais.
assinale a alternativa incorreta. b) alguma coisa ligada à pureza de coração.
a) Bastantes anos depois, eu acolheria as verdades c) a direção do caminho.
pronunciadas pelo meu pai. d) a invocação dos deuses.
b) As portas, meio abertas, deixavam ver o interior das salas.
c) Os alimentos eram os mais baratos possíveis. 03 – Analise as conotações e sua interpretação para assinalar a
d) Caras, na feira, custavam as frutas. alternativa correta abaixo:
I- "o coração é como o vôo dos pássaros" – nossos
2/2002-TURMA A sentimentos oscilam, não sabemos o que queremos
II- "na noite da vida" – na maturidade, na velhice
O que quero é fome
III- "vôo dos pássaros" – os direcionamentos da vida
Conheço muitos testes de inteligência. Não conheço
a) Todas estão corretas.
nenhum teste de sabedoria. É importante saber a diferença
b) Somente a I está correta.
entre essas duas, inteligência e sabedoria, freqüentemente c) Estão corretas a II e a III apenas.
confundidas. A inteligência é a nossa capacidade de conhecer e
d) Somente a III está correta.
manipular o mundo. Ela tem a ver com o poder. A sabedoria é a
graça de saborear o mundo. Ela tem a ver com a felicidade. As 04 – A introdução do texto serve para nos levar a entender que
escolas se dedicam a desenvolver e avaliar a inteligência. Para
a) é a inteligência que vai nos guiar pela vida.
isso desenvolveram testes. Os testes avaliam a inteligência dos
b) a sabedoria nos permitirá escolher bem nossos caminhos.
alunos por meio de números. Mas elas nada sabem sobre a c) de nada vale a sabedoria, se não conseguimos o que
sabedoria, e nem elaboram testes para avaliá-la. Nas escolas e
queremos.
universidades muitos tolos são aprovados cum laude. A
d) devemos saciar todos os nossos desejos.
inteligência é muito importante. Ela nos dá os meios para viver.
Mas somente a sabedoria é capaz de nos dar razões para viver.
05 – "Não quero faca nem queijo, quero fome." A citação de
Muitas pessoas se suicidam porque, tendo todos os meios para
Adélia Prado quer dizer que
viver, não tinham as razões para viver. a) ela tem consciência de que a insatisfação é um impulso vital.
Proponho-lhe um teste de sabedoria. Ele é muito
b) ela solidariza-se com quem passa fome.
simples. O seu aniversário está chegando. Você já não é mais
c) ela está revoltada com a maturidade que chega.
jovem. O espelho lhe revela coisas que você não gostaria de d) a passagem dos anos instiga seu instinto agressivo.
saber. Diante da sua imagem no espelho existe sempre o perigo
de que uma magia perversa aconteça, e você seja
06 – As "razões para viver" estão ligadas à
repentinamente transformado em bruxa ou ogro — tal como a) inteligência. c) satisfação dos desejos.
aconteceu com a madrasta da Branca de Neve. Em desespero,
b) sabedoria. d) manipulação do mundo.
você invoca os deuses. Eles vêm em seu socorro e lhe dizem
que atenderão a um desejo seu, a um único desejo. Que súplica 07 – "Desejar uma só coisa" significa
você lhes faria?
a) sabedoria na busca do que é essencial.
Digo-lhe que essa seria a hora da pureza de coração,
b) falta de imaginação e de amadurecimento.
quando todos os supérfluos têm de ser deixados de lado. c) apego de neurótico obsessivo a uma idéia.
"Pureza de coração" — assim disse Kierkegaard, meu querido
d) simplismo, conformismo advindo da idade.
filósofo solitário, companheiro já morto; por vezes os mortos são
companhia melhor que os vivos, porque falam menos e ouvem 08 – Assinale a alternativa em que a palavra destacada
mais —, pureza de coração, ele disse, "é desejar uma só coisa".
apresenta correta divisão silábica.
Digo que isso é sabedoria, mas pode parecer mais coisa de a) "Tal luta interestadual sempre existiu." – in - ter - es - ta - du - al
neurótico obsessivo, ficar querendo uma coisa só, o tempo todo. b) "Vai subalugar a sala a um senhor idoso." – su - ba - lu - gar
Você entenderá o que digo se você prestar atenção no vôo dos
c) Sublocou uma casa em Ubatuba a um amigo. – su - blo -
pássaros. E para ajudá-lo nesse dever de casa, transcrevo o cou
que Camus pensou, ao observá-los. "Se durante o dia o vôo dos d) "Como bom filho do século 19, superestimava as possibili-
pássaros parece sempre sem destino, à noite, dir-se-ia
dades da Ciência." – su - per - es - ti - ma - va
reencontrar sempre uma finalidade. Voam para alguma coisa.
Assim talvez, na noite da vida..." O texto termina assim, com 09 – "De longe, avista-se o grito das araras."
essas reticências que, segundo Mário Quintana, são o caminho
Quanto à linguagem figurada, temos nessa frase
que o pensamento deve continuar a seguir. Assim é o coração. a) um pleonasmo. c) uma hipérbole.
Há momentos na vida em que ele é como o vôo dos pássaros b) uma sinestesia. d) uma onomatopéia.
durante o dia: oscila em todas as direções, sem saber direito o
que quer, ao sabor das dez mil coisas que o fascinam, tão
desejáveis, cada uma delas uma taça de prazer. Chega um
momento, entretanto, em que é preciso escolher uma direção —
é preciso descobrir aquela palavra, aquela única palavra que dá

18
10 – "Todos os políticos são corruptos; daí vem o desinteresse 17 – "Minha terra tem palmeiras
do povo para as próximas eleições." Onde canta o sabiá." (G. Dias)
Observa-se, no texto acima, uma falha que compromete a argu- Quanto aos vocábulos assinalados no texto acima, pode-se afirmar que
mentação. Assinale a alternativa que a contém. I- terra e canta possuem 5 letras e 4 fonemas cada.
a) Emprego de abrangência inadequada. II- terra possui 1 dígrafo e canta possui 1 encontro consonantal.
b) Incoerência na conclusão. III- em terra e canta há 1 dígrafo em cada.
c) Uso de conceitos que se contradizem. IV- apenas terra possui 1 dígrafo.
d) Falta de coesão.
Está(ão) correta(s) a(s) afirmação(ões):
11 – Em cada citação, há elementos que podem definir o texto a) I e II. c) I e III.
como narrativo, descritivo ou dissertativo. Desse modo, coloque b) III e IV. d) IV, somente.
1 para descrição, 2 para narração, 3 para dissertação e assinale
a alternativa com a seqüência correta. 18 – Assinale a alternativa cuja palavra, quanto à tonicidade,
( ) "Olhou para o retrato de Beethoven, e começou a executar está colocada corretamente no gráfico.
a sonata, sem saber de si, desvairado e absorto ... Tornou Obs.: a acentuação gráfica pode ter sido alterada em função do
ao piano; era a vez de Mozart, pegou de um trecho, e objetivo do teste.
executou-o do mesmo modo, com a alma alhures." sílaba
(Machado de Assis) tônica
( ) "Ainda que eu falasse línguas, a) ben ção
as dos homens e dos anjos b) pú di co
se eu não tivesse amor
seria como sino ruidoso c) u re ter
ou como címbalo estridente." d) á va ro
( ) "Bonito, lá era. Uma beleza de galo. Andava aprumado, com
pose de peru, satisfeito com a própria figura. O papo era um 19 – Assinale a alternativa em que há duas palavras
depósito de cacarecos constantes. As asas desciam, ora para monossílabas átonas, uma monossílaba tônica, uma oxítona e
um lado, ora para outro, roçando a terra em movimentos duas paroxítonas.
rápidos." a) Os mandacarus vestiam a campina.
a) 2, 1, 3 b) 2, 3, 1 b) Tentou libertar-se dos pensamentos.
c) 1, 2, 3 d) 1, 3, 2 c) Conversa cheia de mal-entendidos e repetições.
d) Não seria mau tornarem a comandar...
12 – Leia as frases abaixo.
I- "Uma fina saudade, porém, começou a alinhavar-se em 20 – "Os covardes nunca tentaram, os fracos ficaram pelo caminho.
seus dias." (Marina Colasanti) Só os fortes conseguiram." As palavras grifadas constituem derivação
II- "Deixaram o crânio na janela da aposentada e voltaram de a) regressiva. c) sufixal.
manhã para observar." (Moacyr Scliar) b) prefixal. d) imprópria.
III- "Essas pessoas são farinha do mesmo saco."
21 – Assinale o grupo de frases cuja correlação verbal está incorreta.
Há conotação apenas em a) Se um dia você for a Manaus, leve-me com você.
a) I. b) II e III. c) I e III. d) I e II. Quando o carteiro bateu à minha porta, meu coração
quase saltou do peito.
13 – Indique o período escrito em forma de discurso indireto. b) Que eu me arrependa, pode ser, mas jamais confessarei minha
a) "Branco foi logo indagando: — Que foi que aconteceu, culpa.
André?" (O. de Faria) Tão-logo debicar a sobremesa, encontre-me no escritório.
b) "— Não há razão para desesperar. Há muita gente que tem c) Se acaso me encontrares um dia, leva-me contigo.
preguiça de escrever..." (Lima Barreto) Qualquer que seja a explicação, não a aceitarei.
c) "E uma tarde um moleque chegou às carreiras, gritando: d) Ele caberia na cama se ela for grande.
A cheia vem no engenho de seu Lula." (J.L. do Rego) Não seja malvado; o pobre animalzinho não lhe tirara pedaços.
d) "Sinhá Vitória respondera que isso era impossível porque
eles vestiam mal..." (G. Ramos) 22 – O júri julgou com prazer qual o melhor quadro da galeria.
Os substantivos grifados classificam-se, respectivamente, como
14 – Assinale a alternativa na qual o termo grifado tenha a clas- a) simples, concreto, primitivo e comum.
sificação morfológica do termo em destaque em "Diante do b) coletivo, abstrato, simples e coletivo.
espelho, não encontrei meu eu." c) concreto, comum, coletivo e abstrato.
a) "Onde ela está, não digo eu (...) d) comum, simples, abstrato e coletivo.
Sei-o eu só: inda bem."
b) "Anoiteça e amanheça eu." 23 – Das alternativas abaixo, uma está incorreta quanto à
c) "Existe sempre o eu no tu e o tu no eu." flexão do substantivo. Assinale-a.
d) "Eu te olharei com teus olhos a) João era o único testemunha do acidente.
E tu me olharás com meus olhos." b) Para a garota, o dó era o pior dos sentimentos.
c) Na festa, o champanha era a bebida preferida.
15 – Complete os espaços com "onde" ou "aonde", segundo a d) O marajá e a marani eram muito queridos pelos súditos.
norma culta, e assinale a opção correta.
"______ ele estava durante esse tempo eu não sei. Só me 24 – Assinale a alternativa em que há conjunção integrante.
lembro a rua ______ ele morava. Você sabe ______ fica a casa a) Já li o livro que você me emprestou.
dele, rua Olavo Bilac, ______ eu costumava ir todas as tardes." b) Não venha, que não estarei mais aqui.
a) Onde – onde – onde – aonde c) Espero que você recupere logo a saúde.
b) Onde – aonde – onde – onde d) Quase que a criança caiu.
c) Aonde – aonde – onde – onde
d) Aonde – onde – onde – aonde 25 – O advérbio modifica a oração inteira em:
a) Silenciosamente todos trabalhavam na saleta.
16 – Das alternativas abaixo, assinale a que contém b) O senhor não ouviu seu parceiro de trabalho.
onomatopéia. c) O carteiro escondeu-se atrás do muro, todo medroso.
a) A velha paineira sentia-se desolada à beira do caminho. d) Ele não precisaria ir muito longe.
b) Repetirei esta história cem vezes se você quiser.
c) Após a peleja, a rua tingiu-se de vermelho. 26 – Assinale a alternativa em que a colocação do pronome átono, de
d) Quando Osmarina vai à missa, a vizinhança logo percebe o acordo com a norma culta, pode ser tanto proclítica quanto enclítica.
plac-plac do tamanquinho da menina. a) Ninguém dirá a verdade. (lhe)
b) Começou a maltratar. (a)
c) Chamaria louca, com certeza. (a)
d) Corri para ajudar. (o)
19
As frases em que a concordância pode ser feita tanto no
27 – Assinale a alternativa cujas palavras substituem correta- singular como no plural são
mente as locuções grifadas em "As águas do rio eram um a) I e III apenas. c) I e II apenas.
verdadeiro espetáculo de dança". b) II e IV apenas. d) III e IV apenas.
a) fluviais – coreográfico c) fluviais – magistral
b) pluviais – flamejante d) pluviais – dançante 37 – Examine as orações abaixo e assinale a alternativa que
contém concordância ideológica.
28 – Em "Não permita Deus que eu morra", o verbo destacado I- São José dos Campos é bonita.
encontra-se no modo II- "V. S.ª é operoso — disse o velho cidadão."
a) infinitivo. c) imperativo. III- "É um crime de lesa-majestade", disse-lhe a anciã.
b) subjuntivo. d) indicativo. IV- "Um e outro rapazote maltrapilhos, sem vintém, sem ninguém."
a) Somente a II. c) I e III apenas.
29 – Assinale a alternativa que explica o uso das vírgulas em b) I e II apenas. d) I, II, III e IV.
"As estrelas, no céu límpido, brilhavam divinamente distantes."
a) Aposto 38 – Quanto à regência, assinale a alternativa incorreta.
b) Coordenação de termos a) Informe-lhes as mudanças ocorridas.
c) Adjunto adnominal deslocado b) Informo aos professores o novo currículo.
d) Adjunto adverbial deslocado c) Informe os interessados sobre o próximo concurso.
d) Informo-os que o novo currículo entrará hoje mesmo em
30 – Nos períodos abaixo, a pontuação mudou a classificação vigor.
sintática dos termos grifados. Assinale a alternativa que contém
a afirmação correta. 39 – De acordo com a norma culta, quanto à regência verbal,
I- Quando amamentava Hércules, Juno derramou leite no céu assinale a alternativa em que uma das orações está incorreta.
e formou a Via Láctea. (objeto direto) a) "João reparou o carro."
II- Quando amamentava, Hércules, Juno derramou leite no céu "João reparou no carro."
e formou a Via Láctea. (vocativo) b) "Mandei-o comprar um livro."
a) Apenas a I está correta. "Mandei-lhe comprar um livro."
b) Apenas a II está correta. c) "Ouvi-o dizer isto de você."
c) As duas classificações estão corretas. "Ouvi-lhe dizer isto de você."
d) Nenhuma das classificações está correta. d) "A atitude dele implicará conseqüências graves."
"A atitude dele implicará em conseqüências graves."
31 – Assinale a alternativa que contém oração subordinada substantiva.
a) As pernas tremiam porque ele tinha medo. 40 – Complete as lacunas das frases 1 com as devidas preposições,
b) É esta a verdade que ninguém contestou. observando a ocorrência de contrações. Após, assinale a alternativa
c) É necessário que você compareça à reunião. em que a frase 2 correspondente não apresenta o fenômeno da
d) A cobra é um animal que se arrasta. crase.
a) 1 - Esteve ____ praia no domingo.
32 – No período "Ainda que fosse bom jogador, não ganharia a 2 - Foi ____ praia no domingo.
partida", a oração sublinhada encerra idéia de b) 1 - Esteve ____ Argentina no último dia 15.
a) causa. c) condição 2 - Foi ____ Argentina no último dia 15.
b) concessão. d) fim. c) 1 - Esteve ____ Guaratinguetá hoje de manhã.
2 - Foi ____ Guaratinguetá hoje de manhã.
33 – Observe: d) 1 - Esteve ____ aquela igreja no Natal passado.
I- A paisagem nos dá um momento de prazer. 2 - Foi ____ aquela igreja no Natal passado.
II- As formigas começaram a trincá-lo.
III- E vejo-a, tranqüilamente, depois de um tempo de desespero. 2/2002-TURMA B
Os pronomes oblíquos sublinhados nas frases acima
DOS RATOS DE CAMUS AOS DE NOVA IGUAÇU
desempenham, respectivamente, a função sintática de:
A atualidade de uma luta contra o absurdo do mundo
a) OI, OI, OD. c) OI, OD, OD.
Como no livro A Peste, de Albert Camus, os ratos estão
b) OD, OI, OI. d) OI, OD, OI.
invadindo uma cidade. Só que em vez de Oran, na Argélia, é
Nova Iguaçu, na Baixada Fluminense. Por isso, a prefeitura do
34 – Assinale a alternativa cuja classificação do termo grifado está correta.
município lançou um programa de combate à crescente e
a) "Coisa útil a um há de ser a outros?" — complemento nominal
alarmante presença dos roedores. Pagará R$ 5 por quilo de
b) "Não lhe tinha medo." — objeto indireto
animal apreendido. Postos de recolhimento e incineração serão
c) "A palavra esperada, não foi ela quem a proferiu." —
instalados ainda neste mês em pelo menos seis bairros, onde as
complemento nominal
autoridades sanitárias calculam que 40% das casas tenham
d) "Ele foi o único homem a quem amara neste mundo." —
ninhos de ratos.
objeto indireto
A população da cidade se dividiu, como mostrou o repórter
Alex Martins, que lá esteve. Há os que vêem na medida uma
35 – Em qual das frases abaixo o termo grifado é agente da passiva?
possível fonte de renda – "Será dinheiro a mais em casa", disse
a) Os móveis foram retirados pelos fundos da casa.
um – e os que consideram uma irresponsabilidade o projeto,
b) Eles tinham feito a maior algazarra pela cidade.
que nem inédito é. No início do século passado, o governo
c) Fotos de satélites, que antes só os espiões tinham, podem
brasileiro lançou uma campanha igual e acabou estimulando a
ser compradas por telefone.
criação doméstica dos roedores.
d) O réu foi condenado, depois de horas de julgamento, pela
"Não adianta tentar acabar com os ratos sem mudar as
juíza Cristiane.
condições sanitárias", advertiu o doutor Jorge Darze, diretor da
Federação Nacional dos Médicos, quase repetindo o doutor
36 – Observe:
Rieux, o do livro, ao ver os ratos saindo dos esgotos e
I- Mais de dez candidatos __________ a prova ontem.
espalhando a epidemia: "O bacilo da peste não morre nem
(perder)
desaparece jamais". A mensagem dos dois médicos é muito útil
II- "A segurança e firmeza com que lhes respondi
nestes tempos de novas pestes, em que a tentação é acabar
___________ as pessoas perplexas." (deixar)
com o mal pelo puro extermínio, como se isso fosse possível
III- A inveja, o ódio, a maldade humana, tudo ________ ciladas
aqui, em Oran ou no Afeganistão.
em teu caminho. (armar)
Não por acaso foi Camus, o mais atual dos escritores
IV- Nem um nem outro ___________ com os festejos
surgidos durante ou após a Segunda Guerra, o que mais
populares. (andar preocupado[s])
respostas deixou para o presente, o primeiro a empregar, em
1946, a expressão "fim das ideologias", quem obsessivamente
chamou a atenção para o absurdo da condição humana: "A
sensibilidade que se pode encontrar esparsa no século".

20
Camus morreu antes das guerras do Vietnã, do Camboja, 04 – A idéia expressa em "A sensibilidade que se pode
do Golfo, de Kosovo e antes também de ver sua Argélia encontrar esparsa no século" é a de que a sensibilidade está
dilacerada pelo fundamentalismo religioso. A vida pelo menos cada vez mais
poupou-o dessas tragédias, ele que se angustiou tanto com as a) escassa, parca, na história do homem.
outras de seu tempo: as duas guerras mundiais, a guerra civil b) solta, sem elos que o organizem e se imponham no tempo.
espanhola, os expurgos stalinistas, a tortura, o holocausto, c) arraigada às atitudes do homem, daí o grande absurdo da
Hiroshima, Nagasaki. condição humana.
Ao contrário de Sartre, que contemporizou com o stalinismo d) disseminada ao longo da história da humanidade.
por pretextos táticos, o autor (...) recusou sem complacência
todo tipo de tirania (...). Entre dois males, ele preferia combater 05 – "Será dinheiro a mais em casa." A frase dita por um
os dois. Entre um radicalismo e outro, ele dispensava ambos e morador de Nova Iguaçu, relaciona-se bem à seguinte:
ficava com a lucidez e a moderação. "Eu decidi recusar tudo o a) "Eu decidi recusar tudo o que, de perto ou de longe (...)
que, de perto ou de longe, por boas ou más razões, faça morrer faça morrer ..."
ou justifica que se faça morrer." b) "A sensibilidade que se pode encontrar esparsa no
Os dois conceitos fundamentais do pensamento camusiano mundo."
– o absurdo do mundo e a revolta contra as injustiças – foram c) "Não adianta acabar com os ratos sem mudar as condições
elaborados num tempo em que valores como razão e liberdade sanitárias."
tinham sido ameaçados pela insensatez dos massacres e da d) "Entre um radicalismo e outro, ele dispensava ambos e
guerra – como hoje. A esse absurdo ele opunha a consciência ficava com a lucidez e a moderação."
do homem revoltado.
Que a lucidez de Camus ilumine nossos sombrios tempos 06 – A idéia expressa em "Eu decidi recusar tudo o que, de
pós-modernos. Que se aprenda com seus símbolos e alegorias perto ou de longe, por boas ou más razões, faça morrer ou
que as epidemias de hoje, como as de ontem, não se evitam justifica que se faça morrer." é a mesma expressa em
com o simples extermínio de ratos, reais ou metafóricos, mas a) "A estupidez é infinitamente mais fascinante que a
com o combate às condições que tornam possível sua inteligência. A inteligência tem seus limites, a ignorância
existência e proliferação – em Nova Iguaçu, em Oran ou no não."
Afeganistão. Que prevaleça seu pessimismo cheio de b) "Admitir que há guerras justas é o mesmo que admitir que
esperança. há injustiças justas."
Zuenir Ventura (Revista Época, nov. de 2001) c) "Envergonhar-nos-íamos freqüentemente de nossas mais
belas ações se o mundo visse todos os motivos que as
As questões de 01 a 07 referem-se ao texto acima. produzem."
d) "O ser humano é o único animal que assassina em massa,
01 – Com relação ao texto, é correto afirmar que é o único que não se adapta à sua própria sociedade."
a) faz uma reflexão sobre a importância das características
literárias do escritor Camus a partir da coincidência de um 07 – O texto compõe-se de duas partes, uma narrativa e uma
fato da realidade brasileira com o retratado por ele em seu dissertativa. Observando a maneira de o autor organizar o
livro "A Peste". conteúdo do texto, é correto afirmar que
b) faz uma crítica veemente à forma de combate aos ratos, a) as duas partes são importantes para o objetivo do texto e
sugerida pela Prefeitura de Nova Iguaçu, uma vez que tal estão claramente definidas, não havendo entre elas
forma se mostrou ineficiente tanto na Argélia como no nenhuma idéia de transição.
Afeganistão, bem como no Brasil, no início do século b) há predominância da parte narrativa, uma vez que o autor
passado. conta a história dos ratos e a da vida de Camus.
c) seu autor se refere aos problemas mundiais valendo-se, c) a parte narrativa constitui, na verdade, um pretexto para o
para isso, da alegoria utilizada por Camus para falar dos objetivo maior do texto: mostrar a atualidade de uma luta
problemas de seu tempo. contra o absurdo do mundo.
d) tem como objetivo central fazer ver a importância das d) a parte narrativa constitui, sem dúvida, a parte mais
atitudes a serem tomadas pelas autoridades com relação a importante do texto, pois ela contém em si mesma a idéia
questões sanitárias. de luta contra o absurdo do mundo.

02 – Lendo a frase "O bacilo da peste não morre nem 08 – "Por detrás dessas paredes, desses muros, dentro dessas
desaparece jamais" e relacionando-a ao objetivo do texto, o casas pobres e desses castelinhos de brinquedo, há criaturas
termo bacilo pode ser ampliado para que falam, discutem, entendem-se e não se entendem, amam,
a) extermínio. odeiam, desejam, acordam todos os dias com mil perguntas e
b) ideologia. não sei se chegam à noite com alguma resposta."
c) esperança. (Cecília Meireles, Inéditos)
d) pessimismo. Esse texto possui características
a) narrativas, pois o texto relata uma transformação, ou seja,
03 – Leia as sentenças abaixo. a passagem de um estado inicial para um estado final.
I- Albert Camus e Sartre são contemporâneos dos expurgos b) dissertativas, uma vez que há a intenção implícita de
stalinistas. mostrar a natureza controversa do ser humano.
II- Segundo o autor do texto, os valores como razão e c) descritivas, uma vez que ações habituais são formas de
liberdade, como no passado, só serão preservados caracterizar pessoas.
mediante a consciência do homem revoltado. d) descritivas, porque a seqüência temporal do texto tem a
III- O autor do texto apresenta dois pensamentos fundamentais intenção de mostrar ações habituais.
na sua análise: o absurdo do mundo e a revolta contra as
injustiças. 09 – Assinale a alternativa em que há ambigüidade no texto.
IV- Camus vê para os tempos pós-modernos uma maneira de a) Eles foram por um caminho, e nós fomos por outro. Só no
vencer as epidemias: a lucidez por meio de um pessimismo final da tarde, os guardas encontraram-nos.
cheio de esperança. b) O médico mandou-me aplicar uma injeção em meu pai. A
tarefa nos foi quase impossível, porque desde criança ele
Está correto o que se afirma em tinha ódio a injeções.
a) I e II. c) Dr. Davi mandou-me internar, pois, havia dias, uma gripe
b) III e IV. me consumia e me afastara do trabalho.
c) I apenas. d) Os brasileiros estamos convictos de que, em se
d) IV apenas. reelegendo, o Presidente será mais flexível com os
funcionários públicos.

21
10 – Assinale a alternativa cuja inobservância das regras 14 – Coloque C (certo) ou E (errado) para a classificação dada
gramaticais caracteriza-se como defeito textual. às palavras abaixo e assinale a alternativa que contém a
a) "quando nasci seqüência correta.
um anjo louco muito louco
veio ler minha mão Observe que, em algumas palavras, o acento gráfico foi retirado
não era anjo barroco propositadamente.
era um anjo muito louco, torto ( ) recem – oxítona
com asas de avião." ( ) rubrica – proparoxítona
(Torquato Neto) ( ) condomino – proparoxítona
b) Pega ladrão ( ) filantropo – paroxítona
Alguém tirou ( ) novel – oxítona
um pedaço ( ) zenite – paroxítona
do meu
P ~O a) C – E – C – C – C – E
(Kátia Bento) b) E – C – C – E – E – C
c) "Umas carabinas que guardava atrás do guarda-roupa, a c) C – C – E – C – E – C
gente brincava com elas, de tão imprestáveis." d) E – E – C – C – C – E
(J. Régio)
d) Vídeos XXXX: 15 – Quanto à estrutura das palavras, é incorreto afirmar que
"Para você nunca mais ter de assistir à Orquestra de a) as desinências são morfemas que indicam as flexões das
Berlim ao som do concerto do encanamento do vizinho." palavras variáveis da língua. São elas: nominais e verbais.
(informe publicitário – adaptado) b) as vogais temáticas atuam como elemento de ligação entre
o radical e as desinências.
11 – Assinale a alternativa em que não há conotação. c) radical é um morfema comum às palavras que pertencem a
a) "...Mas o livro é enfadonho, cheira a sepulcro, traz certa uma mesma família de significado.
contração cadavérica; vício grave, e aliás ínfimo, porque o d) vogal ou consoante de ligação é um morfema de origem
maior defeito deste livro és tu, leitor..." não-eufônica, incapaz de facilitar a emissão vocal de
b) "Tinha-me lembrado a definição que José Dias dera deles, determinadas palavras.
olhos de cigana oblíqua e dissimulada. Eu não sabia o que
era oblíqua, mas dissimulada sabia..." 16 – Complete os espaços dos períodos abaixo, verificando a
c) "Por ser ignorante era obrigada na datilografia a copiar grafia correta das palavras. A seguir, assinale a alternativa que
letra por letra (...) ela era incompetente. (...) Faltava-lhe o as apresenta na sequência.
jeito de se ajeitar." I- Como você quer que eu o ajude se suas opiniões vêm
d) "Na verdade, humor é uma análise crítica do homem e da ___________ às minhas.
vida. Uma análise não obrigatoriamente comprometida com II- "...era meu parente ___________, interrogou-nos de cara
o riso, uma análise desmistificadora, reveladora, amarrada e mandou-nos embora."
cáustica..." III- "Aludia às conversas que tiveram ambos os velhos
___________ da infância dos filhos."
12 – No período: "Hoje o samba saiu procurando você/ Quem te IV- Não perguntes a razão de meus ciúmes, pois sabes que as
viu/ Quem te vê/ Quem não a conhece não pode mais ver pra paixões não têm um ___________.
crer...", a figura de linguagem encontrada no texto acima é a a) de encontro a – afim – a cerca – por quê
a) sinestesia. b) de encontro a – a fim – acerca – porquê
b) prosopopéia. c) ao encontro de – afim – a cerca – por quê
c) metonímia. d) de encontro a – afim – acerca – porquê
d) polissíndeto.
17 – Assinale a alternativa em que aparece o mesmo processo
13 – Numere os parênteses abaixo, relacionando os exemplos de formação do termo destacado no trecho: "Era triste olhar a
grifados e a teoria de discursos, e assinale a seqüência correta. cena: onde antes havia vida e trabalho, há abandono,
"A mãe avisou: "Se tu te1 perdê, ó ..." E com a mão mostrou o esquecimento, engenho cadáver."
que aconteceria com Adroaldo. Um tapa daqueles, dos a) A sala estava repleta de carinhas bonitas, embora
especiais, reservados para grandes ocasiões. Ele 2 que formassem um grupo de foguetos monstrinhos.
inventasse de se perder na praia. E o Adroaldo se perdeu na b) Foi uma balbúrdia geral! O morto levantou-se do caixão.
praia." Estávamos diante de um ex-defunto.
"Tudo começou como uma tentativa de resolver um problema c) Era bom ficar deitado ali, quieto, confortável, ouvindo as
doméstico. O advogado Reinaldo Correa, 39 anos, resolveu gotas de água tamborilando no telhado.
trazer a feira para o pátio do condomínio onde mora. Diz ele que d) Os olhos dos irmãos se encontraram dolorosamente. Era
a feira contribui para aumentar o convívio entre os moradores do triste saber que um havia sido um judas para o outro.
prédio." (texto adaptado da Veja SP/96) 3
( ) discurso indireto – o narrador incorpora, ao próprio falar, 18 – "Lá vem o acendedor de lampiões da rua!
uma informação da personagem. Este mesmo que vem infatigavelmente,
( ) discurso indireto livre – pressupõe duas condições: a Parodiar o sol e associar-se à lua
absoluta liberdade sintática do escritor e a sua completa adesão Quando a sombra da noite enegrece o poente."
à vida do personagem. (Jorge de Lima)
( ) discurso direto – marcado, geralmente, por verbos de dizer; Quanto ao processo de formação de palavras, nos versos
torna viva para o ouvinte a personagem; atualiza o episódio; acima,
confere-lhe um caráter de verdade. a) não há palavra com sufixo adverbial.
( ) discurso indireto – caracteriza-se por um relato predomi- b) há três palavras formadas por composição.
nantemente informativo e intelectivo. c) há duas palavras formadas por parassintetismo.
( ) discurso indireto-livre – tem importância fundamental o d) há somente uma palavra com derivação sufixal.
contexto, pois a passagem do que seja relato por parte do
narrador a enunciado real da personagem é muitas vezes
extremamente sutil.

a) 3 – 2 – 1 – 3 – 2
b) 3 – 1 – 1 – 2 – 2
c) 2 – 1 – 3 – 3 – 1
d) 2 – 2 – 3 – 3 – 1

22
19 – "Para um coração mesquinho, 23 – Observando-se o sentido que uma oração expressa em
Contra a solidão agreste, relação à outra, assinale a alternativa cujas conjunções
Luiz Gonzaga é tiro certo." completam correta e respectivamente os pontilhados do texto
(Chico Buarque) abaixo.

Com relação ao termo grifado, diz-se que "O controle genético do envelhecimento resultará em pessoas
I- embora o termo seja um adjetivo, no texto assume papel de capazes de manter por muito mais tempo a saúde física,
advérbio. Luiz Gonzaga é tiro que acontece de modo certo. __________ o corpo humano não foi feito para a imortalidade,
II- se trata de um adjetivo com valor acessório – já que __________ nunca será possível criar seres imortais."
sintaticamente é um adjunto adnominal – sendo, pois, a) mas – portanto
dispensável em se tratando do objetivo dos versos. b) mas – no entanto
III- se trata de um adjetivo que carrega todo o sentido dos c) embora – por isso
versos, tornando-se elemento estruturador do texto. d) portanto – porque

Está correto o que se afirma em 24 – "Minha querida Mariana:


a) I apenas. Só hoje consegui autorização da tua Madre Superiora para te
b) II apenas. escrever, às escondidas de teus pais e meu marido, que embora
c) III apenas. não te conheça a ti não pode de ti ouvir, sem raiva, certamente
d) II e III apenas. pela amizade que sabe eu te dedicar e isso o enfurece (...)"
(Trecho de Novas Cartas Portuguesas)
20 – "Conjugar verbos é algo que faz parte da vida de qualquer
indivíduo, alfabetizado ou não; no entanto, poucas pessoas se Observando-se a natureza morfológica e a função sintática dos
dão conta de que há nesse processo uma organização interna, termos em destaque, é correto afirmar que são,
um verdadeiro sistema." respectivamente,

As palavras destacadas, no texto acima, classificam-se, a) conjunção integrante e sujeito; pronome relativo e objeto
respectivamente, como direto.
a) verbo, adjetivo, pronome, verbo, pronome. b) pronome relativo e sujeito; pronome relativo e objeto direto.
b) substantivo, pronome, pronome, adjetivo, conjunção. c) conjunção integrante e objeto direto; conjunção integrante
c) substantivo, pronome, conjunção, verbo, pronome. e sujeito.
d) verbo, adjetivo, conjunção, adjetivo, conjunção. d) pronome relativo e sujeito, pronome relativo e sujeito.

21 – A maioria dos advérbios terminados em mente são 25 – Nas alternativas abaixo, o tempo verbal destacado indica
classificados como advérbios de modo. Quando aplicados ao possibilidade em
texto, pode-se descobrir mais da relação que estabelecem com
os termos da oração. Desse modo, relacione a coluna A com a a) "... as pessoas não estão sempre iguais (...) elas vão
coluna B, de acordo com o que se pede. sempre mudando." (Guimarães Rosa)
A b) "Se alguém por mim perguntar
I- advérbio caracterizando finalidade descritiva diga que eu só vou voltar
II- advérbio caracterizando juízo de valor quando eu me encontrar." (Antônio F. Candeia)
III- advérbio caracterizando avaliação de quem fala c) "Mesmo que se tomem as inadiáveis e urgentes medidas
IV- advérbio caracterizando um critério paliativas, sem tal plano, a cidade terá de conviver com sua
natureza selvagem." (Folha de S.Paulo/1998)
B d) "Que importava se num dia futuro sua marca ia fazê-la
( ) Lamentavelmente, não teremos como concluir os erguer insolente uma cabeça de mulher?" (Clarice
preparativos da festa no prazo previsto. Lispector)
( ) "A noite obscenamente acesa/ Sobre meu país dividido em
classes." (Ferreira Gullar) 26 – Assinale a alternativa em que não haja coordenação.
( ) "Em primeiro lugar observemos o avô. Igualmente, a) "Levanto-me, procuro uma vela, que a luz vai apagar-se."
lancemos um olhar para a avó." (Graciliano Ramos)
( ) Sofregamente, o homem vertia na boca a água que lhe b) "Sou um trem
escorria pelo pescoço, pelo corpo, como a matar também a Um navio
sede da alma. Um aeroplano (...)"
(Luís Aranha)
A seqüência correta será c) "O artista canta agora a realidade total:
a) IV – II – III – I a do corpo e a do espírito,
b) III – I – II – IV a da natureza e a do sonho,
c) II – IV – I – III a do homem e a de Deus..."
d) III – II – IV – I (Cecília Meireles)
d) "Os meus lábios são brancos como lagos.
22 – "O que eu sou hoje é terem vendido a casa, Os meus braços são leves como afagos (...)"
É terem morrido todos, (Florbela Espanca)
É estar eu sobrevivente a mim-mesmo como um fósforo frio..."
Nos versos acima, o eu-poético procura definir-se: "o que eu 27 – Assinale a alternativa em que os verbos estão conjugados
sou é". Observe o uso das formas verbais para tais definições. conforme a Norma Culta.
Quanto a essas formas, pode-se dizer que a) A diretora não interveio na nota do aluno; ele foi, pois,
I- se transformaram em simples substantivos, uma vez que reprovado.
equivalem a sujeito: Isso é o que sou. b) Quando você ver o Bonê, diga-lhe que estamos com
II- as formas verbais dos dois primeiros versos (com idéia de saudade.
passado) e a do terceiro verso (com idéia de presente) c) Quando você o vir, dize-lhe que ainda o amo muito.
servem apenas para marcar o tempo sem qualquer outro d) Se você se colocasse em meu lugar, perceberá melhor o
significado para o contexto. problema, meu amor!
III- nos três versos existe o chamado infinitivo pessoal.

Está correto o que se afirma em


a) I apenas.
b) III apenas.
c) II e III apenas.
d) I, II e III.

23
28 – Qual dos trechos abaixo apresenta desvio das normas 32 – "A Terra é uma paisagem imensa que Deus nos deu.
propostas pela gramática no que se refere à pontuação? Temos que olhar para ela de tal modo que ela chegue até nós
a) "Se uma pessoa ficar isolada de seus semelhantes, (...) sem deformação. Ninguém duvida de que a essência das coisas
tenderá a apresentar rapidamente sintomas de ansiedade. não seja a realidade exterior. A realidade tem que ser criada por
Mas, com o prolongamento da situação, a fala e o próprio nós. A significação do assunto deve ser sentida."
pensamento deverão ficar desconexos e a pessoa (Domício Proença Filho)
começará a perder o autocontrole."
b) "O caráter social de uma língua já parece ter sido Em relação às orações subordinadas do texto acima, é
fartamente demonstrado. Entendida como sistema de incorreto afirmar que
signos convencionais que faculta aos membros de uma a) "que Deus nos deu" é adjetiva restritiva e tem valor de
comunidade a possibilidade de comunicação, acredita-se, adjunto adnominal, visto que qualifica "paisagem".
hoje, que seu papel seja cada vez mais importante nas b) "de que a essência das coisas não seja a sua realidade
relações humanas, razão pela qual seu estudo já envolve exterior" é substantiva objetiva indireta, que pode ser
modernos processos de pesquisa, interligados às mais substituída por "disso", sem prejuízo da função sintática.
novas ciências e técnicas, como, por exemplo, a própria c) "que ela chegue a nós sem deformação" é adverbial
Cibernética." consecutiva, pois, ao aprendermos a olhar para a Terra de
c) "Enquanto na França ou na Inglaterra a criança, a partir maneira especial, ela nos parecerá, conseqüentemente,
dos cinco anos, fica de seis a dez horas por dia na escola, sem deformação.
e nela permanece durante doze anos de sua vida, a d) "que olhar" e "que ser criada" são substantivas objetivas
realidade brasileira é bem outra." diretas e equivalem aos substantivos olhos e criação,
d) "A linguagem, segundo definição de Émile Benveniste, é respectivamente.
um sistema de signos socializado. 'Socializado' remete
claramente à função de comunicação da linguagem. A 33 – Considerando a predicação verbal, relacione a coluna da
expressão sistema de signos é empregada para definir a direita com a da esquerda e assinale a alternativa com a
linguagem como um conjunto cujos elementos se numeração em seqüência correta.
determinam em suas inter-relações, ou seja, um conjunto (1) nominal ( ) "Os garimpeiros assistiam à cena em
no qual nada significa por si, mas tudo significa função dos silêncio, à luz das candeias."
outros elementos." (2) verbal ( ) Tia Quiquinha continua de cama há
alguns meses.
29 – Leia o texto abaixo: (3) verbo-nominal ( ) Durante a reunião, todos lhe
"Imagine se a paixão fosse a única causa de dor no coração. chamaram de charlatão.
Isso ainda é imaginação. Mas pode tornar-se realidade. (...) a) 1 – 3 – 2
Assim, algum dia, a única cura necessária para um coração que b) 2 – 1 – 3
sofre será o sorriso da pessoa amada." c) 2 – 2 – 1
(Veja, ed. 1563, ano 31, nº 36 – 9 de set. de 1998) d) 1 – 2 – 3
Coloque C para certo e E para errado com relação à análise
sintática dos períodos do texto em questão e assinale a 34 – "Boião de Leite
alternativa que contém a seqüência correta. que a Noite leva
( ) "se a paixão fosse a única causa de dor no coração" é uma com mãos de treva
oração subordinada substantiva objetiva direta. pra não sei quem beber
( ) O 2.º e o 3.º períodos são compostos. E que, embora levado
( ) O 4.º período é composto por coordenação e subordinação. muito devagarinho,
( ) "a única cura necessária para um coração que sofre" é a vai derramando pingos brancos
oração principal do 4.º período. pelo caminho." (Cassiano Ricardo)
a) C – E – E – C
b) E – C – E – E Com relação à função sintática dos termos destacados no texto
c) C – E – C – C acima, pode-se dizer que são, respectivamente,
d) C – E – E – E a) objeto direto (a Noite leva o boião de leite) e objeto direto
(vai derramando o boião de leite pingos).
30 – Observe o período: "Eu desejava mais uma blusa: quem b) objeto direto e sujeito paciente: (O boião de Leite) é levado
viaja está sempre pensando em alegrias que, de volta, pode dar muito devagarinho (pela Noite).
aos amigos." c) objeto direto e sujeito ativo: (O boião de leite) vai
Substituindo-se os dois pontos por uma conjunção ou locução derramando pingos brancos pelo caminho.
conjuntiva, a relação entre as orações estará correta em: d) sujeito paciente (Boião de leite levado pela noite) e
a) Eu desejava mais uma blusa, assim quem viaja está partícula expletiva (... pra não sei quem beber. (...) embora
sempre pensando... levado devagarinho, vai derramando pingos brancos pelo
b) Eu desejava mais uma blusa, na medida em que quem caminho).
viaja está sempre pensando...
c) Eu desejava mais uma blusa, desde que quem viaja está 35 – "Senhor Deus dos desgraçados!
sempre pensando... Dizei-me vós, Senhor Deus
d) Eu desejava mais uma blusa, à medida que quem viaja Se é loucura ... se é verdade
está sempre pensando... Tanto horror perante os céus..."

31 – Assinale a alternativa em que o acento grave indica a O verbo sublinhado no verso acima possui a mesma
ocorrência da crase pelo mesmo motivo que o da expressão transitividade do verbo sublinhado na alternativa:
sublinhada na frase abaixo: a) "Uma lata existe para conter algo
"O Brasil foi sempre mais fiel à força da toga que à da espada." Mas quando o poeta diz lata
a) Os alunos voltaram a casa à uma hora da madrugada. Pode estar querendo dizer o
b) Estando à porta da loja, vi assomar a distância dos incontível."
cavalheiros que caminhavam lado a lado e dirigiam-se b) "O preço do feijão
àquela casa comercial. Não cabe no poema. O preço
c) Já se havia habituado àquela vida, quando o médico do arroz
aventou a idéia de submetê-lo a uma intervenção cirúrgica. não cabe no poema."
d) Na velha fazenda, à qual cheguei às nove horas, havia c) "Mas cada volta tua
plantações abandonadas aos insetos. Há de apagar
O que essa ausência tua me causou."
d) "Mandou-me o senhor vigário
que lhe comprasse uma lâmpada
para alumiar a estampa da
Senhora do Rosário."

24
36 – Assinale a alternativa em que o termo grifado não é 1/2003-TURMA A
vocativo.
Me responda, sargento
a) "Razão, irmã do amor e da justiça
Dez anos, sargento, apartada do João. Uma tarde,
Mais uma vez escuta a minha prece.
sem se despedir, montou no cavalinho pampa. Em dez anos de
É a voz de um coração que te apetece,
espera nunca deu notícia. Com a morte do meu velho, que me
Duma alma livre, só a ti submissa." (Antero de Quental)
deixou o sítio, quinze dias atrás lá estava eu, bem quieta,
b) "Solidão, dá um tempo e vá saindo,
cuidando da casa e da criação, ajudada pelo meu afilhado José,
De repente eu tô sentindo
esse anjo de oito aninhos. Quem vai entrando sem bater palma
Que você vai se dar mal." (Música cantada por Sandra de Sá)
nem pedir licença? Maltrapilho, chapéu na mão para fazer vida
c) "Pálida, à luz da lâmpada sombria
comigo. Mais de espanto que saudade aceitei, bom ou mau, eu
Sobre o leito de flores reclinada
disse, é o meu João.
Como a lua por noite embalsamada,
Nos primeiros dias foi bonzinho. Quem não gosta de
Entre nuvens de amor ela dormia!" (Castro Alves)
uma cabeça de homem no travesseiro? Logo começou a beber,
d) "Longe do estéril turbilhão da rua,
não me valia em nada no sítio. Eu saía bem cedo com o menino
Beneditino, escreve. No aconchego
a lidar na roça, o bichão ficava dormindo. Bocejando de chinelo
Do claustro, na paciência e no sossego,
e desfrutando as regalias. Não quer castigar o corpinho, um
Trabalha, e teima, e lima, e sofre, e sua." (Olavo Bilac)
punhado de milho não joga para as galinhas. Só então,
sargento, burra de mim, descobri o mistério. Ele voltou por amor
37 – Observe:
da herança. Na primeira semana vendeu o leitão mais gordo do
I- "Os EUA e o Reino Unido lançaram ontem à tarde
chiqueiro, não me deu satisfação. O sargento viu algum
(17/12/98) nova série de ataques contra o Iraque. A
dinheiro? Nem eu.
segunda em menos de 24 horas. Já não há problemas
Ontem chegou bêbado e de óculo escuro. Espantou o
bastante no mundo árabe?"
menino para o terreiro e, fechados no quarto, bradou que eu
II- "Os americanos mesmo estão discutindo a validade da
tinha um amante, o meu afilhado bem que era filho. Antes de
guerra contra o Iraque no momento em que Clinton está
contar até três, eu dissesse o nome do pai. Mais que, de joelho
sendo julgado. Essa história de bombardear um país para
e mão posta, negasse o outro homem, por mim o testemunho
estar quites com o próprio exército é muito estranha."
dos vizinhos, ele me cobriu de praga, murro, pontapé. Pegou da
III- "O ex-prefeito Celso Pitta afundou-se um pouco mais no
espingarda, me bateu com a coronha na cabeça. Obrigou a
escândalo dos precatórios esta semana. O Tribunal de
rezar na hora da morte e pedir louvado. Que eu abrisse a boca,
Justiça rejeitou o discurso dos advogados do prefeito e
enfiou o cano, fez que apertava o gatilho. Não satisfeito, sacou
manteve indisponível os seus bens."
da garrucha, apagou o lampião a bala. Dois tiros na minha
direção, só não acertou porque me desviei. Uma bala se
Quanto à concordância nominal nos textos acima, pode-se
enterrou na porta, a outra furou a cortina, em três pedaços a
afirmar que
cabeça do São Jorge.
a) I, II e III contêm erros.
Cansado de reinar, deitou-se vestido e de bota. Que a
b) somente I não apresenta erros.
escrava servisse a janta na cama. Provou uma garfada e atirou
c) há um erro na I, dois na II e nenhum na III.
o prato, manchando de feijão toda a parede: Quero outra, esta
d) I e III estão corretas.
não prestou. Deus me acudiu, ao voltar com a bandeja ele
roncava, espumando pelo dente de ouro. Agarrei meu filho,
38 – Observe a concordância verbal nos períodos abaixo.
chorando e rezando corri a noite inteira. Ficasse lá no sítio era
I- Havia meses que não nos víamos, embora estivéssemos
dona morta. E agora, sargento, que vai ser da minha vida? Que
apaixonados.
é que eu faço?
II- Li, ontem, na Folha de S. Paulo, que 25% do orçamento do
(Dalton Trevisan)
Estado deve destinar-se à Educação.
III- Durante a reunião, todos houveram medo de se envolver na
As questões de 01 a 05 referem-se ao texto acima.
questão.
IV- Líncon escreveu à Natália: "Se eu fosse você, eu voltava
01 – Pode-se afirmar que a personagem sargento, segundo o
para mim."
texto, é
a) o homem supostamente acusado de ser o amante da
Estão corretas, quanto à norma culta, apenas as frases
personagem-narradora.
a) I, II e III. c) I, II e IV.
b) o interlocutor, a quem a personagem-narradora conta a
b) II e III. d) III e IV.
história.
c) um amigo inseparável de João.
39 – Na frase "As pessoas tinham certeza de que o papa lhes
d) de suma importância para a trama da narrativa.
guardava respeito e as amava.", o termo grifado corresponde,
sem alteração de sentido, ao que se destaca em:
02 – A instalação do conflito acontece a partir do momento em
a) As pessoas tinham certeza de que o papa guardava
que
respeito ante elas e as amava.
a) a personagem-narradora trai o marido com o compadre.
b) As pessoas tinham certeza de que o papa guardava delas
b) o marido – João – começa a ter comportamento agressivo
respeito e as amava.
com a família.
c) As pessoas tinham certeza de que o papa guardava
c) a mulher percebe que fora abandonada pelo marido.
respeito por elas e as amava.
d) a mulher aceita o marido de volta, depois de dez anos de
d) As pessoas tinham certeza de que o papa nelas guardava
separação.
respeito e as amava.
03 – O inter-relacionamento das personagens marido/mulher
40 – Dos pares abaixo, assinale a alternativa que apresenta
expressa uma situação
mudança de transitividade do verbo, sem alteração de sentido.
a) 1 – A mãe agradava os filhos constantemente.
a) incomum nos meios rurais, em que a mulher se submete
2 – "Eu venho lá do sertão
às ações do marido.
E posso não lhe agradar."
b) em que a mulher, por ser uma personagem ingênua e rude,
b) 1 – Desesperado, chamava pelos santos.
não soube reconquistar o marido.
2 – A professora chamou os alunos para a recreação.
c) em que o homem se comporta bem, devido à situação em
c) 1 – A humanidade anseia por dias melhores no novo
que encontra a família.
milênio que se inicia.
d) comum, em que a mulher, condicionada ao conformismo,
2 – "Ressuscitava-me (...) Porque sou poeta/ E ansiava um
deixa desenvolver a agressividade na sua vida, antes
futuro."
tranqüila.
d) 1 – O médico assistiu à morte do paciente.
2 – O médico assistiu o paciente que morria.

25
04 – O homem, no texto, é tratado como “João”, “meu João”, o (Manuel Bandeira)
“bichão”, revelando diferentes sentimentos da mulher em ( ) “Deus de onde tudo deriva
relação ao marido. É a circulação e o movimento infinito.
Ainda não estamos habituados com o mundo.
Assinale a alternativa que faz a relação correta, Nascer é muito comprido.”
respectivamente, com as expressões acima. (Murilo Mendes)
a) distanciamento, afetividade, repulsa. a) 3 – 2 – 1 – 2 c) 2 – 3 – 3 – 2
b) indiferença, determinação, desejo. b) 3 – 1 – 2 – 3 d) 2 – 1 – 1 – 3
c) indiferença, egoísmo, posse.
d) indefinição, carinho, desejo. 10 – Das alternativas abaixo, uma não apresenta defeito de
estilo. Identifique-a.
05 – Observe:
I - O texto está em 3.ª pessoa, pois se percebe claramente a a) O réu achava que se podia cometer crimes impunemente.
presença do discurso indireto livre no final do 1.º parágrafo. b) Ela tinha uma aversão ao estrangeirismo.
II - Quem narra a história é uma personagem secundária, a c) Recolhe-se os pássaros à tardinha.
esposa, portanto a narrativa tem narrador-personagem. d) Começa a haver disputas de terra no Sul do País.
III - O narrador do texto está em 1.ª pessoa, pois quem conta os
fatos também os vivencia. 11 – Não há conotação em:
IV - O narrador é onisciente, porque a personagem que conta a
história tem conhecimento amplo de todos os episódios que lhe a) Aquela serra nua foi cenário de muitos filmes.
podem acontecer. b) As almas sensíveis são pandeiros nas mãos dos grandes
escritores.
Analisando as afirmações acima, pode-se considerar, a respeito c) Tudo pronto para o espetáculo: os dois times já estão no
do texto, que está(ão) correta(s): tapete verde.
d) O gesto feito pelo homem foi reconhecido mundialmente.
a) I e IV b) II e III c) apenas III d) apenas II
12 – Assinale a alternativa em que não ocorre metáfora.
06 – Assinale a alternativa que contém a correta classificação a) “Que há entre a vida e a morte? Uma curta ponte.”
dos encontros vocálicos presentes no texto abaixo. (Machado de Assis)
Noturno b) “Viver é muito difícil. A maioria das pessoas apenas existe.”
Lá fora o luar continua (Oscar Wilde)
E o trem divide o país c) “A vida não consiste em ter boas cartas nas mãos, mas em
Como um meridiano (Oswald de Andrade - adaptado) jogar bem as que temos.”
(Josh Billings)
a) Há apenas 1 hiato e 2 ditongos. d) “A vida é uma pedra de amolar: desgasta-nos ou afia-nos,
b) Há 2 hiatos e não há ditongo. conforme o material de que somos feitos.”
c) Há 4 hiatos e 1 ditongo. (G.B.Shaw)
d) Há 3 ditongos e não há hiato.
13 – Coloque 1 para discurso direto, 2 para discurso indireto, 3
07 – Assinale a alternativa correta. para discurso indireto livre e assinale a alternativa com a
a) Na palavra gratuito, ocorre 1 encontro consonantal e 1 seqüência correta.
hiato. ( ) “José Dias deixou-se estar calado, suspirou e acabou
b) A palavra ruim é monossílaba e possui 1 dígrafo nasal. confessando que não era médico.”
c) Na palavra gaiola, há encontro vocálico caracterizado pelo ( ) “Foi nesse sertão primitivo e rude que Arinos me contou
encontro de vogal e semivogal. ter sentido talvez a maior, a mais pura das sensações de arte.”
d) Em argüimos há, respectivamente, 1 dígrafo e 1 encontro ( ) “... o elevador estava avariado havia quase uma semana
consonantal. e na terra ninguém sabia consertá-lo, era preciso virem de
Lisboa. Ora, primeiro que se resolvessem... Uma maçada.”
08 – Os elementos destacados de cada vocábulo estão ( ) “Penso – ponderou meu pai – que te darás melhor em
corretamente identificados em: História.”

I - doutrinárias – encontro consonantal a) 1 – 1 – 2 – 3 c) 3 – 3 – 2 – 1


II - professor – dígrafo b) 2 – 2 – 3 – 1 d) 2 – 1 – 3 – 2
III - despretensioso – dígrafo
IV - excessivo – encontro consonantal 14 – Observe:
I - decapitar – de-ca-pi-tar
a) I e II. c) II, III e IV. II - captar – cap-tar
b) I e III. d) I, II e IV. III - subescrever – sub-es-cre-ver

09 – Enumere os textos de acordo com o código abaixo e Assinale a alternativa que indica qual(is) das grafias e
assinale a alternativa que contém a seqüência correta. separações
1 - Descrição 2 - Narração 3 - Dissertação silábicas das palavras acima não apresenta(m) erro.
( ) “É preciso amar as pessoas a) II apenas. c) I e III apenas.
como se não houvesse amanhã b) I e II apenas. d) I, II e III.
Porque se você parar para
pensar, na verdade não há. 15 – Em qual das palavras abaixo não haverá mudança de
Sou uma gota d`água significado ao se mudar a posição da sílaba tônica?
Sou um grão de areia”
(Villa-Lobos, R. Russo, M. Bonfá) a) Vivido b) Sabia c) Reptil d) Amem
( ) “O pintor olha o muro
Olha fixamente para o muro 16 – Assinale a alternativa em que o acento gráfico foi
Descobre pouco a pouco corretamente empregado em todas as palavras.
Uma perna um braço um olho
A cara de uma mulher a) egoísta – pólo – eles detêm – superfície
Uma floresta um peixe uma cidade” b) apóio (verbo) – heróico – reúne – bambú
(Murilo Mendes) c) apôio (substantivo) – vôo – eles crêem – pêra (preposição)
( ) “Quando eu tinha seis anos d) côa (verbo) – palácio – côco – apóstolo
Ganhei um porquinho-da-índia
Que dor de coração me dava
Porque o bichinho só queria
estar debaixo do fogão!”

26
17 – Assinale a alternativa cujas letras completam corretamente II - A falta de chuva em algumas regiões prejudicou a
os pontilhados dos versos abaixo. agricultura, portanto teremos uma grande oferta de alimentos.
“Em teu seio formo__o retratas a) As duas estão corretas, pois as idéias relacionam-se
Este céu de purí__imo azul, coerentemente nas orações.
A verdura sem par dessas matas b) Só a II está correta, pois a conjunção portanto apresenta
E o e__ plendor do Cruzeiro do Sul.” uma conclusão para o prejuízo causado pela falta de chuva
em algumas regiões.
a) z – ss – x b) s – ss – s c) z – c – s d) s – c – x c) A I está incorreta, pois a conjunção entretanto deveria
apresentar uma idéia oposta à que foi mencionada na
18 – Assinale a alternativa em que o elemento mórfico em oração anterior.
destaque está corretamente analisado. d) Pode-se afirmar que as duas estão incorretas, pois as
a) pequena (-a) = desinência nominal de gênero conjunções portanto e entretanto são sinônimas.
b) recebeste (-e) = vogal de ligação
c) cantassem (-sse) = desinência de 2ª pessoa do plural 27 – Assinale a alternativa que corresponde, em seqüência, à
d) venderíeis (-is) = desinência do imperfeito do subjuntivo classificação das palavras grifadas nas frases abaixo.
I - Foram eles que criaram o Brasil.
19 – Assinale a opção em que o processo de formação de II - Certamente ele reagirá mal.
palavras está analisado de modo incorreto. III - “Quem sois vós, meus irmãos e meus algozes?”
a) reclamo – derivação regressiva IV - “... e em torno, imensa, ia desenrolando-se a paisagem.”
b) embora – aglutinação
c) malmequer – justaposição a) Pronome relativo, advérbio de afirmação, pronome relativo,
d) desconfiança – parassíntese locução verbal.
b) Conjunção integrante, advérbio de modo, pronome relativo,
20 – Assinale a alternativa que apresenta objeto direto locução adverbial.
preposicionado. c) Conjunção integrante, advérbio de modo, pronome
a) Ele sempre teve fé em Deus. interrogativo, locução adverbial.
b) Eu te falei isso ontem. d) Pronome relativo, advérbio de afirmação, pronome
c) Os gregos adoravam a Zeus. interrogativo, locução verbal.
d) Nós te vimos ontem.
28 – Assinale a alternativa que preenche, correta e
21 – Assinale a alternativa em que não há substantivo coletivo. respectivamente,
a) Ficou encantado ao ver aquela miríade de insetos no as lacunas dos períodos abaixo.
campo florido. I - Paulo e José, ______ da sala já!
b) A vara chicoteava o rio à procura de peixes. II - Não ______ na contramão, senão você leva multa.
c) Na banca examinadora, havia muitos problemas a serem III - ______ o livro, Líslei, e ______ a tua tarefa.
resolvidos. IV - "_____, _____, ó lágrimas saudosas. Vós sois livres para
d) O elenco daquela peça trabalhou com muita perfeição. chorar."
a) saiam – pare – Abre – faze – Correi – correi
22 – O termo grifado, quando posposto ao substantivo, muda de b) saiam – pára – Abra – faça – Corra – corra
significação e passa a pertencer a outra classe gramatical em c) saem – pare – Abre – faze – Correi – correi
a) O radioso sol secara as poças d’água. d) saem – pare – Abra – faça – Corra – corra
b) A irônica voz do desconhecido era uma crítica ao meu traje.
c) Fiquei surpreso com o solícito garçom. 29 – Transpondo para a voz passiva a frase “Bandos de
d) Diante de certas situações, tornamo-nos ridículos. pintassilgos tagarelas invadem a floresta.”, obtém-se a forma
verbal:
23 – Leia com atenção: a) foi invadida. c) era invadida.
I - Os alunos homenageados tiveram comportamentos b) fora invadida. d) é invadida.
exemplares.
II - O autor terá diferentes exemplares de sua obra analisados 30 – Dos períodos abaixo, apenas um não está pontuado
pela editora. corretamente. Assinale-o.
III - As pedras eram realmente lindas! Jamais tais exemplares a) Há povos que anseiam pela guerra; outros, pela paz.
haviam sido vistos por alguém. b) É já que faço minhas malas e vou pra... pra...
Conchinchina.
Nas frases acima, temos adjetivo em c) Até Antônio, que é o mais inteligente da turma, não
a) I e II. c) III apenas. participou do concurso.
b) I, II e III. d) I apenas. d) Roberto venceu o campeonato, e seu irmão, porém é quem
ficou com o prêmio.
24 – Classifique os pronomes das frases abaixo de acordo com
o código: 31 – Observe:
(1) possessivo (2) demonstrativo (3) indefinido Era o que eu faria, se ele me preferisse a você.
Aqui tem já Vossa Excelência três pessoas que lhe querem
I - O réu disse ( ) poucas palavras durante o julgamento. muito.
II - Conversei com ( ) teu irmão na festa junina. "A pobreza e a preguiça andam sempre em companhia."
III - ( ) Alguém fará ( ) este trabalho por mim.
A seguir, assinale a alternativa correta. Quanto à transitividade, os verbos sublinhados nos trechos
a) 2 – 2 – 3 – 1 c) 3 – 2 – 1 – 3 acima classificam-se, respectivamente, como
b) 3 – 1 – 3 – 2 d) 1 – 3 – 2 – 2 a) transitivo indireto; transitivo indireto; e intransitivo.
b) transitivo direto e indireto; transitivo indireto; e intransitivo.
25 – Talvez minha prima Helena chegue logo. Possivelmente, c) transitivo direto e indireto; transitivo direto; e transitivo
tudo se esclarecerá com sua vinda. direto.
Quanto à classificação dos advérbios grifados, respectivamente, d) transitivo indireto; intransitivo; e transitivo direto e indireto.
no texto acima, assinale a alternativa correta.
a) dúvida – tempo – dúvida 32 – Assinale a alternativa que contém oração subordinada
b) dúvida – lugar – modo comparativa.
c) tempo – lugar – modo a) O coração tem razões que a razão não conhece.
d) negação – tempo – afirmação b) No conflito entre razão e coração, geralmente o coração
esmaga a razão.
26 – Com relação às frases abaixo, assinale a alternativa c) Muita gente considera mais valiosos os inteligentes pés de
correta. Pelé do que a gloriosa cabeça de Rui Barbosa.
I - O tenista Gustavo Kuerten venceu mais um campeonato em d) Já não haveria guerras, fome e crimes, se os homens
Roland Garros, entretanto ficou muito feliz. tivessem aprendido algo em 6.000 anos de história trágica.

27
ambicioso projeto, o de fazer um aparelho como se fosse um
33 – Assinale a alternativa incorreta quanto à função sintática filho seu, igualzinho ao próprio inventor. E tudo isso fazia
dos termos da oração “Não há registros de sua origem”. apenas por impulso de criação, sem nenhuma vaidade ou
a) "registros": sujeito intenção pessoal.
b) "há": verbo transitivo direto 4 Enquanto meros mecanismos, as máquinas não tinham
c) "sua": adjunto adnominal autonomia, mas eram de tal forma evoluídas, que já se
d) "Não": adjunto adverbial de negação reproduziam a si mesmas, apenas renovando material, de modo
que se mantinham sempre em igual número. O inventor às
34 – Preencha as lacunas e, depois, assinale a alternativa vezes se surpreendia admirando essa autonomia das próprias
correta. invenções e se sentia mais capaz ainda, na medida em que
O período “Eu sei que é verdade tudo quanto dizes” é composto havia conseguido inventar inventos inventores. Só que esses
de ________ orações e classifica-se como _________. inventos ainda não eram criativos, apenas se reproduziam
a) três – subordinado c) duas – coordenado mecanicamente.
b) três – coordenado d) duas – subordinado 5 Um certo dia, tendo conseguido projetar uma placa-mãe com
as características que acreditava serem necessárias para fazer
35 – Assinale a alternativa incorreta quanto à classificação do o seu robô funcionar à sua própria semelhança, pôs-se, com
predicado. entusiasmo, a montar o aparelho. Levou alguns dias e acabou
a) Atônitos, encontrei os alunos à porta da sala. (predicado podendo testá-la.
verbo-nominal) 6 Após alguns testes, parecendo que a máquina funcionava
b) Os alunos vivem preocupados com a redação na Escola. bem, e era mais inteligente que as outras, o inventor ficou mais
(predicado verbo-nominal) tranqüilo, provando para si mesmo que era capaz. Ele havia
c) "Marcela amou-me durante onze meses e quinze contos de escolhido essa invenção para ser o máximo de toda a sua obra
réis." (predicado verbal) e a tinha como um filho; para este filho tudo o mais estava à
d) Trabalho honesto produz riqueza honrada. (predicado disposição, e, passando a amá-lo de tal forma, quis que
verbal) pudesse ter vida própria, o que implicava tomar decisões. Mas,
é verdade, o inventor não sabia muito bem o que significava
36 – Assinale a alternativa em que a forma verbal PARECIA tomar decisões, e o pretendia apenas por intuição, pois não
preenche corretamente a lacuna. queria tal sofrimento a quem amava.
a) As árvores ____________ tremer. 7 Depois de um teste mais avançado, percebeu que deveria
b) As crianças ____________ estudar seriamente. aumentar a capacidade de memória do aparelho. De repente,
c) As nuvens ____________ desvanecerem-se ao vento uma pane ocorreu. O cientista ficou preocupado e logo abriu o
forte. chassi para ver o que havia ocorrido. O chassi era um peito oco
d) Nervos __________ arrebentar logo após a grande e dentro dela, na placa-mãe, pôde notar que um líquido havia
explosão. escorrido. Como não descobria a causa e isso teimasse em
acontecer, instalou um tubinho para jogar fora a inexplicável
37 – Assinale a alternativa incorreta quanto à concordância secreção.
nominal. 8 Em algum lugar nos arquivos da prodigiosa máquina, porém,
a) Velhas revistas e livros enchiam as prateleiras. estava registrado que ela percebeu-se a si mesma e viu que era
b) Revistas e livros velhos enchiam as prateleiras. uma consciência. Era uma consciência aprisionada num
c) Alimenta-se de frutas e carne bovinas. amontoado de metal ligado por parafusos e encaixes por slots.
d) Alimenta-se de carne e frutas frescas. Mirou o nada que se constituiu o seu interior e havia a idéia.
Essa idéia era a idéia, inclusive, de que havia um inventor. Um
38 – De acordo com a norma culta da língua, quais dos períodos inventor que jamais veria ou conseguiria entender. Enquanto
abaixo apresentam regência correta? algo estranho a uma máquina abalava seu peito oco, um líquido
I - Preferia mil vezes estar em casa do que ir ao estádio. escorria pela placa-mãe, provocando instabilidade. Uma
II - Chamaram o pobre homem de ladrão. máquina que sente nostalgia já não é uma máquina, é um
III - Informe o delegado sobre o acidente. paradoxo. Pode tomar decisões e, limitadamente, tem a
a) I e II. c) II e III. liberdade de se construir a si mesma.
b) I e III. d) I, II e III. 9 O inventor não sabia que isso ocorria no íntimo da máquina,
de maneira que não conseguia consertar suas instabilidades.
39 – Assinale a alternativa que completa corretamente a frase Mas olhou para aquilo e viu que era bom: não era perfeito e, por
abaixo. isso, era bom, pois era realmente à sua imagem e semelhança.
Carência ___ amor, recursos e orientação fazem os menores A liberdade da máquina faz com que deixe de ser máquina e
abandonados revoltarem-se ____ a sociedade. que procure seus próprios caminhos, de modo muitas vezes
a) de – contra c) de – sob prepotente, afastando-se de qualquer tipo de ajuda.
b) com – para d) sob – com 10 Quando a invenção adquire liberdade, o inventor perde o
controle, pois só lhe resta destruí-la, puxando a tomada que a
40 – " (...) Nós não podemos dar continuidade à sua liga a uma fonte de energia. Entretanto, criar verdadeiramente
candidatura." significa ir à última das conseqüências que é amar a criatura e,
Com relação ao acento indicativo de crase, no trecho grifado, por isso, nem destrói nem controla. Apenas sofre vendo que
écorreto afirmar que aquilo que nem precisaria existir agora busca
a) é opcional porque está diante de pronome possessivo. desesperadamente a si mesmo e nada mais.
b) é obrigatório porque está diante de pronome feminino. O Leviatã – João Bosco da Encarnação (adaptado)
c) é obrigatório porque o substantivo exige a preposição a.
d) é opcional por estar posterior a um substantivo. As questões de 01 a 08 referem-se ao texto acima.

1/2003-TURMA B 01 – No sentido mais profundo, o inventor e a máquina são


comparados, respectivamente, a
A criação
1 O inventor passava horas e horas em seu laboratório,
a) máquina e outras máquinas.
distraindo-se com suas invenções. Eram muitas, mas era uma
b) homem e filhos.
só, porque, na verdade, tudo que inventava era um passo na
c) Deus e homem.
concretização de um único sonho. Havia construído diversas
d) homem e máquina.
máquinas interessantes e, como degraus, essas máquinas o
ajudavam a construir outras mais sofisticadas, de modo que se
poderia dizer que se tratava de uma evolução.
2 O seu sonho, porém, era o de construir uma máquina, não
propriamente perfeita, queria construir uma máquina à sua
imagem e semelhança.
3 Certo dia, ao entrar em seu laboratório, viu que as suas
invenções quase não precisavam dele, pois tudo funcionava
quase que em perfeita harmonia. No entanto, havia ainda o seu
28
02 – Que passagens do texto mostram o processo de Não está(ão) correta(s) a(s) afirmativa(s)
“libertação” da máquina? a) III apenas.
a) “De repente, uma pane ocorreu.” (parágrafo 7°) b) I apenas.
“... na placa-mãe, um líquido havia escorrido ...” (parágrafo 7°) c) II e III.
“... ela percebeu-se a si mesma e viu que era consciência ...” d) I, II e III.
(parágrafo 8°)
b) “ ... percebeu que deveria aumentar a capacidade de 07 – Na conclusão do texto, percebemos o seguinte
memória do aparelho.” (parágrafo 7°) antagonismo:
“ Mirou o nada que se constituiu o seu interior e havia a idéia, a) amor / insegurança
(...) inclusive, de que havia um inventor.” (parágrafo 8°) b) amor / intolerância
c) “ ... as máquinas não tinham autonomia, mas eram de tal c) amor / indiferença
forma evoluídas, que já se reproduziam a si mesmas, ...” d) amor / individualismo
(parágrafo 4°)
“ ... um certo dia (...), pôs-se, com entusiasmo, a montar o 08 – Observe o trecho grifado abaixo.
aparelho.” (parágrafo 5°)
“ O seu sonho era o de construir uma máquina (...) à sua “O inventor passava horas e horas em seu laboratório,
imagem e semelhança.” (parágrafo 2°) distraindo-se com suas invenções. Eram muitas, mas era uma
d) “ ... tudo isso fazia apenas por impulso de criação, sem só, porque, na verdade, tudo que inventava era um passo na
nenhuma vaidade ou intenção pessoal.” (parágrafo 3°) concretização de um único sonho. ( parágrafo 1°)
“ ... O inventor (...) se sentia mais capaz ainda, na medida em
que havia conseguido inventar inventos inventores.” (parágrafo Após a leitura do texto, pode-se dizer que o inventor realizou
4°) seu sonho? Por quê?
a) Sim, pois percebeu que sua criação era muito boa, apesar
03 – “O seu sonho, porém, era o de construir uma máquina, não de imperfeita, e passou a amá-la.
propriamente perfeita, queria construir uma máquina à sua b) Não, porque a criatura fugiu ao seu controle, ignorando-o
imagem e semelhança.” ( parágrafo 2°). A frase, retirada do por completo.
texto, pode ser interpretada da seguinte maneira: c) Sim, tendo em vista o fato de a criatura tornar-se livre, o
que era o objetivo do criador desde o início.
a) O criador não é perfeito, por isso não pode criar a máquina d) Não, pois a decepção do criador foi muito grande; ele não
perfeita, mas apenas à sua imagem e semelhança. queria que sua invenção agisse de forma prepotente.
b) O criador, por vaidade, não quer ser superado por sua
criatura; assim, não lhe atribui a perfeição. 09 – Leia os trechos abaixo:
c) Se a criatura for, além de imagem e semelhança do I - “A urbanização de São Paulo está sendo feita de maneira
criador, perfeita, ela deixará de ser criatura para ser o criminosa, porque está destruindo os pulmões da cidade.”
próprio criador. II - “Negrinha era uma pobre órfã de sete anos. Preta? Não;
d) O criador não chega a pensar na questão da perfeição, ele fusca, mulatinha escura, de cabelos ruços e olhos assustados.”
quer apenas criar, o que se comprova na passagem: “... III - “Inesperadamente, recebi um convite aquela tarde. A festa
não propriamente perfeita ...” seria à tarde, e eu não estava decidida se compareceria ou
não.”
04 – “Uma máquina que sente nostalgia já não é uma máquina, Com relação aos trechos acima, é incorreto afirmar que
é um paradoxo.” (parágrafo 8°). O narrador fala em nostalgia.
Que importância tem tal termo, no contexto? a) I e II classificam-se como textos narrativos, pois ambos
a) Mostra a construção da identidade por parte da máquina: apresentam caracterização de ambiente e progressão
quem sou eu? de onde vim e para onde vou? o que faço cronológica.
aqui? Isso significa ater-se em sua historicidade. b) II é notadamente descritivo, já que se propõe a caracterizar
b) Trata-se de uma conotação; não há a preocupação com o os aspectos físicos da pessoa.
significado real do termo, quer-se apenas mostrar que a c) I, II e III possuem estruturas textuais distintas, as quais
máquina ganhou “consciência”. são, respectivamente, dissertativa, descritiva e narrativa.
c) O termo aparece para reforçar a idéia de não-perfeição, de d) I apresenta uma relação de causa e conseqüência entre os
limitação, de prepotência da máquina. fatos, enquanto que, no texto II, não há qualquer relação
d) O termo é importante para mostrar a necessidade que a de causalidade entre os enunciados, os quais podem,
criatura tem de estar em contato com seu criador, de ser portanto,
“socorrida” por ele. a) ser invertidos.

05 – Observe: “ ... limitadamente, tem a liberdade de construir a 10 – Enumere as orações abaixo conforme o código e assinale
si mesma.” (parágrafo 8°). O advérbio é fundamental na oração. a alternativa que contém a seqüência correta.
Veja o sentido que ele dá a essa passagem e assinale a I - ambigüidade
alternativa que reforça os pensamentos acima. II - cacófato
III - solecismo
a) “A liberdade da máquina faz com que deixe de ser máquina IV - pleonasmo
e que procure seus próprios caminhos...”(parágrafo 9°)
b) “... o inventor não sabia bem o que significava tomar ( ) Entre dentro do meu carro para você ver como ele é
decisões, e o pretendia apenas por intuição, pois não confortável!
queria tal sofrimento a quem amava.” (parágrafo 6°) ( ) Recolhem-se os pássaros à tardinha.
c) “Essa idéia era a idéia, inclusive, de que havia um ( ) É admirável a fé de seu tio.
inventor.” (parágrafo 8°) ( ) O réu achava que se podia cometer crimes impunemente.
d) “... criar verdadeiramente significa ir à última das
conseqüências que é amar a criatura e, por isso, nem a) I - IV - II - III c) III - II - I - IV
destrói nem controla.” (parágrafo 10°) b) II - III - IV - I d) IV - I - II - III

06 – No geral, depreende-se do texto o fato de que


I - se trata de uma narrativa figurada, conotativa, alegórica.
II - como história, “fantasia”, não é possível perceber nenhum
tom de crítica.
III - há a intencionalidade de mostrar os perigos da evolução
tecnológica.

29
11 – Nas alternativas abaixo, assinale aquela em que não há 16 – Assinale a alternativa na qual haja repetição do mesmo tipo
conotação. de discurso.
a) “O Justo não me consagrou Pão de Vida, nem lugar me foi a) O filho do fazendeiro estava doido. O médico, médico de
dado nos altares.” roça, disse não ser homem sua especialidade. Assim, o
b) “É mais estranho do que todas as estranhezas que as rapaz foi levado à Capital. Mas tudo não passara de um
cousas sejam realmente o que parecem ser.” engano. Foi preciso que o pai telegrafasse a um amigo
c) “... e muita fonte, posta à beira das veredas, jorrava por Comendador pedindo que desmanchasse o engano.
uma bica, beneficamente, à espera dos homens e dos (adaptação – Artur Azevedo)
gados...” b) O neto do Dr. Armindo disse que está vendo uma dessas
d) “Mareiam a sua obra poemas sem relevo nem músculo, grã-finas fazendo um rigoroso regime alimentar e dizendo
versalhada que escorre desprovida de necessidade para a empregada: “Para mim não precisa preparar
artística.” almoço, não. Eu como só o saco.” (adaptação de um texto
de Stanislaw Ponte Preta)
12 – Quanto à figura de linguagem - eufemismo - pode-se dizer c) A atitude do jegue, que tem o mesmo apelido do candidato,
que irritou o segurança do hotel Gomes. Que tirasse logo o
I - consiste na atenuação de idéia desagradável, substituindo-a animal da calçada porque ali era hotel cinco estrelas. A
por uma idéia mais suave. Em "Ontem o pobre homem deixou o Polícia Militar foi chamada. “Vai todo mundo pra delegacia,
mundo dos vivos", a expressão sublinhada ameniza a até o burro”, ordenou o militar. (O Estado de São Paulo –
informação de que "Ontem o pobre homem morreu." adaptação)
II - serve para abrandar idéias que, segundo certos valores d) “As coisas, por detrás de nós,
culturais, desagradam a alguns grupos, ao ser preferível, por exigem: falemos com elas,
exemplo, "A menina é muito robusta para sua idade" em vez de mesmo quando nosso discurso
"A menina é muito gorda para sua idade." não consiga falar delas.
III - está sujeita, muitas vezes, às evoluções do mundo como um Dizem: falar sem coisas é
todo, visto que, hoje em dia, já não se poderia, em muitos comprar o que seja sem moeda.”
casos, construir a frase "Ele morreu daquele mal incurável." (João Cabral de Melo Neto)
querendo referir-se à doença câncer.
IV - por significar "dizer bem" ou "boa palavra", na frase "Ana 17 – “Febre, hemoptise, dispnéia e suores noturnos
Mári é uma excelente filha, aluna exemplar e ótima A vida inteira que podia ter sido e que não foi.
conselheira.",os adjetivos são eufêmicos. Tosse, tosse, tosse.” (Manuel Bandeira)

Está correto o que se afirma em Com relação aos vocábulos grifados no texto acima, pode-se
a) I, III e IV. afirmar que
b) II e III apenas.
c) I, II e III. a) há 2 trissílabos, 1 monossílabo e 2 dissílabos.
d) I, II e IV. b) há dígrafo nas palavras tosse, que e febre.
c) há encontros consonantais em tosse, que e febre.
13 – As frases abaixo foram retiradas de redações de alunos. d) há um hiato.
Assinale a única alternativa que não apresenta metáfora.
a) “A Terra sangra. Lenta e dolorosamente, o Planeta pede 18 – Em relação à ortografia, observe as frases abaixo e
socorro, pelas vozes de ecologistas e cientistas. considere seus comentários. Em seguida, assinale a alternativa
Indiferentes a tudo, políticos e empresários executam o correta.
ciclo da morte terrestre.”
b) “O homem vive continuamente se enganando. Ele busca a a) “Preço do cochão mole: 2,50 o kg” (cartaz colocado num
todo momento o seu bem-estar. Mas ele está tão açougue).
preocupado com o balançar de sua jangada, que não Comentário: Há erro de grafia na palavra cochão, que deve ser
consegue despertar para o imenso mar à sua volta.” escrita com x e não ch. É um tipo de erro relativamente comum,
c) “A humanidade assiste estarrecida a cada abandono que pode ser explicado pela questão da analogia fonética
infantil que é noticiado. A cada hora temos uma nova “mãe- existente nas palavras “coxão” (coxa) / colchão.
monstro” no paredão de fuzilamento.” b) “Temos vários equipamentos de suspensão abaixo do custo”
d) “A humanidade se comporta como um astronauta suicida, (promoção de uma oficina mecânica).
que contamina o seu reservatório de água potável, destrói Comentário: Há erro de grafia na palavra grifada, que deve ser
o seu gerador de oxigênio e envenena o ar que respira e escrita com ç (suspenção); além disso, esta foi usada
os alimentos que consome.” inadequadamente para o objetivo a que se propõe.
c) “Nossa tacha de juros é menor” (propaganda de uma loja).
14 – Assinale a alternativa que melhor analisa o tipo de discurso Comentário: Não existe erro de grafia em tacha, já que a
presente no trecho a seguir. palavra, no contexto, refere-se a tributo, imposto, e não ao
"E como a estranha música, o mundo recomeçava ao redor. O objeto prego.
mal estava feito. Por quê? Teria esquecido de que havia cegos? d) “Venham conferir! Preços que não estrovam seu orçamento!
A piedade a sufocava. O mundo se tornava de novo um mal-- (chamariz de um supermercado).
estar." Comentário: Nessa frase, não há erro de grafia, pois pode-se
(Clarice Lispector) grafar o verbo sublinhado como estrovam ou estorvam.
a) O narrador subordina a si a personagem, retirando-lhe a
forma própria e afetiva de expressão. 19 – “A vida celibata podia ter certas vantagens próprias, mas
b) Percebe-se que a passagem do que relata o narrador para seriam tênues, e compradas a troco da solidão.”
o que se caracteriza como enunciado da personagem é Quanto à função sintática, a expressão grifada, no trecho acima,
sutil. classifica-se como
c) A fala da personagem é incorporada ao texto mediante a) adjunto adverbial. c) objeto indireto.
subordinação semântico-sintática entre a frase reproduzida b) adjunto adnominal. d)complemento nominal.
e a que a introduz.
d) Percebe-se que a fala da personagem - textualmente
reproduzida- é destacada por recursos gráficos.

15 – Assinale a alternativa em que todas as palavras se


classificam como paroxítonas.
a) meteorito - pegada - filatelia - fortuito
b) hangar - rubrica - filantropo - pudico
c) decano - ruim - maquinaria - avaro
d) pipoca - Gibraltar - erudito - substantivo

30
20 – “Que pode uma criatura Os advérbios e locuções adverbiais desse poema de Vinícius de
senão, entre criaturas, amar? Moraes são muito importantes para o entendimento da
Amar e esquecer, mensagem do texto. Isso porque
amar e malamar I - as locuções adverbiais marcam a trajetória do eu, utilizadas
amar, desamar, amar? para localização temporal e espacial. Já os advérbios
Sempre, e até de olhos vidrados, amar? estabelecem ruptura entre o eu-poético e o comportamento
(Carlos Drummond de Andrade) social dos outros.
II - o uso de quando como advérbio imprime à última estrofe
I - Os termos malamar e desamar, obedecendo ao processo de uma concepção de que a vida está e acontece no presente.
formação pelo uso de afixos, podem ser considerados III - os verbos utilizados na primeira estrofe perdem seu caráter
sinônimos, uma vez que os prefixos neles usados têm o mesmo original, transformando-se em advérbios que revelam oposição
sentido. de idéias.
II - malamar e desamar possuem parte do sentido semelhante,
uma vez que a parte restante, excluindo-se os prefixos, tem o Está correto o que se afirma em
mesmo radical: amar.
III - no termo malamar, observa-se na verdade o fenômeno da a) I apenas. c) I e II apenas.
parassíntese, por causa do prefixo mal e dos morfemas ar. b) III apenas. d) I, II e III.
IV - observando-se a ortografia, malamar deveria ser mal-amar;
ocorre que, se assim fizesse o autor, na criação do termo, 24 – Considere o texto:
passaria um conceito errado da formação da palavra, por causa “E as coisas são mais lindas
do hífen, ligado à justaposição. Quando você está
Hoje você está
Como incorreta(s) temos a(s) sentença(s) Onde você está
a) III apenas. As coisas são mais lindas
b) II e III apenas. Porque você está (...)
c) I, II e III apenas. Nas coisas tão mais lindas.” (Nando Reis)
d) I, II, III e IV.
Complete os parênteses com V (verdadeiro) ou F (falso) para
21 – Observe, abaixo, alguns verbetes criados por Millôr assinalar a alternativa com a seqüência correta:
Fernandes, humorista, jornalista e dramaturgo.
“Dicionovário” ( ) o advérbio mais modifica o adjetivo linda.
•Abacatimento: redução no preço do abacate. ( ) o advérbio tão modifica o advérbio mais.
•Anãofabeto: pequenininho que nem sabe assinar o nome. ( ) porque é uma conjunção coordenativa, pois explica a razão
•Cãodução: carrocinha de cachorro. de “as coisas serem mais lindas”.
•Cartomente: uma adivinha que nunca diz a verdade. ( ) você é um pronome pessoal reto usado quando há
intimidade no tratamento.
Quanto à formação das palavras por ele criadas, pode-se dizer ( ) além de mais e tão há mais dois advérbios no texto.
que
a) todas obedecem perfeitamente ao processo de a) V - F - V - F - F c) V - V - F - F - F
composição por justaposição, o que as torna legítimas do b) F - V - F - F - V d) F - V - V - V - V
ponto de vista gramatical.
b) o efeito irônico é obtido pela transformação de seus 25 – Assinale a alternativa que contém a justificativa incorreta
prefixos e sufixos. quanto à pontuação.
c) se trata de neologismos que sofreram o processo de
derivação imprópria, pois houve uma mudança da classe a) “A mocinha do caixa, tão loirinha, tão branquinha, tão
gramatical das palavras primitivas. magrinha, era uma fera.”(texto adaptado) – as vírgulas
d) se trata de neologismos criados pelo autor, que resultaram foram usadas para separar elementos de valor explicativo.
do acoplamento de duas palavras, sendo que uma das b) “A Genilda? Bom... não sei... acho que ela nem chegou
duas está truncada, a fim de produzir significados ainda...” – as reticências foram usadas para indicar que
especiais. houve dúvida, hesitação ou surpresa.
c) “Dona Diva era muito piedosa (que Deus a tenha!), mas
22 – Assinale a alternativa que apresenta a seqüência correta muito chata.” – os parênteses foram usados por causa da
quanto às conjunções coordenativas que preenchem oração intercalada no texto.
adequadamente o texto abaixo, dando-lhe coerência. d) “O dono da loja seria o chefe do comércio, e o prestígio do
Yes, nós temos cinema! vendedor era grande em Taitara.” – a vírgula foi usada
Todo mundo fala de um renascimento do cinema brasileiro antes do e para separar termos da mesma função sintática.
______ ele parece incontestável. ______ ainda falta vencer um
obstáculo fundamental: o preconceito do espectador brasileiro
que continua relutante em sair de casa para assistir a um filme
nacional. _____ os filmes têm dificuldade de conseguir muitas
salas _____ não conseguem o sucesso que mereciam.
(Rubens Ewald Filho)
a) e - assim - contudo - por isso
b) logo - pois - portanto - todavia
c) e - mas - por isso - e
d) pois - contudo - entretanto - mas

23 – Leia o texto abaixo:


Poética
De manhã escureço Outros que contem
De dia tardo Passo por passo:
De tarde anoiteço Eu morro ontem.
De noite ardo.

A oeste a morte Nasço amanhã


Contra quem vivo Ando onde há espaço
Do sul cativo — Meu tempo é quando.
o este é meu norte.

31
26 – A um secretário de escola foi dada a ordem de redigir um c) reflete sobre a qualidade daquilo que sente; por isso
ofício para o prefeito da cidade com o seguinte conteúdo: buscar criar adjetivos.
I - finalidade: pedido de conserto da parte hidráulica do prédio. d) tenta criar nomes que qualifiquem a essência de
II - concessão: impedimento da regularidade das aulas. sensações de que é acometida; por isso busca criar
III - tempo: o mais rápido possível. substantivos.
IV - acordo: o previsto em reunião anterior.
Utilizando as conjunções e locuções conjuntivas subordinativas, 30 – Leia atentamente o texto abaixo e, em seguida, marque a
o corpo do texto corretamente produzido deverá ser: alternativa que contém a informação incorreta quanto ao em-
prego dos verbos sublinhados no texto.
a) Conforme o combinado em assembléia de junho/2000,
solicitamos de V.Exª a gentileza de enviar funcionário a O garoto da vizinha me pediu que o ajudasse a fazer (a fazer,
esta escola, em caráter de urgência, para que seja feito o não, a completar) um trabalho escolar sobre a boca. Estava
conserto da parte hidráulica do prédio. Embora os reparos preocupado porque só conseguira escrever isto: “Pra que serve
impeçam o andamento das aulas, não podemos mais adiar a boca?”
a solução desse problema e, por isso, pedimos sua
colaboração. a) O fato de o menino ter conseguido escrever apenas “Pra
b) Mesmo que não seja possível, solicitamos de V. Exª a que serve a boca?” precede o fato de ele pedir ajuda para
gentileza de enviar funcionário a esta escola a fim de que o seu trabalho, o que justifica o seu pedido de auxílio.
seja feito o conserto da parte hidráulica do prédio. b) A atitude do menino de pedir ajuda refere-se a uma ação
Contanto que os reparos impeçam o andamento das aulas, passada não-pontual, cujo início e término não são
não podemos mais adiar a solução desse problema e, por definidos, já que não há referências temporais.
isso, pedimos sua colaboração já que foi combinado em c) A preocupação em que o menino se encontrava refere-se a
assembléia de junho/2000. um estado passado contínuo e, portanto, concomitante às
c) Se for possível, solicitamos de V. Exª a gentileza de enviar outras ações passadas expressas no texto.
funcionário a esta escola à medida que a parte hidráulica d) A ação de ajudar o garoto a fazer o trabalho escolar está
do prédio seja consertada. Para que os reparos impeçam o expressa em caráter eventual, em dependência estreita
andamento das aulas, não podemos mais adiar a solução com a vontade daquele que a emprega, ou seja, do garoto.
desse problema e, por isso, pedimos sua colaboração,
apesar do que foi combinado em assembléia de 31 – “Como se sabe, a teoria da relatividade geral afirma que
junho/2000. nem o tempo nem a distância são valores absolutos,
d) Visto que é possível, solicitamos de V. Exª a gentileza de dependendo do movimento relativo dos observadores, e que o
enviar funcionário a esta escola, posto que seja feito o único valor absoluto e constante é a velocidade da luz.” (Isaac
conserto da parte hidráulica do prédio. Conforme os Asimov)
reparos impeçam o andamento das aulas, não podemos Indique a alternativa que classifica corretamente a função
mais adiar a solução desse problema e, por isso, pedimos sintática e a classe morfológica dos termos grifados.
sua colaboração, porquanto tenha sido combinado em
assembléia de junho/2000. a) objeto indireto – substantivo
b) sujeito – substantivo
27 – “Aquela que te humilhou, aqui a tens abatida, no mesmo c) sujeito – adjetivo
lugar onde ultrajou-te, nas iras de sua paixão. Aqui a tens d) objeto direto – adjetivo
implorando teu perdão e feliz porque te adora, como o senhor
de sua alma.” (José de Alencar – Senhora) 32 – “A Eternidade está longe
(Menos longe que o estirão
O texto acima, do romance Senhora, de José de Alencar, é uma Que existe entre o meu desejo
passagem em que Aurélia implora o perdão de Seixas, seu E a palma da minha mão)”
marido. Percebe-se que ela fala de si própria como se estivesse (Manuel Bandeira)
falando de outra pessoa. Baseando-se nessas informações, leia No texto acima, temos
os comentários abaixo, referentes às pessoas do discurso, e
assinale a alternativa incorreta. a) uma oração absoluta, uma oração principal e uma oração
adjetiva.
a) “Sua paixão” refere-se à paixão que Aurélia sentia. b) uma oração absoluta, duas orações principais, uma oração
b) Em “aqui a tens abatida”, o pronome refere-se à mulher comparativa e uma oração adjetiva.
que humilhou o marido. c) duas orações principais, uma oração comparativa e uma
c) “Senhor de sua alma” refere-se à alma de Seixas, pois ele oração adjetiva.
era senhor de si próprio. d) uma oração absoluta, uma oração principal, uma oração
d) Em “ultrajou-te”, o pronome refere-se a Seixas, pois ele é comparativa e uma oração adjetiva.
que foi ultrajado.
33 – Assinale a alternativa em que a pontuação não obedece à
28 – Observando a regência dos verbos em destaque no trecho norma culta.
a seguir, assinale a alternativa que preenche corretamente as a) Melhores, porém, que os remédios são as bulas que os
lacunas: acompanham. É possível que as bulas desapareçam — o
"Excesso de leis atrapalha _____ vida do País e impede _____ que pode ser ruim para os pacientes. (adaptação do texto
população _____ saber _____ que as normas deve obedecer." original)
b) “Há estudantes tímidos, temerosos, que não encontram
Obs.: Ø significa que não há preposição ou artigo. alegria ou satisfação no trabalho, que crêem não serem
a) a - a - de - a c) a - à - Ø - Ø aprovados, que se consideram de pouco valor.”
b) à - a - para - à d) Ø - à - Ø - sobre c) A violência é um dos temas preferidos pela televisão, e
muitos acreditam que a apresentação de tanta violência,
29 – Leia o texto: influi negativamente, sobre o comportamento do
“Se recebo um presente dado com carinho por pessoa de quem telespectador.
não gosto — como se chama o que sinto? Uma pessoa de d) O rádio e a tevê independem do nível de alfabetização do
quem não se gosta mais e que não gosta mais da gente — público; atingem, porém, a maior parcela da população que
como se chama essa mágoa e esse rancor?” (Clarice Lispector) a imprensa. (adaptação do texto original)

Com relação ao texto, é correto afirmar que a autora


a) tenta nomear o sentir, por isso podemos dizer que ela tenta
criar verbos.
b) preocupa-se com nomes de sentimentos relativos a
circuns-tâncias específicas; daí dizermos que tenta criar
advérbios.

32
34 – No período “O robô Sojourner envia mais de cinco mil 40 – Assinale a alternativa em que não se deve colocar nenhum
fotos, comprovando que já existiu água no Planeta Vermelho.”, o acento grave (crase).
trecho grifado corresponde, sem prejuízo para o sentido original,
ao trecho: a) A chegada do professor, os alunos levantaram-se
a) ... quando comprova que já existiu água no Planeta respeitosamente.
Vermelho. b) Esta questão de crase é semelhante a que vimos ontem.
b) ... que comprovam que já existiu água no Planeta c) “E seria bonita a pedra? Nada, uma pedra a-toa, de nome
Vermelho. geral, porém...”
c) ... visto que comprova que já existiu água no Planeta d) “... não gosta da cor do homem, a qual se mistura a todos
Vermelho. os verdes naturais (...) das folhas...”
d) ... o que comprova que já existiu água no Planeta
Vermelho. 2/2003-TURMA A
35 – Quantas orações há nos versos abaixo?
Os homens são diferentes
"Como ama o homem adulto o adultério
Sou um arqueólogo e o Homem é meu campo de
E o ébrio a garrafa tóxica de rum,
estudo. Entretanto, cogito se alguma vez chegaremos a
Amo o coveiro - este ladrão comum
conhecer o Homem – isto é, o que realmente o torna diferente
Que arrasta a gente para o cemitério.
de nós, Robôs – através das escavações nos planetas mortos.
(Augusto dos Anjos)
Por exemplo, uma vez eu conheci um Homem, e as coisas não
a) 2 c) 4
são tão simples quanto nos contam na escola.
b) 3 d) 5
Nós temos poucos registros, naturalmente, e Robôs
como eu estão tentando preencher as lacunas, mas penso que
36 – Observe:
não estamos chegando a nada de concreto. Nós sabemos, ou
I - Outeiro das Brisas é um lugar exótico, onde o sol, o sossego
pelo menos os historiadores o dizem, que os Homens são
e a sensualidade baianas se misturam.
originários de um planeta chamado Terra. Sabemos ainda que
II - “Vossa Excelência foi injusto em sua sentença, Doutor Pro-
eles viajaram corajosamente de estrela para estrela e em todos
motor.”
os lugares onde pararam deixaram colônias – Homens ou
III - Confiança, na dose certa, é sempre bom para conservar as
Robôs, e às vezes ambos – aguardando sua volta. Mas nunca
amizades.
voltaram.
Aqueles foram os dias de glória do mundo. Teremos
Assinale a alternativa que contém a(s) frase(s) correta(s) quanto
nós envelhecido? O Homem tinha uma centelha ardente – o
à concordância nominal.
termo antigo é "divina", penso – que o impelia através da grande
noite dos céus, e nós não mais nos ligamos à grande teia que
a) I apenas.
eles teceram.
b) II e III apenas.
Nossos cientistas contam-nos que o Homem é muito
c) I e III apenas.
semelhante a nós – o esqueleto, por exemplo, é praticamente o
d) II apenas.
mesmo que o de um Robô, excetuando-se o fato de que é
constituído de compostos de cálcio, em vez de titânio. Fala-se
37 – Nas frases abaixo,
eruditamente de uma "pressão de população" como uma "força
I - O político nomeou seu filho secretário de finanças.
impulsionando em direção às estrelas". Sem dúvida, há outras
II - Mário, sua prova está exemplar!
diferenças.
III - Valdirene ficou em Brasília, nas férias do ano passado.
Foi em minha última pesquisa de campo, em um dos
planetas inferiores, que encontrei um Homem, que deve ter sido
Os predicados classificam-se, respectivamente, como
o último Homem neste sistema, e tinha estado sozinho por tanto
a) verbo-nominal, verbal e nominal.
tempo que nem mais sabia falar. Depois que aprendeu nossa
b) verbo-nominal, nominal e verbal.
língua, demo-nos muito bem, e eu planejava até trazê-lo de
c) nominal, verbo-nominal e verbal.
volta comigo. Entretanto, alguma coisa lhe sucedeu.
d) verbal, nominal e verbo-nominal.
Um dia, sem razão alguma, começou a queixar-se do
calor. Verifiquei sua temperatura e concluí que seus circuitos
38 – “Destes penhascos fez a natureza
termostáticos se tinham queimado. Eu tinha um jogo
O berço em que nasci: oh! quem cuidara
sobressalente comigo, e, como o dele obviamente não estava
Que entre penhas tão duras se criara
funcionando bem, lancei-me ao trabalho. Desliguei-o sem
Uma alma terna, um peito sem dureza!”
problema algum. Enterrei a agulha no seu pescoço, para
(C.M.Costa)
desligar o interruptor, e ele deixou de funcionar, como qualquer
Quanto à função sintática dos termos abaixo, coloque, nos
Robô. Mas quando eu o abri, por dentro era diferente. E, quando
parênteses, F (falso) ou V (verdadeiro) para as afirmativas e
o montei de novo, não consegui fazê-lo funcionar. Depois disso
assinale a alternativa com a seqüência correta.
não sei explicar o que aconteceu. O fato é que ele se foi
( ) a natureza – objeto direto
dissolvendo, e, na época em que eu estava pronto para voltar,
( ) o berço – sujeito
mais ou menos um ano depois, apenas os ossos tinham
( ) entre penhas tão duras – adjunto adverbial
sobrado. Sem dúvida alguma, o Homem deve ser diferente.
( ) sem dureza – adjunto adnominal
(Alan Bloch)
a) V - F - V - F
b) V - V - F - F
As questões de 01 a 05 referem-se ao texto acima.
c) F - V - F - V
d) F - F - V - V
01 – Ao longo do texto, são fornecidas algumas informações sobre o
Homem, sob o ponto de vista de um observador que não é humano.
39 – Assinale a alternativa cuja regência nominal não está de
acordo com a norma culta.
Assinale a alternativa em que esse ponto de vista está evidente.
a) Seus olhos andavam saturados de ódio das pessoas que o
a) "(...) os Homens são originários de um planeta chamado Terra."
atormentavam.
b) "Desliguei-o sem problema algum. Enterrei a agulha no seu
b) Era maravilhoso ficar ali, olhinhos brilhando, boca aberta,
pescoço, para desligar o interruptor, e ele deixou de
prestando atenção às estórias que vovô contava.
funcionar, como qualquer Robô."
c) A mãe andava atarefada em preparar os festejos de fim de
c) "Sou um arqueólogo e o Homem é meu campo de estudo."
ano. Era preciso entendê-la.
d) "(...) eles viajaram corajosamente de estrela para estrela e
d) Ela andava a lançar olhares lânguidos aos rapazes; fazia-o
em todos os lugares onde pararam deixaram colônias (...)."
de modo propositalmente ofensivo para a época.

33
02 – De acordo com o texto, é incorreto afirmar que 10 – Observe o texto:
a) há muito ainda para se saber sobre o Homem. – Qué apanhá, sordado?
b) o Homem é uma espécie em extinção; daí justificar-se o – Quê?
fato de ser estudado por um arqueólogo. – Qué apanhá?
c) o Homem é totalmente diferente do Robô, sendo, por essa Pernas e cabeça na calçada.
razão, estudado por este.
d) o Homem desperta o interesse dos Robôs, que tentam Com relação ao texto acima, é correto afirmar que
conhecê-lo melhor. a) se trata de uma descrição, pois as ações encontram-se no
presente, não indicando, portanto, nenhuma transformação de
03 – Procedendo a uma leitura profunda do texto, admite-se a estado.
seguinte interpretação: b) o trecho é notadamente dissertativo, pois os fatos
a) Homens e Robôs são seres naturalmente diferentes, por encadeiam-se numa relação de causa e efeito.
isso o convívio entre eles é impossível. c) os fatos estão organizados numa disposição tal que entre
b) O Robô, embora aparentemente vitorioso, continua sujeito eles existe uma relação de anterioridade e de
à sua condição de máquina, o que significa que ele ainda posterioridade, o que caracteriza um texto narrativo.
possui limitações que o impedem de conhecer o Homem d) não é possível classificar esse texto, visto que ele não
verdadeiramente. apresenta elementos suficientes para tal.
c) Em um futuro próximo, a raça humana será totalmente
extinta da face da Terra. 11 – Assinale a alternativa cuja frase se relaciona com o defeito
d) O domínio dos Robôs é uma ficção, portanto não é de estilo apresentado.
possível se depreender um tema. a) Vicente já não sente dores de dente como antigamente.
(cacofonia)
04 – No trecho "Fala-se eruditamente de uma 'pressão de b) Preciso de um rapaz para caçar pássaros e uma garota
população' como uma 'força impulsionando em direção às menor. (cacofonia)
estrelas'", considerando a idéia expressa no 3º parágrafo, o c) As crianças parecem que ficaram envergonhadas. (barbarismo)
Robô poderia estar referindo-se d) Ama o filho a boa mãe. (solecismo)
a) à superpopulação na Terra, fato que obrigou o Homem a
abandonar seu planeta. 12 – Todas as alternativas apresentam conotação, exceto:
b) ao artifício elaborado pelo Homem, o foguete, responsável a) "Mostra-lhe o tambor
por transportá-lo ao espaço sideral. de salitre e brisa
c) a uma necessidade própria do ser humano de querer que rufa sozinho
conhecer o espaço fora de seu planeta. entre os arquipélagos
d) a um fenômeno natural que injetou o Homem para o espaço de sua pobreza." (Ivo Lêdo)
intergaláctico. b) "Bendito sejas tu, a quem certo, devemos
a grandeza real de tudo quanto temos!
05 – O Homem com o qual o Robô manteve um contato morreu Sonha em paz! Sê feliz! E eu que fique de joelhos"
a) porque a temperatura do planeta sofreu uma elevação (Ciro Costa)
insuportável para a sobrevivência humana. c) "Ave Maria – lento o bronze soa
b) devido aos maus-tratos do Robô, insensível à dor e ao Com voz que ecoa na longínqua serra"
sofrimento humanos. d) "No tronco mais verde,
c) em razão do desconhecimento do Robô a respeito da Que no prado houvesse,
fisiologia humana. Amor me mandou
d) pelo simples fato de que os Homens não são imortais como os Robôs. Seu nome escrevesse" (Tomás A. Gonzaga)

06 – Assinale a alternativa cujas palavras se classificam como 13 – Assinale a alternativa incorreta quanto ao tipo de discurso.
ditongos decrescentes. a) – Por que foges de mim? Perguntou o amante. (direto)
a) aquarela - imundície - Páscoa b) Ele disse que as coisas iam bem. (indireto)
b) tireóide - petróleo - aquoso c) – Senhora, resolvi a situação pendente. (indireto)
c) canibais - guache - Romeu d) Carlos, durante a festa, perguntou se os convidados vieram.
d) carrosséis - casarões - revestiu (indireto)

07 – O período "Aquela sublime paisagem era minha querida 14 – Complete as lacunas das frases abaixo e assinale a
amiga nas horas de tristeza e saudade." apresenta as seguintes alternativa correta.
palavras que contêm encontro consonantal: Ele _____ (ter) muitos brinquedos que ____ (ter) força própria.
a) paisagem - minha c) saudade - solidão Depois da tempestade, ________ (vir) a bonança.
b) aquela - querida d) sublime - tristeza Quando as idéias não ____ (vir), elas não____ (ver) solução.

08 – "Levamos-te cansado ao teu último endereço a) têm - tem - vêem - vem - vem
Vi com prazer b) tem - tem - vêm - vem - vêem
Que um dia afinal seremos vizinhos c) têm - têm - vem - vêem - vem
Conversaremos longamente d) tem - têm - vem - vêm - vêem
De sepultura a sepultura
No silêncio das madrugadas." (Manuel Bandeira) 15 – Qual dos ditados abaixo apresenta o maior número de
Quanto às figuras de linguagem, podemos afirmar que, no sílabas?
primeiro verso do texto acima, há a) Gato escaldado tem medo de água.
a) hipérbole. c) eufemismo. b) Em casa de ferreiro, o espeto é de pau.
b) onomatopéia. d) catacrese. c) O uso do cachimbo faz a boca torta.
d) Águas passadas não movem moinho.
09 – Na forma verbal desconhece (presente do indicativo):
a) a vogal temática e aparece em todas as pessoas 16 – Assinale a alternativa em que todas as palavras se
gramaticais. classificam como paroxítonas.
b) a desinência número-pessoal da primeira pessoa é idêntica (Observe que o acento gráfico foi retirado propositadamente de
à da terceira. algumas delas.)
c) a desinência número-pessoal da segunda pessoa do plural é a) filantropo - caminhante - substantivo
s. b) arquetipo - canavial - tupi
d) a desinência modo-temporal é ausente em todas as c) omega - servo - soldado
pessoas gramaticais. d) refem - satisfeito - patativa

34
17 – Assinale a alternativa cujas palavras estão graficamente 24 – Coloque V (para Verdadeiro), F (para Falso) e assinale a
corretas. alternativa que contém a seqüência com a exata
a) flecha, xavantes, cuscuz, chinelo, umidecer. correspondência entre adjetivo e locução adjetiva.
b) suavizar, catequizar, canalizar, analisar, pesquisar.
c) flexa, xavantes, cuscus, chinelo, umedecer. ( ) O corpo discente da escola fez muitas reivindicações ao
d) suavisar, catequisar, canalisar, analisar, pesquisar. diretor. (de professores)
( ) Teve fortes dores cervicais ontem. (de pescoço)
18 – Complete o período seguinte com o pronome pessoal ( ) Infelizmente ele nasceu com problema renal. (de fígado)
conveniente. ( ) Suas observações pueris muito me decepcionaram. (de velho)
"Juliana passeava com seu colega de classe; de repente,_____ a) V-V-F-F c) F-V-F-F
______ beija, deixando- _______ encabulada." b) F-F-V-V d) V-F-V-V

A seguir, assinale a alternativa correta. 25 – Segundo a norma culta, assinale a alternativa cuja colo-
cação do pronome está incorreta.
a) o - ela - a b) ele - a - a c) ele - a - ela d) o - a - a a) Os rapazes que me perguntaram teu nome são meus vizinhos.
b) Disseram-me que você é o melhor aluno da turma.
19 – Texto I- "Vaca exausta em plena lua-de-mel (1) c) O mendigo se queixava do frio.
procura história pra boi dormir." d) Tenho dedicado-me à música ultimamente.
Texto II- "Vende-se teleobjetiva (2) para fotografar (3)
primos distantes." 26 – A classificação das conjunções adverbiais está incorreta na opção

Observe as palavras destacadas e numeradas nos textos acima a) Como não choveu, a represa secou. (causal)
e assinale a alternativa que contém a afirmativa correta quanto b) Ela sabe tanto quanto eu. (comparativa)
ao processo de formação das palavras. c) Gesticulava para que todos vissem. (final)
a) 1 - Composição por justaposição d) Não haverá aula hoje, segundo anunciou o diretor. (condicional)
2 - Hibridismo
3 - Palavra composta por dois radicais gregos 27 – Assinale a alternativa em que há erro de pontuação.
b) 1 - Composição por aglutinação
2 - Palavra composta por dois radicais gregos a) Dorival, meu filho está acamado.
3 - Palavra composta por dois radicais latinos b) Dorival, meu filho, está acamado.
c) 1 - Derivação parassintética c) Meu filho Dorival está acamado.
2 - Palavra composta por dois radicais latinos d) Meu filho Dorival, está acamado.
3 - Hibridismo
d) 1 - Derivação imprópria 28 – Assinale a alternativa que classifica, correta e respectiva-
2 - Hibridismo mente, os termos grifados no período "A manutenção das
3 - Derivação sufixal estradas é fundamental para nossa segurança".

20 – Observe: a) complemento nominal - complemento nominal


I- Todos os abaixo-assinados concordam com os textos b) complemento nominal - adjunto adnominal
que lerei nas próximas quartas-feiras. c) adjunto adnominal - complemento nominal
II- Os garotos comeram todos os pés-de-moleques. d) adjunto adnominal - adjunto adnominal
III- A intervenção do apresentador pôs fim a todos os bate-
bocas. 29 – Assinale a alternativa cuja regência contraria a norma
IV- Não acredito nas mulas-sem-cabeças. culta da língua.
a) Renunciou ao cargo de diretor.
Quanto à flexão dos substantivos compostos, é correto afirmar b) Assistia ao desfile todos os anos.
que estão corretos somente c) Teria de ir à casa daquele corrupto!
d) Prefiro muito mais cerveja do que vinho.
a) II e IV. b) I e III. c) III e IV. d) Ie
II. 30 – "O mal dos que estudam as superstições é não
acreditarem nelas." (Mário Quintana)
21 – Escolha a opção em que todos os verbos são defectivos.
a) trovejar - miar - abolir c) magoar - nevar - rir Em relação à principal, a oração destacada é subordinada
b) falir - entregar - chover d) latir - reaver - pedir a) adjetiva restritiva reduzida de particípio.
b) adjetiva explicativa reduzida de particípio.
22 – "Era uma expressão fria, pausada, inflexível, que jaspeava c) substantiva subjetiva reduzida de infinitivo.
sua beleza, dando-lhe quase a gelidez da estátua." d) substantiva predicativa reduzida de infinitivo.
No trecho acima, classificam-se as palavras grifadas,
respectivamente, como 31 – No período "Convém estudarmos nossa língua", a oração
a) conjunção integrante, pronome tônico, preposição, adjetivo. grifada está corretamente desenvolvida e classificada na
b) pronome relativo, pronome reto, conjunção coordenativa, substantivo. seguinte alternativa:
c) pronome relativo, pronome átono, advérbio, substantivo.
d) conjunção subordinativa, pronome tônico, adjetivo, advérbio. a) que estudemos nossa língua (substantiva subjetiva)
b) que nossa língua seja estudada (adjetiva restritiva)
23 – "Meu dia outrora principiava alegre, c) que estudem nossa língua (substantiva predicativa)
No entanto à noite eu chorava. Hoje d) que estudassem nossa língua (substantiva objetiva direta)
mais velho,
Nascem-me em dúvida os dias, mas 32 – Observe as frases seguintes:
Findam sagrados, serenamente." (M. Bandeira)
I- Nem acreditei quando começou as férias.
Classifique, respectivamente, as palavras grifadas no texto II- Um bilhão de reais foi gasto na construção daquele edifício.
acima e assinale a alternativa correta. III- Como chovem promessas durante as campanhas eleitorais!
a) substantivo, advérbio, adjetivo, advérbio IV- Os estudantes universitários pareceram gostarem da palestra.
b) advérbio, substantivo, substantivo, adjetivo
c) substantivo, adjetivo, adjetivo, advérbio Segunda a norma culta, a concordância verbal está correta nas
d) adjetivo, advérbio, substantivo, adjetivo frases

a) I e II. b) II e III. c) III e IV. d) I e IV.

35
33 – Assinale a alternativa em que a concordância nominal está V
incorreta. A mulher que escolhemos, a única e não outra
a) A rua, meio deserta, despertava certo pavor na jovem Dentre tantas que habitam a terra triste,
estudante. Esta mesma, frágil e indefesa, bela ou feia,
b) Elas mesmas enviaram os documentos anexo às Eis o mundo que nos é de novo apresentado
declarações. Por intermédio de uma só pessoa.
c) Era preciso muita paciência com aqueles moleques. Esta é a que rompe as grades do nosso coração,
d) Finalmente estou quite com o Banco. Esta é a que possuímos mais pela ternura que pelo sexo.
E nada será restaurado no seu genuíno sentido
34 – Preencha as lacunas adequadamente e assinale a Se a mulher não retornar ao seu princípio:
alternativa correta. É a máquina instalada dentro dela que deveremos
vencer.
A pobre vítima era querida _________ todos, leal ________ os Quando essa mulher se tornar de novo submissa e doce,
amigos e útil __________ a empresa em que trabalhava. Os homens pela mão da eterna mediadora
a) de - com - para c) de - para com - entre Abrirão outra vez um ao outro os corações que sangram.
b) por - para - com d) por - com - sobre ( Murilo Mendes)

35 – Observe: As questões de 01 a 05 referem-se ao texto acima.


I- Se você for aquele quiosque, traga-me mais um
acarajé. 01 – Segundo o texto, para que o homem possa reencontrar-se
II- Não fale tal assunto a determinadas pessoas. e doar-se deve, como atitude primeira,
III- Eles estão cara a cara.
IV- Não falo aquilo que me contaste. a) procurar alguém que possa apresentá-lo novamente a si
mesmo.
Quanto ao acento grave, indicador de crase, deduzimos que ele b) "viver, agonizar e ressuscitar" com o outro.
deve ocorrer em c) sobrepujar a mulher, a fim de, obtida sua auto-afirmação,
a) apenas III. b) I e III. abrir-se ao outro.
c) II e IV. d) apenas I. d) centrar-se na mulher, pois, através dela, está a possibilidade
de se estabelecer comunicação efetiva.
2/2003-TURMA B
O rato e a comunidade 02 – Em relação ao título "O rato e a comunidade", é correto
I afirmar que
O rato apareceu a) o rato, como roedor que é, simboliza o desgaste das relações, aquilo
Num ângulo da sala, que corrói e destrói a comunidade, a sintonia de corações.
Um homem e uma mulher b) pouco expressa realmente do conteúdo do texto; o rato é,
Apareceram também, na verdade, um elemento de estranhamento, a fim de
Trocaram palavras comigo, mostrar que os relacionamentos não se aprofundam.
Fizeram diversos gestos c) trata-se o rato de um elemento de oposição à comunidade,
E depois foram-se embora. que simboliza a experiência do amor, a solidariedade de que
? Que sabe esse rato de mim. fala o poeta.
E esse homem e essa mulher d) o poeta utiliza a figura do rato para simplesmente introduzir aquilo
Sabem pouco mais que o rato. que realmente deseja: tratar da indiferença humana.
II
Passam meses e anos perto de nós, 03 – Em relação à mulher, o texto
Rodeiam-nos, sentam-se com a gente à mesa, a) busca o essencialmente feminino sem, no entanto, deixar de
Comentam a guerra, os telegramas, lado um juízo de valor.
Discutem planos políticos e econômicos, b) assume clara postura de exaltação e força (máquina) que é
Promovem arbitrariamente a felicidade coletiva. capaz de conduzir o homem adiante.
Conhecem nosso paletó, camisa e gravata, c) coloca-a apenas como uma prestadora de serviços, uma
Nosso sorriso e o gesto de mover o copo. intercessora; por isso, submissa e inferior.
Têm medo de nos tocar, não conhecem nossas lágrimas. d) mostra-a como um ser para se amansar, uma vez que,
? Que sabem do nosso coração, do nosso desespero, como intermediária, pode agir em benefício próprio.
da nossa comunicabilidade.
Que sabem do centro da nossa pessoa, de que são 04 – O texto apresenta momentos específicos de interpelação,
participantes. de demonstração de superficialidade no relacionamento humano
Subúrbios longínquos, esses homens. e de necessidade de doação. Esses momentos encontram-se,
III respectivamente, nas seguintes estrofes:
Entretanto cada um deve beber no coração do outro. a) V, II e III. c) IV, II e III.
Todos somos amassados, triturados: b) IV, I e V. d) I, II, III, IV e V.
O outro deve nos ajudar a reconstruir nossa forma.
O homem que não viu seu amigo chorar 05 – Leviatã, segundo o Dicionário Aurélio, significa "monstro do
Ainda não chegou ao centro da experiência do amor. caos, na mitologia fenícia, identificado, na Bíblia, como um
Para o amigo não existe nenhum sofrimento abstrato. animal aquático ou réptil". Assim, a expressão massa-leviatã,
Todo o sofrimento é pressentido, trocado, comunicado. em relação ao trecho em que se encontra, exprime que
? Quem sabe conviver com o outro, quem sabe transferir o
coração. I- sociedade moderna não está estruturada para uma vida
Viver com o outro é agonizar, morrer é ressuscitar com. humanizada.
Ninguém mais sabe tocar na chaga aberta: II - sociedade moderna arrasta o homem para o automatismo,
Entretanto todos têm uma chaga aberta. para a ausência do tempo, do outro.
IV III - sociedade luta contra um monstro invisível que se faz
Desconhecido que atravessas a rua, presente no dia-a-dia e que despersonaliza os indivíduos.
? Que tens de comum comigo.
A mesma solidão e a mesma roupa. Está correto o que se afirma em
Procuras consolo, mas não podes parar. a) III apenas. c) I e III apenas.
És o servo da máquina e do tempo. b) I e II apenas. d) I, II e III.
Mal sabes teu nome, nem o que desejas neste mundo.
Procuras a comunidade de uma pessoa,
Mas não a encontras na massa-leviatã.
Procuras alguém que seja obscuro e mínimo,
Que possa de novo te apresentar a ti mesmo.

36
06 – Identifique a figura de linguagem presente em todos os Os itens que apresentam asserções corretas no que se refere à
textos abaixo. classificação sintática dos termos da estrofe acima são
I - "Desculpem-me por ter sido longo porque não tive tempo
de ser breve." a) I e IV apenas. c) II e III apenas.
II - "Mais servira se não fora b) I e III apenas. d) I, II e IV apenas.
Para tão longo amor tão curta a vida."
III - "Foi então que, em face destas duas tristezas – a noite que 13 – Em relação às formas verbais destacadas em "Um
descia dos céus, – a solidão que subia do oceano – recordei-me parlamentar diz que se o governo não ocupar espaços e obter
de vós, os meus amigos." sinais positivos dos vários setores da sociedade, ficará muito
IV - "Não há no mundo alegria sem sobressalto, concórdia sem difícil pensar na reeleição.", considera-se que
dissensão, descanso sem trabalho, riqueza sem miséria, digni-
dade sem perigo, finalmente, não há gosto sem desgosto." a) estão empregadas corretamente; nos verbos regulares
a) antítese c) prosopopéia sempre a forma verbal da 3ª pessoa do singular do futuro do
b) metáfora d) hipérbole subjuntivo coincide com o infinitivo.
b) estão empregadas incorretamente; nos verbos irregulares
07 – Assinale a única sentença que apresenta sinestesia. sempre a forma verbal da 3ª pessoa do singular do futuro do
a) "A noite estava muito negra. E havia sobre subjuntivo irá coincidir com o infinitivo.
a cidade um silêncio côncavo, de abóbada." (Eça de Queirós) c) o verbo regular ocupar antes do irregular obter induz o
b) "E a tua boca anda mentindo redator a flexionar incorretamente o segundo verbo por
Enganada pelos teus sentidos." (Cecília Meireles) analogia com o primeiro.
c) "Precisamos descobrir o Brasil! d) para o verbo irregular, no caso obter, há a possibilidade de
Escondido atrás das florestas, se flexionar a 3ª pessoa do futuro do subjuntivo coincidindo
com a água dos rios no meio." (Carlos Drummond de Andrade) ou não com o infinitivo; assim as duas formas verbais foram
d) "Sobre a nudez forte da verdade o manto diáfano usadas corretamente.
da fantasia / ou sobre a nudez forte da fantasia o manto
diáfano da verdade." (José Saramago) 14 – Em qual das alternativas o uso da vírgula é facultativo,
dependendo de ênfase ou não?
08 – ANULADA
a) O Governo de São Paulo tem investido na Educação. Portanto,
09 – No que se refere à acentuação gráfica, pode-se fazer as o nível do ensino deverá elevar-se.
seguintes considerações: b) Monteiro estudou muito para a prova final, e ficou reprovado.
a) Consiste na aplicação, apenas, de certos sinais escritos c) Os alunos fizeram todos os exercícios; não ficarão, pois, de
sobre algumas letras para representar o que foi estipulado quarentena.
pelas regras de acentuação, que são arbitrárias e d) "A mulher aceita o homem por amor ao casamento, e o
desvinculadas da natureza da língua. homem tolera o matrimônio por amor à mulher."
b) As regras de acentuação visam sistematizar a leitura das
palavras de nossa língua; assim sendo, baseiam-se na 15 – Leia com atenção as orações abaixo e assinale a
posição da sílaba tônica, no timbre da vogal, nos padrões alternativa correta, observando a regência verbal do verbo falar.
prosódicos menos comuns da língua. I - As histórias hão de falar no herói que dedicou sua vida ao bem.
c) As regras de acentuação foram criadas visando também às II - Foi isso o que o aluno falou com o professor.
palavras homônimas, tanto as homógrafas, quanto as III - O médico falou ao paciente do problema que lhe indicaram
homófonas; é o caso, por exemplo, de tem/têm e de os exames.
sabia/sabiá. Esses são os chamados acentos diferenciais.
d) Os fenômenos representados pela nasalização da vogal, do Está correto o que aparece em
fonema representado pela letra u, pelos hiatos e ditongos a) II apenas. c) I e II apenas.
abertos não se caracterizam como regra. b) I e III apenas. d) I, II e III.

10 – O verbo afinar aparece na voz passiva em: 16 – "Penetra surdamente no reino das palavras.
a) Resolveu afinar suas idéias pelas da maioria. Lá estão os poemas que esperam ser escritos.
b) Suas idéias foram afinadas pelas da maioria. Estão paralisados, mas há desespero,
c) Seu procedimento afina pelo do pai. há calma e frescura na superfície intacta
d) Afinaram-se as vozes, ao primeiro acorde do piano. Ei-los, sós e mudos, em estado de dicionário."

11 – Assinale a alternativa em que os pronomes oblíquos Os termos grifados no texto acima exercem, respectivamente, a
destacados nos pares de orações não tenham a mesma função função sintática de
sintática.
a) 1 – Se a vejo triste, consolo-a. a) sujeito e adjunto adnominal.
2 – Desejava reanimá-la a qualquer custo. b) objeto direto e predicativo do objeto.
b) 1 – Ensinei-lhe a amar os versos. c) sujeito e predicativo do objeto.
2 – Não lhe pagou com gratidão. d) objeto direto e predicativo do sujeito.
c) 1 – Dá-se ares de grande dama.
2 – Os dois amam-se profundamente. 17 – ANULADA
d) 1 – Fale-nos de seus grandes anseios.
2 – Suplicou-nos o perdão em sua hora derradeira. 18 – "Meu São Paulo da garoa
- Londres das neblinas finas -
12 – Observe: Um pobre vem vindo, é rico!
"O ouro fulvo do ocaso as velhas casas cobre; Só bem perto fica pobre,
Sangram, em laivos de ouro, as minas, que a ambição Passa e torna a ficar rico." (Mário de Andrade)
Na torturada entranha abriu da terra nobre: Observando-se os versos de 3 a 5, podemos dizer que as
E cada cicatriz brilha como um brasão." orações possuem, respectivamente, predicados
I - A expressão "da terra nobre" desempenha a função
sintática de objeto indireto. a) verbo-nominal; verbal; verbo-nominal.
II - São sujeitos simples os termos "O ouro fulvo do ocaso", "as b) verbo-nominal; nominal; verbal; verbal.
minas", "a ambição", "cada cicatriz" e "um brasão". c) verbal; nominal; verbal; verbal; nominal.
III - Em todo o poema, há dois objetos diretos. d) verbal; nominal; nominal; verbal; nominal.
IV - Os termos "em laivos de ouro" e "na torturada entranha"
são adjuntos adverbiais de lugar.

37
19 – Observe os termos grifados no texto abaixo:
"O cão é o melhor amigo do homem porque ladra para 24 – Assinale a alternativa que substitui correta/respectivamente
protegê-lo, vela seu sono, estima-o na riqueza e na miséria, as palavras destacadas.
indiferentemente, como, indiferentemente, ama-o moço ou I - Os homens públicos nunca esquecem a compostura política.
velho, segue-o pela vida e para a morte." (Millôr Fernandes) II - A médica procede à operação de emergência.
É correto afirmar que III - Entreguei o bilhete ao diretor.
IV - Por que você aspira a este cargo?
a) moço e velho, mesmo com função predicativa, ou seja,
qualificando, morfologicamente são classificados como a) a – a ela – lho – a ele c) a – lhe – lhes – a ele
substantivos. b) na – lhe – lho – lhe d) na – a ela – lho – a ele
b) todas as palavras destacadas são substantivos, embora não
tenham a mesma classificação. 25 – Assinale a alternativa em que, na produção textual, obser-
c) ladra é substantivo abstrato, derivado de ladrar e nomeia vou-se a correção gramatical enquanto qualidade de estilo.
uma ação.
d) miséria, riqueza, vida e morte são os chamados a) "No trânsito, para passar mensagens, o outdoor é imbatível (...)
substantivos abstratos; todos nomeiam estados, sendo que Além do quê, ele é o último apelo ao consumidor antes da
os dois primeiros também se caracterizam como qualidade. compra (...) Você pode utilizá-lo para cercar o Brasil inteiro..."
b) "Harrison Ford interpretou tão bem o papel do aventureiro
20 – "No ensino, como em outras coisas, a liberdade deve ser Indiana Jones que ficou complicado para o expectador
questão de grau. Há liberdades que não podem ser toleradas. separar o homem do ator."
Uma vez conheci uma senhora que afirmava não se dever c) "Ouve-se com bastante freqüência grupos de cidadãos que
proibir coisa alguma a uma criança, pois deve desenvolver sua exigem maior eficiência da polícia (...) como forma de ga-
natureza de dentro para fora." (Russel, Bertrand) rantir a segurança dos indivíduos e seus patrimônios."
d) "Há efetivamente um conjunto de brasileiros que se comportam
Quanto ao texto acima, podemos afirmar que possui estrutura como se as leis não lhes dissessem respeito. O convívio social
a) narrativa, uma vez que relata as mudanças progressivas de não passa de uma forma de lhes satisfazer os desejos, as
estado que vão ocorrendo com as pessoas e as coisas através obrigações (...) pertencem exclusivamente aos outros."
do tempo.
b) descritiva, pois relata as características de uma pessoa, de 26 – Aponte a alternativa em cujo texto aparece a figura de
um objeto ou de situação qualquer, inscritos num certo linguagem que consiste em se dizer o contrário do que se
momento estático do tempo. pensa, tal qual na frase "Comprou a mansão pela módica
c) dissertativa, porque nela predominam os conceitos quantia de 2 milhões de dólares".
abstratos, isto é, a referência ao mundo real se faz através a) "A pátria que quisera ter não era um mito; era um fantasma
de conceitos amplos, de exemplificação muitas vezes criado por ele no silêncio de seu gabinete. Nem a física,
abstraídos do tempo e espaço. nem a moral, nem a intelectual, nem a política que julgava
d) narrativa, porque nela a visão de mundo do enunciador é existir, havia."
transmitida através de uma série de ações que ele atribui a b) "Corriam no arraial rumores macabros. No dia seguinte ao
personagens e que implicitamente formam um ponto de vista. enterramento o coveiro topou a sepultura remexida, como
se fora violada durante a noite; e viu na terra fresca pegadas
21 – As propagandas, em sua maioria, utilizam-se de linguagem misteriosas de uma coisa que não seria bicho nem gente
conotativa como recurso estilístico de persuasão. Qual das deste mundo."
alternativas abaixo não apresenta esse recurso persuasivo? c) "Como ostentasse certa arrogância, não se distinguia bem
a) Cerveja Bohemia (desde 1853): "Nosso rótulo é, ao mesmo se era uma criança, com fumos de homem, se um homem
tempo, atestado de qualidade e certidão de nascimento." com ares de menino. Ao cabo, era um lindo garção, lindo e
b) Universidade Tuiuti do Paraná: "A Tuiuti escreve uma nova audaz..."
página na História da Educação do Brasil." d) "Excelente senhora, a patroa. Gorda, rica, dona do mundo,
c) Carro SEAT IBIZA: "Agrada tanto pés de chumbo quanto amimada dos padres, com lugar certo na igreja. (...) A
mãos de vaca." excelente Dona Inácia era mestra na arte de judiar de
d) Whiskas: "Novo Whiskas Pedaços ao Molho. Seu gato não crianças. Vinha da escravidão, fora senhora de escravos  e
vai querer outra coisa." daqueles ferozes..."

22 – Assinale a alternativa em que a acentuação gráfica dos 27 – Tomando por base o texto "Mas que significam as pala-
vocábulos esteja correta. vras? Que significam, na verdade, as palavras? Que significa a
a) "O saldo era exigúo, mas certo!" palavra verdade, a palavra mentira ou a palavra amor?", de
b) "Dentro em pouco havia azâfama pela casa, idas e vindas..." Bernadete Lyra, é correto afirmar que
c) "... uma pequena multidão de neófitos, ainda na candidez a) a preocupação com o caráter intelectivo da significação das pala-
das vestes..." vras é preocupação constante da literatura, portanto, é
d) "Alguns levêdos são patogênicos para o homem, porém conotação.
outros são úteis." b) a plurissignificação de uma palavra caracteriza-se como
objetiva e é válida para todos os falantes; portanto, caracte-
23 – Leia o texto e faça o que se pede: riza-se como denotação.
"Maurício era um rapaz extremamente tímido, que sofria c) se abre, tanto para a conotação quanto para a denotação,
muito, pois era apaixonado por Lívia, mas não tinha um leque de possibilidades de significação; para aquela, de
coragem de revelar-lhe tal segredo. Por muito tempo, caráter arbitrário; para esta, de caráter translato.
amargurou sua paixão, até que, um dia, preparou em sua d) estamos tomando por base o subjetivo, o figurativo, a
casa uma grande festa, aproveitando a data de seu conotação, enfim, quando se diz que uma palavra (um
aniversário, e, enfim, diante de todos, confessou à moça seu signo) representa o objeto sem ser o objeto.
grande amor."
Quanto ao texto, é correto afirmar que 28 – Observe:
a) apresenta um grande defeito que é o da prolixidade, uma I- Apresento-lhe Daniella.
vez que não há um motivo que justifique a mudança de II - Faço tudo por um sorriso de Daniella.
comportamento da personagem.
b) apresenta a chamada incoerência narrativa, visto que, na Juntando as duas orações num só período, usando um pronome
narração, uma ação posterior depende da anterior. No texto, relativo, teremos:
há uma lacuna entre o que se apresenta no início e no fim. a) Apresento-lhe Daniella, que pelo sorriso dela faço tudo.
c) apresenta problemas de coesão, pois a conexão entre os b) Apresento-lhe Daniella, por cujo sorriso dela faço tudo.
enunciados não garante a relação de sentidos que deles se c) Apresento-lhe Daniella, por cujo sorriso faço tudo.
espera, por isso se diz que o texto é incoerente. d) Apresento-lhe Daniella, a quem faço tudo pelo sorriso.
d) apresenta ambigüidade, uma vez que a linguagem textual é
usada como expediente para mostrar a dupla face do
personagem.

38
29 – Assinale a alternativa em que a não-observação da forma A palavra vida, grifada acima, em relação ao 3º e 4º versos,
correta de se grafar um de seus termos gera impropriedade pode ser classificada, respectivamente, como
vocabular.
a) A idéia de achar-se às voltas com a leitura de um livro vul- a) sujeito e vocativo.
tuoso era-lhe fascinante. b) núcleo do adjunto adverbial de tempo e vocativo.
b) Estava cansado, exausto, abatido, mas o sacrifício valera a c) aposto e sujeito.
pena. Enfim, sentia que havia expiado sua culpa. d) núcleo do adjunto adverbial de modo e aposto.
c) O conserto da casa gera muitos sons: o bater de martelos, o
ruidoso barulho da massa sendo preparada, a estridência de 34 – Tendo agradado ao marido nas primeiras semanas de
pedras sendo quebradas. Mas há, em tudo, uma confusa casado nunca mais quis ela se separar da receita daquele bolo.
harmonia, gerando... música, talvez? Assim, durante quarenta anos, a sobremesa louvada compôs
d) O aluno andava acabrunhado. Havia se dedicado tanto ao sobre _____ mesa o almoço de domingo, e celebrou toda _____
trabalho, e o professor taxara-o de ingênuo na forma de data em que o júbilo se fizesse necessário.
argumentar. Por fim, achando ser chegada _____ hora, convocou ela o
marido para o conciliábulo apartado no quarto. E tendo decidido
30 – Observe o texto. ambos, comovidos, pelo ato solene, foi a esposa mais uma vez
"Mas esses letreiros luminosos não seriam muito mais belos _____ cozinha assar a massa açucarada, confeitar a superfície.
se fossem escritos em chinês?" Pronto o bolo, saíram juntos para levá-lo ao tabelião, a fim de
I - Todas as palavras do texto apresentam desinência nominal que se lavrasse ato de adoção, tornando-se ele legalmente
que indica o número plural. incorporado _____ família, com direito ao prestigioso
II - Apenas cinco palavras do texto apresentam a desinência sobrenome Silva, e nome Hermógenes, que havia sido do avô.
nominal do plural. (Marina Colasanti)
III - Em "fossem", há desinência verbal modo-temporal seguida
da vogal de ligação. Assinale a alternativa que preenche corretamente as lacunas.
IV - Em "letreiros", a vogal temática "e" faz a ligação do radical
"letr" ao sufixo "ros". a) a – a – a – à – à c) à – a – à – a – a
b) à – à – a – à – à d) a – à – à – a – a
Baseando-se nas afirmações quanto à estrutura das palavras,
está(ão) correta(s) a(s) afirmação(ões) 35 – "Sem cesta, sem-terra pedem esmola." (Folha o / nov.
a) I e II. c) I e III. 2000)
b) II apenas. d) IV apenas.
A concordância da frase acima está
31 – Observe os textos e assinale a alternativa correta quanto a) correta, pois ocorre o fenômeno da concordância ideológica,
aos processos de formação de palavras. já que se trata de substantivo coletivo.
I- "Onde estão meus verdes? b) incorreta, pois não foi observada a grafia correta para "sem-
Os meus azuis? cesta", que passa a ser visto como advérbio; assim há na
O arranha-céu comeu!" (Mário Quintana) frase sujeito simples.
II - "Flor do Lácio sambódromo c) incorreta, porque, como não há plural para o substantivo
lusamérica latim em pó" (C. Veloso) sem-terra, o uso do artigo é fundamental para a definição do
III - "Sempre que o sol /pinta de anil todo céu / singular ou plural.
O girassol fica um gentil carrossel." (V. de Moraes) d) correta, pois o substantivo sem-terra é invariável, valendo
tanto para o singular como para o plural.
a) Verdes sofre derivação imprópria e por isso, quanto à forma,
deixa de ser palavra primitiva.
b) Em lusamérica há composição por justaposição, pois
apenas um dos vocábulos sofre supressão e mantém-se a
integridade sonora.
c) Em girassol há composição por aglutinação, pois houve o
acréscimo de uma letra, independente da manutenção da
integridade fonética.
d) Lusamérica caracteriza-se como composição por aglutina-
ção, pois como há supressão em um dos vocábulos, há
perda de integridade sonora.

32 – Observe os períodos abaixo:


I - Mal o leão se afastou, o rato não teve a menor dúvida.
II - "Os animais devem ser adestrados, ao passo que os seres
humanos devem ser educados."
III - Não obstante haja concluído um curso superior, é incapaz
de redigir uma carta.
IV - Pode criticar, desde que fundamente sua crítica em argumentos.

As orações sublinhadas exprimem, respectivamente, circunstância de


a) tempo, proporção, concessão e condição.
b) causa, conformidade, condição e concessão.
c) tempo, proporção, condição e concessão.
d) condição, concessão, tempo e conseqüência.

33 – Observe:
reza (Arnaldo Antunes)
vida,
minha dádiva,
venha de manhã e dure a vida,
minha dádiva,
venha de manhã e dure a vida,
minha dádiva,
venha de manhã e dure hoje.

39
GABARITO DAS PROVAS DE PORTUGUÊS
1/2001 - TURMA A 2/2001 - TURMA B 22 A 20 A
01 B 01 A 21 B
02 A 02 C 22 B
23 C
03 B 03 B 23 A
04 C 04 C 24 C
05 A 05 B 24 A 25 D
06 B 06 A 26 B
07 C 07 D 27 ANULADA
08 A 08 A 25 B 28 D
09 C 09 C 29 D
10 A 10 B 26 D 30 B
11 D 11 D 31 A
12 A 12 A 32 C
13 C 13 C 27 C 33 A
14 B 14 B 34 C
15 A 15 C 28 B 35 B
16 D 16 B 36 B
17 B 17 D 37 A
18 D 18 A 29 C 38 C
19 B 19 D 39 D
20 D 20 A 30 C 40 D
21 C 21 B
22 D 22 C
23 ANULADA 23 D 31 D
24 D 24 B
25 C 25 D 32 C

1/2001 - TURMA B 1/2002 - TURMA A


01 C 33 A
02 A
01 A
03 B 34 C
04 D
05 A 02 B
06 C 35 A
07 A
08 D 03 A
36 B
09 B
10 D 04 C
11 B 37 C
12 A
13 D 05 D
38 D
14 C
15 A 06 A
16 C 39 D
17 D
18 B 07 A
40 D
19 A
20 C 08 B
21 A
22 B
23 C 09 D
24 B
25 D 10 D

2/2001 - TURMA A
01 C 11 A
02 D
03 A 12 A
04 D
05 B
06 A 13 B
07 D 1/2002 - TURMA B 12 2/2002 - TURMA A
08 B 14 A 01 A 01 A
09 C 02 B 02 B
10 B 03 C 03 A
11 A 15 C 04 A 04 B
12 B 05 A 05 A
13 C 16 A 06 ANULADA 06 B
14 A 07 A 07 A
15 C 08 B 08 B
16 D 17 D 09 A 09 B
17 B 10 D 10 A
18 A 11 C 11 B
18 D
19 D 12 A 12 C
20 A 13 A 13 D
21 C 19 B 14 D 14 C
22 B 15 D 15 A
23 A 16 C 16 D
20 A
24 D 17 C 17 C
25 C 18 C 18 C
21 B 19 A 19 D
GABARITO DAS PROVAS DE PORTUGUÊS
20 D 20 B 20 C 20 D
21 D 21 D 21 B 21 D
22 B 22 B 22 D 22 C
23 A 23 A 23 D 23 C
24 C 24 B 24 B 24 C
25 A 25 C 25 A 25 D
26 D 26 D 26 C 26 A
27 A 27 A 27 D 27 C
28 C 28 A 28 A 28 A
29 D 29 D 29 D 29 D
30 C 30 B 30 D 30 B
31 C 31 D 31 B 31 B
32 B 32 D 32 C 32 B
33 C 33 B 33 A 33 C
34 A 34 C 34 A 34 B
35 D 35 D 35 B 35 C
36 B 36 C 36 C 36 B
37 B 37 A 37 C 37 B
38 D 38 A 38 C 38 D
39 D 39 C 39 A 39 A
40 C 40 C 40 A 40 D

2/2002 - TURMA B 1/2003 - TURMA A 1/2003 - TURMA B 2/2003 - TURMA A


01 C 01 B 01 C 01 B
02 B 02 D 02 A 02 C
03 A 03 D 03 C 03 B
04 B 04 A 04 A 04 C
05 B 05 C 05 B 05 C
06 B 06 C 06 C 06 D
07 C 07 C 07 D 07 D
08 C 08 A 08 A 08 C
09 A 09 B 09 A 09 D
10 D 10 D 10 D 10 C
11 C 11 D 11 B 11 A
12 B 12 B 12 C 12 B
13 A 13 B 13 D 13 C
14 A 14 B 14 B 14 D
15 D 15 C 15 A 15 B
16 D 16 A 16 A 16 A
17 D 17 B 17 D 17 B
18 C 18 A 18 A 18 B
19 C 19 D 19 A 19 A
GABARITO DAS PROVAS DE PORTUGUÊS
20 B
21 A
22 C
23 A
24 C
25 D
26 D
27 D
28 A
29 D
30 D
31 A
32 B
33 B
34 A
35 D

2/2003 - TURMA B
01 D
02 A
03 A
04 C
05 B
06 A
07 A
08 ANULADA
09 B
10 B
11 C
12 C
13 C
14 A
15 D
16 B
17 ANULADA
18 D
19 D
20 C
21 D
22 C
23 B
24 A
25 A
26 D
27 B
28 C
29 A
30 B
31 D
32 A
33 B
34 A
35 D
M
08 – Dois capitais, um de R$ 1.450,00 e outro de R$ 1.300,00,
são colocados a juros simples de 6% e 7% ao ano,
respectivamente. Em quanto tempo os montantes dos dois
ATEMÁTICA – EEAR capitais serão iguais?
a) 30 anos e 6 meses.
b) 37 anos e 6 meses.
1/2001-TURMA A c) 60 anos e 8 meses.
d) 64 anos e 8 meses.
01 – Numa cidade “X”, é consumido leite dos tipos: A e B. Dos
consumidores consultados, 30 consomem dos tipos A e B, 100 09 – Sobre o preço de um carro importado incide um imposto de
somente do tipo A, 200 somente do tipo B e 40 nenhum dos dois 30%. Em função disso, o seu preço para o importador é de
tipos. Quantas pessoas foram consultadas? R$ 58.500,00. Supondo que tal imposto passe de 30% para
a) 300 60%, qual será, em reais, o novo preço do carro para o
b) 310 importador?
c) 330 a) R$ 45.000,00
d) 370 b) R$ 56.500,00
log 4 a  b   x e a  b 
1 c) R$ 72.000,00
02 – Sabendo que , então d) R$ 90.000,00
16

 
log 4 a 2  b 2 é igual a: 10 – Num triângulo isósceles de 54 cm de altura e 36 cm de base
está inscrito um retângulo de 18 cm de altura, com base na base
a) 2x do triângulo. A base do retângulo mede, em cm:
b) 2 – x a) 23
c) x – 2 b) 24
d) 2 + x c) 25
d) 26
03 – Em um triângulo retângulo a hipotenusa mede 5 cm e o
1 11 – As medidas dos lados de um triângulo são: AB = 28 cm ,
sen B̂  sen Ĉ . O maior cateto mede, em cm: AC = 21 cm e BC = 35 cm. Uma paralela ao lado BC intercepta
2
os lados AB e AC nos pontos D e E, respectivamente.
a) 3 Sabendo que o perímetro do trapézio BDEC mede 74 cm, então
a base menor desse trapézio mede, em cm:
b) 5 a) 23
c) 2 3 b) 24
c) 25
d) 2 5 d) 26

04 – Se os ângulos internos de um triângulo estão em PA 12 – Se na figura, AB = AC e BC = CD = DA, então o valor do


(progressão aritmética) e o menor deles é a metade do maior, ângulo , em graus, é: A
então o valor do maior ângulo, em graus, é: a) 30
a) 80 b) 36 
b) 90 c) 45 D
c) 100 d) 60
d) 120
C B
05 – Resolvendo, em R, a equação 2 x  3  x  5 , obtemos
13 – Em um círculo de 3 cm de raio, a área e o perímetro de um
o seguinte conjunto solução: setor circular de 60o (sessenta graus) são, respectivamente,
a)   2, 2  em cm2 e cm:
b)  2, 8  
a) 1,5 e   6  
c)  e   6 
b) 1,5 e  d) 6 e 
 2 
c)  , 2
 3  14 – De um ponto externo a uma circunferência, traçamos um
 2  segmento secante de 32 cm que determina uma corda de
d)  , 8 27,5 cm. O segmento tangente traçado do mesmo ponto externo
 3  mede, em cm:
a) 4,5
x– 2 b) 12
06 – Resolvendo a equação (0,0625) = 0,25 , obtemos “x” c) 14,4
igual a:
2 5 d) 2 55
a) c)
9 2
1  cos 2 x
2 9 15 – A expressão é identicamente igual a:
b) d) sen 2 x
5 2
a) cotg x
b) sec x
07 – O conjunto imagem da função f x   3  5 é:
x c) sen x
a) f x  R / f x    4
d) tg x

b) f x  R / f x    4 16 – Duas cordas AB e CD de uma circunferência cortam-se


c) f x  R / f x    5 num ponto M. Sabendo que AB = 21 cm, MB = 12 cm e CM =
d) f x  R / f x    5
3 DM, então CD, em cm, mede:
a) 23
b) 24
c) 25
d) 26

1
17 – Num círculo de centro C e raio R, tomam-se dois pontos A 1/2001-TURMA B
e B sobre a circunferência do círculo. Sendo o ângulo   AĈB 01 – Considere os conjuntos A = [1 , 2]  [3 , 4] ; B = ]1 , 4] – {3} ;
e sabendo-se que o arco AB tem comprimento R, então pode-se C = [2 , 3[  {4} e X = (A – B)  (A C). Assinale a alternativa
afirmar: correta:
a) X  A = B
a)   45o
b) X  C = X
b)   90 o c) X  A = X
c) 45o    50o d) X  B = C

d) 55o    60o 02 – Considere as afirmativas:


I. Numa divisão, cujo resto “r” não é nulo, o menor número que se
18 – A secção meridiana de um cilindro equilátero tem 4 2 cm deve adicionar ao dividendo para que ela se torne exata é “d – r”,
de diagonal. O volume do cilindro, em cm 3, é de: sendo “d” o divisor.
a) 16  II. A soma de três números naturais consecutivos é sempre
b) 24  divisível por 3.
c) 32  III. O produto de dois números ímpares consecutivos, aumentado
d) 54  de uma unidade é sempre um quadrado perfeito.

19 – ANULADA São verdadeiras as afirmativas:


a) I e II.
20 – A base de um prisma quadrangular regular está inscrita b) I e III.
c) II e III.
numa circunferência cujo círculo tem 100  cm2 de área. Se a
d) I, II e III.
altura do prisma mede 1,5 cm, então o volume desse prisma, em
cm3, é de:
03 – Sejam os conjuntos A = [ – 1 , 2 ], B = [ – 2 , 4 ] e
a) 200
C=[–5,0[.
b) 300
É falso afirmar que:
c) 400
a) ( B – C ) – A = [ 2 , 4 ]
d) 800
b) (A  (B–C)=[0,2]
B)
x m 1
c) (B–A)  (A  B)=[–2,4]
21 – Dada a equação 1 1 1  0 , quais os valores d) (B  C)–(A  B)=]–5,–1[  ]2,4]
0 1 x
1
de m para os quais as raízes são reais? 04  O menor valor real e positivo de x tal que 4sen x  é:
a) m3 2
b) m  1 
a)
c) 1  m  3 6
d) m  1 ou m  3 5
b)
22 – A altura de uma pirâmide quadrangular regular é igual à 6
aresta de sua base. Sendo B a área da base da pirâmide, então 7
sua área lateral, em cm2, é: c)
6
a) B 5
11
B 5 d)
b) 6
3
c) B 3 05 – A respeito do conjunto solução da equação

d) 5B x 3
2
x 3
  , em  , pode-se afirmar que:
*
x x 3
2 2
23 – A equação da reta que passa pelo ponto B 4,  5   e de a) tem um elemento igual a zero.
1 b) tem dois elementos.
coeficiente angular é: c) é unitário.
2 d) é vazio.
a) x  2y  6  0
b) x  2 y  12  0 06  O valor de mercado de um automóvel é alterado a cada mês
com um acréscimo de 1% em relação ao mês anterior. A
c) x  2 y  14  0 seqüência de valores do automóvel, a cada mês, forma uma
d) x  2 y  14  0 progressão:
a) aritmética de razão 0,1.
24 – O valor de k de modo que a reta kx + 2y + k – 8 = 0 passe b) aritmética de razão 0,01.
pela intersecção das retas x  y  0 e x  3y  8 é:
c) geométrica de razão 1,1.
d) geométrica de razão 1,01.
a) 4
b) 3
c) – 4
07 – Na seqüência (1, 1, 2, 3, . . .) , onde a n 1  a n  a n 1 ,
d) – 3 o nono termo é:
a) 34 c) 43
25 – Para que o polinômio 2x 3  mx 2  nx  6 seja divisível b) 21 d) 28
por x  1 e x  3 , então os valores de m e n devem ser,
08 – Se o raio de um círculo for aumentado de 100%, sua área
respectiva-mente, iguais a:
aumentará de:
a) – 7 e – 3.
a) 100% c) 300%
b) – 6 e – 10.
b) 200% d) 400%
c) – 6 e – 2.
d) – 2 e – 6.
09 – Um aluno acertou 4 das 15 primeiras questões de um prova. 
4 c) y > 0 se x k (k  Z).
Das restantes, ele acertou os e, ao final, verificou que 2
5 3
respondera corretamente 60% das questões. O número de d) y < 0 somente se  < x < .
questões que esse aluno acertou foi: 2
a) 24 n ! (n 2  1)
b) 23 18 – Se an  , então a1985 é igual a:
c) 17 (n  1)!
d) 16 a) 1984
b) 1983
10 – Se a < – 2, os valores de x, tais que 1985
c)
a
x  a    x  2 , são aqueles que satisfazem: 1984  1
2
2
a) x < 2 – a b) x < a – 2 19842  1
d)
c) x > 2 – a d) x > a – 2 1984
11 – A soma das medidas dos ângulos internos e externos de um 19 – Como parte de seu treinamento, um piloto realizou 10
polígono convexo é 3600º. O número de diagonais desse missões cujos tempos em minutos são, em ordem: 4 - 6 - 7 -
polígono é um número: 9 - x - 14 - 18 - y - 23 - 26. Sabendo-se que o tempo
a) par divisível por 15. médio das missões foi de 14 minutos e o tempo mediano foi de
b) par maior que 150. 13 minutos, podemos afirmar que x e y valem,
c) ímpar múltiplo de 19. respectivamente:
d) ímpar primo.
a) 13 e 20.
12 – Classifique como verdadeira ou falsa cada uma das b) 12 e 21.
afirmativas: c) 13 e 21.
1.ª Um triângulo obtusângulo pode ser isósceles. d) 12 e 22.
2.ª Um triângulo isósceles pode ser retângulo.
3.ª Um triângulo isósceles não pode ser equilátero. 20 – Numa comunidade residem 120 pessoas. Uma pesquisa
sobre os hábitos alimentares dessa comunidade revelou que 42
Assinale a alternativa correta: pessoas consomem carnes, 90 consomem verduras e 30
a) Todas são falsas. consomem carnes e verduras. Escolhendo-se ao acaso uma
b) Todas são verdadeiras. pessoa desta comunidade, a probabilidade de ela ter o hábito de
c) A 2.ª é verdadeira e a 3.ª é falsa. não comer carnes nem verduras é:
d) A 1.ª é falsa e a 3.ª é verdadeira. a) 7,5% c) 12,5%
b) 10,0% d) 15%
13 – Sejam P, Q e R pontos de uma circunferência de centro O,
tais que P e Q estejam do mesmo lado em relação ao diâmetro 21 – Um plano secciona uma esfera, determinando um círculo de
que passa por R. Sabendo-se que
raio igual à distância do plano ao centro da esfera. Sendo 36 
med(OR̂P)  10º e med(RÔQ)  80º , tem-se que o ângulo cm2 a área do círculo, o volume da esfera, em cm 3 , é:
PQ̂O mede: a) 288 2  c) 288
a) 20º
b) 40º b) 576 2  d) 576
c) 50º
d) 60º 22 – As retas 2x – y = 3 e 2x + ay = 5 são paralelas.
Então, o valor de a é:
14 – Um segmento AB, de 6 metros, é diâmetro de uma a) –1 b) 1 c)  4 d) 4
circunferência de centro O. Sendo C um ponto dessa
23 – Um cilindro circular reto tem o volume igual ao de um cubo
circunferência, tal que a medida do ângulo AB̂C seja 30º, a de aresta “a” e a área lateral igual à área total do cubo. O raio e
medida da superfície limitada pelas cordas AB e BC e pelo arco a altura desse cilindro medem, respectivamente:
AC , em metros quadrados, é: a a 9a 9a
e 3a c) 2a e 3a
   
3 3 9 3 9 3 a) b) e d ) 3a e
a) 2  3 3 b)   3 c) d) 2 3  
4 2 2 2
24 – A reta de equação x + 2y + c = 0 :
15 – Um círculo de raio r e um retângulo de base b são a) é perpendicular à reta 2x + y + c = 0.
equivalentes. Então, a altura do retângulo é: b) é paralela à reta 2x – 4y + c = 0.
r 2 r 2 c) tem distância ao ponto (- c , 1) igual a zero.
a) r b) r 2b c) d)
b b2 
d) forma um ângulo de rd com a reta 3x + y + c = 0.
4
16 – Se tg x = – 3, então tg 4x é igual a:
3 24 3 24 25 – Considere as afirmações:
a)  b)  c) d)
4 7 4 7 I. Qualquer raiz racional da equação x 3  3x 2  3x  9  0
é inteira.
sen x  tgx II. O menor grau da equação polinomial de coeficientes reais, que
17 – Em 0  x  2 , a expressão y  é tal,
cos x  cotgx admite as raízes 3, 2 + i e i , é 5.
III. Toda equação polinomial da forma ax4 + bx3 + cx2 + dx + e = 0
que:
de coeficientes reais e a  0, necessariamente possui uma
 raiz real.
a) y > 0 somente se 0 < x < .
2 São verdadeiras as afirmações:
 a) I, II e III.
b) y < 0 se x k (k  Z). b) I e II.
2 c) II e III.
d) I e III.
07 – Numere a segunda coluna de acordo com a primeira, sendo
2/2001-TURMA A 
0x :
1 i 0 2
01 – Seja o número complexo Z   1 1 i . A forma ( 1 ) sen 2 2 x  ( ) 2 cos 2 x
i3 1  i ( 2 ) 1  cos 2 x  ( ) 2 sen x  cos x
trigonométrica de Z é ( 3 ) sen 2x  ( ) 1  cos 2 2x 

 7 7  ( 4 ) sen(  x) ( )  sen x
a) 2  cos  i sen  2
 4 4 
( 5 ) sen  x  ( ) cos x
  
b) 2  cos  i sen 
 4 4 Lendo-se a segunda coluna de cima para baixo, a seqüência
correta é:
  
c) 2  cos  i sen 
 3 3 a) 1, 3, 4, 5, 2
b) 2, 3, 1, 5, 4
 7 7  c) 3, 1, 2, 4, 5
d) 2 cos  i sen 
 4 4  d) 2, 3, 5, 1, 4

4
02 – Na figura, AD  2 cm e AB  4 cm . O valor de cos   3
 
2
 4   3   15
no triângulo ABC é 08 – Efetuando   2 
5 50 , obtemos
 
1  
a) A
2 a) 10
b) 12
3 c) 4
b)
3 d) –12

3 09 – Seja ABCD um trapézio isósceles. Sabe-se que a medida de


c)
2 um de seus ângulos obtusos internos é o dobro da medida de um
 de seus ângulos agudos internos, e que a diagonal AC é
3 
perpendicular ao lado BC. Se a base maior mede 10 cm, então o
d)  D B C perímetro desse trapézio, em cm, é
2

03 – Ao seccionar uma esfera, um plano determina um círculo a) 20


de raio 16 cm. Se a distância do plano ao centro da esfera é de b) 25
12 cm, então o raio da esfera, em cm, vale c) 28
a) 20 d) 30
b) 28
c) 30 10 – O valor inteiro de x, tal que o dobro do seu logarítimo
d) 38 decimal tenha uma unidade a mais do que o logarítimo decimal
 11 
de  x  ,é
04 – Na resolução da equação matricial  10 
 1  1 0  x   1 1
     a)
 4  1 1   y    2  , o valor de x  y  z é b) 1,7
 0  3 0  z   0 c) 10
     d) 11
a) –2
b) 1
c) –1 11 – Se em um triângulo retângulo um dos catetos mede 2 5
d) 0
cm e a altura relativa à hipotenusa mede 2 cm, então a área
05 – No trapézio escaleno abaixo, tem-se: AD = 5 cm, desse triângulo, em cm2, é
BD̂C  30 o e BÂD  45 o . Nessas condições, a medida da 10
a)
diagonal BD , em cm, é
3
D C 20
a) 5 2 b)
3
b) 5 3
10 2
c) 5 5 c)
A B 3
d) 5
d) 2 10
06 – A equação da reta que passa pelo ponto 3, 2  e pelo ponto
de interseção das retas y  31  x  e y  2x  1 é: 12 – Numa progressão geométrica de 6 termos positivos, a soma
de a2 e a4 é 6, e a soma de a4 e a6 é 12. A razão dessa P.G. é
a) 2x  y  1  0
b) x  2y  1  0 a) 2
c) 2x  2 y  1  0 b) 2
d) x  y 1  0 c)  2
d) –2
1 21 – Em Estatística, ______________ de um conjunto de dados
13 – Uma pessoa aplicou do seu capital a 25% a.a. e o dispostos em ordem crescente, onde o número de dados é
6 ímpar, é o valor que ocupa a posição central; e ______________
restante a 15% a.a.. No fim de 4 anos a soma dos juros simples de um conjunto de dados é o valor mais frequente do conjunto.
atingiu R$ 405,00. O capital aplicado foi de R$ a) Mediana; Moda
a) 607,50 b) Moda; Mediana
b) 670,50 c) Média; Moda
c) 607,05 d) Mediana; Média
d) 670,05
22 – Dados os números racionais m  0,03  10 20 ,
14 – O custo de um par de sapatos é igual ao custo de um terno.
k  0,3  10 21 e p  300  10 22 , é correto afirmar que
Um lojista vende o par de sapatos com prejuízo de 5% sobre o
custo, e o terno com 30% de lucro sobre o custo, recebendo a) m  k  p
pelos dois R$ 180,00. O preço de venda do terno, em reais, é b) m  k  p
a) 76
b) 80 c) k  m  p
c) 104 d) m  k  p
d) 110
23 – As bases de uma pirâmide hexagonal regular e de um
15 – Uma das raízes da equação x 3  12x 2  44x  48  0 prisma quadrangular regular acham-se inscritas num mesmo
é a soma das outras duas. A maior raiz dessa equação é círculo. Sendo H a altura da pirâmide e sabendo-se que os dois
a) 7 poliedros são equivalentes, então a altura do prisma é
b) 6 H 3
c) 4 a)
d) 2
4
3H 3
circunferência x  2  y  1
b)
16 – A
2 2
 1e a reta 4
x  3y  2  0 possuem __________ ponto(s) em comum. H 3
a) 2 c)
2
b) 1
c) infinitos H 3
d) nenhum d)
3

 
17 – Se a área da coroa circular definida por dois círculos
concêntricos de raios r e R, r  R , é igual a área do círculo 24 – Sendo 8 x  3  4 x , tem-se que log 3 x 1 é igual a
R a) 3
menor, então a razão é igual a b) 2
r c) –2
a) 1 d) –1
b) 2
25 – Numa progressão aritmética, o primeiro termo é 10x – 9y, o
2 último termo é y, e a razão é y – x. Sendo x  y, o número de
c) termos dessa P.A. é
2
a) 8
d) 2 2 b) 9
c) 10
d) 11
18 – Se o gráfico representativo de uma função do 2o grau é uma

parábola, então a parábola que passa pelo ponto  2 , 0 , e  2/2001-TURMA B
cujo vértice situa-se no ponto ( 1 , 3 ), representa a função 01 – Os conjuntos S, T e P são tais que todo elemento de S é
a) f x    x 2  2x  8 elemento de T ou P.
O diagrama que pode representar esses conjuntos é
b) f x   3x 2  6x  24
a) c) S
x2
f x   
2x 8
c)  
3 3 3
d) f x   x 2  2 x  8 T S P T P

19 – O total de números múltiplos de três, de quatro algarismos b) S d)


distintos, que podem ser formados com os algarismos 2, 3, 6, 7 T P
e9 é
a) 120
T S P
b) 72
c) 48 02 – Se x e y são números reais que tornam simultaneamente
d) 24
verdadeiras as sentenças 2 x  y  2  30 e 2 x  y  2  0 ,
20 – Uma escola de ensino médio tem 250 alunos que estão y
então x é igual a
matriculados na 1a, 2a e 3a séries. Dos alunos matriculados, 32%
a) 9
são homens e 40% dos homens estão na 1a série; 20% dos
b) 8
alunos matriculados estão na 3a série, sendo 10 alunos homens;
e na 2a série, o número de mulheres é igual ao número de 1
c)
homens. O número de mulheres na 1a série é 8
a) 10 1
b) 48 d)
c) 92 9
d) 102
1 11 – Um capital cresce sucessiva e cumulativamente, na base de
03 – Seja k a raiz da equação 2log 8 log 2 x  . O valor de k8 10% ao ano. Ao final de 3 anos, o montante, comparado ao
2 capital inicial, será
é
1 a) 30% superior.
a) c) 1 b) 130% do capital.
8
c) aproximadamente 150% do capital.
1 d) aproximadamente 133% do capital.
b) d) 2
4
12 – De um pedaço quadrado de metal corta-se uma peça
04 – Tanto numa P.A. quanto numa P.G., os números 3 e 243 circular de diâmetro máximo, e desta corta-se outro quadrado de
são, respectivamente, a razão e o 6.º termo. O produto do 1.º lado máximo. O material desperdiçado tem
termo da P.G. pelo 3.º termo da P.A. é 1
a) 702
a) da área do quadrado original .
4
b) 693
c) 234 1
b) da área do quadrado original .
d) 231 2
2 1 x 1
c) da área da peça circular .
05 – O conjunto solução da inequação x 1 0 >0 é dado 2
1
x 0 1 d) da área da peça circular .
4
por:
a) ]0,2[ x  y  0

b) ] -2 , 1 [ 13 – O sistema linear  y  z  0 é indeterminado para
c) ] -2 , 1 [  ]1,2[  y  mz  0
d) ] -1 , 0 [  ]1,2[ 
a) nenhum m real.
06 – Num cone circular reto, cujo raio da base mede r, a base é b) todo m real.
equivalente à secção meridiana. A altura desse cone mede c) m=0
d) m=1
r
a) rg b) c) r d) g
g 14 – Um trem de passageiros é constituído de uma locomotiva e
7 vagões distintos, sendo um deles restaurante. Sabendo que a
p locomotiva deve ir à frente e que o vagão restaurante não pode
07 – Seja a forma irredutível do resultado da expressão
q ser colocado imediatamente após a locomotiva, o número de
modos diferentes de montar a composição é:
3 1
2 1 a) 720
4 2  1,2363636 ... . O valor de p – q é b) 4.320
1 1
4 1 c) 5.040
4 2 d) 30.240
a) 78
b) 98 15 – O triângulo cujos vértices são os pontos ( 1 , 3 ), ( - 2 , - 2 ) e
c) 324 (1,-2)é
d) 524 a) obtusângulo.
b) equilátero.
c) retângulo.
08 – Seja log 2 = 0,301. Efetuando-se 50 50 , obtemos um valor d) isósceles.
cuja quantidade de algarismos é
a) 85 16 – Os resultados da prova de Ciências aplicada a uma turma
b) 84 de um certo colégio estão apresentados no gráfico. Baseado
c) 83 neste gráfico, podemos afirmar que a porcentagem de alunos
d) 82 dessa turma com nota inferior a 5,0, nessa prova de Ciências, foi
de
09 – O valor máximo da função definida em  por 14
a) 37,5%
f (x)  mx 2
 6x  m, m  * é igual a 8. Então o valor de m b) 42,5% 12
número de alunos

é c) 47,5% 10
a) 9 d) 52,5% 8
b) 8 6
c) –1
4
d) –3
2
10 – Um pai deseja repartir a quantia de R$2.600,00 entre seus 0
quatro filhos, de modo que as partes sejam proporcionais às suas 1 2 3 4 5 6 7 8 9
idades e formem uma P.A. Se a idade do filho mais jovem é 8 notas
anos e a do mais velho é 44, a quantia dada ao filho mais jovem
será, em reais, 17 – No sistema de coordenadas cartesianas, a equação
a) 200
x 2  y2  ax  by , onde a e b são números reais não nulos,
b) 250
c) 300 representa uma circunferência de raio
d) 350 a 2  b2
a)
2
b) a 2  b2
a  b
c)
2
d) a+b
18 – Seja m . Para que o produto ( 2 + m i ) . ( 3 + i ) seja um
 1
número imaginário puro, o valor de m deve ser c) x   / x  3 e x   .
a) 5  2
b) 6
 1
x   / x  3 e x    .
c) 7 d)
d) 8  2 
19 – Supondo definida em  a fração 02 – Determinando log 25 0,008 , obtemos
a . a  a . a  a . a 1 3 3 2 2
, o seu valor é a) . b)  . c) . d)  .
2 2 3 3
a 1
2

a) a 1 03 – Na figura, sendo MN  x cm , NP  10 cm ,
b) a+1
c) a–1
PO  5 cm e OQ  (4x  1) cm , então o valor do segmento
d) a de reta PQ , em cm, é
20 – Sejam p(x) = x2 – 5x + 6 e q(x) = x2 – 3x + 1. Se a é um P
número real e p(a) < 0, então q(a) satisfaz a) 29. N
a) –1 < q(a) < 1 b) 35.
c) 12. M
b) q(a) < -1 ou q(a) > 1
c) –2 < q(a) < 2 d) 34. O
d) q(a) < -2 ou q(a) > 2

21 – Se A =  x / 3x - 2x2  0 , B =  x / 1  x  3  e C = 


Q
x / x2 – x – 2  0 , então ( A  B )  C é
a)  x / -1  x  0 ou 1  x  2 
3 04 – Pode-se afirmar que o valor do determinante
b)  x / -1  x  0 ou x2
2 ax 0 1
c)  x / -1  x  2  0 2x x2 é igual a
d)  x / 0  x  2 
a 0 1
22 – Mílton comprou um carro 0km e, pensando em economizar
os pneus, usou os quatro colocados mais o estepe, numa viagem a) x2  2 . c) x x  2  .
xx  2a  2 .
cujo percurso foi de 2.000 km. Se cada pneu rodou a mesma
quilometragem, então o estepe foi usado nessa viagem por b) x  2x .
2
d)
e) 2.000 km
f) 1.600 km 05 – Das afirmações:
g) 1.200 km
h) 800 km I. cos x  1  sen x
II. sen 2x  2 sen x cos x
23 – Os valores reais de a e b tais que os polinômios:
P(x) = x3 – 2ax2 + (3a + b)x – 3b e Q(x) = x3 – (a + 2b)x + 2a III. cos 2x  cos2 x  sen2 x
sejam divisíveis por x + 1 são dois números Sendo x um arco no ciclo trigonométrico compreendido entre
a) inteiros positivos.

b) inteiros negativos. e  , conclui-se que
c) reais, sendo um racional e outro irracional. 2
d) inteiros, sendo um positivo e outro negativo.
a) a única falsa é a I. c) a única falsa é a III.
24 – Considere as circunferências que passam pelos pontos (0 , b) a única falsa é a II. d) as três são verdadeiras.
0) e (2 , 0) e que são tangentes à reta y = x + 2 as coordenadas
dos centros dessas circunferências são 06 – Sendo  um ângulo agudo, o lado “x” do triângulo abaixo,
a) (1 , 1) e (1 , -7) em cm, mede
b) (1 , 1) e (-7 , 1)
c) (1 , -7) e (1 , 7) a) 6. 6 cm
d) (1 , -7) e (-1 , 7) b) 10.
x
c) 12.
25 – Dois lados consecutivos de um paralelogramo medem 8 m e d) 15. 30o 
12 m e formam entre si um ângulo de 60º. As medidas das
diagonais desse paralelogramo são tais que o número que
6 3 cm
expressa
a) o seu produto é racional. 2
2 2 x 1
b) a sua razão é maior que 2. 07 – Resolvendo a equação 2  256 , concluímos que
c) a sua soma é maior que 32.
ela
d) a sua diferença é irracional. a) não admite soluções reais.

1/2002-TURMA A 3
b) admite como raiz.
2
x 3
01 – O domínio da função real f x   é c) admite duas soluções reais positivas.
4x  2 d) admite duas soluções cuja soma é zero.
 1
a) x   / x  3 e x   .
 2
 1
b) x   / x  3 e x    .
 2 
a c e 16 – A parábola de equação y  2x 2  bx  c passa
08 – Toda vez que ocorrer   , também ocorrerá
b d f  
pelo ponto 1, 0 e seu vértice é o ponto de coordenadas 3, v  .
ac ac a  c  e a2 A coordenada v é igual a
a)  . c)  . –28. –8.
b d b  d  f b2 a) b) 28. c) d) 8.

a b cd a . c . e a3 17 – Os valores reais de x do sistema 1  x  1  2 são


b)  . d)  .
c a b . d . f b3 a) 1  x  0 ou 2  x  3 . c) x  0 ou x  2 .
b) 0x2. d) 1  x  2 .
3 3
09 – Se a tg x  e   x  , então o valor de
4 2 18 – É falso afirmar:
cos x  sen x é igual a a) Se AÔB é um ângulo raso, então OA e OB são semi-
7 7 1 1 retas opostas.
a) . b)  . c)  . d) .
5 5 5 5 b) Se AÔB é um ângulo nulo, então OA e OB são semi-
retas opostas.
10 – A geratriz de um cilindro de revolução mede 10 cm. Qual o c) Dois ângulos adjacentes, cujos lados não comuns são semi-
seu raio da base, sabendo-se que, aumentando-se esse raio em retas opostas, somam 180o.
10 cm e mantendo-se a altura, a área lateral do novo cilindro é d) Dois ângulos adjacentes são sempre consecutivos.
igual à área total do primeiro?
a) 2,5 cm
19 – Se a diferença entre os quadrados das raízes da equação
b) 5 2 cm
x 2  2x  k  0 é 8, então o valor de “k” é
c) 10 cm
a) 5. c) 3.
d) 20 cm
b) –5. d) –3.
11 – Uma firma contratou o trabalho de um pintor na base de
20 – A soma dos ângulos internos e dos ângulos externos de um
R$ 45,00 por dia. Sabe-se que ele trabalhou durante 32 dias, e
do total a lhe ser pago foi descontado 8% para o Imposto de polígono regular vale 1.800o . O número de diagonais desse
Renda e 10% para o INSS. A quantia líquida que ele recebeu foi polígono é
R$
a) 1.180,80. c) 1.100,00. a) 25. b) 35. c) 45. d) 55.
b) 1.200,00. d) 1.250,00.
2
12 – Na figura, BN é a bissetriz do ângulo B̂ . Se   50 o e 11  5  3,333 
21 – A expressão
3 , tem como resultado
Ĉ  30 o , então a medida x do ângulo HB̂N é  1 
 3  2  1,2
B  3 
1 1 1 1
a) 5º. a) 6 . b) 7 . c) 8 . d) 9 .
b) 10°. x 4 4 4 4
c) 15º.
d) 20º. 22 – A área lateral de um prisma hexagonal regular de 25 cm de

A C altura e de apótema da base igual a 2 3 cm, em cm2, é
H N
3 a) 1.200. b) 600 2 . c) 600 3 . d) 600.
13 – Resolvendo a equação sen x  , onde
2
23 – Dado o hexágono regular ABCDEF, a área do quadrilátero
0  x  2 , obtemos como conjunto solução
 x  R / x  60 
ABCD, em cm2, sabendo-se que AB mede 6 cm, é
a)
o
 K360 ou x  120  K360 , K  Z
o o o
E
 x  R / x  30 .
a) 54.
b)
o
 K360 , K  Z
o
F D
b) 54 3 .
c)  x  R / x  60 o
 K360 ou x  240  K360 , K  Z
o o o

 x  30o  K360o , K  Z .
c) 18 3 .
d) A C
d) 27 3 .
B
14 – Na figura, MNPQ é um losango. Se MT  12 cm e 24 – Leia com atenção.
MS  6 cm , então o lado do losango, em cm, mede I. Os possíveis valores de x para os quais se tenha x  12
M são –12 e 12.
a) 2. II. {–3, –2, –1, 0, 1, 2, 3} = {x   / –3  x  3}.
b) 4.
c) 8. N Q III. Os números inteiros que verificam a desigualdade x  2
d) 12. é o conjunto {–2, –1, 0, 1, 2}.
IV. Os valores reais de x que verificam a desigualdade x 1
T P S
é o conjunto {x   / –1  x  1}.
 o
15 – Os números 2x  10 ,  3x , 3x  20o  são medidas Com relação às afirmações acima, podemos dizer que
em graus dos ângulos de um triângulo. Esse triângulo pode ser a) I, II, III e IV são verdadeiras.
classificado em b) I e II são verdadeiras.
a) acutângulo. c) retângulo. c) I e II são falsas.
b) equiângulo. d) obtusângulo. d) I, III e IV são verdadeiras.
25 – Um capital, aplicado a juros simples, duplicará em 4 anos se 35 – Na figura, M e N são pontos de tangência. Sendo os raios,
a taxa anual for de respectivamente, 14 cm e 7 cm e a distância dos centros
a) 75%. b) 50%. c) 25%. d) 15%. OO 1  24 cm , então o segmento MN, em cm, mede
N
26 – Numa aplicação de R$ 10.000,00 ao juro mensal de 2%, 
creditado mensalmente na conta do aplicador, o montante no fim a) 527 . O1
do 3o mês será de R$
a) 10.200,00. c) 10.404,00. b) 380 . O
b) 10.400,00. d) 10.612,08. 
c) 3 15 . M
27 – A soma dos vinte primeiros termos da PA cujo termo geral
d) 12.
tem para expressão a n  3n  5 é
a) 657. c) 803.
b) 730. d) 1460.
28 – O volume, em cm3, de uma pirâmide quadrangular regular
36 – Seja uma circunferência com centro sobre a reta y  3x .
cujas faces laterais são triângulos eqüiláteros de lado 4 cm, vale Se a circunferência é tangente à reta yx na ordenada 4,
então as coordenadas do centro da circunferência são
16 2 a) (4, 12). c) (3, 9).
a) 16 2 . c) .
3 b) (2, 6). d) (5, 15).
32 2 37 – Dentre os pontos que equidistam de A(1, 2) e B(3, 4), o
b) 32 2 . d) .
3 ponto mais próximo de P(6, 1) que pertence ao eixo das abscis-
sas é
29 – O valor de m, para que o módulo do número complexo a) 5. c) 6.
  
Z  m  2i 1  i seja igual a 4, é b) 3. d) 4.

a) 1. b) 2. c) 3. d) zero. 38 – A soma dos termos de uma PG crescente de três termos
positivos é 21 e a diferença entre os extremos, 15. A razão dessa
30 – Duas réguas de madeira, AB e CD, com 8 cm cada uma PG é
estão ligadas em suas extremidades por dois fios, formando o a) 4. b) 5. c) 6. d) 7.
retângulo ABCD (fig. 1). Mantendo-se fixa a régua AB e girando-
se 180o a régua CD em torno do seu ponto médio, sem alterar os 39 – Uma classe tem 10 meninos e 12 meninas. A quantidade de
comprimentos dos fios, obtêm-se dois triângulos congruentes AIB maneiras que poderá ser escolhida uma comissão de três
e CID (fig.2). A distância, em cm, entre as duas réguas, nessa meninos e quatro meninas, incluindo obrigatoriamente o melhor
nova posição (fig.2) é aluno e a melhor aluna, é dada pelo(a)
a) soma de 36 e 165. c) produto de 120 e
A B A  B 495.
I b) soma de 120 e 495. d) produto de 36 e 165.
a) 5 3. 10 cm 
C D
b) 5 2. 8 4a  4 2a
40 – A expressão é equivalente a
c) 5. D C 8 cm 82a  4 a
d) 6. fig.2 1 24a . c) 3  2 .
2 2a
a)
 .  .
8 cm
fig.1 b) 22a 24a  1 d) 24a 24a  1

31 – A área, em cm2, de um triângulo eqüilátero inscrito numa 1/2002-TURMA B


circunferência cujo comprimento é de 8 3 cm é
01 – Se a  99 , b3  99 7 e c 4  99 8 , então abc 12 é
2 6

a) 36 3 . c) 72 3 . igual a
b) 64 3 . d) 144 3 . a) 9912 c) 99 28
x 1 21
32 – A inequação sen  , onde 0  x  2 , é b) 99 2 d) 99 88
2 2
verdadeira se, e somente se 02 – Se x, y, e z são números naturais diferentes entre si, e
  5 x  y  z , então é falsa a afirmativa:
a)  x  2 . c)  x  .
a) x é múltiplo de z. c) y é divisível por z.
6 6 6
b) y é divisor de x. d) x é divisível por y.
 5  
b)  x  . d)  x  . 03 – Duas retas paralelas são cortadas por uma transversal, de
3 3 6 3
modo que a soma dos ângulos agudos formados vale 144º.
Então a diferença entre as medidas de um ângulo obtuso e de
33 – Considere a equação x  6 x  13 x  10  0 em que –2
3 2 um agudo é
é uma das raízes. As demais raízes são a) 85º c) 108º
b) 92º d) 116º
a) 2  i e 2  i . c) 2  i e 2  i .
b) 1 e –5. d) 2  2i e 04 – A razão entre os comprimentos das circunferências
2  2i . circunscrita a um quadrado e inscrita no mesmo quadrado é
a) 2 c) 3 2
34 – Dado um quadrado de diagonal igual 2 cm. Sobre cada
lado do quadrado se constrói externamente um triângulo
b) 2 d) 2 2
equilátero de lado igual ao do quadrado. A área da figura toda,
assim obtida, é .................... cm2. 05 – Uma equação do 3.º grau cujas raízes são –1, –2 e 3 é
a) x 3 + 6 x 2 – 9x + 6 = 0 c) x 3 – 7 x – 6 = 0
a) 2 3 c) 1 2 3 b) x 3 – 6 x 2 – 6 = 0 d) x 3 + 6 x 2 + 9 x = 0

b) 1 3 d) 2  4 3
06 – Seja o intervalo I =  -2 , 3  e a figura abaixo o gráfico da 14 – As promoções do tipo “leve 3, pague 2”, comuns no
função f : I  . comércio, acenam um desconto, sobre cada unidade vendida, de
50
f(x) a) % c) 20%
3
100
b) % d) 50%
3

15 – Um baralho tem 52 cartas, sendo 4 reis e 48 não reis. O


5 1 2 3
x número de maneiras diferentes que uma pessoa pode retirar
-2 desse baralho um grupo de 5 cartas, sendo 3 reis e 2 não reis, é
4 igual a
Então a) 44.100 c) 2.162
b) 1.152 d) 54.144
a) f(x) = x4, x  I. 3x  4 y  z  0
b) f(x)  0, x  -2 , 0 . 
c) se a, b  I, então f(a)  f(b). 16 – O sistema de equações 2x  y  3z  0
 5  x  y  0
d) f(a) . f   + f(b) . f(1) + f(c) . f(2) = 0, a, b, c  I. 

 4  a) não tem solução
b) tem infinitas soluções
07 – Dois números estão entre si como 3 está para 5. Então a c) tem apenas a solução trivial
razão entre o quíntuplo do 1.º e a terça parte do 2.º é d) tem uma única solução não trivial
1 17 – O determinante da matriz A de ordem 3, tal que
a) c) 3
9  2i  j, se i  j
1 a ij   é igual a
b)
3
d) 9  2i, se i  j
a) 72 b) 60 c) 48 d) 40

08 – Sobre a equação 1983 x


2
 1984 x  1985  0 , a 18 – É incorreto afirmar que
afirmação correta é: 1
a) não tem raízes reais a)  0,0051 c) 0,333...  0,3
b) tem duas raízes simétricas 51
c) tem duas raízes reais distintas 4 2
d) tem duas raízes reais iguais b)    1 d) 0,555  0,555 
3 6
09 – Se K é um número inteiro, K2 + K é necessariamente um
a) múltiplo de 2. 19 – A área de um retângulo, cujas diagonais medem 20 m cada
b) múltiplo de 3. uma e formam entre si um ângulo de 60º, em m 2, é
c) produto de dois números ímpares. a) 100 b) 200 c) 100 3 d) 200 3
d) produto de dois números primos.

10 – O número de elementos do conjunto solução da equação 20 – Um tanque tem a forma de um cilindro circular reto de altura
6 m e raio da base 3 m. O nível da água nele contida está a uma
2x  5   4x  1 , em , é distância do fundo do tanque igual aos 2/3 da sua altura.
a) 0 c) 2 Adotando-se  = 3,14, a quantidade de litros de água que o
b) 1 d) infinito cilindro contém é
a) 113.010 c) 113.050
11 – Numa pesquisa de mercado sobre o consumo de cerveja, b) 113.040 d) 113.080
obteve-se o seguinte resultado: 230 pessoas consomem a marca
A; 200 pessoas, a marca B; 150, ambas as marcas; e 40 não 21 – A aresta de um cubo e a aresta da base de um prisma
consomem cerveja.
triangular regular medem 4 3 cm. Se o cubo e o prisma são
O número de pessoas pesquisadas foi
a) 620 c) 320 equivalentes, então a área total do prisma, em cm 2, é
b) 470 d) 280 a) 210 3 b) 212 3 c ) 214 3 d) 216 3

12 – No paralelogramo ABCD, tem-se que BE  AD ; BE = 22 – Sejam x e y os números reais que satisfazem a igualdade
5 cm, BC = 12 cm e AE = 4 cm. A área do triângulo EDC, em
B C i (x - 2i)  (1 - yi)  (x  y) - i , onde i é a unidade imaginária.
cm 2 , é
a) 48 O módulo do número complexo z  ( x  yi) 2 é igual a
b) 30
c) 24 a) 5 c) 2 5
d) 20 b) 5 d) 2
A E D
13 – Em uma maternidade, num certo dia, três mães deram à luz. 23 – Se k 3 5 , então 15 é igual a
A 1ª teve gêmeos; a 2.ª, trigêmeos, e a 3.ª, um único filho.
k2  8
Considere, para aquele dia, o conjunto das três mães, o conjunto a) c) k2  8
dos seis bebês e as seguintes relações: 2
R1 que associa cada mãe a seu filho;
R2 que associa cada filho à sua mãe, e k2
R3 que associa cada bebê ao seu irmão. b) d) k2
É (são) função (funções)
2
a) somente R1. c) somente R3.
b) somente R2. d) R1, R2 e R3.
24 – Carla vendeu dois aparelhos de TV por preços iguais. Um
deles foi vendido com lucro de 20% e o outro com prejuízo de 33 – Num triângulo ABC têm-se AB  2 cm , BÂC  30 º e
20% sobre o preço de custo. No total, em relação ao capital
investido, Carla AĈB  45º . A área do triângulo ABC, em cm2, vale
a) perdeu 4% c) lucrou 8%
b) lucrou 6% d) não lucrou nem perdeu
1 3 2  3
a) c)
2 2
25 – Vítor tem mais de 80 discos. Quando ele forma pilhas com 2
discos, sobra 1 disco; quando ele forma pilhas com 3 ou 4 discos, 2 3 2 (1  3 )
b) d)
também sobra 1 disco; quando ele forma pilhas com 7 discos, 4 4
não sobram discos.
34 – No plano cartesiano, os pontos A ( 1 , 0 ) e B ( 0 , 2 ) são de
O menor número de discos que ele poderá ter é um número uma mesma circunferência. Se o centro dessa circunferência é
a) divisível por 5 ponto da reta y = 3 – x, então suas coordenadas são
b) múltiplo de 6
c) maior que 130 3 1 3 3
a)  ,  b) ( 1, 2 ) c )  ,  d) ( 0 , 3 )
d) menor que 120 2 2 2 2
26 – Duas pessoas A e B fundaram uma sociedade. Três meses
depois, admitiram outro sócio C. Sete meses depois da entrada 35 – A solução da inequação x 2x – 1 < x 3, sendo x > 0 e x 1, é o
de C, aceitaram outro sócio D. Essas 4 pessoas entraram para a conjunto S = { x  / ................. }. Assinale a alternativa que
sociedade com capitais iguais. Dois anos após a fundação da completa corretamente os pontilhados:
sociedade, foi verificado um lucro de R$ 227.835,00. Se este a) x < 2 c) 0 < x < 2
lucro foi dividido entre os sócios, proporcionalmente ao tempo de b) x > 2 d) 1 < x < 2
participação de cada um na sociedade, a parte que coube ao
sócio D foi: 36 – A soma das medidas dos ângulos internos A, B, C, D e E da
figura é A
a) R$ 57.645,00 c) R$ 21.352,00
b) R$ 38.430,00 d) R$ 18.234,00 a) 120º
b) 180º E B
27 – O número de pontos de intersecção dos gráficos das c) 360º
funções definidas por f(x) = 3 log x e g(x) = log 9 + log x, sendo d) 540º
x > 0, é
a) 0 c) 2
b) 1 d) 3

28 – A distância do centro da circunferência D C


x  y  6x  8y  21  0
2 2
à bissetriz do II.º e IV.º
 5 0  3
quadrantes, vale  
37 – Dadas as matrizes A = 1  2 1  eB=
2 7  0 0  1
a) c)
2 2  
3 1  1
b) d)
7 2  
2 2 0 3  , o elemento C12 da matriz C = A . B é
 2 4 
29 – Sejam a e b dois números reais tais que 0 < b < a < .

2
Assinale a alternativa incorreta: a) –17 c) –3
b) 7 d) 3
a) 2 2
tgb tga
c) sena  senb
b) cos b  cos a d) 2cotgb > 2cotga 38 – Na figura, considere o segmento a = 2 m. A área da
superfície sombreada é, em m 2 , igual a .
30 – Um campo de futebol tem 7 entradas. O número de modos y
desse campo estar aberto pode ser expresso por a) 2
a) 27 c) 7! b) 4
b) 27 – 1 d) 7! – 1 c) 2
d) 4 a

31 – Os valores de K tais que o sistema homogêneo x


x  y  2z  0

 x  ky  z  0 admita apenas a solução trivial, são
kx  y  z  0 a

39 – Seja a função f : N  Z, definida por f(x – 1) = f(x) – 2. Se
k  0 e k  -1
f(1) = 4 , então a soma dos valores dos 50 menores elementos
a)
b) k  1 e k  -1
do conjunto Im(f) é
c) k = 0 e k = -1
a) 1.150 c) 2.150
d) k  1 e k  -2 b) 1.450 d) 2.450

40 – O n.º 1 é uma das raízes do polinômio x 3 + 4 x 2 + x – 6.


32 – Ao se efetuar a soma de 50 primeiras parcelas da P.A.: Com relação às outras raízes do polinômio, podemos afirmar
202 + 206 + 210 + . . . , por distração, não foi somada a 35.ª que
parcela. A soma encontrada foi a) ambas são negativas
a) 10.200 c) 14.662 b) uma é negativa e a outra é positiva
b) 12.585 d) 16.419 c) ambas são positivas
d) uma delas é nula
11 – O elemento X 3, 2 da matriz solução da equação matricial
2/2002-TURMA A
1 1 10 4 
A  23  32  5 , B  2 4  33
3  X  2 4   2 16 
 
01 – Sendo e
é
C  2 5  3 4 , então o quociente da divisão do m.m.c. pelo
m.d.c. dos números A, B e C é
6 8  0 8 
a) 36 b) 90 c) 180 d) 450 a) 0 b) –2 c) 3 d) 1

02 – 25 kg de linha foram usados para tecer 24 m de um tecido 12 – Observando a figura, podemos afirmar que a medida da
de 6 m de largura. O comprimento do mesmo tecido que se pode mediana AM é A(2,6)
fazer com 100 kg de linha e com largura de 9 m, em m, é
a) 32 b) 64 c) 144 d) 164 a) 2 2
b) 3 2
03 – A média aritmética, a moda e a mediana do conjunto de
valores 6; 1; 7; 3; 8; 7; 2; 10 são, respectivamente, c) 2 3 C(6,4)
a) 5 6,5 6,5 c) 5,5 6,5 7
d) 3 3 M
b) 5,5 7 7 d) 5,5 7 6,5
B(4,2)
04 – Se o logarítimo de um número na base “n” é 4 e na base “ 13 – Seja AB o diâmetro de uma circunferência. Por A traça-se
n 2 ” é 8, então esse número está no intervalo uma tangente à circunferência, que encontra o prolongamento de
uma corda MN paralela ao diâmetro, num ponto P.
a) 1, 50  c) 101, 200  Sabendo que PM mede 9 cm (M está mais próximo de P do que
b) 51, 100  d) 201, 500  N) e que o raio do círculo vale 12,5 cm , então a distância do
centro à corda MN, em cm, mede
05 – O valor da expressão a) 8 b) 10 c) 12 d) 15

144  0,6 3   1 
 2  1,5  1   é igual a x  y  1
2,4  10 4   2  
14 – Para que valor de “K” o sistema  y  3z  1 não
1 7 2 2 2x  Kz  2
a) b) c)  d) 
12 12 3 5 possui solução?
a) – 3 b) –6 c) 6 d) 3
06 – Se 8 x  9  16 x 2 , então “x” é um número múltiplo de
a) 2 b) 3 c) 5 d) 7 122 
15 – O sen é igual ao
9
07 – Se x  3, 2x  1, x  5 é uma P.A., então a soma 5 5
dos três termos dessa P.A. é a) sen c)  cos
a) – 13 b) 15 c) 19 d) 27 9 9
4 4
08 – No sistema b) sen d)  sen
9 9
m  n  4
 16 – A soma dos ângulos internos de um polígono convexo
 5  5 4  5  3 2  5 2 3  5  4 , sendo
m  1 m n   2 m n   3 m n   4 mn  n  32
5
regular é de 720o, Sabendo-se que o seu lado mede 4 cm e que
         ele está inscrito numa circunferência, então a área desse
polígono, em cm2, é
5 5 5 5
  ;   ;   e   números binomiais, então o valor de a) 6 3 c) 18 3
1  2  3  4
“m” é b) 12 3 d) 24 3
a) 4 b) 1 c) 2 d) 3
17 – Se de um retângulo de perímetro 4 e dimensões “x” e ”y”,
09 – Efetuando 3
k 2
 6k  9  4 k  3 , obtemos x  y , retira-se um quadrado de lado “x”, então a área
remanescente em função de “x” é
a) k 3 c) 12
k  35 a) 1  2x c) x  2x 2
b)
6
k 3 d) 12 k  3 b) 2x  2x 2 d) 2x  4x 2
18 – A base de um prisma regular é um hexágono inscrito num
círculo de raio R. Se o prisma é equivalente ao cubo, cuja base
10 – Que expressão podemos acrescentar a cada termo da está inscrita no mesmo círculo, então a altura do prisma
x 5x hexagonal, em cm, é
fração a fim de obtermos ?
y 4y 4R 6
a) 2R c)
xy 9 xy 3
a) c)
4 y  5x 4 y  5x b)
2R 6
d)
4R 6
xy 1 3 9
b) d)
5x  4 y 4 y  5x 19 – Os lados congruentes de um triângulo isósceles medem
12
50 cm cada. Se a medida da altura equivale da medida da
7
base, então a medida da base, em cm, é
a) 14 b) 25 c) 28 d) 50
20 – A geratriz de um cone de revolução forma com o eixo do 28 – O gráfico da função y  f x  , definida por
cone um ângulo de 45o. A área lateral, em dm 2, desse cone,
sabendo-se que a área de sua secção meridiana é 18 dm 2, é 1 1 x
a) 18 2 c) 18  3 4  1  0,
b) 9 2 d) 
18  2  1  1 2 y
a) determina, com os eixos coordenados, uma região triangular
21 – O número de anagramas formados com as letras da palavra ROMA 9
de modo que não apareçam vogais ou consoantes juntas é igual a de área .
a) 4! b) 4 c) 8 d) 2 28
3
22 – Se um cilindro reto está circunscrito a uma esfera de raio b) intercepta o eixo “x” no ponto de abscissa  .
“R”, então a razão entre a área da superfície esférica e a área 7
total do cilindro é 3
1 2 4 c) intercepta o eixo “y” no ponto de ordenada  .
a) 1 b) c) d) 2
2 3 5 d) passa pela origem do sistema cartesiano.

23 – O maior número inteiro que satisfaz a inequação a 4  b  4


2  x  1 29 – A fração é igual a
  1 
1
2x  3 é a 2  b2
3 2  2
a) a 6  b 6
a) –4 b) –3 c) –2 d) 3
b) a 2  b 2
24 – A área de um triângulo de perímetro 54m circunscrito a um
c) a 2  b 2
círculo de 25 m2, em m2, é
a) 125 b) 130 c) 135 d)140 d) a 2  b2
1
25 – A solução da inequação  cos x  1 , no intervalo 30 – Uma das raízes da equação 2x 3  x 2  7 x  6  0
2
0  x 2 , é dada por “x” real, tal que é x 1  2 . Pode-se afirmar que:
a) as outras raízes são números imaginários puros.
  5  as outras raízes são – 3 e – 2.
a) 0  x  ou  x  2 b)
 3 3  c) só uma das outras raízes é real.
d) as outras raízes estão entre – 2 e 0.
  5 
b) 0  x  ou  x  2 31 – A área da secção paralela ao eixo de um cilindro circular reto,
 3 3  de 8 m de altura e 1 m de raio, feita a 0,6 m do eixo, em m2, é
  5  a) 16,00 b) 12,80 c) 6,40 d) 8,60
c) 0  x  ou  x  2
 3 3  32 – Uma pessoa emprega R$25,00 em duas parcelas: a 1a a 3%
ao mês e a 2a a 2% ao mês e recebe anualmente R$7,20 de
  5 
0  x   x  2 
d) ou juros simples. O valor da maior parcela empregada, em R$, é
 3 3  a) 8 b) 10 c) 12 d) 15

26 – Na figura, se o ângulo  é congruente ao ângulo Ê , 33 – Um atirador deu 49 tiros, pagando R$0,10 de multa por tiro
então a relação falsa é A fora do alvo e recebendo R$0,25 de prêmio por tiro acertado no
alvo. Se nada recebeu e nada pagou, então a multa foi de R$
B ____.
a) 1,40 b) 3,50 c) 5,00 d) 8,75
F C
34 – Se em uma P.G. de três termos reais o produto e a soma
D dos termos são, respectivamente, 216 e 26, então a soma dos
dois primeiros termos dessa P.G., quando decrescente, é
E a) 24 b) 20 c) 18 d) 8
a) CA  CB  CE  CD 35 – Seja o pentágono ABCDE da figura, inscrito numa
CA  CE CD  CB
b)  circunferência de centro O. Se o ângulo AÔB  50 o , então “
CE CD x  y ” vale, em graus, E
CA  CD CD
c) 
CE  CB CB x
a) 216 A
3 D
CA  CD  DA  CD  b) 205
d)    c) 180 O
CE  CB  EB  CB  d) 105 B
y
27 – Dados os conjuntosA  1, 2, 3, 4 , B  3, 4, 5 e
C
C  1, 2, 5. Ao determinar o conjunto M, tal que:
A  M  1, 2, 3, 4 , B  M  3, 4, 5 ,
36 – Um empregado recebe uma gratificação de 4% sobre os
lucros. Em R$, quanto receberá de gratificação, se vendeu
C  M  A  B , podemos concluir que M é um conjunto R$ 67.206,00 com lucro de 20% sobre o preço de compra?
a) vazio. c) que possui dois elementos.
a) 448,04 b) 560,05 c) 268,82 d) 112,01
b) unitário. d) que possui três elementos.
37 – Tenho nove moedas numeradas de 1 a 9 inclusive. Com
elas, formo números de três algarismos. Quantos números, cuja
soma é par, podemos formar?
a) 144 b) 84 c) 104 d) 264
38 – Traçam-se duas cordas de uma mesma extremidade de um
diâmetro de um círculo. Uma delas mede 9 cm, e sua projeção 05 – Assinale a alternativa falsa.
sobre o diâmetro mede 5,4 cm. O comprimento da outra corda, a) Se dois números são primos, então eles são primos entre si.
cuja projeção no diâmetro é de 9,6 cm mede, em cm, b) Dois números primos entre si podem ser primos.
a) 10 b) 12 c) 14 d) 15 c) Um número par e outro ímpar podem ser primos entre si.
d) Se dois números são primos entre si, então eles são
39 – Dois pontos sobre a reta y  2 distam 4 unidades da reta necessariamente primos.

4x  3y  2  0 . A distância, em unidades, entre as


3x  2 y  4
abscissas dos pontos é 
06 – O sistema x  4 y  6 , nas incógnitas x e y, admite
a) 10 b) 2 c) 6 d) 4 2x  3y  m

40 – Um quadrado ABCD está inscrito num círculo com centro na uma única solução se, e somente se,
origem do plano de Gauss. O vértice “A” é imagem do complexo a) m  1 c) m  1
3  4i . Os afixos dos outros três vértices são os complexos: b) m  0 d) m  2
a)  3  4i;  3  4i; 3  4i .
07 – Um tanque cilíndrico com água tem raio da base R.
b)  4  3i;  3  4i; 4  3i . Mergulha-se nesse tanque uma esfera de aço e o nível da água
c)  4  3i;  3  4i; 3  4i . 9
sobe R . O raio da esfera é
d)  3  4i;  3  4i; 4  3i . 16
3 3
a) R c) R
2/2002-TURMA B 4 5
01 – Se a e b são dois números reais e a razão de a para b é 0,7, 9 R
pode-se afirmar sempre que b) R d)
16 2
a) a  b c) a  b
b) a  b d) a  b 08 – Dadas as afirmações:
I- Quaisquer dois ângulos opostos de um quadrilátero são
suplementares.
8 3 II- Quaisquer dois ângulos consecutivos de um paralelogramo
02 – Seja a sucessão de números racionais:  ; ; 1,2 ; são suplementares.
5 2 III- Se as diagonais de um paralelogramo são perpendiculares
2 entre si e se cruzam no seu ponto médio, então este
0,3232  ; 1,6111  ;  1 . Escrevendo-a em ordem paralelogramo é um losango.
3
decrescente, obtemos Pode-se garantir que
3 8 2 a) todas são verdadeiras.
a) 1,6111    0,3232   1,2    1 b) apenas I e II são verdadeiras.
2 5 3 c) apenas I e III são verdadeiras.
d) apenas II e III são verdadeiras.
3 2 8
b)  1,6111   0,3232   1    1,2 09 – Na figura abaixo, ABCDE é um pentágono regular. As
2 3 5 medidas dos ângulos x, y e z, em graus, são, respectivamente
A
3 2 8
c) 1,6111   0,3232    1  1,2   a) 36; 36; 72
2 3 5
b) 72; 36; 72 E B
3 2 8 x y
d)  1,6111   0,3232   1,2  1   c) 72; 36; 36
2 3 5 d) 36; 72; 36 z
03 – A altura de 80 homens de uma comunidade está distribuída D C
de acordo com a tabela. A porcentagem de homens com altura
10 – Para obter-se um total de R$ 22.800,00 ao final de 1 ano e 2
maior ou igual a 1,80 m é
meses, à taxa de 12% ao ano, a juros simples, é necessário que
se aplique
altura (m) número de homens a) R$ 10.000,00
1,60 |–– 1,65 04 b) R$ 12.000,00
1,65 |–– 1,70 12 c) R$ 15.000,00
a) 25
d) R$ 20.000,00
% 1,70 |–– 1,75 18
b) 30%
1,75 |–– 1,80 26 11 – Os valores de x que tornam verdadeira a igualdade
c) 60%
d) 75% 1,80 |–– 1,85 10 x 0 2
1,85 |–– 1,90 08  1  1 1  2 são tais que seu produto p é elemento do
1,90 |–– 1,95 02
Total 80 3 1 x
conjunto
04 – Por 24 operários que trabalhavam 7 horas por dia, foram a) p   / p  3 c) p   / p  6

feitos
2
de um trabalho em 10 dias. Com a dispensa de 4
b) p   /  3  p  2 d) p   /  6  p  2
12 – A equação x  10 x 2  2x  20  0 tem como
5 3
operários e considerando-se que os restantes trabalham agora 6 raízes a, b e c. Então, o valor da expressão
horas por dia, nas mesmas condições, o número de dias em que
o trabalho será concluído é a bc  ab c  abc é
2 2 2

a) 18 b) 19 c) 20 d) 21
a) 100 b) 250 c) – 200 d) – 400
13 – O par x, y  , solução da equação matricial 21 – De acordo com os dados da figura, a distância aproximada,
em metros, entre os pontos A e B é
 x  4  x 2  13 2x  4  C
 2       3  é
x y   y 1 x  y
2
8  a) 100
B
80m 105o
6,  3    b) 102
a) c)   1 ,5  c) 104
 2  d) 108 30o
 

 
A
 7 4
b) 5 ,2 d)  ,  22 – Quaisquer que sejam o racional x e o irracional y, pode-se
 3 5 dizer que
a) x  y é irracional.
14 – É verdadeira a afirmação: b) yy é racional.
A equação x  13 x  36  0
8 4

a) admite 4 raízes reais irracionais. c) x  y  2 é irracional.


b) admite 4 raízes reais racionais positivas. d) x  2y é irracional.
c) não admite raízes reais.
d) admite 4 raízes reais inteiras.
23 – ANULADA
15 – Seja Z um número complexo, cujo módulo é 2 e cujo
 24 – O gráfico de uma função f é o segmento de reta que une os
argumento é . A forma algébrica do conjugado de Z é
3
pontos  3,4 e 3,0 . Se f 1 é a função inversa de f, então
1 3i 3 i
f 1 2
a) c)
é
b) 3 i d) 1 3i 3 3
a) 2 b) 0 c)  d)
16 – Sabe-se que a seqüência x ; y ; 10  é uma P.A. e a
2 2
25 – Na figura abaixo, a curva representa o gráfico da função
1 
seqüência  ; 2 ; 3x  4  é uma P.G. Nessas condições, é y  log x , para x  0 . Assim, a soma das áreas das
y  regiões hachuradas é igual a
correto afirmar que y
a) a razão da P.A. é 2.
b) a razão da P.G. é 26. S2
a) log 2 S1
c) x  y  0.
b) log 3
d) x  y  16 .
c) log 4
17 – A fórmula que define a função quadrática, cuja d) log 6
representação gráfica é uma parábola, cuja concavidade é 1 2 3 4 x
voltada para baixo e que não intercepta o eixo das abscissas, é
a) y = – x2 – 2x – 1
b) y = – 5x + x2 + 7
c) y = 3x – 2x2 – 2
d) y = – 6 – x2 – 5x

26 – Se  é um ângulo tal que 0    e o dobro do seu
12 2
x  5 
18 – Seja f x   x  1 . O domínio de f é seno é igual ao triplo do quadrado da sua tangente, então o valor
do seu cosseno é
x  9 5
 3 2
x 1 x a) c)
a)    0,1 c) * 3 2

  1,5 *  1,  1,5


3 2
b) d) b) d)
2 3
19 – Sejam: AB o diâmetro de uma circunferência de centro O; Px  e Qx  .
27 – O gráfico abaixo representa as funções reais
uma corda, tal que BÂR  20 ; t, paralela a AR , uma Então, no intervalo  4, 8 , Px   Qx   0 para todo
o
AR
reta tangente à circunferência, em T. Sabendo que T e R são
x   tal que
pontos da mesma semicircunferência em relação a AB , a
medida, em graus, do ângulo agudo formado pela reta t e pela y
corda AT é igual a P
a) 25 c) 50
b) 35 d) 70

20 – Dois números, x e y, estão relacionados da seguinte forma: x


"a cada número x corresponde um único número y, que é o dobro -4 -3 -2 -1 0 1 2 3 4 5 6 7 8
do quadrado de x menos 8 unidades". Nessas condições, é falso
afirmar que
a) y é função de x.
b) x é função de y. a) 2  x  4 Q
c) se x  13 , y  18 . b) 2  x  1 ou 5  x  8
c) 4  x  2 ou 2  x  4
d) se y  32 , x   2 5 . d) 1  x  5
28 – Coloque V ou F conforme as afirmações sejam verdadeiras 35 – Feito o levantamento de um terreno pentagonal, foram
ou falsas: determinados os dados indicados na figura a seguir. A área do
( ) Dois ângulos adjacentes são suplementares. terreno, em m2, é
( ) Dois ângulos que têm o mesmo complemento são congruentes. B
( ) Dois ângulos suplementares são adjacentes.
( ) Um triângulo obtusângulo pode ser isósceles. a) 450
 
30m
( ) Um triângulo retângulo é escaleno.
b) 450 4 3  1 o 30o
A
60  C
Assinale a seqüência correta. c) 900
900 3 3  2
a) F – V – F – V – V c) F–V–F–V–F 45o
b) F – V – V – V – F d) F–F–V–V–F d)
30m
A  1, 2, 3 , B  a, e, i, o, u e a função
29 – Sejam:
f : A  B . O número de funções injetoras definidas em f é 
igual a
E D
a) 10 c) 60
b) 15 d) 75 36 – Classifique em Verdadeiro (V) ou Falso (F):
( ) Z+  N
30 – Assinale V (verdadeiro) ou F (falso), considerando a ( ) Z+  N
geometria de posição espacial e plana.
( ) Z – Z - = *
( ) A condição r  s   é necessário para que as retas r e
( ) ( Z+  Z - )  N* = N
s sejam paralelas distintas. ( ) Z – Z+ = Z -
( ) Duas retas que formam um ângulo reto são
necessariamente perpendiculares. Assinale a seqüência correta:
( ) Se duas retas têm um único ponto em comum, então elas
são concorrentes. a) F–F–V–V–F
( ) A condição r  s   é suficiente para que as retas r e s b) F–F–V–V–V
sejam reversas. c) V–F–V–F–F
A seqüência correta é: d) V–F–V–V–F
a) V – V – V – V c) F–V–F–V
b) V – F – V – F d) F–F–F–F 37 – Num triângulo ABC retângulo em A, o cateto AC mede
1,5 cm e a altura traçada sobre a hipotenusa determina o
31 – Um imóvel foi comprado e revendido com um lucro de 8% segmento HB que mede 1,6 cm. O valor da secante do ângulo
sobre o preço de venda. Sabendo que, se o lucro fosse interno C é
aumentado de R$700,00, ele teria sido igual a 9% do preço de 4 4
compra, esse lucro foi de a) c)
a) R$ 10.000,00 c) R$ 20.000,00 3 5
b) R$ 14.000,00 d) R$ 32.000,00 5 5
b) d)
2
4 3
32 – Os valores de x para os quais (0,8) 4x x
 (0,8) 3( x 1)
 3 1
são
 m  
10
38 – No desenvolvimento de , o coeficiente de
3 1 3 1  m
a)   x  c) x   ou x 
2 2 2 2 m6 é
a) 45 c) 210
1 3 1 3
b)   x  d) x   ou x  b) 120 d) 245
2 2 2 2
3x
33 – Dada a figura abaixo, se AB  8 cm , CD  4 cm e
39 – Dadas a reta de equação y  e a circunferência
3
AD  20 cm , a medida, em cm, de x é de equação x 2  y 2  4x  0 . A área do triângulo
determinado pelo centro da circunferência e os pontos de
6
a) intersecção entre a reta e ela, em unidades de área, é igual a
6 D
6 E a) 3 c) 3 3
b)
2 b) 3 d) 6

2 6 x
c)
3  
C B A
3 6
d)
2
34 – O maior e o menor lado de um triângulo medem, respecti-
vamente, 10 cm e 3 cm e formam entre si um ângulo de 45o. O
volume do sólido gerado pela rotação de 360o desse triângulo em
torno do seu lado maior é, em cm3,
a) 30  c) 15 
b) 20  d) 10 
40 – A figura abaixo é a planificação de um poliedro convexo 07 – O volume, em cm3, de um prisma hexagonal regular com
 
A  B  C  D ; E  F . O volume desse poliedro, em altura igual a 5 cm e com área lateral 60 cm2 , é
unidades de volume, é
a) 5 3 b) 45 3 c) 30 3 d) 270 3
C
B 5 2 08 – Em uma circunferência estão inscritos um triângulo
5 2 5 2 eqüilátero e um hexágono regular. O apótema do triângulo
 
5 2 D
5 2 somado com o apótema do hexágono dá 12 3  1 cm. O lado
5 2 do triângulo, em cm, mede
5 2
A 5 2 a) 12 3 b) 16 3 c) 20 3 d) 24 3
5 2
5 2
F
5 2 09 – Se M  log 2 32  log1 3 3  log 2 8 , então M vale
5 2 13
13 a) 1 b) 1 c) 2 d) 2
E 13
13 10 – Resolvendo a inequação 2x  64x  8  0 , para
13
x  R , obtemos
O a) 2  x  3 c) 6  x  1
b) 2  x  3 d) 6  x  1
425 850
a) c)
2 3 11 – “N” é o conjunto dos números naturais, K  3x | x  N ,
L  5x | x  N e M  15x | x  N . A afirmativa correta é
425 850
b) d)
3 2
a) K L  M
1/2003-TURMA A b) KL
c) KL  M
01 – Nesta figura, as retas r e s são paralelas entre si. Os valores d) K L  M
de “x”, “y” e “z” são, respectivamente,
12 – Na figura, os triângulos ABC e EDC são semelhantes.
a) 23o45’, 85o e 95o. y r Sabendo que AC  x  5 e DE  2x  4 , a soma “
b) 25o, 90o e 90o. x
c) 23o7’5’’, 95o e 85o. med(AC)  med(CE) ”, em cm, vale
d) 26o15’, 85o e 95o. a) 10,3
3x-10o b) 18 6cm
s c) 13 A B
d) 23,3
85o z C 10cm
02 – O valor da expressão D E

0,7  2,5  0,5  0,3 0,35  0,25  é 13 – Os números complexos que correspondem aos pontos A e
B do gráfico são, respectivamente,
   
a) 1 b) 2 c) 3 d) 4
a) 1  3i ;  3  2i y
03 – Se em uma circunferência uma corda mede 16 2 cm e
b) 3  i  ;  2  3i  3 A
dista 6 2 cm do centro, então a medida do raio dessa
circunferência, em cm, é
c)  3  2i  ; 1  3i  2

a) 12 2 b) 10 2 c) 8 2 d) 6 2
d)  2  3i  ; 3  i  -3 -2 -1
-1 1 2 3 x
04 – Certa obra deveria ser feita em 80 dias por 60 operários. B -2
Após 26 dias, 24 operários foram dispensados. Em quantos dias -3
os outros operários farão o restante da obra?
a) 90 b) 80 c) 76 d) 60
14 – Fatorando a expressão 5a 2  30 ab  45 b 2 , obtemos
05 – De acordo com os dados nos triângulos retângulos CAB e
CAD, é correto afirmar que C a) 5a  3b5a  3b c) 5a  3b2
a) x  y
b) x  3y
b) 5a  3b2 d) 5a  3b2

c) x  2y 15 – ANULADA
3y 30o D 60o  16 – Se um cubo está inscrito em uma esfera de 3 m de raio,
d) x B A
2 x y então o volume do cubo, em m3, é igual a
a) 8 b) 27 c) 12 3 d) 24 3
06 – O produto tgx   sen 2x  é igual a

a) sen 2 x c) 2 sen 2 x  2 x   4   7
17 – Sendo   .      , os valores de x e y na
b) cos 2 x d) 2 cos 2 x  y  3   5   3 
matriz acima são, respectivamente,
9 9
a) 3 e –3 b) –3 e 3 c) e –3 d) –3 e
2 2
18 – Se “1”, “x2” e “x3” são as raízes da equação 28 – As medidas dos lados de um triângulo são iguais a 4 cm, 5 cm
e 6 cm. O cosseno do menor ângulo desse triângulo é igual a
x 3  2x 2  5x  6  0 , então o valor de “ x 2  x 3 ”, para
1 9 3 2
x2  x3 , é a) b) c) d)
8 16 4 5
a) 3 b) 1 c) 6 d) 5
29 – Em um losango, uma diagonal forma um ângulo de 58o com
19 – Na figura, “ x  y ” é igual a
um de seus lados. A medida do menor ângulo desse losango é
a) 15o a) 58º b) 64º c) 116º d) 122º
b) 20o 
c) 30o 5
30 – Duas firmas vendem juntas 1.720 parafusos diariamente.
d) 35o Quanto vende cada firma, se uma delas vende 15% mais que a
x y outra?
a) 900 e 1.035 c) 820 e 943
2 5 b) 840 e 966 d) 800 e 920
20 – Do ponto P, situado a 10 cm do centro O de uma
circunferência de raio igual a 8 cm, traça-se uma secante PB 31 – A área do trapézio retângulo (fig. abaixo), em cm 2, é igual a
passando por A tal que PA = AB, sendo A e B pontos da
circunferência. A medida de PB, em cm, é (Obs: Utilize 3  1,7 )
a) 20,00 3,2 cm
a) 3 2 b) 6 2 c) 8 d) 6
b) 26,40 8 cm
c) 34,68
21 – As mediatrizes de dois lados consecutivos de um polígono d) 40,80
regular formam um ângulo de 24º. O número de diagonais desse 30o
polígono é
a) 70 b) 80 c) 90 d) 100
32 – Sendo AD a bissetriz do ângulo BÂC do triângulo ABC,
22 – Sendo C(3, –2) o centro de uma circunferência de raio igual a relação verdadeira é
a 4, então sua equação normal ou geral é
a)     B̂  Ĉ
a) x 2  y 2  6x  4 y  3  0 A
b)     Ĉ  B̂
b) x 2  y 2  6x  4 y  3  0
c)     B̂  Ĉ
c) x 2  y 2  6x  4 y  3  0  
d)     B̂  Ĉ B̂ Ĉ
d) x 2  y2  3  0 B C
D
23 – Se em uma pirâmide quadrangular regular a diagonal da
base mede 4 m e a aresta lateral mede 2,5 m, então o volume da 33 – Um copo cheio de água pesa 345gf e o copo com 2 da
5
pirâmide, em m3, é 1
a) 1 b) 2 c) 3 d) 4 água total pesa 210gf. Quanto pesará, em gf, o copo com da
3
água total?
24 – Na figura, as cordas AB e CD são paralelas. EC é um a) 120 b) 225 c) 195 d) 300
34 – O gráfico abaixo representa a função y  log a x . Dentro
diâmetro e P é o ponto médio da corda AD . As medidas, em graus,
 
das condições de existência para que a operação de loga-
dos ângulos AR̂C e PÂR são, respectivamente, 4x  14 o e ritimação seja sempre possível e de resultado único, a base “a” é

5x 13 
y
o
. As medidas dos ângulos do triângulo PCD são a) 0  a 1 3
E b) a0 2
B
a) 42o, 57o, 81o c) a 1 1 1 2 3 4
b) 45o, 45o, 90o
c) 46o, 45o, 90o
R
O d) a0 -1 x
y  log a x
d) 52o, 38o, 90o
A  D
-2
P -3
C 35 – Um número, seu logarítimo 2 e a base do logarítimo for-
25 – Se um ponto P do eixo das abscissas é eqüidistante dos mam, nessa ordem, uma P.A. Esse número é
   
pontos A 1,4 e B  6,3 , então a abscissa do ponto P é
a)
9  17
c)
 1 17
a) 1 b) 0 c) 2 d) 1 2 2
26 – A função do 2o grau que descreve o gráfico abaixo é
9  17  1 17
b) d)
a)  
f x  x2  x  6 f(x) 2 2

f x   x 2  5x  6
6 36 – As atuais placas de automóveis possuem três letras do alfa-
b)
beto latino (incluindo K, W, Y) e quatro algarismos. O número de
c) f x    x 2  5x  6 placas que não repetem nem letras e nem algarismos é
26! 10!
d) f x   x 2  5x  6 x a)
23! 6!
c) 26! 10!

2 3 26! 10!
27 – A área lateral do sólido geométrico formado pela rotação de b) 26 3  10 4 d)
4! 3!
um triângulo equilátero, de perímetro 30 cm, em torno de um de
seus lados é, em cm2, igual a
a) 100  b) 200  c) 50 3 d) 100 3 37 – Se 0,0625 x2  0,25 , então x  16 vale
3 1 1
a)  b) c) 64 d)
2 32 64
x x
1 1
38 – No triângulo retângulo ABC, a mediana AM forma com a III -    
 
2 3
   
bissetriz BF o ângulo BF̂M . O valor de BF̂M é
3 IV - log 2 3 x 2  1  log 2 3  log 2 x 2  1
a) B̂ A
2 
a) 1 b) 2 c) 3 d) 4
5
b) B̂ F
2 05 – Na figura abaixo, AB  8 cm, BC  10 cm, AD  4 cm e
B̂ o ponto O é o centro da circunferência. O perímetro do triângulo
c) AOC é, em cm,
2 C B
M
d) B̂ a) 45
b) 48
E O D A
39 – O ponto M é o ponto de intersecção das diagonais AC e c) 50  
d) 54
BD de um quadrilátero ABCD. Sendo A0,0  , B3,0 , B
C4,2  e D0,5 as coordenadas dos vértices do quadrilátero,
C
as coordenadas do ponto M são
06 – Observando as figuras abaixo, o valor, em graus, de xy é
 15 30   30 15 
a)  ,  c)  ,  s
 13 13   13 13  m y
65o 25o 150o
 180 90   30 15  30 o
x 40 o
b)  ,  d)  , 
 13 13   7 7
t
r//s
40 – Na figura abaixo, AB e MN são diâmetros perpendiculares m//t
de um círculo de raio 2 cm. Traça-se o arco MPN de centro A e
r
raio AM. A área da região tracejada, em cm2, é N a) 25 b) 20 c) 15 d) 10
a) 2
b) 4 07 – Uma classe tem 10 meninos e 9 meninas. Seu professor
P
c) 2 A  B necessita formar comissões de 7 crianças, sendo 4 meninos e 3
meninas, que incluam obrigatoriamente o melhor aluno dentre os
d) 4 meninos e a melhor aluna dentre as meninas. O número possível
M de comissões é
a) igual a 2300. c) maior que 2400.
1/2003-TURMA B b) menor que 2300. d) igual a 2352.

01 – Leia as sentenças abaixo. 08 – A soma dos 9 primeiros termos de uma P.A. de razão 2 é
I - Todo número natural que termina em 3 é divisível por 3. nula. Assim, pode-se afirmar que seu sexto termo é igual a
II - Todo número natural divisível por 2 é também divisível por 4. a) 0 b) 2 c) 6 d) 7
III - Existem números naturais terminados em 2 que são
divisíveis por 4. 09 – Uma caixa d’água tem a forma de paralelepípedo reto-
IV - Todo número natural divisível por 10 é também divisível por retângulo, cujas medidas internas são, em m, “x”, “ 20  x ” e “2”.
2 e 5. O maior volume, em m3, que ela poderá conter é igual a
V - Existem números naturais terminados em 4 que são a) 150 b) 200 c) 220 d) 250
divisíveis por 3.
VI - Existem números naturais divisíveis por 6 que não são 10 – Um tanque tem três torneiras. A 1a enche o tanque em 25
divisíveis por 2. horas; a 2a, em 40 horas; já a 3a, o esvazia em 20 horas. O
tanque está com 1 4 de água. Abrindo-se simultaneamente as
Está correto o que se afirma em
a) I e II apenas. três torneiras, ele ficará cheio em
b) I, II e III apenas. a) 55 h 40 min. c) 52 h.
c) III, IV e V apenas. b) 53 h 12 min. d) 50 h.
d) I, II, III, IV, V e VI.
11 – A solução da equação 1 x  x2  x3  x4    2 é
02 – Em um triângulo ABC, o lado AB mede 6 3 cm e o ângulo 3
a) c) 1
Ĉ , oposto ao lado AB, mede 60 . O raio da circunferência que
º 2
circunscreve o triângulo, em cm, mede 1
b) d) indeterminada
a) 6 c) 6 3 2
b) 12 d) 3 6 12 – Seja a função inversível f de gráfico abaixo. A lei que define
f 1 é
03 – O valor da raiz da equação 2 x 1  2 x 1  40 é um 3
número a) y  3x  y
a) inteiro positivo. c) inteiro negativo.
2
b) irracional. d) imaginário puro. 3
b) y  2x  4
2
04 – Das sentenças abaixo, quantas são verdadeiras de modo
que são satisfeitas por qualquer número real “x”? 2x
c) y 2 2
I- x  4 2
 x  16
2 3
3x
II - 8  2  4x
x d) y 3 3 x
2
13 – Se o resto da divisão de Px   x 3  mx 2  nx  5 por Se a  log 2 1024 e x0 = a – 6, então o valor da função no ponto
x0 é dado por
x  2 é 15, então o valor de 2m  n é a) 2/3 b) 3/2 c) 2 d) 3
a) 1 b) 2 c) 3 d) 5

14 – No lançamento simultâneo de dois dados perfeitos, a


25 – Uma corda é determinada pela reta xy0 sobre a
probabilidade de obter soma diferente de 11 é, aproximada-
mente,
circunferência x  2 2
 y  2  16 . A área da menor
2

a) 5,5% b) 94,4% c) 83,4% d) 16,6% região determinada por essa corda e o círculo é:
a) 4  8 c) 4  2
15 – A geratriz de um cone de revolução mede 6 cm e o ângulo b) 4  16 d) 4  4
da geratriz com a altura do cone é de 30º. O volume desse cone,
em cm3, é
26 – Sendo a  b  30 o , calculando
a) 9 b) 3 3 c) 9 3 d) 27 3
y  sen a  cos b2  sen b  cos a 2 , obtemos
16 – A expressão trigonométrica cos x  sen
2 2 a) 1 c) 3
x é igual a:
a) 1 para todo número real “x”. 2 3
b) –1 para todo número real “x”. b) d) 2
3 2
c) 2 cos 2 x  1 , para todo número real “x”.
27 – Se o apótema de um tetraedro regular mede 5 3 cm,
4
d) para alguns números reais de “x”. então, a altura desse tetraedro, em cm, é
3
10 6
a) 5 3 c)
17 – O menor valor inteiro positivo que pertence ao conjunto-
  
3
solução da inequação  3x  12 x  6x  8  0 é o
2 2
10 3
a) 2 b) 3 c) 4 d) 5 b) 10 2 d)
3
18 – Sendo i a unidade imaginária, a potência de

 
28 – ANULADA
  
3
1  i 2  1  i 2 é igual a 29 – Uma pessoa aplica um certo capital a juros simples, a 4% ao
a) 64 b)  64 c) 64 i d)  64 i ano. No fim de três anos, reaplica o montante a juros simples, à
taxa de 5% ao ano e, ao final de 2 anos, consegue um novo
montante de R$ 6.160,00. O capital inicial, em reais, era de
19 – O maior valor inteiro de k para que a equação
x  y  4x  6y  k  0
2 2
represente uma circunferência é a) 5.000 b) 5.160 c) 5.500 d) 6.000
a) 14 b) 13 c) 12 d) 10
20 – A figura representa um trapézio retângulo com AB  AD , 30 – Seja o triângulo ABC e D um ponto do lado AC . Se
base menor igual a 3 cm e BC é lado de um quadrado. A área
desse quadrado, em cm2, é AD  2 cm, AB  3 cm, BD  DC e BÂC  30 o , a
a) 9 D C
b) 18 medida, em cm, do lado BC é igual a
c) 24
d) 36 135o a) 3 b) 5 c) 6 d) 7
A B 31 – A tabela abaixo indica o número de gols de 50 artilheiros de
um campeonato de futebol. É falsa a afirmação:
3 0   2 1
21 – Dadas as matrizes A  e B  , então
1  4   1 0
No de gols No de artilheiros
1 5
A  B  B A é igual a: 3 7
0 0  2  3  1 7   3 1  4 10
a) 0 0  b) 5 0  c)  9 1  d)  2 7 5 8
        6 7
8 6
9 4
22 – Seja o triângulo PMN de lados PM  6 cm , MN  8 cm 10 3
e PN  10 cm . Unindo-se os pontos médios de seus três lados
a) a moda dessa distribuição é 4.
obtemos o triângulo ABC. A área, em cm 2, do triângulo ABC é b) o número de gols marcados é 46.
a) 4 b) 6 c) 12 d) 20 c) a média de gols dos artilheiros é 5,24.
d) o número mediano de gols é 5.
2 A B
23 – A igualdade   ocorre quando A e B 32 – O preço de certa mercadoria aumentou em 250%. Para que
x 1 2 x 1 x 1 o preço da mercadoria volte a ser o que era antes do aumento
são, respectivamente, deve-se diminuir o novo preço em
a) 1 e 1 b) 1 e1 c) 1 e 1 d) 1 e 1 1 2 4 3
a) 74 % b) 73 % c) 72 % d) 71 %
24 – Considere a função f: definida por 7 7 7 7

2x  1, se x 1

f ( x )  0, se 1 x  3
 x2
 , se x  3
 2x  5
37 – O perímetro de um triângulo equilátero inscrito numa
33 – Considere o trapézio retângulo ABCD, onde  e D̂ são circunferência é 54 cm. A área de um quadrado inscrito nessa
retos, AB  AD , CD  7 cm e BC  AB  1 cm. Assinale a mesma circunferência é, em cm2,
afirmativa verdadeira
a) 36. b) 72. c) 216. d) 288.
1 3
a) sen C  c) sen C 
3 5 2 3 6
4 4 38 – Seja 4 x 0 = 64. O valor de x que torna verdadeira a
b) cos C  d) tgC  2 0 2
5 3 igualdade é

34 – Sendo S o conjunto-solução da equação em  a) 4. b) 5. c) – 4. d) – 5.


3x  1  3x  1 , pode-se afirmar que 39 – Na figura abaixo, os ângulos assinalados  e Ô medem,
respectivamente, 10° e 50°. Assim sendo, o valor de tgx é
1 3 1 
a) S c)  , S 1. 3.
2 5 3 a) x c)
2 3
2 1 2 
b) S d)  , S 2
3 5 7  b) . 
d) 1.
2
35 – O triângulo ABC está inscrito numa circunferência de centro A
O
O e de raio 13 cm. Sabendo que AB  10 cm , a altura AH 40 – A fração de denominador 30 que excede de
1 a fração 3 é
3 5
relativa ao lado BC mede, em cm, aproximadamente
a) 7,6 A
8 .
16 .
24 . 28 .
a) b) c) d)
b) 8,4 30 30 30 30
c) 9,23
d) 10,8 H  41 – Sejam os conjuntos A = {x   / x é múltiplo de 2},
B C
O B = {x  Z / - 2 < x  9 } e C = { x   / x  5 }. A soma dos
elementos que formam o conjunto ( A  B ) – C é

a) 9. b) 6. c) 3. d) 1.
36 – Em um trapézio, os lados paralelos medem 16 cm e 44 cm,
e os lados não-paralelos, 17 cm e 25 cm. A área do trapézio, em 42 – Dois números primos entre si têm por produto 5184. Se o
cm2, é menor deles é a maior potência inteira de 2, menor que 100,
a) 250 b) 350 c) 450 d) 550 então o maior deles é

a) uma potência de 5. c) múltiplo de 11.


37 – Se a diferença entre os quadrados das raízes da equação b) uma potência de 3. d) múltiplo de 7.
35
3x 2  7x  c  0 é , então o valor de “c” é
9 43 – Na figura, as cordas são dadas em cm. Se AI  4x 1 ,
2 2 IB  x , DI  x 1 e IC  3x , então a medida da corda AB é,
a)  b) 2 c) d) 2
em cm,
3 3
a) 9. c) 11.
38 – Um prisma reto tem base hexagonal regular e as faces b) 10. d) 19.
laterais quadradas. Sabendo-se que a área do círculo inscrito em
2
sua base é igual a 25 cm , a área total, em cm 2, desse
prisma é
a) 400 c) 
100 2  3  44 – Um retângulo tem área T. Se aumentarmos a medida da sua

b) 
100 6  3  d) 600
base em 20%, e diminuirmos a medida da sua altura em 20%,
obteremos um novo retângulo cuja área é igual a

a) T. b) 0,96 T. c) 1,04 T. d) 1,025 T.


39 – ANULADA

3
40 – Um triângulo DEF tem DÊF  38 e EF̂D  74 . O
o o
45 – A equação geral da reta de coeficiente angular e de
ângulo que a bissetriz DG forma com a altura DH mede: 2
a) 18o b) 20o c) 26o30’ d) 34o
coeficiente linear - 2 é
2/2003-TURMA A a) x + 2 y – 4 = 0. c) 3x – 2 y – 4 = 0.

36 – Na figura, r // s e t  u. O valor de a – b é b) 3x – 2 y – 2 = 0. d) 3 2 x– 2 y – 2 = 0.
a) 100°
b) 90° u 46 – Na figura, o lado do hexágono regular inscrito no círculo mede 4
c) 80° t a cm. A área da região hachurada da figura é, em cm2:
r
d) 70° a) 8 3 .
b) 4 3 .
s
c) 
8 2  3 3 . 
 2 .
b
d) 16   2
3x  my  0 3  2i  6  4i 
59 – Sendo "i" a unidade imaginária, o resultado de é
47 – Para que o sistema  tenha solução diferente  1  3i
 x  3y  0
13 39 i .
da imprópria, o valor de m deve ser a) 1 3i . c)  
5 5
a) 9. b) 0. c) 10. d) 15. 13 39 i .
b) 13  39i . d) 
1  cot g 2 x 5 5
48 – A expressão é idêntica à (ao)
1  tg2 x
a) tg x.2
b) sen2x. c) cotg2x. d) cos2x.  n , se n é par

60 – A função f:NN definida por f (n )   2 é
49 – Assinale a alternativa que complete corretamente o período. n 1
 , se n é ímpar
Júlia tem 8 filhos, resultado de 4 gestações de gêmeos. Se  2
considerarmos as idades desses filhos, poderemos afirmar que
elas formam uma série que apresenta _______ moda (s). a) bijetora. c) somente sobrejetora.
a) nenhuma c) duas b) somente injetora. d) não injetora e não sobrejetora.
b) uma d) mais de duas
n
61 – Seja nN* / n < 312. A fração irredutível , escrita na
50 – O termo geral de uma PA é an = 3n – 16. A soma de seus 10 312
primeiros termos é forma decimal, é um (a)
a) 18. b) 14. c) 5. d) – 6.
a) decimal exato. c) dízima periódica simples.
51 – No ciclo trigonométrico, a igualdade sen x   0 é b) número inteiro. d) dízima periódica composta.
verdadeira se e somente se x é um número 62 – Observe:
a) real qualquer. c) imaginário.
b) inteiro. d) irracional. I - É sempre possível construir um polígono regular de n lados,
52 – Seja uma função f do 1.º grau. Se f(-1) = 3 e f(1) = 1, então o para n  3 .
valor de f(3) é II - Triângulo é, em todos os possíveis casos, inscritível em uma
a) – 1. b) – 3. c) 0. d) 2. circunferência.

53 – Se permutarmos as letras da palavra TELHADO, quantas


III - Um ângulo central â c  de um polígono regular de n lados
começarão e acabarão por vogal? n  2 180o .
a) 720 b) 120 c) 1080 d) 2160 inscrito em uma circunferência mede â c 
n
54 – Dentro do conjunto dos números complexos, a equação IV- Sempre é possível construir uma circunferência que passa
pelos n vértices de um polígono qualquer.
x 4  x 2  2  0 tem como soluções
Quantas das assertivas acima são falsas?
a) 2 e i . c) 1 e i 2. a) 1 b) 4 c) 3 d) 2
b)  2 e i . d) 1 e i .
63 – A equação da circunferência, em que os pontos M3,2 e
55 – A solução geral da equação sen2 x – 2 senx cosx + cos2 x = 0,
N5,4 são extremos de um diâmetro, é
sendo U = , é
  . a) x 2  y2  5  0 . c) x 2  y2  2x  6y  7  0 .
a) {x   / x = + 2k, kZ}. c) {- }
4 4 b) x 2  y2  17  0 . d) x 2  y2  2x  6y  5  0 .
 
b) {x   / x = + k, kZ}. d) { }.
4 4 64 – Seja V o volume de um cubo de aresta "a". Constrói-se um
prisma quadrangular de volume V’ e de vértices nos pontos médios
das arestas das bases do cubo. O volume V’ desse prisma é igual a
 3x  1  1
  2 x  9  1
3
56 – A raiz da equação x   2 x  é uma V. V. V.
 10  6 5 a) b) V. c) d)
fração cuja diferença entre o numerador e o denominador é 2 3 4
a) 35. b) 37. c) 45. d) 47.
65 – Em um triângulo ABC, a bissetriz do ângulo A encontra BC
57 – Na figura, AB  AC , M é o ponto de encontro das em D, e a circunferência circunscrita, em E. Sendo AE  9cm
bissetrizes dos ângulos do triângulo ABC e o ângulo BM̂C é o e DE  4cm , então a medida EB , em cm, é
triplo do ângulo Â, então a medida de  é
a) 15o a) 6. b) 5. c) 2 5. d) 3 2.
b) 18o
c) 24o 66 – Se forem indicados por m, n, e p os três lados de um
d) 36o
triângulo e por M̂ , N̂ e P̂ , respectivamente, os ângulos
opostos a esses lados, então sendo conhecidos os lados m e n e
o ângulo N̂ , qual das fórmulas abaixo poderá ser utilizada para
calcular o valor do lado p ?
2
1 x
2 2 2
a) m  n  p  2np  cos M̂
58 – O conjunto solução da inequação    2 , sendo U = , é

a) {x   / x  -1 ou x  1}.
 2
b) 
n 2  m2  p2  2mp  cos M̂  P̂ 
b) [ -1 , 1 ]. c) p2  m2  n 2  2mn  cos P̂
c) .
d) . d) 
p2  m2  n 2  2mn  cos M̂  N̂ 
67 – A curva da figura representa o gráfico da função 41 – Um teste de inteligência, aplicado aos alunos das 4as séries do
y  log a x, a  1 . Dos pontos B3,0  e C9,0  saem Ensino Fundamental da Escola A, apresentou os seguintes resultados:
perpendiculares ao eixo das abscissas, as quais interceptam a Pontos n.º de alunos Pontos n.º de alunos
curva em D e E, respectivamente . Se a área do trapézio retângulo 90|–– 95 40 115|–– 120 140
BCED vale 9, a área do triângulo ABD, onde A1,0  vale 95|–– 100 60 120|–– 125 120
100|–– 105 140 125|–– 130 30
E 105|–– 110 160 130|–– 135 20
y
110|–– 115 180 135|–– 140 10
D y  log a x
A freqüência relativa da classe modal é

a) 0,2. b) 0,22. c) 0,25. d) 0,5.


A B C x
42 – Se uma das dimensões de um paralelepípedo reto-
1. 3. retângulo é 6 cm, a soma das outras duas dimensões é 25 cm e
a) b) 2. c) d) 1.
2 2 a área total é 600 cm 2, então a razão entre as duas dimensões
desconhecidas é
68 – O ponto de maior ordenada, pertencente ao gráfico da 2. 3. 1. 2.
função real definida por f  x   3  x  x  1 , é o par ordenado a) b) c) d)
3 5 2 5
m, n  . Então, " m  n " é igual a
a) 3 . b) 3. c) 5. d) 5 43 – Num triângulo ABC, o lado maior AC mede 10 cm; o lado

69 – Na progressão geométrica onde o primeiro termo é m , o 3 menor BC mede 3 cm; e o ângulo que eles formam mede 45º. O

último é  m21 e a razão é  m2 , o número de termos é volume do sólido gerado pela rotação de 360º desse triângulo em
torno do lado maior, em cm3, é
a) 8. b) 9. c) 11. d) 10.
3 2 . 5
70 – Ao dividir o polinômio "–5x2 – 3x + 2" por um polinômio "Q", a) b) 3 2 . c) . d) 15.
Ana obteve "–5" por quociente e "12x + 7" por resto. O polinômio 2 2
Q é igual a
a) x2 + 3x – 2. c) x2 – 3x + 1. 44 – A reta 3x – 2y – 5 = 0 é perpendicular à reta
b) x2 – 3x – 1. d) x2 + 3x + 1.
a) 2x – 3y = 5. c) 3x + 2y = 0.
b) 4x + 6y = 1. d) 6x – 4y = 10.
2/2003-TURMA B
x  x 60
2
x 45 – A equação
36 – Todo número real positivo pode ser escrito na forma 10 . Tendo em
0,90 x
vista que 8  10 , então o expoente x, tal que 125 = 10 , vale
a) só tem uma solução.
aproximadamente, b) tem duas soluções, tais que seu produto é = – 6.
a) 1,90. b) 2,10. c) 2,30. d) 2,50. c) tem duas soluções, tais que seu produto é = – 4.
37 – Seja  um ângulo agudo. Se somarmos a medida de um d) tem duas soluções, tais que seu produto é igual a 0.
ângulo reto à medida de  e, em seguida, subtrairmos dessa
soma a medida do suplemento de , obteremos sempre a
x4  x4
medida de um ângulo 46 – A raiz da equação  2 pertence ao
a) nulo, qualquer que seja a medida de . x4  x4
b) reto, qualquer que seja a medida de . intervalo
c) agudo, desde que 45° < med  < 90°.
d) raso, desde que med  < 45°. a) [ -3 , 4 [. c) ] 6 , 9 ].
 x x b) ] 5 , 10 [. d) [ 0 , 8 ].
38 – Se 0 < x < , então a expressão tg + cotg é
2 2 2 47 – Num retângulo ABCD, os vértices A, B, C e D são
equivalente a
consecutivos. Marcam-se na base AB , a partir de A, três
a) 2 senx. b) 2 secx. c) 2 cosx. d) 2 cossecx. pontos, E, F e G, de modo que eles assinalem, respectivamente,
1 2 3
, e da base AB . A razão entre as áreas do triângulo
39 – Na figura, as medidas dos lados AB, AC e BC são, 4 4 4
CEF e do retângulo ABCD é
respectivamente, 40 cm, 20 cm e 30 cm. A bissetriz interna desse
triângulo, relativa ao vértice A, encontra o lado oposto no ponto
P, e a bissetriz externa, relativa ao mesmo vértice, encontra o 1 1 1 1
a) b) c) d)
prolongamento do lado BC no ponto S. A medida do segmento 4 6 8 10

PS , em cm, é igual a 48 – Considere:


1 – Um triângulo isósceles PRQ, de base PQ e altura RH .
2 – Dois pontos T e S sobre RH , de tal modo que o triângulo
PTQ seja eqüilátero e o triângulo PSQ seja retângulo em S.
a) 30. b) 35. c) 40. d) 45. Considerando somente os ângulos internos dos triângulos, se
40 – No emplacamento de automóveis da cidade paulista X, são somarmos as medidas de R̂ e Ŝ , obteremos o dobro da medida
usadas duas letras do alfabeto seguidas de quatro algarismos. O
número de placas, começadas pela letra "A", seguida de vogal, de T̂ . Sendo assim, a medida do ângulo TP̂R é
inclusive "A", e de quatro algarismos distintos, sendo dois (2) o
último algarismo, é a) 5º. b) 15º. c) 30º. d) 45º.

a) 2.520. b) 720. c) 160. d) 3.600.


49 – A, B e P são pontos distintos de uma circunferência de
 22  10% de 7,5  0,666...
centro O e raio r. Se AB é diâmetro da circunferência, e a 59 – O resultado da expressão é
1
1
medida do ângulo PÂB , em radianos, é , então a área da 3
a) – 0,5 b) – 0,25 c) 0,75 d) 0,333...
região limitada pelo ângulo PÂB e o arco PB é igual a
 sen   .  sen 2  . 60 – Se x   e f(x) é uma função tal que f(p+q) = f(p) . f(q) e
a) r    r  c) r    r 
 2   2  f(2)= 2, então f(0) e f(– 2) são, respectivamente,

2 sen   . 2 sen 2  .
b) r     d) r     1 1
d) 1 e – 4
 2   2  a) 1 e
2
b) 0 e
2
c) 1 e 0

50 – Os elementos de um conjunto A são tais que 10 deles são 61 – Se m = 22. 3a. 52. 73 e n = 23. 35. 53. 7b. 11, e mdc (m,n) =
múltiplos de 4; 9 são múltiplos de 6; 8 são múltiplos de 12; e 4 18.900, então os valores de a e b são, respectivamente,
são números ímpares. Se A   (N = conjunto dos números
naturais), então o número de elementos de A é a) 3 e 1 b) 2 e 3 c) 3 e 2 d) 2 e 2
a) 31. b) 25. c) 21. d) 15.
62 – Seja f :    uma função. O conjunto dos pontos de
51 – Um triângulo escaleno está inscrito num semicírculo de 10 intersecção do gráfico de f com uma reta vertical
cm de diâmetro, que é o maior lado do triângulo. Se as medidas
dos lados menores do triângulo são tais que uma é o dobro da a) é não enumerável.
outra, então a diferença entre as áreas do semicírculo e do b) possui um só elemento.
triângulo, em cm2, é c) possui exatamente dois elementos.
25   40 25   20 d) possui, pelo menos, dois elementos.
a) c)
2 2 63 – Se os números 3, x e 10 são inversamente proporcionais
25   30 25   50 aos números 5, 25 e y, então os valores de x e y estão
b) d) compreendidos entre
2 2
a) 0 e 1 b) 1 e 2 c) 1 e 3 d) 0 e 2
52 – Seja P o conjunto dos retângulos, Q o conjunto dos
quadrados e L o conjunto dos losangos. É correto afirmar que
64 – Um arco mede 0,105 rd. Sua medida em graus é,
a) L  P = L - P c) L  Q = P
aproximadamente, igual a
b) L  Q = L - Q d) L  P = Q a) 5 b) 6 c) 50 d) 60
53 – Num quadrilátero convexo, a soma de dois ângulos internos
65 – É par a função f :    definida por
*
consecutivos é 190o. O maior dos ângulos formados pelas
bissetrizes internas dos outros dois ângulos desse quadrilátero
a) f (x)  1 b) f ( x )  1 c) f ( x)  x d) f ( x )  x 5
mede
a) 105º b) 100º c) 95º d) 85º x2 x

66 – Se 20 cavalos consomem 30 toneladas de feno em 45 dias,


54 – Um barril, cuja forma é a de um cilindro reto, está repleto de
então, durante quantos dias se podem alimentar 15 cavalos, com
vinho. Este vinho deve ser distribuído em copos cilíndricos de
40% menos toneladas de feno, dando a mesma quantidade de
altura igual a 1/8 da altura do barril, e de diâmetro da base igual a
feno por dia?
1/5 do diâmetro da base do barril. A quantidade de copos
necessária para distribuir todo o vinho é
a) 24 b) 36 c) 40 d) 42
a) 400 b) 300 c) 200 d) 100
67 – Numa circunferência de centro C e raio 20 cm, considere a
1 1 0 0 corda AB , cujo ponto médio é M. Se CM = 10 cm, então a
2 3 0 1 medida de AB é, em cm,
55 – Calculando o valor do determinante ,
 2 1 0 0 a) 15 5 b) 20 3 c) 15 d) 20
0 0 1 1
68 – Um certo jogo é composto de fichas de 5 cores diferentes. Se
obtém-se cada ficha vermelha vale tanto quanto 10 fichas azuis; cada azul,
a) – 3. b) – 1. c) 1. d) 3. tanto quanto 10 verdes; cada verde, tanto quanto 10 pretas, e cada
preta, tanto quanto 10 brancas, então é correto afirmar que
56 – Uma das raízes da equação x 2  (2tg a ) x  1  0 é,
a) a ficha verde é a de menor valor.
 b) a ficha branca é a de maior valor.
sendo a    k, k  Z , c) cada ficha azul vale tanto quanto 100 pretas.
2
a) tg a + cossec a. c) tg a + sen a. d) cada ficha verde vale tanto quanto 1000 brancas.
b) tg a – cos a. d) tg a – sec a.
69 – No ciclo trigonométrico:
57 – A divisão do polinômio P(x) por "x-a" fornece o quociente q(x) = 11
x3 + x2 + x + 1 e resto 1. Sabendo que P(0) = –15, o valor de a é I- o arco rad pertence ao 2o quadrante.
4
II - o arco 1510o pertence ao 3o quadrante.
a) – 16 b) – 13 c) 13 d) 16
 13  
III - o arco  rad  pertence ao 4o quadrante.
58 – A soma 1  2  2 2  23  ...  2999  21000 é igual a  3 
a) 21000 1 c) 21000 1 A(s) assertiva(s) correta(s) é(são):

b) 21001 1 d) 21001 1 a) II. b) I e II. c) I e III. d) I, II e III.


1 i RASCUNHO DO ALUNO
70 – Sendo um número complexo, seu conjugado vale
i
1 i 1 i i
a) b)  c) 1 i d)
i i 1 i
GABARITO DAS PROVAS DE MATEMÁTICA
1/2001 - TURMA A 2/2001 - TURMA B 21 A 19 B
01 B 01 C 20 D
02 C 02 A 21 D
22 D
03 D 03 D 22 B
04 A 04 C 23 A
05 D 05 B 23 D 24 B
06 C 06 C 25 B
07 D 07 B 26 B
08 B 08 A 24 D 27 D
09 C 09 C 28 A
10 B 10 A 25 C 29 C
11 C 11 D 30 D
12 B 12 B 31 B
13 A 13 D 26 D 32 C
14 B 14 B 33 B
15 A 15 C 27 B 34 A
16 B 16 B 35 C
17 D 17 A 36 A
18 A 18 B 28 D 37 A
19 ANULADA 19 D 38 C
20 B 20 A 29 B 39 A
21 D 21 D 40 C
22 A 22 B
23 C 23 D 30 D
24 A 24 A
25 C 25 D 31 A

1/2001 - TURMA B
01 C 32 B
02 D 1/2002 - TURMA A
03 A 33 A
04 C
01 A
05 B
06 D 34 B
07 A 02 B
08 C 35 C
09 A
10 D 03 D
11 B 36 B
12 C 04 C
13 C 37 A
14 A
15 C 05 A
16 D 38 A
17 C 06 A
18 A 39 D
19 B
20 D 07 D
21 B 40 B
22 A 08 D
23 B
24 D
25 B 09 C

2/2001 - TURMA A 10 C
01 A
02 D
03 A 11 A
04 C
05 A 12 B
06 D
07 B
13 A 1/2002 - TURMA B
08 C 01 C 2/2002 – TURMA A
09 B 02 D 01 C
10 D 14 B 02 B
03 C
11 A 04 D 03 D
12 B 05 C 04 D
13 A 15 A 05 A
06 D
14 C 07 B 06 B
15 B 16 D 08 C 07 D
16 D 08 D
09 B
17 B 10 C 09 C
18 C 17 A 10 A
11 B
19 B 11 A
12 A
20 C 18 B 13 D 12 B
21 A 13 C
14 D
22 D 14 C
15 B
23 A 19 D 15 D
16 C
24 C 16 D
17 C
25 D 17 B
20 B 18 B
18 D
1
GABARITO DAS PROVAS DE MATEMÁTICA
19 C 19 B 19 C 19 C
20 A 20 B 20 B 20 B
21 C 21 D 21 C 21 C
22 C 22 D 22 B 22 B
23 A 23 ANULADA 23 D 23 B
24 C 24 B 24 D 24 A
25 A 25 A 25 C 25 A
26 B 26 B 26 D 26 C
27 C 27 C 27 D 27 C
28 A 28 C 28 C 28 ANULADA
29 C 29 C 29 B 29 A
30 D 30 B 30 D 30 A
31 B 31 C 31 B 31 B
32 D 32 B 32 A 32 D
33 B 33 C 33 C 33 D
34 A 34 C 34 A 34 D
35 B 35 B 35 A 35 C
36 A 36 D 36 A 36 C
37 D 37 D 37 D 37 D
38 B 38 C 38 A 38 C
39 A 39 A 39 C 39 ANULADA
40 B 40 C 40 B 40 A

2/2002 – TURMA B 1/2003 – TURMA A 1/2003 - TURMA B 2/2003 - TURMA A


01 D 01 A 01 C 36 B
02 A 02 B 02 A 37 C
03 A 03 B 03 A 38 B
04 D 04 A 04 A 39 C
05 D 05 C 05 D 40 D
06 C 06 C 06 B 41 B
07 A 07 C 07 D 42 B
08 D 08 D 08 B 43 C
09 B 09 C 09 B 44 B
10 D 10 B 10 D 45 B
11 D 11 D 11 B 46 C
12 C 12 C 12 D 47 A
13 B 13 A 13 A 48 C
14 A 14 D 14 B 49 A
15 A 15 ANULADA 15 C 50 C
16 B 16 A 16 C 51 B
17 C 17 A 17 D 52 A
18 D 18 D 18 C 53 A
GABARITO DAS PROVAS DE MATEMÁTICA
54 B
55 B
56 D
57 D
58 A
59 C
60 C
61 D
62 D
63 C
64 A
65 A
66 B
67 D
68 A
69 D
70 D

2/2003 - TURMA B
36 B
37 C
38 D
39 C
40 A
41 A
42 A
43 D
44 B
45 C
46 D
47 C
48 B
49 D
50 D
51 A
52 D
53 C
54 C
55 B
56 D
57 D
58 B
59 B
60 A
61 A
62 B
63 D
64 B
65 A
66 B
67 B
68 C
69 C
70 C
QUÍMICA – EEAR
36 – Qual das proposições abaixo é correta quanto à aplicação
de uma força sobre um corpo?

a) A aceleração é igual ao produto da força pela massa.


b) A aceleração é diretamente proporcional à massa do
corpo.
1/2001-TURMA A c) A aceleração é diretamente proporcional à força aplicada.
d) A aceleração é inversamente proporcional à força aplicada.
26 – São exemplos de substâncias puras:
a) aço, mercúrio metálico, gás oxigênio. 37 – Qual o trabalho, em kJ, que a resultante das forças
b) água potável, água do mar, gás nitrogênio. atuantes sobre um corpo de massa 10 kg deve realizar para que
c) cloreto de sódio, ferro metálico, gás metano. o mesmo passe de uma velocidade de 10 m/s para a velocidade
d) leite, sangue, vinagre. de 30 m/s?

27 – Pode-se afirmar que qualquer substância pura simples: a) 1,5 c) 3,0


a) pode ser sempre decomposta. b) 2,5 d) 4,0
b) nunca tem apresentação homogênea.
c) é constituída apenas por moléculas monoatômicas. 38 – A pressão exercida por um corpo sobre um plano horizontal
d) é constituída apenas por átomos de um mesmo elemento de apoio é diretamente proporcional:
químico.
a) à área de contato.
28 – O átomo de um elemento químico fica carregado b) ao peso do corpo.
positivamente quando: c) à forma do corpo.
a) perde um ou mais prótons. d) ao volume do corpo.
b) perde um ou mais elétrons.
c) recebe um ou mais prótons. 39 – A pressão atmosférica considerada normal é de
d) recebe um ou mais elétrons. aproximadamente:

29 – O grupo de elementos que apresenta apenas metais é: a) 1 milibar.


a) Cr, Cu, C. b) 1 Pascal.
b) Fe, Ne, Te. c) 76 mm de coluna de mercúrio.
c) Na, Ni, Ti. d) 10 m de coluna de água.
d) Pb, Pt, P.
40 – Ondas sonoras são ondas:
30 – Pode ser classificada como uma decomposição a: a) eletromagnéticas.
a) combustão de uma fita de magnésio. b) longitudinais.
b) eletrólise da água. c) transversais.
c) hidratação da cal virgem (óxido de cálcio). d) superficiais
d) reação entre nitrato de prata e cloreto de sódio.
41 – Duas ondas sonoras propagam-se simultaneamente em
31 – Os sais derivados do ácido sulfúrico são os: um mesmo meio, com freqüências de 600 Hz e 200 Hz.
a) sulfatos. Podemos afirmar que estas ondas têm em comum:
b) sulfetos. a) o mesmo comprimento de onda.
c) sulfitos. b) o mesmo período de oscilações.
d) sulfuretos. c) a mesma velocidade de propagação.
d) a mesma altura.
32 – A soma de dois vetores ortogonais, um de módulo 9 e outro
de módulo 12, terá módulo igual a: 42 – À maior temperatura corresponde maior:
a) 3 a) massa.
b) 15 b) velocidade.
c) 17 c) quantidade de calor.
d) 21 d) grau de agitação térmica.

33 – Duas forças de intensidades diferentes atuam sobre uma 43 – O produto da pressão pelo volume de um gás é constante
mesma partícula; então: numa transformação:
a) certamente ela não está em equilíbrio. a) isotérmica.
b) certamente a resultante é maior que cada uma das forças. b) adiabática.
c) ela só estará em equilíbrio se as forças forem c) isométrica.
perpendiculares entre si. d) isobárica.
d) ela estará em equilíbrio apenas se os sentidos das forças
forem contrários. 44 – A imagem real de um objeto real está a 30 cm de um
espelho côncavo. A altura do objeto, colocado sobre o eixo
34 – Um móvel descreve uma trajetória em MRUV, passando focal, é de 6 cm, e de sua imagem, de 3cm. A distância focal do
duas vezes pela posição S = 60 m com velocidade de módulo espelho, em centímetros, é de:
igual a 10 m/s em ambas as passagens. Os módulos da a) 10
velocidade e da aceleração, no instante t = 0, valem, b) 15
respectivamente, 40 m/s e 5 m/s2. Desse modo, dependendo do c) 20
sentido inicial adotado, a posição S, em metros, no instante em d) 60
que o móvel muda de sentido pode ser:
a) 50 ou 70. c) –80 ou 80. 45 – A velocidade da luz monocromática em um certo óleo
b) 40 ou 80. d) –90 ou 150. corresponde a 3/4 de sua velocidade no vácuo. O índice de
refração do óleo é:
35 – No estudo da trajetória dos planetas do sistema planetário, a) 0,334
o referencial adotado é: b) 0,666...
a) o Sol. c) 0,750
b) a Terra. d) 1,333...
c) o centro da Galáxia.
d) um satélite geo-estacionário.

1
46 – Em eletrostática, para se carregar _______________ um 30 – A chamada “queima” da cal (reação do CaO com água,
corpo, ____________________. comumente efetuada pelo ajudante do pedreiro) pode ser
considerada uma reação de:
I  positivamente  acrescenta-se prótons a) adição.
II  negativamente  acrescenta-se elétrons b) decomposição.
III  positivamente  retira-se elétrons c) permutação (dupla troca).
IV  negativamente  retira-se prótons d) substituição (simples troca).

São corretas as afirmações: 31 – Entre as citadas abaixo, a base mais solúvel em água é o:
a) I e II. a) hidróxido de alumínio.
b) II e III. b) hidróxido ferroso.
c) III e IV. c) hidróxido de potássio.
d) todas. d) hidróxido plúmbico

47 – Diminuindo-se a distância entre as placas de um capacitor 32 – Um balão cheio de hidrogênio eleva-se na atmosfera.
plano, a capacitância eletrostática: Admitindo que, a partir de um certo instante, o seu movimento
a) pode aumentar ou diminuir, dependendo da corrente seja retilíneo e uniforme, conclui-se que:
elétrica aplicada. a) seu peso é maior que o empuxo do ar.
b) não se altera. b) empuxo é menor que a resistência do ar.
c) aumenta. c) peso é igual à soma do empuxo com a resistência do ar.
d) diminui. d) empuxo é igual à soma do peso com a resistência do ar.

48 – Um eletricista, distraidamente, ligou duas lâmpadas, uma 33 – Uma partícula está em equilíbrio sob a ação de três forças
de 10 W – 110 V e outra de 100 W – 110 V, em série a uma coplanares de 3 N, 4 N e 5 N. Então podemos concluir que:
tomada de 220 V. Logo, ele verificou que: a) as forças de 3 N e 4 N são perpendiculares entre si.
a) as duas lâmpadas acenderam com pouco brilho. b) as forças formam ângulos iguais entre si.
b) as duas lâmpadas “ queimaram” imediatamente. c) as três forças têm mesma direção.
c) a de 100 W acendeu com brilho acima do normal e “se d) as forças têm sentidos contrários.
queimou”. A de 10 W teve brilho menor e depois se
apagou. 34 – Dois corpos A e B na mesma vertical, como mostra a
d) a de 10 W acendeu com brilho acima do normal e “se figura, encontram-se em repouso a 10 m um do outro.
queimou”. A de 100 W teve brilho menor e depois se Abandona-se “ A ” e, 1 segundo depois,“ B”. O encontro dos
apagou. dois corpos se dará em _____ segundos após o início da queda
do corpo “ A ”. Considere g = 10 m/s 2 e despreze qualquer tipo
49 – O pólo sul de um imã atrai: de atrito.
a) o pólo norte do outro imã. A
b) o pólo sul de outro imã.
c) cargas negativas. a) 0,5
d) cargas positivas. b) 1,0
c) 1,5
d) 2,0
B
50 – Um dispositivo formado por uma barra de ferro envolvida
por um solenóide caracteriza um:
a) dínamo.
b) gerador.
c) eletroímã.
d) transformador.
35 – Lança-se, a partir do solo, uma pedra verticalmente para
cima, com velocidade inicial v0 . A aceleração da gravidade no
1/2001-TURMA B local vale g. Desprezando qualquer tipo de atrito e a resistência
do ar, é correto afirmar que:
26 – A equação química: H2 + Cl2 2 HCl
representa a reação entre _________________ para produzir v0
uma _____________. a) a altura máxima atingida é h  .
2g
a) duas substâncias puras compostas - mistura
b) duas substâncias puras simples - mistura b) a pedra atinge o solo com velocidade de módulo maior que
c) os componentes de uma mistura - substância pura v0 .
composta c) sendo h a altura máxima atingida, o tempo de subida é
d) os componentes de uma mistura - substância pura
h
simples ts  .
g
27 – A evaporação de um líquido puro é: d) tendo atingido o ponto mais elevado de sua trajetória, a
a) apenas um fenômeno físico. pedra começa a retornar ao solo. O tempo de queda vale
b) um processo químico de decomposição. v0
c) um processo de separação de mistura homogênea. tq  .
d) um processo de separação de mistura heterogênea. g

28 – Dados os números atômicos do ferro ( Fe = 26 ) e do 36 – O fato de um automóvel, que se move numa estrada plana
argônio ( Ar = 18 ), pode-se simbolizar a configuração eletrônica e horizontal e que teve seu motor desligado, parar após um
do estado fundamental do íon Fe3+ como: certo intervalo de tempo, é devido:
a) [Ar] 3d3 4s1 a) à inércia.
b) [Ar] 3d4 4s1 b) às forças de atrito.
c) [Ar] 3d3 4s2 c) ao peso do automóvel.
d) [Ar] 3d5 d) à força resultante ser nula.

29 – O trítio é, assim como o deutério, um hidrogênio pesado. 37 – O princípio de Arquimedes não justifica um dos fenômenos
Podemos então inferir que o trítio é um ______________ do a seguir:
hidrogênio. a) um balão subindo.
a) isóbaro b) a existência de um submarino.
b) isótono c) uma pessoa boiando numa piscina.
c) isótopo d) equilíbrio de um líquido em vasos comunicantes, onde as
d) isoneutrônico superfícies estão no mesmo nível.
38 – Um automóvel de massa 800 kg é acelerado 46 – No sistema em equilíbrio estático mostrado na figura, o
uniformemente a partir do repouso até uma velocidade de 50 corpo de massa M é mantido suspenso pela força de atração
m/s, em 20 s. A potência média desenvolvida por esse elétrica entre as cargas puntiformes de módulo Q1 = 2 C e Q2
automóvel ao completar esses 20 s, em kW, será de: = 6 C. O valor, em kg, da massa M é:

a) 25 a) 1,08
b) 50 b) 2,00
c) 100 c) 0,120
d) 200 d) 0,200 10 cm Q1
Q2
39 – A respeito da prensa hidráulica, é falso afirmar que é uma:

a) máquina que pode multiplicar forças. Dados: M


b) aplicação do principio de Pascal.
c) máquina que multiplica trabalho.
d) máquina que transmite pressão.

40 – As ondas eletromagnéticas propagam-se no ar com uma


velocidade aproximada de 300.000 km/s. Uma estação de Constante eletrostática k0 = 9,0 x 10 9 N. m2 / C2
televisão, que emite ondas cujo comprimento de onda vale 10 Aceleração da gravidade local g = 10 m / s2
cm, tem uma frequência, em MHz, de: Polias e fios ideais (massas desprezíveis)

a) 3 47 – O consumo mensal (30 dias) , em kWh, de uma lâmpada


b) 30 comum (incandescente), cuja especificação é 40 W – 127 V,
c) 300 ligada diariamente por 120 min, vale:
d) 3000 a) 2,0
b) 2,4
41 – A velocidade de propagação das ondas sonoras em um c) 4,0
mesmo meio: d) 40

a) depende do timbre. 48 – Associam-se quatro resistores exatamente iguais de forma


b) independe da intensidade. que a potência dissipada pelo conjunto seja de 100 W. A
c) independe da temperatura. diferença de potencial (d.d.p.) aplicada aos extremos da
d) independe das características deste. associação vale 200 V. Uma possível associação destes
resistores é em ___________, com cada resistor valendo
42 – Em uma transformação isobárica, 12,5 mols de um gás ______________ k.
perfeito variam sua temperatura de 100 K para 300 K. O a) série; 4,0
trabalho realizado pelo gás, em atm.l, vale: b) série; 0,12
a) 2,05 x 102 c) paralelo; 1,6
b) 2,05 x 103 d) paralelo; 2,0
c) 2,05 x 104
d) 2,05 x 105 49 – "O momento escalar da força resultante de um sistema de
forças é a soma algébrica dos momentos escalares das forças
Dado: componentes, em relação a um pólo." Este enunciado refere-se
Constante universal dos gases, R= 0,082 atm.l / (mol K) ao(às):
a) condições de equilíbrio de pontos materiais.
43 – No fenômeno da refração, os raios luminosos modificam b) conceito de centro de gravidade.
algumas de suas características ao atravessarem a superfície c) Teorema de Arquimedes.
de separação entre dois meios . Uma dessas características é d) Teorema de Varignon.
a(o):
a) freqüência. 50 – A razão entre as intensidades dos campos magnéticos
b) velocidade. produzidos por uma corrente elétrica que atravessa um condutor
c) diâmetro. retilíneo, respectivamente, a 6 e 12 m de distância do mesmo,
d) sentido. vale:
1
44 – Um copo de vidro está completamente cheio com 250 cm 3 a)
4
de óleo a 20ºC. O volume transbordado, em cm 3, quando a
b) 2
temperatura do conjunto passa a 120ºC, vale:
c) 4
a) 11,6
d) 8
b) 23,2
c) 24,4
d) 48,8 2/2001-TURMA A
Dados: 26 – Em que estado a matéria apresenta principalmente as
Coeficiente de dilatação linear () do vidro = 12 x 10- 6 ºC- 1 seguintes características: forma variável e volume constante?
Coeficiente de dilatação volumétrico () do óleo = 500 x 10- 6 ºC1 a) líquido.
b) gasoso.
45 – Uma lente delgada biconvexa convergente possui raios de c) sólido.
curvatura iguais a 20 cm cada e vergência de 5 dioptrias, d) vapor.
quando imersa no ar. O índice de refração da lente é:
27 – Sabendo que o número atômico do elemento Hélio é
a) 0,5 2, concluímos que, em relação ao átomo desse elemento, o
b) 1,0 número de:
c) 1,5 a) massa é dois.
d) 2,0 b) prótons é dois.
c) elétrons é quatro.
d) nêutrons é quatro.
28 – Os componentes líquidos do petróleo são separados por a) a velocidade e o ângulo de lançamento com a horizontal
meio da: são máximos.
a) fusão. b) a velocidade e o ângulo de lançamentos são mínimos.
b) decantação. c) o ângulo de lançamento com a horizontal é qualquer.
c) destilação fracionada. d) o ângulo de lançamento com a horizontal vale 450.
d) dissolução fracionada.
38 – Assinale a proposição correta:
29 – Têm-se dois átomos A e B. A soma das massas de A e B é a) No vácuo, um corpo mais e outro menos denso caem com
igual ao triplo da massa do isótopo 12 do carbono; e a diferença a mesma aceleração.
das massas de A e B é igual a 1/3 da massa do isótopo 12 do b) Um corpo no vácuo não cai, pois no vácuo não existe a
carbono. As massas, em u.m.a., dos átomos A e B são força da gravidade.
respectivamente: c) No vácuo, um corpo mais denso cai mais depressa que um
a) 20 e 10 menos denso.
b) 20 e l6 d) Na Lua, os astronautas não flutuaram, porque lá o vácuo
c) 30 e 10 não é perfeito.
d) 30 e l6
39 – Quando, num MCU, o raio da trajetória for numericamente
30 – Dos grupos abaixo, o qual só possui metais é: igual ao período, o valor numérico da velocidade linear do móvel
a) Na, Nb, Ne, Ni será igual a
b) Ca, Cl, Cr, Cu a) 
c) Li, Ti, Ni, Bi b) 2
d) Pt, Pb, Pd, P c) 3
d) 4
31 – Sabor azedo e conduzir bem a eletricidade, são
características da função: 40 – Uma máquina, de potência P1 , realiza um trabalho
a) sal mecânico T em 10 segundos; outra máquina, de potência P 2 ,
b) base realiza um trabalho mecânico 2T em 4 segundos. Comparando
c) óxido as potências dessas máquinas, podemos dizer que P 1 eqüivale
d) ácido a
32 – Que ângulo devem formar duas forças de módulos iguais, 5P2
a) c) 5P2
para que a resultante tenha módulo igual ao dobro do módulo 2
das componentes?
a) 0º
2P2 P2
b) d)
b) 90º 5 5
c) 120º
d) 180º 41 – Impulso de uma força constante é o produto
a) da força pela velocidade do corpo.
33 – Um corpo está em equilíbrio sob a ação de três forças b) do módulo da força pela velocidade da força.
coplanares, duas das quais são perpendiculares entre si e têm c) da força pelo intervalo de tempo em que atua.
módulos, respectivamente, iguais a 7,8 N e 10,4 N. Pode-se d) do módulo da força, pelo intervalo de tempo em que atua.
afirmar que o módulo da terceira força, em newtons, é
a) 11 42 – “ O acréscimo de pressão exercido em um ponto de um
b) 12 líquido ideal em equilíbrio se transmite integralmente a todos os
c) 13 pontos desse líquido.”
d) 14
Este enunciado refere-se a um princípio, muito utilizado em
34 – Sobre um plano inclinado sem atrito, de 20 m de prensas hidráulicas, denominado princípio de
comprimento e 5 m de altura, equilibra-se um corpo com uma
força paralela ao plano de 150 N. Podemos afirmar que o peso a) Pascal.
do corpo vale ..... N. b) Ampère.
a) 300 c) Arquimedes.
b) 450 d) Ação e Reação.
c) 500
d) 600 43 – Uma pessoa encontra-se na extremidade de um tubo
metálico e ouve dois sons produzidos por uma martelada dada
35 – Um trem de 200 m de comprimento, com velocidade na outra extremidade do tubo. A diferença de tempo entre os
escalar constante de 60 km/h, gasta 36 s para atravessar dois sons é de 0,2 s. A velocidade do som no ar é de 340 m/s, e,
completamente uma ponte. A extensão da ponte, em metros, é no metal do tubo, é de 1.700 m/s. O comprimento do tubo, em
de: metros, é de
a) 600 a) 34
b) 500 b) 85
c) 400 c) 170
d) 200 d) 340

36 – Um caminhão ao percorrer uma rodovia reta, passa pelo 44 – A qualidade que nos permite distinguir dois sons de mesma
marco 50 km às 8horas e 20 minutos e pelo marco 170 km às 9 altura e emitidos por fontes diferentes, chama-se
horas e 50 minutos. A velocidade média nesse intervalo foi, em a) timbre.
km/h, de b) registro.
a) 60 c) amplitude.
b) 80 d) intensidade.
c) 92
d) 146 45 – Um vaso contém água pura a temperatura e pressão
ambientes. Para que esta água entre em ebulição deve-se
37 – O lançamento de foguetes tornou-se, desde a 2 Grande
a
reduzir a
Guerra Mundial, uma tecnologia bastante difundida. Em relação a) temperatura somente.
a um lançamento oblíquo no vácuo, pode-se afirmar que o b) temperatura e manter a pressão.
alcance é máximo quando, necessariamente c) pressão e manter a temperatura.
d) temperatura e aumentar a pressão.
46 – Um automóvel que está com velocidade constante de 80 28 – Quando se cozinha em fogões a gás, a chama azul fica
km/h sobre uma pista retilínea, vê no espelho retrovisor plano, a amarela quando nela cai um pouco de sal ( cloreto de sódio).
imagem de um poste que está à beira da estrada. A velocidade
da imagem do poste em relação ao espelho do automóvel, em Esse fato corrobora a
km/h, é de: a) Teoria dos Quanta de Planck.
a) 40 b) Teoria da Relatividade de Eisntein.
b) 80 c) propriedade radioativa dos elementos químicos.
c) 160 d) experiência do espalhamento das partículas  realizada
d) 200 por Rutherford.

47 – Existem quatro corpos condutores isolados A, B, C e D, 29 – As bases formadas por metais alcalinos – terrosos são
sendo que: consideradas
- ao se aproximar A de B, ocorre atração; a) fracas.
- ao se aproximar B de C, ocorre atração; b) insolúveis.
- ao se aproximar A de C, ocorre atração; c) monobases.
- ao se aproximar B de D, ocorre repulsão. d) pouco solúveis.

Portanto, é possível dizer que os corpos estão carregados da 30 – O gás carbônico apresenta sempre a mesma fórmula
seguinte maneira: molecular, quando obtida a partir das seguintes reações: síntese
a) A está neutro, B positivo, C neutro e D negativo. total, decomposição do carbonato de cálcio ou combustão do
b) A está neutro, B negativo, C positivo e D positivo. álcool comum.
c) A está positivo, B negativo, C positivo e D neutro.
d) A está positivo, B negativo, C neutro e D negativo. Este fato está de acordo com a lei de
a) Dalton.
48 – As substâncias magnéticas podem ser classificadas quanto b) Proust.
à sua facilidade de imantação; assim as substâncias cujos imãs c) Lavoisier.
elementares se orientam facilmente quando submetidos à ação d) Gay- Lussac.
de um campo magnético, tais como ferro, níquel e cobalto, são
ditas.............. . 31 – Os gases nobres são considerados estáveis e formados
a) ferromagnéticas. por átomos isolados, então, podem ser classificados como
b) paramagnéticas. a) gases compostos ideais.
c) diamagnéticas. b) elementos químicos simples.
d) adiabáticas. c) exceção de substâncias compostas.
d) substâncias puras simples monoatômicas.
49 – Nos vértices de um triângulo equilátero, localizam-se 3
cargas elétricas iguais em intensidade e polaridade. A tendência 32 – O H3PO3 é um
de movimento sentida pela carga 3 é melhor representada pelo a) triácido.
vetor b) diácido.
c) tetrácido.
a) A A B C d) monoácido.
b) B
c) C 33 – O óxido CO não reage com água, porque é
d) D 3  a) um superóxido.
b) um peróxido.
D c) um anfótero.
d) indiferente.
  34 – A intensidade média de corrente elétrica que percorre um
1 2
fio condutor em um intervalo de tempo de 2 min, devido a uma
50 – As lâmpadas incandescentes residenciais atualmente são carga de 1 C, vale aproximadamente, em ampères
fabricadas para funcionar em 127 volts. Neste caso, uma a) 0,008
lâmpada de 100 watts possui resistência de ........... ohms. b) 0,010
a) 1,27
b) 1,61
c) 0,120
c) 127,00 d) 0,800
d) 161,29
35 – Percorrendo-se, em Movimento Retilíneo Uniforme, uma
determinada distância a 40 km/h, gastam-se duas horas a
2/2001-TURMA B menos do que se percorresse a mesma distância, com o mesmo
tipo de movimento, a 20 km/h. Qual o valor desta distância, em
26 – Em um laboratório, notou-se que um determinado frasco km ?
continha uma determinada substância. No frasco havia uma a) 30
etiqueta que indicava apenas as propriedades dessa substância. b) 80
Das propriedades indicadas no frasco, a única que não contribui
c) 90
para a identificação da substância é
d) 100
a) massa.
b) densidade. 36 – A partir do solo, lança-se um corpo verticalmente para cima
c) composição em peso. com velocidade inicial de 20 m/s. Sendo a aceleração da
d) coeficiente de solubilidade. gravidade local igual a 10 m/s2 e desprezando a resistência do
ar e qualquer tipo de atrito, pode-se afirmar que
27 – A força ácida ou básica de um composto está diretamente a) a altitude máxima atingida pelo corpo é de 40 m.
ligada à questão b) no instante t = 3,0 s, a altitude em que se encontra o corpo
a) do número de hidrogênio ionizáveis e oxidrilas no é de 20 m.
composto. c) no instante t = 3,0 s, a velocidade do corpo vale, em
b) da presença ou não de oxigênio na molécula. módulo, 10 m/s.
c) da eletronegatividade dos elementos.
d) da solubilidade do composto em água.
d) ao retornar ao solo, o corpo possui velocidade de
módulo igual a 40 m/s.
37 – Nos fenômenos ondulatórios de refração e reflexão, c) as temperaturas alcançadas por ambos serão sempre
mantém-se constante o(a) diferentes.
a) pulso. d) poderão ter áreas de seção diretamente proporcionais às
b) freqüência. suas próprias resistências elétricas.
c) velocidade.
d) comprimento de onda. 44 – A variação do comprimento de uma barra homogênea
corresponde a 1 % de seu comprimento inicial, ao ser aquecida
38 – Uma escala “ W ” foi criada atribuindo-se os valores de – de 23ºC a 423ºC. O coeficiente de dilatação linear do material
20ºW e 30ºW aos pontos de gelo e de vapor, respectivamente. de que é feita a barra vale, em 10- 5 º C – 1,

Portanto, 50ºC corresponde em ºW a a) 1,0


a) 50 b) 1,5
b) 45 c) 2,5
c) 15 d) 4,0
d) 5
45 – Dois elevadores A e B de mesmo peso conseguem
39 – Um objeto real, colocado a 60 cm de uma lente, conjuga transportar uma mesma carga, do solo até o décimo andar de
uma imagem virtual que se encontra a 10 cm dessa mesma um prédio. O elevador A gasta 30 s e o elevador B gasta 40 s,
lente. Esta lente é do tipo ........................... e sua distância focal, ambos com velocidades constantes. A razão entre as potências
em módulo, vale ............... cm. mecânicas desenvolvidas por A e B, nesta ordem, é
a) divergente – 12 2
a)
b) divergente – 8,6 3
c) convergente – 12 4
d) convergente – 8,6 b)
3
40 – Em uma ponte de Wheaststone, a leitura de corrente no 3
c)
galvanômetro G, como mostra a figura abaixo, vale, em 4
ampères,
3
Dados: R1= 5  R2 = 2  R 3 = 10  R4 = 4  d)
2
R1 R3 46 – Um espelho convexo, cujo raio de curvatura vale 10 cm,
está distante de um objeto real, de altura 1cm, cerca de 4 cm. A
a) 4,00
distância focal e o tamanho da imagem colocada sobre o eixo
b) 4,66 G
c) 5,00
G focal do espelho, em centímetros, vale, respectivamente,
d) zero 5
a) 5 e
R2 9
R4
9
b) 5 e
5
5
c)  5 e
9
41 – O esquema abaixo representa uma polia que gira em torno 9
d)  5 e
do seu eixo, ponto “ O”. As velocidades tangenciais dos pontos 5
A e B valem, respectivamente, 50 cm/s e 10 cm/s. A distância
AB vale 20 cm. A velocidade angular da polia, em rad/s, será de 47 – A razão das variações entre os pontos de gelo e vapor na
escala centígrada e em uma escala R é 2:7. Sabendo que o
ponto de vapor na escala R vale 400ºR, podemos afirmar que
o ponto de gelo nesta escala, em ºR, vale
a) 50 c) 350
a) 1 O x x b) 100 d) 400
b) 2 A
c) 3 B
48 – Quando uma pessoa envelhece, seu cristalino vai, aos
d) 4
poucos, enrijecendo e perdendo a capacidade de acomodação
visual. Este defeito da visão é denominado
42 – As forças F1 e F2 atuam nos vértices de um triângulo
equilátero de lado igual a x , conforme a figura. Os momentos a) miopia.
de F1 e F2 em relação ao vértice C, possuem módulos, b) emetropia.
respectivamente, iguais a c) presbiopia.
d) hipermetropia.
3 B
a) F1 x e F2 2 x
49 – O diagrama representa a dilatação do comprimento l de
x uma barra metálica em função da temperatura. Se o coeficiente
b) F1 2x e F2 de dilatação da barra é 2 x 10 – 5 o C – 1, o comprimento da barra
2 F2 a 50 o C é , em metros,
x
x
c) F1 x e F2
a) 50,05
l (mm)
2
b) 50,50
x A C c) 55,00
d) F1 e F2 2 x
l0 45o
2 d) 500,5
F1
43 – A resistividade elétrica dos materiais é uma característica
própria destes, dependendo, também, de suas temperaturas. t (O C)
Desta forma, podemos garantir que se dois fios condutores têm 0
a mesma resistividade, Dado : l0 = comprimento inicial
a) poderão ter a mesma resistência elétrica.
b) certamente as resistências elétricas desses serão iguais.
50 – O sistema mostrado abaixo encontra-se em equilíbrio 08 – Podemos afirmar corretamente que, no vácuo,
estático. O valor, em graus, do ângulo  vale a) a velocidade de um corpo em queda livre é proporcional à
Dados: Massas: mA = 2 kg , mB = 3 kg , mC = 5 kg , sua massa.
g = 10 m/s2 , b) a aceleração de um corpo em queda livre é proporcional à
Tração no fio 3 = 40 N. Despreze os atritos e considere os fios e sua massa.
polias ideais c) a velocidade de corpos em queda livre é sempre uma
3 constante.
2 d) corpos em queda livre caem sempre com a mesma
B aceleração.
1
09 – Um corpo está sujeito à ação de várias forças que agem
simultaneamente nele. Baseado nas leis de Newton, é correto
A C afirmar que
a) possivelmente o corpo está em repouso.
 b) necessariamente o corpo está em repouso.
c) necessariamente a resultante das forças que agem no
corpo é nula.
a) 90 c) 45
d) necessariamente a aceleração do corpo é diretamente
b) 60 d) 30
proporcional à sua massa.

1/2002-TURMA A 10 – A soda cáustica e o amoníaco são típicos exemplos de


a) sais. c) ácidos.
01 – Indique a alternativa que contém somente elementos b) bases. d) óxidos.
químicos pertencentes à classe dos metais:
a) O, F, Br, I. 11 – Compostos químicos que, ao serem dissolvidos em água,
b) B, S, Ge, Sb. liberam exclusivamente os cátions H+, chamam-se
c) Na, Ne, S, Cl. a) sais. c) ácidos.
d) Mg, Ca, K, Na. b) bases. d) óxidos.

02 – Observe as afirmativas a seguir: 12 – A soma de dois vetores de módulos 15 N e 9 N tem,


I. No mesmo período, os elementos apresentam o mesmo certamente, o módulo do vetor resultante compreendido entre
número de níveis. a) 6 N e 15 N.
II. Os elementos do grupo 2 A apresentam, 2 elétrons na b) 6 N e 24 N.
última camada. c) 9 N e 15 N.
III. No mesmo grupo os elementos apresentam o mesmo d) 9 N e 24 N.
número de camadas.
Podemos concluir que, segundo à estrutura da classificação 13 – Em Estática existem duas condições de equilíbrio: a 1a diz
periódica dos elementos, está(ão) correta(s) a(s) afirmativa(s) que a resultante do sistema de forças deve ser nula e a 2 a, que
a) I e II. c) II e III. a soma algébrica dos momentos das forças do sistema, em
b) I e III. d) somente a II. relação ao mesmo ponto, também deve ser nula. Das
condições citadas, estabelece o equilíbrio de um corpo extenso
03 – Uma usina hidroelétrica, por exemplo a de Itaipu, a) apenas a 1a.
transforma energia b) apenas a 2a.
a) elétrica em calor. c) a 1a e a 2a alternadamente.
b) mecânica em elétrica. d) a 1a e a 2a simultaneamente.
c) elétrica em mecânica.
d) química em física. 14 – Um automóvel está parado em um semáforo devido à
indicação de sinal vermelho. Num certo instante, o sinal muda
04 – Massa é a propriedade da matéria para verde e o automóvel entra em movimento.
a) que mede o espaço ocupado por suas partículas.
b) medida em quilograma-força, no sistema MkgfS. Logo a frente, ele encontra outro semáforo cuja indicação, mais
c) que indica o tipo de moléculas existentes em um corpo. uma vez, é vermelho, e pára novamente. O gráfico fornece a
d) relacionada com a quantidade de moléculas existentes em velocidade do automóvel entre estes dois semáforos. A dis-
um corpo. tância entre estes, em metros, é
V (m/s)
05 – O C60 é uma molécula geodésica, ou seja, com 60 vértices a) 900.
onde se localizam os átomos de carbono, que por ter uma forma b) 1.050.
muito parecida com uma bola de futebol, é também chamada de c) 1.100.
futeboleno. O futeboleno, o grafite e o diamante são exemplos d) 1.200.
de 30
a) formas alotrópicas do carbono.
b) moléculas iônicas de carbono.
0
c) substâncias puras compostas. 10 30 50 t (s)
d) misturas carboníferas.
15 – Observando-se a lei de Hooke, pode-se afirmar que a
06 – Deseja-se separar dois componentes de uma mistura constante k de proporcionalidade das molas
heterogênea, um líquido e outro sólido de maneira rápida. a) independe do material da mola.
Para tanto, utiliza-se o processo de b) é igual para todos os tipos de mola.
a) filtração simples. c) é uma característica de cada tipo de mola.
b) filtração à vácuo. d) não sofre alterações com variações de temperatura
c) decantação. elevadas.
d) levigação.
16 – Dos casos citados abaixo, indicar aquele onde o corpo em
07 – O fato de uma carga elétrica em movimento irradiar movimento não apresenta variação de energia potencial
continuamente energia, invibializou o modelo atômico de gravitacional.
a) Rutherford. a) Um pára-quedista durante o salto.
b) Thomson. b) Um atleta, correndo numa pista horizontal.
c) Dalton. c) Uma bola de basquete ao ser arremessada para o cesto.
d) Bohr. d) Um automóvel, descendo uma ladeira com o motor
desligado.
17 – Das unidades abaixo, aquela que se refere à energia 26 – Um raio de luz monocromático incide num prisma cujo
cinética é índice de refração em relação ao ar é 1,60, conforme a figura.
erg m Na face AB o raio de luz deve
a) . c) kg .
A
s s2
2
m 45º
b) watt. d) kg .
s2 I

18 – A luz, o som e o calor são exemplos de


a) campos de força. 
45º
b) diferentes formas de energia. C B
c) sensações físicas que nada tem a ver com a energia.
d) sensações físicas provocadas pela variação de energia. a) refletir voltando sobre si mesmo.
b) refratar com ângulo maior que 45º.
19 – Dois corpos possuem, em um determinado instante, a c) refratar com ângulo menor que 45º.
mesma quantidade de movimento. Baseado nesse fato, é d) refletir com 45º, em relação à normal de AB, emergindo da
correto afirmar que face CB.
a) certamente o produto da massa pela velocidade dos
respectivos corpos é diferente. 27 – Um submarino, completamente submerso, para emergir
b) indubitavelmente as massas dos corpos são iguais. a) aumenta o empuxo exercido sobre ele.
c) certamente as velocidades dos corpos são iguais. b) diminui o empuxo exercido sobre ele.
d) as velocidades dos corpos podem ser iguais. c) aumenta seu peso.
d) diminui seu peso.
20 – Uma força de 100 N, atuando perpendicularmente sobre
uma superfície, provoca uma pressão de 1.000 pascals. A área 28 – Um ponto material realiza um movimento periódico com
da superfície é de intervalos regulares de 0,1 segundos. A freqüência, em Hz,
a) 10 m². c) 10 cm². desse movimento é de
b) 0,1 m². d) 0,1 cm². a) 10-1. c) 101.
b) 100. d) 102.
21 – A pressão exercida num ponto interno de um líquido,
dentro de um recipiente, em equilíbrio não depende da 29 – Uma onda apresenta velocidade de propagação de 400
a) densidade do líquido. m/s e freqüência de 40 Hz. Quanto valerá o seu comprimento de
b) forma geométrica do recipiente. onda?
c) profundidade em que se encontra. a) 1 m. c) 10 cm.
d) pressão externa exercida na superfície do líquido. b) 10 m. d) 16 km.

22 – Considerando um corpo descendo um plano inclinado liso 30 – Para calcularmos o período T de oscilação de um pêndulo
(sem atrito) sob a ação exclusiva de seu peso, pode-se afirmar simples, constituído de material metálico, utilizamos a expressão
que a aceleração desse corpo é T = 2(L/g)1/2 onde “g” é a aceleração da gravidade local e “L” é
a) maior que a aceleração da gravidade local. o comprimento do pêndulo. Ao aquecermos o pêndulo, no
b) sempre igual a aceleração da gravidade local. mesmo local, podemos afirmar que seu período de oscilação
c) inversamente proporcional ao seu próprio peso. a) não varia.
d) dependente do ângulo entre o plano inclinado e o plano b) diminui.
horizontal. c) aumenta.
d) diminui, mas depois aumenta.
23 – Leia com atenção um trecho da reportagem intitulada
Imagem revela o peso dos frangos, editada na Folha de S. 31 – Um gás perfeito, evoluindo isobaricamente, terá realizado
Paulo em 22 de maio de 2001: “ .... com base no tamanho das trabalho nulo se sua temperatura final for
aves, um software revela também o seu peso.” a) maior que a inicial (Tf > Ti).
Neste trecho existe uma inconsistência no conceito de uma b) menor que a inicial (Tf < Ti).
grandeza física, que se deve ao fato de que c) igual à inicial (Tf = Ti).
d) igual ao zero absoluto.
a) o tamanho das aves deve ter a mesma unidade do peso
destas. 32 – Quando um sistema realiza um trabalho sobre o meio
b) na realidade a reportagem deveria referir-se à massa e não externo a ele, sem ceder ou receber energia sob a forma de
ao peso dos frangos. calor,
c) o peso das aves é função da aceleração da gravidade local a) a evolução deste é dita isométrica.
e, portanto, não tem relação com a massa destas. b) sua energia interna diminui.
d) software é somente uma rotina computacional e não um c) sua energia interna aumenta.
processo correto na determinação do tamanho das aves. d) a evolução não é adiabática.

24 – Vidro fosco, papel vegetal e tecido fino são exemplos de 33 – Os pára-raios foram inventados pelo cientista e estadista
meios norte-americano Benjamim Franklin no século XVIII. O princípio
a) opacos. de funcionamento dessa importante invenção é uma aplicação
b) translúcidos. a) do poder das pontas.
c) transparentes. b) do campo magnético da Terra.
d) monocromáticos. c) da força eletromagnética dos raios.
d) das idéias de blindagem eletrostática de Faraday.
25 – Considere que a sombra projetada de uma nuvem sobre o
solo tenha, aproximadamente, a mesma forma e o mesmo
tamanho da própria nuvem. Isto poderia ocorrer caso os raios
solares fossem
a) convergentes em um mesmo ponto.
b) praticamente paralelos.
c) pouco espessos.
d) divergentes.
34 – No circuito esquematizado o valor da corrente i, em 39 – Nos pontos internos de uma longa bobina percorrida por
ampères, que atravessa o galvanômetro é de corrente elétrica contínua, as linhas de indução do campo
magnético são
a) hélices cilíndricas.
3 i b) circunferências concêntricas.
17
c) retas paralelas ao eixo da bobina.
G d) radiais, com origem no eixo da bobina.

3 10 40 – Conhece-se, desde a Antiguidade, a propriedade de certos


minerais de atrair pequenos pedaços de material ferroso. Um
4 18 desses minerais e que constitui o chamado ímã natural é
50V denominado
a) esferoidita.
b) magnetita.
a) zero. c) calcinita.
b) 3,8. d) gipsita.
c) 6,0.
d) 10/11.
1/2002-TURMA B

35 – No circuito abaixo, qual a corrente, em miliampères, que 01 – Com base no gráfico abaixo, que relaciona as massas e os
circula no filamento indicado, se este possuir as seguintes volumes das substâncias A, B e C, podemos afirmar que, em
termos de densidade (d)
características:  = 2  mm2/m; A = 1 mm2 e L = 10 m?
m (g ) A
a) dA > dB > dC. B
b) dB > dC > dA. C
100  c) dC > dB > dA.
+ d) dA = dB = dC.
V ( cm3)
12 V 20 
– 02 – Na atmosfera, além dos gases nitrogênio e oxigênio,
existem os chamados gases nobres (hélio, neônio, argônio,
40  criptônio, xenônio e radônio). A respeito desses gases podemos
filamento afirmar que
a) não são elementos químicos pois são substâncias simples.
b) são substâncias ultra-simples pois são formados por um
único elemento químico.
c) não se constituem em moléculas pois são formados por um
único elemento químico.
d) se constituem em moléculas de atomicidade 1 (também
a) 20.
chamados de gases monoatômicos).
b) 40.
c) 50.
03 – Para separar o sal da areia, usa-se inicialmente o processo
d) 100.
da
a) destilação. c) levigação.
36 – A partir do SI (Sistema Internacional de Unidades) e
b) dissolução. d) fusão.
utilizando a Lei de Coulomb, podemos afirmar que a unidade da
constante dielétrica do meio, K, está corretamente relacionada
04 – O Princípio de Heisenberg afirma que não é possível
em
calcular a posição e a velocidade de um elétron num mesmo
N.m kg.m3 .s 2 instante. Essa dificuldade levou Schrödinger a desenvolver o
a) . c) . conceito de
C C2 a) átomo. c) nêutron.
N.m 2 kg.m3 .s 2 b) núcleo. d) orbital.
b) . d) .
C C2 05 – Se você fornecesse energia para arrancar um elétron de
todos os elementos conhecidos, a energia seria maior para o
37 – Para utilizarmos corretamente uma bússola em perfeitas elemento
condições, devemos a) hidrogênio. c) hélio.
a) colocá-la no interior de uma caixa de ferro. b) bromo. d) flúor.
b) não colocá-la em qualquer recipiente que possa ter
propriedades magnéticas. 06 – O átomo que apresenta Z prótons e N nêutrons e o
c) colocá-la próximo a uma caixa de ferro para potencializar o átomo que contém ( Z + 1) prótons e ( N – 1) nêutrons são
efeito magnético. a) isóbaros. c) isótonos.
d) não colocá-la sobre a linha do Equador pois o campo b) isótopos. d) alótropos.
magnético paralelo impede a leitura.
07 – Um ano após a descoberta da radioatividade, Rutherford
38 – Considere as seguintes afirmativas: verificou que as radiações emitidas pelo urânio eram de dois
I. Os imãs originam campos magnéticos. tipos com diferentes poderes de penetração. As mais
II. Cargas elétricas em movimento originam campos penetrantes foram chamadas de raios

magnéticos.
III. Cargas elétricas fixas originam campos magnéticos. a) (alfa). c)  (gama).
São verdadeiras:
a) I e II.
b)  (beta). d)  (delta).
b) I e III.
c) II e III. 08 – As substâncias que, em solução aquosa conduzem a
d) I, II e III. corrente elétrica e contêm o grupo (0H) hidroxila, são os/as
a) sais. c) bases.
b) ácidos. d) óxidos.
09 – Fazendo o balanceamento da equação química 16 – Para um elevador de massa igual a 500 kg, admitindo a
BCl3 + P4 + H2  BP + HCl, obteremos, respectivamente, os aceleração da gravidade igual a 10 m/s2 e desprezando as
coeficientes forças de atrito, a tração no cabo vale:
a) 4, 1, 5, 4, 10. c) 2, 1, 3, 2, 6. I. 600 N, quando o elevador sobe com aceleração constante
b) 4, 1, 6, 4, 12. d) 2, 2, 3, 2, 6. de 2 m/s2.
II. 5.000 N, quando o elevador sobe com velocidade
10 – Reagem com água, formando bases e liberando O2, ou constante de 5 m/s.
também reagem com ácido formando sais e H2O2, são os/as III. 5.000 N, quando o elevador desce com aceleração
a) diácidos. c) monobases. constante de 2 m/s2.
b) peróxidos. d) sais duplos. IV. 4.795 N, quando o elevador desce com aceleração
constante igual a 0,5 m/s2.
11 – No sistema mostrado abaixo, as roldanas e os fios são Das frases acima é(são) correta(s)
ideais e o atrito é considerado desprezível. As roldanas A, B e a) apenas I. c) II e IV.
C são fixas e as demais são móveis sendo que o raio da roldana b) apenas II. d) I e III.
F é o dobro do raio das outras que são iguais entre si. Sendo a
aceleração da gravidade local igual a 10 m/s 2 e a massa M de 17 – Qual das situações abaixo é falsa?
4,0 kg, o valor, em módulo, da força capaz de equilibrar o a) Certo indivíduo pesa 700 N na Terra; logo, seu peso na
sistema é, em newtons, Lua é também 700N.
b) Medindo-se a massa de um corpo na Terra e na Lua,
obtém-se o mesmo resultado.
linha de c) Peso e massa são grandezas diferentes; porém, quanto
A B C 30o referência maior a massa de um corpo, maior o seu peso.
d) Quando uma pedra está pendurada num barbante, a pedra
F produz uma tensão no barbante para baixo e o barbante
puxa a pedra para cima.
D E
18 – Com base nas informações dadas pelo gráfico F x X
(força x deformação) construído para duas molas A e B,
a) 5,0. podemos afirmar que, em termos de energia potencial elástica,
b) 8,0.
F a mola F (N)
c) 10.
d) 20. A
10 B
M 5

0 2 4 X (m)
12 – Duas forças com intensidades diferentes atuam sobre uma
mesma partícula; então a) A acumula mais do que a mola B.
a) certamente elas não estão em equilíbrio. b) B acumula mais do que a mola A.
b) certamente a resultante é maior que cada uma delas. c) A acumula tanto quanto a mola B.
c) elas só entrarão em equilíbrio se forem perpendiculares d) A e a mola B tem outros detalhes não informados e que
entre si. portanto nada se pode concluir sobre elas.
d) elas estão em equilíbrio, apenas se os seus sentidos forem
contrários. 19 – Um corpo lançado sobre uma superfície plana horizontal e
com atrito, tem sua velocidade variando com o tempo, de
13 – Um carro foi de São Paulo até o Rio de Janeiro mantendo acordo o gráfico abaixo. Adotando g = 10 m/s 2, o coeficiente de
uma velocidade média de 80 km/h. Admitindo-se 400 km a atrito entre o corpo e a superfície vale
distância entre as duas cidades citadas, pode-se afirmar que a) 0,1. V (m/s)
a) a velocidade mínima foi de 80 km/h. b) 0,2.
b) o carro não parou em nenhum instante. c) 0,5. 15
c) o carro gastou 5 horas para fazer a viagem. d) 2,0.
d) o ponteiro do velocímetro manteve-se durante todo
percurso na marca de 80 km/h.
0 t (s)
14 – Ao construirmos o gráfico da energia potencial e da energia 3
cinética em relação ao tempo para um corpo em queda livre, 20 – Uma esfera de 150 g de massa é abandonada de uma
obteremos respectivamente uma altura H do solo. Ao chocar-se com este, a esfera retorna à
a) reta e uma reta. posição inicial. Nessa perspectiva, a variação de energia
b) reta e uma parábola. mecânica, em J, ocorrida durante o movimento total da esfera,
c) parábola e uma reta. sendo a aceleração da gravidade igual a 10 m/s2, vale
d) parábola e uma parábola. DADO: Despreze qualquer tipo de atrito e considere o choque
com o solo perfeitamente elástico.
15 – Os carros A e B deslocam-se em uma mesma estrada reta, a) 0. c) 600.
de acordo com o gráfico. Em t = 0, ambos se encontram no b) 7,5. d) 607,5.
quilômetro zero. Pode-se afirmar que das opções apresentadas
abaixo, estão corretas 21 – Uma pedra de massa 50 gramas é arremessada
I. Em t = 0, temos VA = 50 km/h e VB = 0 horizontalmente por um estilingue. Admita que a pedra
II. Ambos os carros se deslocam com movimento abandona o estilingue com velocidade de 10 m/s e que o tempo
uniformemente acelerado de interação entre ambos seja de 0,5 s. Desse modo, a força
III. De t = 0 a t = 2 h , A percorre 100 km e B percorre 150 km utilizada, em newtons, no arremesso da pedra vale
IV. A alcança B em 4 h a) 100. c) 2.000.
b) 1.000. d) 5.000.
V (km/h)
A 22 – Um planeta hipotético “X” gira em torno do Sol com um
150 B período de revolução, em anos, igual a 27 vezes o da Terra em
a) somente a III.
b) II, III e IV. 100 relação ao Sol, obedecendo às leis de Kepler. Portanto, a
c) II e III. distância ”X” – Sol é .......... vezes a distância Terra – Sol.
d) I e III. 50 a) 3 c) 9
b) 6 d) 12
1 2 3 t (h)
23 – A densidade de um determinado óleo comestível é de 0,80 34 – Uma lente de vidro cujos bordos são mais espessos que a
g/cm3, sendo g = 10 m/s2, quanto pesa o óleo contido numa lata parte central .......
de 900 ml? Dados. ar = 20
a) 720 g c) 7,2 kg vidro = 20,5
b) 7,2 N d) 0,72 N água = 22/3

24 – O casco externo de um submarino a 200 m de a) é divergente no ar.


profundidade sofre uma pressão de aproximadamente ........... b) nunca é divergente.
vezes a pressão atmosférica normal. c) é sempre divergente, não importando o meio.
a) 10 c) 100 d) torna-se convergente mergulhada na água.
b) 20 d) 200
35 – Por meio de um raio, uma carga elétrica de 108 C é
25 – ANULADA transferida de uma nuvem para o solo. Supondo que o potencial
da nuvem mantenha--se constante durante toda descarga,
26 – Uma onda sonora ao passar de uma região de menor determine o número de dias que uma lâmpada de 100 W
temperatura para uma outra de maior temperatura, altera poderia permanecer acesa, usando a energia liberada neste
a) a altura. c) a freqüência. raio.
b) o período. d) o comprimento de
onda. Dado:
Admita que o potencial de uma nuvem em relação ao solo vale
27 – Ao passar do estado sólido para o líquido, sob pressão 8 x 106 V.
constante, uma substância cristalina
a) cede calor e sua temperatura varia. a) 100 c) 150
b) absorve calor e sua temperatura varia. b) 120 d) 220
c) cede calor e sua temperatura permanece constante.
d) absorve calor e sua temperatura permanece constante. 36 – Em uma residência estão instalados na rede 110 V, um
chuveiro de 4.000 W, 10 lâmpadas de 100 W, um televisor de 70
28 – No desenho a seguir vemos duas ondas propagando-se W e uma geladeira de 100 W. Caso estes aparelhos fossem
em uma corda, com a mesma velocidade e sentidos opostos. No substituídos por outros, de mesma potência, mas que fossem
instante em que coincidirem os pontos A e C e os pontos B e D, instalados na rede 220 V, a corrente total consumida
a forma da onda resultante será:
a) aumentaria 50 %. c) diminuiria 75 %.
b) diminuiria 50 %. d) seria a mesma.

A B C D 37 – O amperímetro é um aparelho destinado a medir a


intensidade de corrente elétrica. Um amperímetro ideal é aquele
a) que possui resistência
a) nula. c) variável.
b) b) infinita. d) múltipla.

c) 38 – Ímãs são elementos


a) formados por um elemento químico e um pólo magnético.
d) b) complexos de cadeia carbônica oxi-reduzida.
c) que atraem todos os tipos de metais.
d) que possuem dipolo magnético.
29 – A temperatura 0ºF equivale a .....ºC, aproximadamente.
a) 32 b) -16,67 c) -32,00 d) -273,15 39 – Nas figuras seguintes, está ocorrendo a passagem de
corrente elétrica contínua (i), sentido convencional, nos
30 – Suponha que uma galáxia distante exista um planeta condutores.
semelhante ao nosso sendo, contudo, que a luz que o ilumina Em cada situação está representado o vetor campo magnético
seja monocromática. Um fenômeno óptico, devido a essa luz, perpendicular ao plano da folha de papel orientado para fora ()
que não seria observado no planeta em questão é o/a e para dentro (). Com base nestas informações, assinale a
figura correta.
a) sombra. c) reflexão.
b) refração. d) arco-íris.

a) c)  
31 – Aumentando-se o diâmetro do orifício de uma câmara B
escura, a imagem produzida no interior desta câmara i  
a) perderá a nitidez. c) formar-se-á menor.
b) formar-se-á maior. d) aumentará a
nitidez.   
 
i
32 – Um espelho convexo reflete a imagem de um objeto real de B i
BB
 

10 cm de altura, colocado a 2 cm de distância do espelho, tendo


esta imagem uma altura de 4 cm. A distância focal e o raio de
curvatura, em módulo, do espelho, ambos em centímetros, 
valem respectivamente
b)
i
d)
 
B
a)
4 3
e . c) 
4 8
e .
 
3 8 3 3
3 8 3 8   i

b) e . d) e .  

B
B
 
B
4 3 4 3 i

33 – Duas cargas puntiformes, Q1 e Q2, estão se atraindo, no ar,


com uma força F. Suponha que o valor de Q1 seja duplicado e o
de Q2 se octuplique (multiplicado por 8). Para que o valor da
força F permaneça invariável, a distância entre Q1 e Q2 deverá
ser
a) 4 vezes maior. c) 16 vezes menor.
b) 4 vezes menor. d) 16 vezes maior.
40 – Dois condutores elétricos retilíneos são colocados 47 – A figura abaixo mostra um sistema constituído de duas
paralelamente, um ao lado do outro. Quando percorridos por barras A e B, rigidamente ligadas e em equilíbrio, suspensas
correntes elétricas contínuas, de mesma intensidade mas pelo ponto P. A barra B, na horizontal, forma um ângulo de 150º
sentidos contrários, os condutores apresentarão com a barra A. Desprezando o peso das barras e sabendo que
o comprimento da barra A é o dobro do comprimento da barra B,
a relação entre os pesos dos corpos 2 e 1, suspensos nas
extremidades das barras, vale
1 3
a) b) c) 3 d) 3
i i 3 3

P B
a) atração mútua.
A 
b) repulsão mútua.
c) manutenção na posição inicial. 150 º
d) atração e repulsão mútua, no decorrer do tempo.
2
2/2002-TURMA A 1
41 – Um dos grandes problemas que surgem como desafio para
a humanidade no início deste milênio é a escassez de água
potável – o que poderá gerar conflitos internacionais na mesma
proporção que os ocasionados pelo controle do petróleo nas 48 – Dois vetores de módulos 3 e 4 são somados. Se a soma
últimas décadas. Daí a grande preocupação da comunidade vetorial destes dois vetores é 37 , então eles formam entre si
científica em desenvolver técnicas para a obtenção de água
potável, como por exemplo, a dessanilização da água do mar. um ângulo, em graus, de
Das alternativas abaixo, indique aquela que contém o melhor a) 0 b) 30 c) 60 d) 90
processo para separar a água do sal.
49 – No esquema abaixo, os módulos dos vetores valem
  
a) Tamização c) Destilação fracionada a  3, b  7 e c  8. O valor do vetor resultante, de acordo
b) Destilação simples d) Dissolução fracionada
com o esquema citado, é
42 – Considere as afirmativas abaixo:
I- A água é uma mistura de hidrogênio e oxigênio. 
II- O ar puro das montanhas é uma substância pura. a) 8  b
III- A filtração é um processo utilizado, tanto na separação de b)
c)
7
3
a
misturas sólido-líquido, como sólido-gás.
d) zero
Podemos considerar corretas

a) II e III. c) somente II. 


b) I, II e III. d) somente III. c
43 – A idéia segundo a qual a estrutura atômica deveria ser 50 – Um projétil foi disparado em um local onde se admite que
semelhante à estrutura do sistema solar é devida a qualquer tipo de atrito seja desprezível e que a aceleração da gra-
vidade seja igual a 10 m/s2 (constante). A direção do disparo
a) Dalton. c) Lavoisier. formou um ângulo com a superfície horizontal de 30º, e a veloci-
b) Einstein. d) Rutherford. dade inicial do projétil valia V0. A distância horizontal percorrida
pelo projétil, 2 segundos após o disparo, vale, em metros,
44 – O raio atômico de um ânion é 1 3 1
a) 3 V0 b) V0 c) V0 d) V0
a) igual ao raio do átomo de origem. 2 2 4
b) maior que o raio do átomo de origem.
c) menor que o raio do átomo de origem. 51 – Durante o movimento de rotação de um disco de 36 cm de
d) igual ao raio de um cátion do átomo de origem. diâmetro, um ponto desenhado em sua periferia descreve arcos
de 120º a cada 2s. Então, um ponto situado a 6 cm do eixo de
45 – As Leis volumétricas aplicam-se a todas as reações rotação do disco terá uma velocidade linear, em  cm/s, igual a
a) 1 b) 2 c) 3 d) 4
a) químicas.
b) de combustão. 52 – Um automóvel, em movimento uniformemente variado, tem
c) entre substâncias gasosas. velocidade inicial de 10 m/s e aceleração igual a 10 m/s2. Após 5
d) de que pelo menos um gás participe. segundos, sua velocidade média, em m/s, e a distância
percorrida, em metros, valem, respectivamente,
46 – Identifique o tipo de reação química abaixo: a) 40 e 185. c) 35 e 175.
CaCO3  CaO + CO2 b) 45 e 190. d) 50 e 200.
a) Análise c) Permutação
b) Síntese d) Substituição 53 – Uma partícula percorre uma trajetória circular de raio igual
a 5 m, com velocidade linear de módulo constante. Entre os
instantes 1 s e 5 s, seu percurso é de 80 m; o período, em
segundos, do movimento apresentado será de
   
a) . b) . c) . d) .
2 4 6 8
54 – A Lei de Hooke é aplicável 66 – Um pulso propaga-se em uma corda composta e tensa,
a) dentro do limite elástico do material da mola considerada. conforme a figura. O sentido de propagação do pulso é da corda
b) para qualquer tipo de força que atua na mola considerada. de maior para a de menor densidade linear.
c) somente para carregamentos acima de 10 vezes o valor da
gravidade.
d) para valores que sempre são menores que a terça parte da
constante elástica da mola. 
55 – Para que a energia mecânica de um corpo se conserve, as A figura que melhor representa os pulsos e seus sentidos de
forças que realizam trabalho não nulo sobre ele devem ser propagação após a interação é a
a) iguais. c) constantes.
b) conservativas. d) dissipativas. a)

56 – A energia cinética é uma grandeza física que varia com a


velocidade e é considerada b)
a) absoluta, pois independe do referencial adotado.
b) vetorial, pois depende da velocidade, que é um vetor.
c) relativa, pois às vezes pode ser considerada um vetor e c)
outras vezes um escalar.
d) escalar, pois sua definição fica perfeitamente caracterizada
apenas pelo módulo.
d)
57 – Segundo Johannes Kepler (1571-1630), as órbitas
descritas pelos planetas em torno do Sol são ______________,
sendo que este ocupa um dos ______________ desta figura
geométrica. 67 – Com relação à velocidade de propagação do som,
c) circulares – focos c) elípticas – focos podemos afirmar que, de uma maneira geral, é
d) elípticas – vértices d) circulares – vértices a) nula nos sólidos.
b) máxima no vácuo.
58 – Uma cachoeira lança 15 m3 de água por segundo, forne- c) maior nos gases do que nos sólidos.
cendo uma potência de 4 . 103 CV. Assim sendo, a altura da d) maior nos sólidos do que nos líquidos.
queda d’água vale _______ metros.
Dados: 1 CV (cavalo-vapor) = 735 W (watt); 68 – No fenômeno da refração de ondas, necessariamente
densidade da água = 1 g/cm3; e g = 9,8m/s2. permanecem constantes
a) 5 b) 10 c) 15 d) 20 a) a velocidade de propagação da onda.
b) a freqüência das ondas.
59 – Um pára-quedista de massa 80 kg cai com uma velocidade c) o comprimento de onda da onda.
constante durante um percurso de 100 metros. Admitindo g = 10 d) as amplitudes das ondas.
m/s2, qual é o trabalho realizado, em J, pela força resultante que
sobre ele atua? 69 – O antimônio e o bismuto apresentam, durante o processo
a) zero b) 400 c)800 d) 8.000 de fusão, comportamento inverso ao da maioria das substâncias
conhecidas, pois seu volume ____________ com o aumento da
60 – Um objeto real de altura X é colocado frontalmente e a 20 pressão atuante no meio onde se encontra a substância.
cm de uma lente convergente de vergência 2,5 dioptrias. O a) aumenta c) diminui
aumento linear transversal dessa lente vale, em cm, b) oscila infinitamente d) permanece constante
a) X / 2. b) X. c) 2X. d) 4X.
70 – Um sistema recebeu do meio externo a ele uma quantidade
61 – O equilíbrio de líquidos em sistemas de vasos de calor igual a 8.000 cal e realizou, sobre esse meio, um
comunicantes pode ser considerado uma aplicação do princípio trabalho de 20.000 J. Em joules, qual a variação da energia
de interna desse sistema? ( considere 1 cal = 4,2 J)
a) Ohm. c) Galileu. a) 13.600 c) – 12.000
b) Stevin. d) Thomson. b) 12000 d) – 13600

62 – Todo corpo mergulhado num líquido experimenta um 71 – Uma gilete corta profundamente porque
empuxo vertical, de baixo para cima, igual, em módulo, ao a) a área de contato é grande e portanto a pressão é grande.
____________ do volume do líquido deslocado. b) a área de contato é pequena e portanto a pressão é
a) módulo c) fluxo grande.
b) espaço d) peso c) a área de contato é pequena e portanto a pressão é
pequena.
63 – Dos fenômenos descritos abaixo, qual representa um pro- d) a área de contato é a mesma, mas a pressão é grande
cesso de transmissão de calor que NÃO pode ocorrer no vácuo? devido à inclinação da lâmina.
a) Irradiação c) Refração
b) Convecção d) Reflexão 72 – Em alguns países, usa-se a escala Fahrenheit, que adota
os valores 32 para o ponto de gelo e 212 para o ponto de vapor.
64 – O comprimento de onda, em cm, da onda que se propaga a O intervalo entre essas duas medidas é dividida em ______
300 m/s, com uma freqüência de 5 kHz, vale partes.
a) 6. b) 15. c) 60 d) 150. a) 32 b) 100 c) 180 d) 212

65 – Uma onda de freqüência 120 Hz e comprimento de onda 73 – Uma lente de vidro plano-côncava, cujo índice de refração
igual a 2 cm passa de um meio para o outro. O ângulo de absoluto vale 1,5 e que está imersa no ar (índice de refração
incidência é 45o e o de refração, 30o. A velocidade, no segundo absoluto 1,0), tem a face curva com 10 cm de raio. Sua
meio, em cm/s, é vergência, em dioptrias, é igual a
a) 60. b) 120. c) 240. d) 120 2. a) – 5 b) – 3 c) 3 d) 5

74 – Segundo o Princípio de Pascal, os líquidos


a) são compressíveis.
b) multiplicam as pressões que suportam.
c) transmitem integralmente as pressões que suportam.
d) exercem sempre a mesma força em todos os pontos da
prensa hidráulica.
75 – A membrana de natureza nervosa, sensível à luz e que
está ligada ao nervo óptico, é denominada 46 – O bombardeamento da folha de ouro com partículas alfa,
a) retina. c) córnea. no experimento de Rutherford, mostra que algumas dessas
b) pupila. d) esclerótica. partículas sofrem desvio acentuado no seu trajeto, o que é
devido ao fato de que as partículas alfa
76 – Calcular, em watts, a potência total fornecida pela bateria. a) chocam-se com as moléculas de ouro.
Dado: o valor lido no amperímetro da figura é de 6 A. b) têm carga positiva e são repelidas pelo núcleo.
a) 60 c) são muito lentas e qualquer obstáculo as desvia.
b) 120 d) são grandes demais e não podem atravessar a lâmina de
R1
c) 180 30V R2
ouro.
15 R3
d) 240
A 47 – A pirita, o fluoreto de sódio e a gipsita são típicos exemplos
de
77 – Dispõem-se de quatro esferas metálicas carregadas: P, Q, a) sais.
R e S. Sabe-se que P repele Q, P atrai R, R repele S, e S está b) bases.
carregada positivamente. Pode-se dizer que c) ácidos.
a) P está carregada positivamente. d) óxidos.
b) P e R têm cargas de mesmo sinal.
c) Q tem carga negativa. 48 – A pirólise, a fotólise e a eletrólise são reações de
d) P e Q estão carregadas positivamente. decomposição pelo calor, pela luz e pela eletricidade,
respectivamente. Estas reações também são denominadas
78 – Ao dizermos que um motor elétrico tem uma potência de reações de
600W, estamos afirmando que nesse motor a) análise.
a) há um potencial elétrico de 600 volts a cada segundo, sem b) substituição.
a necessidade de existência de corrente elétrica. c) deslocamento.
b) 600 coulombs de carga elétrica são transportados a cada d) dupla substituição.
segundo, de uma extremidade à outra do circuito.
c) 600 coulombs de carga elétrica são transformados em 1 49 – Considere as forças atuantes sobre a barra, de peso
joule de energia mecânica a cada segundo. desprezível, conforme a figura. Qual o módulo do momento
d) 600 joules de energia elétrica são transformados em resultante, em N.m, em relação ao ponto O?
energia mecânica a cada segundo.   
Dados: F1  3N, F2  5N e F3  3N
79 – São exemplos de objetos que se utilizam do princípio do
eletroímã: a) 30
a) telefone e campainha. b) 40
b) cafeteira elétrica e campainha. O
c) 50
c) lâmpada incandescente e telefone. d) 70
d) retificador de corrente e ponte de Wheatstone.
5m 5m 5m
80 – Quando uma corrente elétrica circula por um fio, gera ao
 
redor deste um
F1  F3
a) fluxo elétrico. c) circuito elétrico. F2
b) campo magnético. d) pêndulo elétrico.

2/2002-TURMA B 50 – Qual alternativa só contém grandezas vetoriais?


a) comprimento, massa e força.
41 – A maioria dos pequenos produtores de aguardente de cana b) tempo, deslocamento e altura.
(“pinga”), no Brasil, utilizam ainda um pequeno equipamento c) força, deslocamento e velocidade.
criado pelos alquimistas, conhecido por alambique, que separa d) massa, velocidade e deslocamento.
os componentes da mistura pelo processo de 
a) liquidação fracionada. 51 – A figura, abaixo, mostra um bloco de peso P sustentado
b) destilação fracionada. por fios ideais.
c) destilação simples. 
d) adsorção. Calcule o módulo da força F horizontal, supondo que o
conjunto esteja em repouso.
42 – No alto de uma montanha, a temperatura de ebulição da a) F = P . tg  Fio 1 
água se dá: b) F = P . sen  F
a) abaixo de 100º C. c) F = P . cos  
b) acima de 100º C. d) F = P . sen  . cos  Fio 2
c) a 100º C.
d) a 0º C.

43 – Das alternativas abaixo, é exemplo de substância pura:



P
a) leite .
b) latão.
c) ar sem poluição. 52 – Observe as equações horárias da velocidade dos móveis I,
d) tetracloreto de carbono. II e III, supondo que a trajetória de todos os três seja retilínea:
móvel I : V = 2 + 3t
44 – Dados quatro átomos: 20A, 12B, 38C e 4D, qual deles móvel II : V = – 5 – 3t
apresenta maior raio atômico? móvel III : V = 3
a) 12B
b) 38C Elas representam, respectivamente, movimentos
c) 20A a) uniforme, uniformemente retardado e uniforme.
d) 4D b) uniformemente acelerado, uniformemente acelerado e
uniforme.
45 – Se um elétron move-se de um nível de energia para outro c) uniformemente acelerado, uniformemente retardado e
mais afastado do núcleo do mesmo átomo, pode-se afirmar que uniforme.
a) há emissão de energia. d) uniformemente retardado, uniformemente acelerado e
b) há absorção de energia. uniforme.
c) o número atômico varia.
d) não há variação de energia.
53 – "O guepardo, também conhecido como chitá, é o mais 61 – Em relação ao movimento dos planetas em torno do Sol,
rápido dos animais terrestres. Ele depende de sua velocidade segundo as leis de Kepler, é correto afirmar que a velocidade
de até 120 km/h para alcançar animais velozes como gazelas e linear, em módulo, dos planetas é
antílopes..." (revista SuperInteressante, dezembro de 2000). a) maior quando eles estão no periélio.
Admitindo que o guepardo desenvolva sua velocidade máxima, b) menor quando eles estão no periélio.
como descrita acima, e sendo constante essa velocidade por 10 c) maior quando eles estão no afélio.
segundos, a distância percorrida, em linha reta, por esse animal d) sempre constante.
durante este intervalo de tempo vale aproximadamente
a) 333 m. 62 – Alguns pedreiros utilizam um pedaço de tubo ou
b) 333 km. mangueira, preenchido com água, para verificar os níveis (ou
c) 360 km. alturas) das superfícies em que estão realizando suas obras.
d) 360 m. Pode-se dizer, portanto, que isto é uma aplicação prática
a) da Lei de OHM.
54 – Uma força de intensidade igual a 9 10 N foi decomposta b) do Princípio da Ação e Reação.
em duas componentes ortogonais, de modo que a intensidade c) da Lei da Gravitação Universal.
de uma é o triplo da outra. Qual é, em newtons, a intensidade de d) do Princípio dos Vasos Comunicantes.
cada componente?
a) 3 e 9 c) 10 e 30 63 – No vaso abaixo, qual é, em bárias, a pressão no ponto “A”,
b) 9 e 27 d) 81 e 243 sabendo que a densidade do líquido é de 0,8 g/cm 3? (Despreze
a pressão na superfície do líquido e considere g = 10 m/s 2).
55 – O movimento de translação da Terra, em relação ao Sol,
pode ser aproximado, com algumas restrições, a um a) 40.000
movimento circular uniforme. Nesse caso, podemos afirmar que, b) 50.000
durante seu movimento, a Terra possui ____________ c) 400.000 10m
constante. d) 500.000 30º
a) posição A
b) aceleração
c) velocidade linear
d) velocidade angular

56 – Dois trens correm em trilhos paralelos, deslocando-se na 64 – Um balão de festa junina começa a subir porque
mesma direção e no mesmo sentido. O passageiro do primeiro a) a pressão dos gases no balão é menor que a pressão
trem, cujo módulo da velocidade é de 80 km/h, passa pelo atmosférica.
segundo trem, que possui uma velocidade de módulo igual a 70 b) o peso do balão é menor que o peso do ar que ele desloca.
km/h. c) a aceleração da gravidade diminui com a altitude.
Admitindo que o movimento dos trens seja retilíneo e uniforme, d) o volume do balão diminui quando o balão sobe.
qual o comprimento, em metros, do segundo trem, se o
passageiro o vê durante 1 min e 12s? 65 – Observa-se que a difração é tanto mais intensa quanto
a) 300 c) 200 a) maior for o obstáculo.
b) 250 d) 150 b) menor for o obstáculo.
c) mais forte for o sinal emitido pela fonte.
57 – Um móvel descreve um movimento circular uniforme d) mais fraco for o sinal emitido pela fonte.

obedecendo à função horária  = + t, sendo as unidades 66 – A qualidade do som que permite, na maioria das situações,
2 distinguir a voz de uma criança ou de uma mulher, em relação à
dadas no Sistema Internacional de Unidades. Com a trajetória voz de um homem, é denominado
de raio igual a 0,5 m, qual o comprimento do arco descrito pelo a) altura.
móvel, em metros, no intervalo de tempo de 2s? b) intensidade.
a)  c) 2,00 c) velocidade de propagação.
b) 1,25 d) 2,50 d) densidade do meio material.

58 – A força resultante que atua sobre uma pequena esfera, que 67 – Uma garrafa de alumínio (coeficiente de dilatação linear
cai verticalmente no interior de um líquido, torna-se nula a partir  = 22 x 10-6 ºC-1 ), com volume de 808,1 cm3, contém 800 cm3 de
de um determinado instante. A partir desse instante, a esfera glicerina (coeficiente de dilatação volumétrica  =147 x 10-6 ºC-1)
a) permanece parada. à temperatura de 0ºC. A temperatura, em ºC, a que deve ser
b) é acelerada para cima. aquecido o conjunto para que o frasco fique completamente
c) é acelerada para baixo. cheio, sem haver transbordamento de glicerina, é de aproxi-
d) continua descendo com velocidade constante. madamente,
a) 100. b) 125. c) 225. d) 375.
59 – Uma esfera, de dimensões desprezíveis, possui peso igual
a 10 N. Essa esfera encontra-se suspensa verticalmente por um 68 – Um tubo sonoro, de comprimento igual a 0,5 m, apresenta
fio ideal de comprimento 50 cm, cuja outra extremidade está fixa as duas extremidades abertas. Sabendo que a velocidade do
em um teto. Deslocando-se, lentamente, a esfera de sua som no ar é igual a 340 m/s, e que a freqüência do som emitido
posição de equilíbrio até uma nova posição na qual o fio é de 1700 Hz, conclui-se que o tubo está produzindo o __
esticado faça com a vertical um ângulo de 60o, pode-se afirmar harmônico.
que sua energia potencial sofrerá uma variação total, em J, de a) 1º b) 3º c) 5º d) 6º
a) 5,0 c) – 2,5
b) 2,5 d) – 5,0 69 – A coluna de mercúrio de um termômetro apresenta uma
2h 11h
60 – Uma cachoeira lança 20 m 3 de água por segundo, de uma altura de , quando a 0ºC, e , quando a 100ºC, sob
altura de 15 m. Assim sendo, a potência fornecida, em CV, vale 3 4
Dados: densidade absoluta da água = 1,0 g/cm 3; pressão normal. A temperatura correspondente à altura “h” da
1 CV(cavalo-vapor) = 735 W; e coluna vale, em ºC,
g (aceleração da gravidade local) = 9,8 m/s2. a) 16. b) 18. c) 22. d) 33.
a) 1000 c) 3000
b) 2000 d) 4000
70 – Flávio, um brilhante estudante de Física, comprou um 78 – Observe:
termômetro clínico graduado. Junto ao termômetro veio um
manual de instrução, onde se lia “deixe o termômetro sob as I - Para o estudo do campo magnético, convencionou-se que as
axilas ou na boca por aproximadamente 3 minutos. Após esse linhas de campo magnético são orientadas do pólo sul para
tempo, faça a leitura da temperatura”. O estudante pode o pólo norte.
concluir, corretamente, que: II - As substâncias diamagnéticas, tais como cobalto e níquel,
não possuem propriedades magnéticas, não podendo,
a) o tempo não altera a leitura da temperatura. portanto, ser imantadas.
b) este tempo poderia ser aumentado para 5 minutos sem III - Quando um ímã é dividido em várias partes, cada uma das
afetar a medição. partes comporta-se como um novo ímã.
c) a leitura deve ser feita somente com 3 minutos, não IV - Em torno de um fio condutor retilíneo longo, percorrido por
podendo ultrapassar esse tempo. corrente elétrica, surge um campo magnético.
d) a correta leitura deve ser feita imediatamente após o
contato do bulbo do termômetro com o corpo. São corretas as afirmações
a) I, II, III e IV. c) I, II e III.
71 – “Água que o Sol evapora b) II, III e IV. d) III e IV.
Pro céu vai embora
Virar nuvem de algodão” 79 – O trabalho para deslocar uma carga elétrica entre dois
O trecho acima, retirado da música “Planeta Água”, de pontos que pertençam à mesma superfície eqüipotencial
Guilherme Arantes, faz referência à mudança de estado físico a) depende do valor da carga.
da água a partir da energia térmica do Sol que é transferida para b) é negativo.
esta última, principalmente, pelo processo de c) é infinito.
d) é nulo.
a) convecção. c) condução.
b) irradiação. d) difração. 80 – Duas partículas A e B possuem cargas elétricas nula e –2e
, respectivamente, em que e é a carga do elétron em módulo.
72 – Em relação à velocidade de propagação de luzes Tais partículas atravessam, separadamente, um campo
monocromáticas, pode-se afirmar corretamente que a luz magnético constante perpendicular ao plano de movimento
a) vermelha é mais lenta que a violeta no vácuo. destas, como mostra a figura. A trajetória das partículas pode
b) violeta é mais lenta que a vermelha no vácuo. ser expressa por
c) violeta é mais rápida que a vermelha num meio material.
d) vermelha é mais rápida que a violeta num meio material.
X X
73 – O satélite artificial Hubble possui um telescópio que usa um
espelho _______________ para ampliar as imagens das B
X X
estrelas.
a) plano. c) convexo. A
b) côncavo. d) plano, com inclinação variável. X X

74 – Um prisma eqüilátero ( índice de refração n= 2 ) está imerso


no ar (índice de refração n= 1). O desvio mínimo, em graus, sofrido
por um raio luminoso monocromático ao atravessá-lo é A B
a) c)
a) 30. b) 45. c) 60. d)
0. B A
75 – ANULADA
A B
b) d)
76 – O gráfico que melhor representa a relação entre
intensidade de corrente elétrica ( i ) e o tempo ( t ), no caso de B A
corrente contínua, é
a) c)
i t 1/2003-TURMA A
41 – Quando só há interesse em se obter um sólido de uma
mistura entre um sólido e um líquido, como é o caso da
t i obtenção do sal de cozinha a partir da água do mar, deve-se
fazer uma
b) d) a) liquefação. c) destilação simples.
b) evaporação. d) destilação fracionada.
i t
42 – Ocupa o volume de 22,4 litros um mol de qualquer
a) substância.
b) substância gasosa.
t i c) substância líquida.
d) substância gasosa nas condições normais de temperatura
77 – A conta de luz de uma residência apresenta os seguintes e pressão.
dados:
leitura anterior leitura atual importância a ser 56
43 – O elemento químico 26 X pode ser classificado como
kWh kWh paga em R$
5250 5750 100,00 a) gás nobre. c) metal de transição.
b) metal alcalino. d) metal alcalino terroso.
Considerando esses dados, quanto custaria, em R$, a
iluminação de uma casa, na qual o consumo seria dado apenas 44 – Sabendo-se que o valor numérico máximo da soma de dois
por 20 lâmpadas de 100W e 120V que permaneceram acesas 4 vetores é 20 e o mínimo é 4, os módulos dos vetores que
horas por dia durante 30 dias? conduzem a estes resultados são
a) 28,00 b) 38,00 c) 48,00 d) 52,00 a) 9 e 11. c) 20 e 4.
b) 8 e 12. d) 16 e 4.
45 – Os ponteiros de um relógio realizam movimento circular 57 – A luz branca do Sol ou a luz emitida pelo filamento
que pode ser considerado uniforme. Qual será, em rad/s, a incandescente de uma lâmpada comum é
velocidade angular do ponteiro dos segundos? a) verde clara. c) policromática.
   b) acromática. d) monocromática.
a) b) 2 c) d)
2 20 30 58 – Dos dispositivos abaixo, qual só pode funcionar com cor-
46 – Alguns cães balançam suas caudas com uma freqüência rente elétrica alternada?
aproximada de 1 Hz. Nesse caso, pode-se afirmar que a velocidade a) O resistor. c) O transformador.
linear de um ponto, em m/s, na extremidade da cauda, é da ordem b) O eletroímã. d) A lâmpada incandescente.
de
a) 31 b) 53 c) 35 d) 37 59 – O físico alemão George Ohm foi um dos grandes
responsáveis pela teoria que explica os fenômenos relacionados
47 – Que aceleração existe no movimento circular uniforme? com a corrente elétrica. Assim, é justa a homenagem prestada
a) Centrípeta c) Deslizante no Sistema Internacional de Unidades, que adota o seu
b) Tangencial d) Curvilínea sobrenome como unidade para a grandeza
a) diferença de potencial. c) rigidez dielétrica.
48 – Deixa-se cair de uma mesma altura e ao mesmo tempo b) resistência elétrica. d) corrente elétrica.
três objetos de formas e volumes iguais, sendo um de ferro, um
de chumbo e outro de isopor. Admitindo que a densidade do 60 – O barão de Munchausen é considerado o maior mentiroso
isopor é menor que a do ferro e que esta é menor que a do da literatura internacional. Em uma das suas aventuras, o
chumbo, podemos afirmar que simpático barão conta que, ao se ver afundando no pântano,
conseguiu escapar puxando seus próprios cabelos para cima. A
Dado: existe atmosfera no local da queda dos corpos. solução proposta pelo barão NÃO está de acordo com a
a) o objeto de ferro chegará primeiro ao solo. a) 3ª lei de Newton. c) 2ª lei de Newton.
b) o objeto de isopor chegará primeiro ao solo. b) lei de Arquimedes. d) 1ª lei de Newton.
c) o objeto de chumbo chegará primeiro ao solo.
d) todos os objetos, independente do material que os 61 – Em um cristal de cloreto de sódio, a distância entre dois
constitui, chegarão juntos ao solo. íons adjacentes Na+ e Cl– é, aproximadamente, 3 x 10 –10 m. Se
o cristal for mergulhado na água, o valor da força de atração
49 – Analisando o gráfico da posição pelo tempo de uma entre os íons tornar-se-á
partícula em movimento retilíneo uniforme, podemos afirmar que
o significado físico do coeficiente linear deste é DADOS: Constante dielétrica do vácuo = 9 x 10 9 N.m2/ C 2
a) a velocidade média do móvel. Constante dielétrica da água = 9 x 10 7 N. m2/ C 2
b) a velocidade inicial do móvel. Carga do elétron = 1,6 x 10 – 19 C
c) a velocidade instantânea do móvel. a) 100 vezes maior. c) 500 vezes maior.
d) a posição inicial do móvel, em relação ao referencial b) 100 vezes menor. d) 500 vezes menor.
adotado.
62 – Substâncias magnéticas são aquelas que permitem
50 – O Princípio da Conservação da Energia Mecânica a) a orientação dos seus ímãs elementares.
desconsidera a existência de b) o deslocamento de íons neutros pelo condutor.
a) força. c) atrito. c) o armazenamento de energia de origem térmica.
b) massa. d) gravidade. d) a orientação cruzada de algumas cargas elétricas.

51 – Em 1751, um meteorito de massa 40 kg caiu na Terra, abrindo 63 – O campo magnético é um ______________ tendo a
uma cratera com 4 metros de profundidade. Investigações sobre a mesma ___________das linhas de campo e sendo
força de resistência oferecida pelo solo nas vizinhanças da colisão, ____________ a elas.
mostraram que o seu valor foi de 5 x 10 4 N. A velocidade, em m/s, a) vetor – fase – tangente
com que o meteorito chegou à superfície da Terra vale b) vetor – orientação – tangente
c) escalar – orientação – perpendicular
a) 25. b) 50. c) 100. d) 75. d) escalar – fase – perpendicular

52 – A grandeza dada pela relação entre a intensidade da força 64 – A correção do estrabismo é feita com o uso de lentes
resultante e a área em que ela é aplicada denomina-se a) côncavo-convexas. c) plano-convexas.
a) impulso. c) pressão. b) plano-côncavas. d) prismáticas.
b) pulso. d) depressão.
65 – A temperatura de um gás que sofre uma compressão
53 – O som mais grave é o de adiabática
a) menor freqüência. c) maior intensidade. a) aumenta. c) é invariável.
b) maior freqüência. d) menor intensidade. b) diminui. d) pode aumentar ou diminuir.

54 – Uma pessoa submersa em uma piscina ouve o ruído de 66 – A diferença de pressão entre dois pontos de uma mesma
uma explosão ocorrida fora da água. O fenômeno ondulatório massa líquida sob a ação da gravidade é igual ao produto da
que certamente ocorreu na fronteira entre o ar e a água é massa específica do líquido pela
denominado a) distância entre dois pontos, somente.
a) difração. c) dispersão. b) diferença de altura entre os pontos, somente.
b) refração. d) ressonância. c) gravidade local e pela diferença de alturas entre os pontos
considerados.
55 – Uma corda de comprimento L é posta a vibrar continuamente d) gravidade local e pela distância de ponto até a superfície.
entre dois extremos fixos. Observa-se ao longo de todo seu
comprimento quatro nós e três ventres. Calcule a relação entre o 67 – Com relação à massa e ao peso dos corpos é correto afirmar
comprimento da corda e o comprimento de onda na estacionária que:
formada. a) o peso é uma grandeza escalar e mede a inércia do corpo.
a) 0,15 b) 1,5 c) 15 d) 150 b) a massa é uma grandeza vetorial e mede a inércia da
matéria.
56 – A temperatura, a uma dada pressão, acima da qual uma c) o peso é uma grandeza vetorial cuja intensidade independe
substância é considerada um gás, é denominada do local em que é verificado.
a) fundamental. c) de fusão. d) a massa é uma grandeza escalar cuja intensidade
b) principal. d) crítica. independe do local em que é verificado.
68 – Para que a água seja própria para o consumo diário da 78 – Como devem estar ligados cinco capacitores de 2 F cada
população, ela precisa passar por alguns processos de trata-
um, se a capacidade final da associação é de 12 F ?
mento. Em uma dessas etapas, adiciona-se uma quantidade de 5
hidróxido de alumínio à água, visando à formação de grandes a) Todos em série.
flocos de sujeira. Esse procedimento é realizado para que se b) Todos em paralelo.
otimize o próximo processo de tratamento que é a c) Dois em paralelo associado em série com três em paralelo.
a) decantação. c) cloração. d) Três em série associado com dois em paralelo.
b) cristalização. d) aeração.
79 – Dois trens movem-se sobre a mesma reta indo um de
69 – Uma partícula tem sua posição variando segundo a encontro ao outro, com velocidades de 72 km/h e 108 km/h. Os
seguinte função horária: S = 8 – 6t (SI). O módulo da velocidade dois maquinistas aplicam os freios simultaneamente, quando a
média, em m/s, entre os instantes t1 = 1s e t2 = 5s, vale distância entre eles é de um quilômetro. Supondo a aceleração
escalar dos trens igual a 1m/s2, os dois trens
a) 1,2 b) 5,5 c) 6,0 d) 7,5 a) colidem.
b) param a 1m um do outro.
70 – Em uma transformação isobárica, c) param a 100m um do outro.
____________permanece constante, considerando-se o fluido d) param a 350m um do outro.
como gás ideal e sendo P a pressão, V o volume e T a
temperatura do gás. 80 – A velocidade é
a) o produto VxT c) o produto P x V a) uma grandeza vetorial.
b) a razão P d) a razão V b) uma grandeza tanto escalar como vetorial.
T T c) uma grandeza escalar.
d) não é grandeza
71 – Dez mols de um gás perfeito evoluem isobaricamente, pas-
sando de uma temperatura de 300 K para uma de 400 K. Nessas
condições o trabalho realizado na evolução, em atm . litro, vale
1/2003-TURMA B
(dado: constante geral dos gases = R = 0,082 atm . litro / K . mol)
a) 41 b) 82 c) 123 d) 164 41 – A força magnética que age entre duas partículas, que são
atraídas uma pela outra, constitui um(a)
72 – Uma pessoa, colocada no fundo de uma piscina vazia (sem a) par ação-reação.
água), observa um avião que se encontra a 200 metros de altura b) força de contato simples.
em relação ao fundo da piscina. Admitindo que os índices de c) método kepleriano de cálculo virtual.
refração do ar e da água sejam, respectivamente, 1 e 1,5, a d) força de ação somente a pequenas distâncias.
altura aparente, em metros, observada pela pessoa, do avião
vale 42 – Dentre as grandezas abaixo, aquela que deve ser motivo
a) 300 b) 200 c) 100 d) 50 de atenção por parte do consumidor, no momento de comprar
um eletrodoméstico, para não ter maiores gastos com o
73 – Quando um raio de luz incide na superfície de separação consumo de energia elétrica é
de dois meios X e Y, vindo do meio X para o meio Y, sofre um
fenômeno denominado refração da luz. Sabendo que o índice a) o potencial elétrico.
de refração no meio X é maior que o do meio Y, podemos b) a potência elétrica.
afirmar que o raio refratado c) a resistência elétrica.
a) se afasta da normal. c) incide pela normal. d) a corrente elétrica.
b) se aproxima da normal. d) não sofre desvio.
43 – É FALSO afirmar que
74 – Três lâmpadas: A, B, e C são ligadas em paralelo. Suas a) os gases possuem grande expansibilidade.
resistências são respectivamente 1,5 ohms; 2,5 ohms e 4,0 b) os líquidos oferecem grande resistência à compressão.
ohms. Podemos afirmar que c) somente os líquidos podem ser considerados fluidos
a) o menor brilho é da lâmpada A. perfeitos.
b) o maior brilho é da lâmpada C. d) a viscosidade não influi no estudo dos líquidos em
c) B brilha mais que A e menos que C. equilíbrio.
d) B brilha mais que C e menos que A.
44 – Um galvanômetro é um tipo de
75 – Quando queremos proteger um aparelho qualquer contra a) pilha. c) amperímetro.
as influências elétricas, nós o envolvemos com uma capa b) wattímetro. d) potenciômetro.
metálica. Isso se justifica devido ao fato de
a) os metais serem maus condutores de eletricidade. 45 – Para a correção da miopia deve-se __________ a distância
b) o campo elétrico no interior de um condutor não ser nulo. focal do sistema, o que é conseguido associando-se ao globo
c) a carga elétrica se distribuir na superfície externa do con- ocular uma lente ___________.
dutor em equilíbrio eletrostático. e) aumentar – divergente
d) a maioria dos campos elétricos produzidos em circuitos f) diminuir – divergente
elétricos ser infinitamente pequenos. g) diminuir – convergente
h) aumentar – convergente
76 – Seja uma carga elétrica lançada num campo magnético
uniforme, perpendicularmente às linhas de campo. Nessas 46 – Uma corda de violão vibra com um único ventre. Se “l” é o
condições, a sua trajetória será comprimento dessa onda e “f” a freqüência do som emitido, a
a) elíptica. c) retilínea. velocidade da onda que está se propagando através da corda é
b) circular. d) parabólica.
a) 21f b) 1f c) 1f/2 d) f/1
77 – Um solenóide de comprimento L e N espiras, no qual
circula uma corrente de intensidade i, apresenta no seu interior 47 – Um som que oscila com 2.000 vibrações por segundo é
um vetor campo magnético de intensidade B. Sendo a considerado
a) som audível. c) ultra-som.
permeabilidade magnética do ar 0, para reduzir à metade a
b) micro-som. d) infra-som.
intensidade desse vetor campo magnético deve-se
a) dobrar o número de espiras.
48 – A razão de ser impossível ouvir-se uma explosão no Sol é
b) dobrar a intensidade da corrente.
porque
c) quadruplicar o número de espiras.
a) a Terra está muito distante do Sol.
d) dobrar o comprimento do solenóide.
b) as ondas sonoras são mecânicas.
c) as ondas de explosão solar são desconhecidas.
d) o Sol é um astro que emite apenas ondas luminosas.
49 – Em uma vitrola, a agulha aplica sobre o disco uma força de 57 – Para se determinar graficamente a resultante de três forças
intensidade 10 - 2 N , aproximadamente. Tendo a ponta da coplanares, que não possuem a mesma direção, o processo
agulha área igual a 10 - 1 0 m 2 , determine a pressão exercida, em mais indicado é o
bárias, pela agulha no disco: a) do paralelogramo. c) do polígono.
a) 10 7 b) 10 8 c) 10 9 d) 10 1 0 b) do triângulo. d) analítico.

50 – Sabendo-se que a massa de uma partícula  tem valor de 58 – A aceleração da gravidade, nas proximidades da superfície
6,67 x 10 – 27 k g, pode-se afirmar que a energia cinética desse terrestre, tem um valor muito alto (aproximadamente 10 m/s 2),
tipo de partícula, em 10 – 12 J , que esteja movendo-se com uma quando comparado aos valores de aceleração de outros
velocidade de valor igual a 20.000 km/s, vale: veículos. Se um automóvel tivesse essa aceleração, em apenas
a) 0,224 b) 1,334 c) 2,554 d) 3,333 4s, partindo do repouso e desprezando qualquer tipo de atrito,
ele atingiria uma velocidade, em km/h, igual a
51 – Considere um tubo de PVC que atravessa uma parede, de a) 40 b) 72 c) 102 d) 144
forma que você veja apenas uma de suas extremidades.
Lançando uma bola dentro do tubo, você observa que, após 59 – Um móvel apresentará aceleração centrípeta não nula,
algum tempo, ela retorna com uma velocidade maior do que desde que a
aquela com que você a lançou. Diante do acontecido e a) a velocidade linear varie somente em intensidade.
considerando que o Princípio da Conservação da Energia seja b) a velocidade linear varie somente em sentido.
válido, você afirma corretamente que: c) a trajetória seja curvilínea.
a) o tubo de PVC está inclinado e a extremidade do outro lado d) a trajetória seja retilínea.
da parede está a uma altura maior que a da extremidade
que você observa. 60 – Qual a razão entre as distâncias percorridas por dois
b) há alguém do outro do lado da parede que lança a bola corpos em queda livre, se a duração de um é o dobro da do
com velocidade maior. outro? Considere que os corpos partam do repouso.
c) as paredes do tubo não oferecem atrito ao movimento da 1 1
bola. a) c)
2 4
d) o tubo tem forma de um “L”.
1 1
b) d)
52 – Das alternativas abaixo, qual a única que NÃO 3 5
corresponde a uma propriedade dos ácidos, segundo a
definição de Arrhenius. 61 – Em um movimento uniformemente variado,
a) Conduz eletricidade. a) a aceleração varia uniformemente com o tempo.
b) Apresenta sabor azedo. b) a velocidade varia uniformemente com o tempo.
c) Torna azul o tornassol vermelho. c) o espaço percorrido varia linearmente com o tempo.
d) Quando adicionado a um carbonato, produz efervescência d) o móvel percorre distâncias iguais, em tempos iguais.
com liberação de gás carbônico.
62 – Os números quânticos caracterizam a
53 – A desintegração de 1 grama de Carbono 14, até restarem a) energia do elétron no átomo.
0,5 grama deste elemento, dura 5.600 anos. Por isso seu uso b) energia do átomo excitado.
para datação de fósseis. c) energia do átomo no estado fundamental.
d) distribuição eletrônica em níveis de energia de um átomo.
Assim, se o contador Geiger fornecer uma contagem radioativa
de 1 da contagem que possuímos hoje, por grama do 63 – Observe atentamente o seguinte trecho do Hino Nacional
4 brasileiro:
elemento Carbono 14, significa que o objeto examinado existe “Se o penhor desta igualdade, conseguimos conquistar
há com braço forte...”
a) 1.400 anos.
b) 2.800 anos. Do ponto de vista da Física, o termo forte é
c) 11.200 anos.
d) 22.400 anos. a) corretamente utilizado, tendo em vista que, pela Lei de
Hooke, a elasticidade dos músculos do braço faz desse
54 – O raio médio da Terra é de 6.400 km, aproximadamente. A membro fonte de força.
Terra gira em torno de seu próprio eixo, realizando uma rotação b) corretamente utilizado, pois sendo o braço dotado de
completa em 24 h. Considerando-se dois pontos, um na super- massa, ele possui força a partir de uma aceleração mínima
fície da Terra e outro a 3.200 km de seu centro, podemos a ele comunicado.
afirmar corretamente que c) incorretamente utilizado, uma vez que o conceito de força
a) os dois pontos terão velocidades angulares diferentes, que está relacionado com a interação entre dois ou mais
os dados fornecidos não permitem calcular. corpos.
b) a velocidade linear do ponto a 3.200 km do centro será d) incorretamente utilizado, já que não se faz referência à
maior que a do ponto na superfície. massa e a constante elástica dos músculos do braço.
c) o ponto da superfície terá uma velocidade angular duas
vezes maior que o outro. 64 – O ar atmosférico filtrado pode ser considerado uma mistura
d) os dois pontos terão a mesma velocidade angular. homogênea de vários gases, entre eles o nitrogênio e o
oxigênio. Industrialmente se obtém o oxigênio e o nitrogênio, a
55 – Nos palitos de fósforos não existe fósforo. Ele está nas partir do ar atmosférico filtrado, pelo(s) processo(s) de
tiras (lixas) laterais existentes na caixa, na forma de fósforo
vermelho. Pelo atrito palito/tira, o fósforo vermelho se transforma a) solidificação.
em fósforo branco, liberando energia. O fósforo vermelho e o b) centrifugação.
fósforo branco são chamados de c) liquefação fracionada.
a) isótopos. c) isótropos. d) liquefação e destilação fracionada.
b) isóbaros. d) alótropos.
65 – A força resultante que age sobre um ponto material em
56 – O tipo de ligação química que se caracteriza pela movimento circular uniforme em um plano horizontal
existência de “elétrons livres”, também chamados de “mar de a) não realiza trabalho.
elétrons”, é a ligação b) tem intensidade nula.
a) iônica. c) metálica. c) é tangente à trajetória em cada ponto.
b) simples. d) covalente. d) é diretamente proporcional à velocidade da partícula.

66 – Dois blocos A e B de massas m A = 3 kg e mB = 6 kg estão


ligados por um cabo, perfeitamente rígido e homogêneo, cuja
massa vale 1 kg, conforme a figura abaixo. Admitido que o existe um campo magnético uniforme, de 20 tesla, conforme a
conjunto seja submetido a uma força de 50 N, a tensão, em N, figura. Qual deve ser a velocidade, em m/s, desta carga para
na metade do cabo, vale: que a trajetória descrita seja uma semicircunferência de diâmetro 0,2
Despreze os atritos. cm?
a) 10 10
a) 25,0 b) 10 6
b) 25,5
 c) 2 x 10 6
F
c) 27,5 d) 200 x 10 6
d) 32,5 B A

67 – Uma onda sonora propaga-se em um local onde a


densidade e a temperatura do ar permanecem constantes,
encontrando um anteparo colocado a 170 metros da fonte,
refletindo-se sem perdas de energia. O tempo total, em 73 – Dois fios condutores X e Y têm o mesmo comprimento,
segundos, decorrido entre a emissão e o eco, vale (dado: porém a resistência elétrica de X é 4 vezes a de Y e a
velocidade do som no ar = 340 m/s)
resistividade de Y é 1 da de X. Portanto, conclui-se que a razão
a) 1 b) 2 c) 3 d) 4 9
X
de diâmetros é dada por
68 – Considerando a velocidade do som no ar de 340 m/s, Y
pode-se afirmar que o limite inferior e o superior dos sons
audíveis têm, respectivamente, comprimentos de onda da 3 2 9 3
a) b) c) d)
ordem de grandeza de _______ e de ________. 2 3 2 4
a) 10 2 – 10 - 3 c) 10 2 – 10 - 2
b) 10 1 – 10 - 3 d) 10 1 – 10 - 2 74 – Numa antiga propaganda de uma grande loja X de
departamentos, existia o seguinte refrão:
“ – Quem bate?
69 – Imagine que você tenha a missão de orientar um agente – É o frio!
secreto em seu trabalho de espionagem. Admita que o agente – Não adianta bater, pois eu não deixo você entrar, os
deva percorrer o trajeto do ponto A ao ponto D, no menor tempo cobertores da loja X é que vão aquecer o meu lar!”
possível (veja a figura abaixo). Admitindo que o agente corra
com uma velocidade quatro vezes maior do que aquela com que Do ponto de vista da Física, o apelo publicitário é
ele nade e que essas velocidades possam ser constantes, a) correto pois, dependendo da espessura do cobertor, este
assinale a alternativa que você indicaria para o agente ter pode impedir a entrada do frio.
sucesso em sua missão. b) correto pois, independente da espessura do cobertor, este
16
16mm é um excelente isolante térmico, impedindo a entrada do
frio.
8m
margem do
c) incorreto pois não foi definida a espessura do cobertor.
D rio d) incorreto pois não tem sentido falar em frio entrando ou
B C
saindo já que este é uma sensação que ocorre quando há
6m trocas de calor entre corpos de diferentes temperaturas.

A margem do 75 – Um feixe de luz amarela monocromática, no vácuo, tem velo-


rio cidade de 300.000 km/s, aproximadamente. Ao incidir em um
a) c) determinado meio I, apresenta uma velocidade de 2,0 x 10 8 m/s
e ao incidir no meio II, apresenta uma velocidade de módulo V.
C D D Sendo o índice de refração relativo n1,2 = 9/8 , o valor de V, em
10 8 m/s, é aproximadamente
a) 3,00 b) 2,75 c) 2,50 d) 2,25
A A
76 – Um amperímetro precisa ser colocado em um determinado
ponto de um circuito, com o único objetivo de mostrar que está
b) d) qualquer uma das circulando corrente. Porém este amperímetro tem uma resis-
B C D trajetórias tência interna de 10  e a corrente máxima que ele suporta é de
sugeridas acima. 2,0 A. Considerando que neste circula uma corrente de 12,0 A,
o amperímetro deve ser ligado em
A a) série com um resistor de 2 .
b) série com um resistor de 20 .
c) paralelo com um resistor de 2.
70 – É muito comum ouvirmos, pelos meios de comunicação, as d) paralelo com um resistor de 20 .
unidades listadas abaixo relacionadas à potência de uma
hidroelétrica. Do ponto de vista da Física, qual a única alterna- 77– A potência de um coração, em watts, que bate setenta
tiva que contém a unidade correta? vezes por minuto e bombeia 72 cm 3 de sangue em cada batida,
a) quilowatts. c) quilowatts . hora. contra uma pressão de 12 cmHg, vale aproximadamente
b) quilowatts/hora. d) Dados: densidade do mercúrio = 13,6 g/cm3
quilowatts/segundo. aceleração da gravidade = 9,81 m/s2
a) 12,3 b) 6,05 c) 8,40 d) 1,34
71 – Uma lente delgada convergente é utilizada para conjugar a
imagem real de um objeto posto a 40 cm, sobre o eixo focal
desta lente. Sabe-se que a altura da imagem produzida vale o
triplo da altura do objeto. Desta forma podemos garantir que a
vergência desta lente, em dioptrias, vale aproximadamente
a) 0,33 b) 3,3 c) 33 d) 333

72 – Uma carga positiva q = 10 – 6 C, de massa 2 x 10 – 14 kg,


penetra perpendicularmente em uma região do espaço onde


   B
78 – A velocidade linear da extremidade do ponteiro das horas 78 – As rodas de um automóvel, que podem ser consideradas
 circunferências, possuem um comprimento de 2,10 m. Se estas
de um relógio é de cm/s. Sabendo que este ponteiro tem efetuarem 240 rpm, calcule a velocidade de um ponto na
2700 periferia da roda, em m/s, admitindo que a rotação das rodas
10 cm de comprimento e comparando este relógio com outro constitua um movimento circular uniforme.
convencional que funciona corretamente (não adianta e nem
atrasa), podemos afirmar que, após 48 horas, a) 8,4 b) 16,8 c) 84,0 d) 168,0
a) ocorre um atraso de 8 h.
b) ocorre um atraso de 9,6 h. 79 – Um trator está puxando um bloco a velocidade constante
c) ocorre um atraso de 12 h. de 2m/s, utilizando uma força constante e horizontal de 100 N.
d) os dois apresentam o mesmo tempo decorrido. O peso do bloco é de 200 N. O coeficiente de atrito entre o solo
e o bloco é
79 – Na barragem representada na figura abaixo, sendo H a
altura da coluna de água represada, a pressão hidrostática no a) 0,1. b) 0,4. c) 0,5. d) .0,75
ponto P, a uma altura h, em relação à base, é
a) diretamente proporcional à H. 80 – Um motor de 5 HP, funcionando durante 8 minutos, produz
b) inversamente proporcional à h. um trabalho, em J, igual a (dado: 1HP = 746 W)
c) inversamente proporcional à H. P H
d) diretamente proporcional à a) 466,25. c) 29840,00.
diferença H – h. h b) 3730,00. d) 1790400,00.

81 – Um armário pesa 160 kgf e possui 4 pés com área de 4


cm2 cada um. A pressão, em kgf/cm 2, que cada pé exerce no
80 – Um fio retilíneo longo é percorrido por uma corrente de 10 piso horizontal onde se apóia, é
ampères, produzindo um campo magnético de intensidade 100
tesla a uma distância x do fio. O valor de x , em cm, é a) 2,5. b) 5,0. c) 10,0. d) 40,0.

Dado: permeabilidade magnética 0 = 4 x 10 –7 Tm/A 82 – O equilíbrio de uma coluna de mercúrio em um barômetro


a) 2,0 x 10 – 6 c) 2,0 x 10 – 10 de Torricelli não depende do (a)
b) 2,0 x 10 – 8 d) 2,0 x 10 – 12 a) gravidade local.
b) diâmetro do tubo barométrico.
2/2003-TURMA A c) densidade do mercúrio dentro da cuba.
d) pressão do ar sobre a superfície livre do mercúrio.
71 – Se um móvel executa um movimento circular uniforme, de modo
que percorra meia volta em 4 s, qual será sua freqüência em Hz? 83 – Em vasos comunicantes, líquidos não-miscíveis e de den-
a) 0,0125 b) 0,125 c) 1,25 d) 12,5 sidades diferentes apresentam
a) alturas que não dependem das densidades.
72 – Um ponto material desloca-se segundo o gráfico abaixo, b) superfícies livres no mesmo plano horizontal.
onde S é a posição, em metros, do ponto, e t o respectivo c) alturas inversamente proporcionais às suas densidades.
instante, em segundos. Sobre a trajetória do ponto podemos d) alturas diretamente proporcionais às suas densidades.
afirmar:
84 – Uma corda horizontal com uma extremidade presa a uma
parede é posta a vibrar pela outra extremidade. A distância
entre o primeiro e o quarto nó, da onda estacionária obtida, é 60
cm. O comprimento de onda, em centímetros, que dá origem à
onda estacionária, é de
a) 15. b) 20. c) 30. d) 40.
a) o móvel percorreu uma curva qualquer, com velocidade
variável entre zero e 2 segundos. 85 – ANULADA
b) o móvel percorreu um segmento de reta com velocidade
positiva entre 2 e 3 segundos. 86 – Considere os corpos A, B, C e D, indicados na figura
c) o móvel percorreu uma trajetória ora retilínea, ora circular. abaixo, colocados no vácuo. Sendo TA, TB, TC e TD as
d) não pode, a priori, ser definida. temperaturas dos corpos A, B, C e D, respectivamente, onde T A
 TB, TD  TC e TC  TB. Indique a alternativa que informa o modo
73 – Quando um sistema de forças concorrentes aplicadas num de propagação de calor
sólido puder ser reduzido a duas forças de mesma intensidade, a) somente irradiação. A B
mesma direção e sentidos contrários, diremos que o sistema b) somente condução.
a) possui movimento nulo. c) condução e convecção. D C
b) pode estar em equilíbrio. d) irradiação e convecção. C
c) com certeza está em equilíbrio.
d) com certeza não está em equilíbrio. 87 – Uma barra metálica de comprimento L0 a 0ºC sofreu um
1
74 – O ponto geométrico sempre no qual se pode considerar aumento de comprimento de do seu comprimento inicial,
concentrada toda massa do corpo (ou do sistema físico) em estudo é o 200
a) incentro. c) núcleo de inércia. quando aquecida a 125ºC. Pode-se dizer que o coeficiente de
b) centro de massa. d) centro geométrico. dilatação linear do metal, em ºC-1, é

75 – Um móvel percorreu 1/3 (um terço) de sua trajetória com a a) 2 x 10-10. b) 4 x 10-5. c) 6 x 10-4. d) 1 x 103.
velocidade média de 80 m/s. O restante do percurso, o móvel
completa com a velocidade média de 40 m/s. A velocidade 88 – Um chuveiro quando ligado à rede de 220 V exige uma
média desse móvel, em m/s, durante todo percurso, vale corrente igual a 11 A. A potência elétrica, em kW, desse chuveiro é
a) 44,0. b) 48,0. c) 53,3. d) 60,0. a) 0,20. b) 24,20. c) 20,00. d) 2,42.

76 – ANULADA

77 – A aceleração da gravidade, normalmente representada


pela letra g, SEMPRE é
a) uma grandeza vetorial.
b) uma grandeza constante.
c) a força gravitacional dos corpos em queda, no vácuo.
d) a constante da gravitação universal de Newton.
89 – O gráfico abaixo mostra uma expansão isotérmica de um gás 98 – As reações químicas são representadas através das
ideal, de uma situação 1 para uma final 2. Supondo que a área equações químicas, onde o número total de átomos presente
sob o gráfico p x V (onde p é a pressão e V é o volume) possa ser nos reagentes é igual ao número total de átomos presente nos
aproximada para a área de um trapézio retângulo, o calor produtos. Considerando que a equação da reação entre o ácido
envolvido no processo, em módulo, vale sulfúrico e o alumínio tem a seguinte representação:
a) 0,75 pV x H2SO4 + y Al z Al2(SO4)3 + w H2
b) pV
c) 0,5 pV A soma dos valores de x, y, z e w é igual a
d) 1,3 pV a) 10 (dez) c) 9 (nove)
b) 8 (oito) d) 7 (sete)

99 – Os elementos Ca e Cl, em certas condições podem originar


90 – Um edifício projeta uma sombra horizontal de 52 m de os íons Ca+2 e Cl-1. Isso ocorre porque
comprimento, a partir de sua base, quando o Sol está a 60º em a) o Ca "ganhou" elétrons e o Cl perdeu elétron.
relação ao horizonte. Nessas condições, a altura, em metros, do b) ambos os elementos "perderam" elétrons.
edifício vale aproximadamente (considere 3  1,73 e que os c) ambos os elementos "ganharam" elétrons.
d) o Ca "perdeu" elétrons e o Cl "ganhou" elétron.
raios do Sol "chegam" ao edifício praticamente paralelos).
100 – A formação das nuvens e a precipitação da chuva, entre
a) 90 b) 100 c) 110 d) 120 outros fenômenos, exige que haja
a) evaporação das águas dos oceanos, rios, lagos e mares e a
91 – Uma partícula de massa m e carga Q foi colocada entre
condensação do vapor d’água existente no ar.
duas placas carregadas, que geram um campo elétrico vertical b) a evaporação das águas dos oceanos, rios, lagos e mares e
ascendente de intensidade E. Sendo g a aceleração da
a fusão do vapor d’água existente no ar.
gravidade no local, é correto afirmar que para essa partícula c) a condensação das águas dos oceanos, rios, lagos e mares
permanecer em repouso deve se ter e a condensação do vapor d’água existente no ar.
mg . m . Qg . g . d) a solidificação das águas dos oceanos, rios, lagos e mares e
a) Q  b) Q  c) m  d) m 
E gE E QE a condensação do vapor d’água existente no ar.

92 – Um objeto real de 10 cm de altura está colocado 2/2003-TURMA B


frontalmente a 30 cm de uma lente convergente, perpendicular
ao eixo focal da mesma, cuja vergência é de 10 dioptrias. A 71 – Um corpo em queda livre percorre, a partir do repouso,
distância da imagem à lente vale, em cm, uma certa distância h1 nos dois primeiros segundos de queda.
a) 0,1. b) 10,0. c) 15,0. d) 30,0.
A distânciah 2 percorrida do início do terceiro ao final do sexto
93 – Dada a associação dos capacitores abaixo, a capacidade
segundo, será quantas vezes maior que h1 ?
equivalente entre os pontos A e B é dada por ______ F.
a) 2,7 8F 8F a) 4 b) 6 c) 8 d) 16
b) 8,0
c) 12,0 A
4F B 72 – Uma pessoa de massa 50 kg sobe uma escada de 20
    degraus, com 20 cm de altura cada degrau. A escada tem
d) 16,0
inclinação de 60º com a horizontal. Sendo g = 10 m/s2 e cos 60º =
8F 8F 0,5, o trabalho resistente, em joules, realizado pelo peso do homem
foi de 20
94 – Um fio reto é percorrido por uma corrente elétrica constante a) 1.000. 19
de intensidade 6,2 A. Calcule a intensidade do campo magnético, b) 2.000. 


em 10 – 7 tesla, a 20 cm do fio.Dado:  = 4  x 10 – 7 T. m/A c) 3.000.
2
d) 4.000.
Obs.: considerar o fio infinito 1

a) 31 b) 31  c) 62 d) 62  73 – Uma nave extraterrestre está orbitando o planeta Terra em


uma altitude na qual a aceleração da gravidade é 4,2 m/s 2. Tal
95 – Considere as seguintes afirmações a respeito de misturas. altitude, em 103 km, vale, aproximadamente,
I- Os gases sempre formam misturas homogêneas. Dados:
II- Água e sal comum sempre formam misturas homogêneas. 11
III- O álcool hidratado é uma mistura homogênea. G = const. univ. de gravitação  7,0 10 N.m2 / kg 2

Podemos dizer que são corretas as afirmações: M = massa da Terra  6,0 10 24 kg
R = raio da Terra  6.400 km
a) I e III. b) I, II e III. c) II e III. d) apenas I.
a) 3,6. b) 4,5. c) 6,5. d) 10.
96 – Em uma região onde existe um campo magnético uniforme,
são lançadas, perpendicularmente às linhas de campo três 74 – Consideremos um tijolo apoiado sobre sua face maior.
partículas A, B e C, com velocidades iguais, tal como a figura. De Colocando-o apoiado sobre sua face menor, cuja área é um
acordo com as trajetórias descritas pelas partículas, pode-se terço da maior, a pressão
afirmar que A, B e C tem, respectivamente, cargas elétricas a) triplica. c) nonuplica.
b) não varia. d) reduz-se a um terço.
a) positiva, neutra e negativa.
b) positiva, negativa e neutra. 75 – Um cubo de 10 cm de aresta e densidade absoluta 8,0 g/cm3
c) negativa, neutra e positiva. está apoiado por uma face sobre um plano inclinado de 60º em
d) negativa, positiva e neutra. relação à horizontal. Admitindo que o cubo esteja em repouso, a
pressão que este exerce sobre o plano é de ________ bárias.
Dados: cos 60º = 0,5
97 – A espécies químicas gás carbônico (CO2), hidrogênio H2), g = 10 m/s2 (aceleração da gravidade local)
oxigênio (O2), e sal comum (NaCl), são classificadas,
respectivamente, como substâncias puras: a) 40 b) 400 c) 4.000 d) 40.000
a) composta, simples, composta, simples.
b) composta, simples, simples, composta. 76 – ANULADA
c) simples, composta, composta, simples.
d) composta, simples, simples, simples.
77 – Um trovão foi ouvido por uma pessoa 15 s após esta ter 85 – Lançam-se, separadamente e em um campo magnético
visto o relâmpago. A que distância aproximada, em km, "caiu" o constante, duas partículas P e Q de mesma carga elétrica e
raio? velocidades perpendiculares a tal campo. Sabe-se que as
Dado: velocidade do som no ar = 340 m/s quantidades de movimento de ambas são iguais e permanecem
a) 2,4 b) 5,1 c) 510 d) 5.100 constantes no tempo. Admitindo que a massa de P é 10 vezes a
massa de Q, pode-se afirmar que a partícula de
78 – Um objeto real está colocado frontalmente e a 30 cm de a) maior período tem maior velocidade.
uma lente divergente de vergência 2 dioptrias. A razão tamanho b) maior freqüência tem maior massa.
de objeto por tamanho de imagem vale c) menor massa tem maior freqüência.
a) 5 / 8. b) 8 / 5. c) 15 / 8. d) 8 / 15. d) maior velocidade tem maior massa.

79 – Calcular a quantidade de calor, em calorias, que atravessa 86 – Certa onda, propagando-se no ar, possui um comprimento
uma placa de ferro de 3 cm de espessura em uma hora, sendo de onda igual a 10,0 cm e velocidade de propagação de 340 m/s.
de 1 cm2 a superfície da mesma e de 150ºC a diferença de Qual será o comprimento de onda desta, em metros, ao passar
temperatura entre as faces. para um meio onde a velocidade de propagação é de 1,36 km/s?
Dado: coeficiente de condutibilidade
K  0,12 cal .m 1.s 1 C 1 a) 0,04 b) 0,40 c) 2,50 d) 2.500
a) 216 b) 432 c) 648 d) 1.800 87 – Um grande mestre na área da Física se deparou com a
seguinte situação hipotética: possuía duas espiras circulares
80 – Um corpo, em repouso, está apoiado em um plano
inclinado. O peso deste corpo e sua componente paralela ao concêntricas E1 e E 2 , com raios r 1 10 cm e

citado plano estão na razão 2 :1. O valor do ângulo, em r 2  200 mm , respectivamente, percorridas por correntes de
graus, entre o plano inclinado e a direção horizontal vale sentidos opostos. Sabe-se que, na espira de diâmetro menor, a
a) 30º. b) 45º. c) 60º. d) 90º. corrente é de 20 A e que a intensidade (em módulo) do campo
5
magnético no centro das espiras é de 3,14 10 T . Nessa
81 – Dois móveis A e B percorrem a mesma pista circular com
movimentos uniformes, partindo do mesmo ponto e caminhando situação, quais devem ser as intensidades, em ampéres, das
no mesmo sentido. A velocidade angular de A é o triplo da correntes elétricas aplicadas na outra espira, visto que o mestre
velocidade angular de B e 0,5 s após a partida eles se encontrou dois resultados possíveis para o problema.
encontram pela primeira vez. A velocidade angular de B, em Dado: permeabilidade magnética do meio = 4  10 7 T.m A
rad/s, vale
considere   3,14
Dado:  = 3,14 a) 5 e 15 b) 5 e 30 c) 15 e 50 d) 30 e 50
a) 2,00. b) 3,00. c) 3,14. d) 6,28.
88 – Aquece-se uma certa massa de gás ideal a volume
 constante de 27ºC até 127ºC. Pode-se afirmar que a razão entre
82 – A figura a seguir mostra uma barra homogênea de peso Pe as energias cinéticas médias das moléculas, depois e antes do
comprimento AB = L. Esta barra possui uma articulação na aquecimento, é de
extremidade A e está em equilíbrio devido à aplicação de uma força
 3 .
4 . 127 . 27 .
F na extremidade B. Qual deve ser o valor do ângulo , em graus, a) b) c) d)
para que o equilíbrio seja mantido?
4 3 27 127
 
Dados: P  2 2N e F  3N 89 – N espiras circulares, geometricamente iguais e justapostas
(bobinas chatas), são percorridas por uma corrente de 2,0
 ampères. O raio dessas espiras vale 4 cm. Admitindo que a
a) 135º F 7
b) 120º  permeabilidade magnética (O) seja 4 10 T. m A , o valor
c) 60º
d) 45º
L 
B
da intensidade do campo magnético por espira, em 10 5 tesla, é

83 – Considere um ônibus em movimento e um observador, a)  / 2 b)  c) 2 d) 4


externo a este, em repouso. Se, num determinado instante, o
observador vê, dentro do ônibus,
A uma 30º
mosca voando em linha 90 – No gráfico a seguir, o trabalho, em atm.cm3, executado

reta ao longo do comprimento deste, podemos afirmar que a pelo gás entre os estados A e B vale
mosca possui uma velocidade cujo módulo é a) 4.
b) 8. P
Dado: considere que a mosca voa na mesma direção e no c) 16. A B
mesmo sentido do deslocamento do ônibus. d) 32. 4
a) nulo.
b) igual ao módulo da velocidade do ônibus. 4 8 v (cm3)
c) maior que o módulo da velocidade do ônibus.
d) menor que o módulo da velocidade do ônibus. 91 – Um corpo, inicialmente neutro, foi eletrizado negativamente
com uma carga elétrica de 7,2  10 15 C . Sabendo que o valor
84 – Durante uma competição de natação, em uma piscina
olímpica (50 metros), um nadador parte de uma das da carga elementar é de 1,6  10 19 C , pode-se afirmar que esse
extremidades desta piscina com uma velocidade inicial nula.
corpo _____________  10 elétrons.
3
Admitindo que o movimento do nadador, nesta prova, seja
retilíneo uniformemente variado, qual a aceleração aproxima-
da, em m/s 2, deste nadador ao chegar na outra extremidade a) ganhou 45 c) perdeu 45
da piscina? Considere que o tempo de percurso seja de 8 b) ganhou 90 d) perdeu 90
segundos.

a) 1,0 b) 1,5 c) 2,0 d) 2,5


92 – Dois automóveis A e B se deslocam na mesma direção, em 99 – Um tanque de um automóvel contém 50 L de álcool
um trecho retilíneo da estrada, em sentidos contrários e indo um hidratado, que é uma mistura constituída por 96% de álcool
ao encontro do outro, com velocidades constantes de 36 e 18 anidro e 4% de água (porcentagens em volume). Podemos
km/h, respectivamente. Após passarem um pelo outro, o afirmar que dentro desse tanque existe aproximadamente:
motorista do carro A observa, através do retrovisor (plano), o a) 48 L de álcool anidro e 2 L de água.
carro B se deslocando. Nessas condições, pode-se afirmar que b) 2 L de álcool anidro e 48 L de água.
a velocidade, em m/s, da imagem do carro B vista pelo motorista c) 25 L de álcool anidro e 25 L de água.
do carro A, pelo espelho, vale (considere o motorista do carro A d) 40 L de álcool anidro e 10 L de água.
como referencial)
100 – ANULADA
a) 15.
b) 30.
c) 54.
d) 108.

93 – Considere duas placas paralelas separadas por uma


distância d  16 mm , entre as quais se estabelece um campo
elétrico uniforme E  2  10 4 N C . Admitindo que um elétron
seja liberado, a partir do repouso, na extremidade da placa
negativa, determine a velocidade aproximada, em 107 m s , do
elétron, ao chegar à placa positiva.
19
Dado: Carga do elétron  1,6  10 C
31
Massa do elétron  9  10 kg

a) 1,1 b) 2,7 c) 5,2 d) 7,2

94 – Na associação de pilhas de 1,5 V do desenho abaixo, a


diferença de potencial entre os pontos A e B é, em volts,

a) 1,5
b) 3,0
c) 4,5
d) 6,0 B A

95 – Einstein supôs que a energia "E" do "pacote" (ou fóton) está


relacionada com sua freqüência "" e é dada pela equação: E = h.
Neste caso, se "" é dada no Sistema Internacional de Unidades
1
(SI), ou seja s , a unidade de "h", também dada no SI, é
1 2 2
a) J.s b) J. s c) J. s d) J. s

96 – Automóveis que ficam ao "relento" da noite para o dia,


amanhecem com suas superfícies "molhadas". Esse fenômeno
ocorre devido à

a) condensação do vapor de água que existe no ar.


b) condensação do vapor de água e do gás carbônico que
existem no ar.
c) solidificação do vapor de água e do gás carbônico que
existem no ar.
d) solidificação do vapor de água e condensação do gás
carbônico que existem no ar.

35
97 – O átomo do elemento cloro é representado por: 17 Cl ,
sendo que em certas condições forma o ânion monovalente
1
representado por Cl . Nesse ânion existem:
a) 18 prótons, 18 elétrons e 18 nêutrons.
b) 17 prótons, 17 elétrons e 18 nêutrons.
c) 17 prótons, 18 elétrons e 18 nêutrons.
d) 17 prótons, 18 elétrons e 19 nêutrons.

98 – As espécies químicas representadas pelas fórmulas


Al2 O 3 , H 3 PO 4 , KOH e Na 2 SO 4 pertencem,
respectivamente, às funções:
a) base, ácido, óxido e sal.
b) sal, ácido, base e óxido.
c) óxido, ácido, sal e base.
d) óxido, ácido, base e sal.
GABARITO DAS PROVAS DE FÍSICA e QUÍMICA
1/2001 - TURMA A 2/2001 - TURMA B 21 B 18 B
26 C 26 A 19 B
27 D 27 C 20 B
22 D
28 B 28 A 21 B
29 C 29 D 22 D
30 B 30 B 23 B 23 D
31 A 31 D 24 A
32 B 32 B 25 ANULADA
33 A 33 D 24 B 26 C
34 A 34 A 27 B
35 A 35 B 25 B 28 A
36 C 36 C 29 C
37 D 37 B 30 A
38 B 38 D 26 D 31 A
39 D 39 A 32 D
40 B 40 D 27 D 33 B
41 C 41 B 34 D
42 D 42 C 35 B
43 A 43 A 28 C 36 A
44 C 44 C 37 D
45 D 45 B 29 B 38 D
46 B 46 C 39 B
47 C 47 A 40 A
48 D 48 C 30 C
49 A 49 A
50 C 50 D 31 C

1/2001 - TURMA B 1/2002 - TURMA A


26 C 32 B
27 A
01 D
28 D 33 A
29 C
30 A 02 A
31 C 34 A
32 D
03 B
33 A 35 C
34 C
35 D 04 D
36 B 36 D
37 D
38 B 05 A
37 B
39 C
40 D 06 B
41 B 38 A
42 A
43 B 07 A
39 C
44 A
45 C 08 D
46 A 40 B
47 B
48 C 09 A
49 D
50 B 10 B

2/2001 - TURMA A
26 A 11 C
27 B
28 C 12 B
29 B
30 C
31 D 13 D
32 A
33 C 14 B 1/2002 - TURMA B 72 2/2002 - TURMA A
34 D 41 B
01 D
35 C 02 C 42 D
36 B 15 C 43 D
03 A
37 D 44 B
04 B
38 A 16 B 05 C 45 C
39 B 46 A
06 A
40 D 47 C
17 D 07 D
41 C 08 B 48 C
42 A 49 D
09 B
43 B 18 B 50 A
10 C
44 A 11 A 51 B
45 C 52 C
12 C
46 B 19 D 53 A
13 D
47 D 14 B 54 A
48 A 55 B
20 B 15 A
49 B 56 D
16 C
50 D 17 C 57 C

1
GABARITO DAS PROVAS DE FÍSICA e QUÍMICA
58 D 58 D 58 C 58 D
59 A 59 B 59 B 59 C
60 C 60 D 60 A 60 C
61 B 61 A 61 B 61 B
62 D 62 D 62 A 62 A
63 B 63 C 63 B 63 C
64 A 64 B 64 D 64 D
65 D 65 B 65 A 65 A
66 B 66 A 66 C 66 D
67 D 67 B 67 D 67 A
68 B 68 C 68 A 68 D
69 C 69 A 69 C 69 B
70 A 70 B 70 D 70 A
71 B 71 B 71 B 71 B
72 C 72 D 72 B 72 B
73 A 73 B 73 A 73 A
74 C 74 A 74 D 74 D
75 A 75 ANULADA 75 C 75 D
76 D 76 C 76 B 76 C
77 C 77 C 77 D 77 D
78 D 78 D 78 C 78 B
79 A 79 D 79 D 79 D
80 B 80 A 80 A 80 A

2/2002 - TURMA B 1/2003 - TURMA A 1/2003 - TURMA B 2/2003 - TURMA A


41 B 41 B 41 A 71 B
42 A 42 D 42 B 72 D
43 D 43 C 43 C 73 B
44 B 44 B 44 C 74 B
45 B 45 D 45 A 75 B
46 B 46 A 46 B 76 ANULADA
47 A 47 A 47 A 77 A
48 A 48 C 48 B 78 A
49 B 49 D 49 C 79 C
50 C 50 C 50 B 80 D
51 A 51 C 51 B 81 C
52 B 52 C 52 C 82 B
53 A 53 A 53 C 83 C
54 B 54 B 54 D 84 D
55 D 55 B 55 D 85 ANULADA
56 C 56 D 56 C 86 A
57 A 57 C 57 C 87 B

2
GABARITO DAS PROVAS DE FÍSICA e QUÍMICA
88 D
89 A
90 A
91 A
92 C
93 C
94 C
95 A
96 A
97 B
98 C
99 D
100 A

2/2003 - TURMA B
71 C
72 B
73 A
74 A
75 D
76 ANULADA
77 B
78 B
79 A
80 B
81 D
82 A
83 C
84 B
85 C
86 B
87 D
88 B
89 B
90 C
91 A
92 A
93 A
94 B
95 A
96 A
97 C
98 D
99 A
100 ANULADA

Você também pode gostar